101 Cases in Respiratory Medicine [1 ed.] 9789390020263, 9789352703111

In this rapidly changing global scenario, medical science has changed a lot. Clinical medicine cannot be confined to his

200 42 11MB

English Pages 443 Year 2018

Report DMCA / Copyright

DOWNLOAD PDF FILE

Recommend Papers

101 Cases in Respiratory Medicine [1 ed.]
 9789390020263, 9789352703111

  • 0 0 0
  • Like this paper and download? You can publish your own PDF file online for free in a few minutes! Sign Up
File loading please wait...
Citation preview

101 Cases in Respiratory Medicine

101 Cases in Respiratory Medicine

Supriya Sarkar  MD (Pulmonary Medicine) Professor and Head Department of Chest Medicine College of Medicine and Sagore Dutta Hospital Kolkata, West Bengal, India

The Health Sciences Publisher New Delhi | London | Panama

Jaypee Brothers Medical Publishers (P) Ltd Headquarters Jaypee Brothers Medical Publishers (P) Ltd 4838/24, Ansari Road, Daryaganj New Delhi 110 002, India Phone: +91-11-43574357 Fax: +91-11-43574314 Email: [email protected] Overseas Offices J.P. Medical Ltd 83 Victoria Street, London SW1H 0HW (UK) Phone: +44 20 3170 8910 Fax: +44 (0)20 3008 6180 Email: [email protected]

Jaypee-Highlights Medical Publishers Inc City of Knowledge, Bld. 235, 2nd Floor, Clayton Panama City, Panama Phone: +1 507-301-0496 Fax: +1 507-301-0499 Email: [email protected]

Jaypee Brothers Medical Publishers (P) Ltd 17/1-B Babar Road, Block-B, Shaymali Mohammadpur, Dhaka-1207 Bangladesh Mobile: +08801912003485 Email: [email protected]

Jaypee Brothers Medical Publishers (P) Ltd Bhotahity, Kathmandu Nepal Phone: +977-9741283608 Email: [email protected]

Website: www.jaypeebrothers.com Website: www.jaypeedigital.com © 2018, Jaypee Brothers Medical Publishers The views and opinions expressed in this book are solely those of the original contributor(s)/author(s) and do not necessarily represent those of editor(s) of the book. All rights reserved. No part of this publication may be reproduced, stored or transmitted in any form or by any means, electronic, mechanical, photocopying, recording or otherwise, without the prior permission in writing of the publishers. All brand names and product names used in this book are trade names, service marks, trademarks or registered trademarks of their respective owners. The publisher is not associated with any product or vendor mentioned in this book. Medical knowledge and practice change constantly. This book is designed to provide accurate, authoritative information about the subject matter in question. However, readers are advised to check the most current information available on procedures included and check information from the manufacturer of each product to be administered, to verify the recommended dose, formula, method and duration of administration, adverse effects and contraindications. It is the responsibility of the practitioner to take all appropriate safety precautions. Neither the publisher nor the author(s)/editor(s) assume any liability for any injury and/or damage to persons or property arising from or related to use of material in this book. This book is sold on the understanding that the publisher is not engaged in providing professional medical services. If such advice or services are required, the services of a competent medical professional should be sought. Every effort has been made where necessary to contact holders of copyright to obtain permission to reproduce copyright material. If any have been inadvertently overlooked, the publisher will be pleased to make the necessary arrangements at the first opportunity. Inquiries for bulk sales may be solicited at: [email protected] 101 Cases in Respiratory Medicine First Edition: 2018 ISBN: 978-93-5270-311-1

Dedicated to My friend, philosopher and guide, my teacher Late Professor Subal Kargupta and My beloved future generations of pulmonologists

Preface

As a teacher, I feel, it is my utmost duty to transmit my knowledge and experience to the future generations. I received lots of love, admiration and respect from my students. I have tried to teach them with my best capabilities. Often, I feel, in this teaching–learning process, I have learnt more than I have taught. In my long journey, I have received blessings and affections from my teachers. My teachers have facilitated and guided me in this long teaching–learning process. During this long journey, sometimes I like to believe, ‘I have mastered the art of clinical medicine’. But next day, some of my patients or students have unequivocally showed me the hard reality that I know nothing. Thereby I get enriched by my patients as well as by my students. With their love and respect, my patients by sharing their pains and sufferings have made my journey on clinical medicine interesting and challenging. In this rapidly changing global scenario, medical science has changed a lot. I personally feel that clinical medicine cannot be confined to history and clinical examination, but it should include interpretation of investigations on the backgroud of clinical setting. The book is intended primarily to the postgraduate students and residents. In this book, I have presented a longitudinal view of sufferings of 101 of my patients. I have narrated their story as they were unfolded before me, and I have included my successes as well as my mistakes also. To make readers understand my narratives, I have written chapters on ‘Clinical Approach Respiratory System’ and ‘Clinical Data Analysis’. This book is a small and humble endeavor from my side to transmit my knowledge and experience to future generations of pulmonologists. I will be happy, if my attempts reach them and help them in their day-to-day clinical practice. Education means a combination of knowledge, attitude and practice. Junior doctors have treasures of knowledge, but they do not know how to apply them. I think this book will help them in filling those gaps.

Supriya Sarkar

Acknowledgments

I acknowledge all my teachers Late Professor Subal Kargupta, Late Professor D Sen, Late Professor RS Bhakta, Late Professor Samir Kumar Saha, Professor AG Ghoshal, Professor M Maitra, Professor RS Mukherjee, Professor Sumita Mukherjee, Professor Somenath Kundu, Professor Atin Dey, Dr PS Bhattacharyya and Professor Arup Kumar Kundu, who have taught and encouraged me to enter into the field of clinical medicine. I am especially thankful to Professor Atin Dey, who has helped me in writing the chapter on clinical approach. I am also thankful to Professor Arup Kumar Kundu, who have encouraged me and taught me the art of writing. I acknowledge my colleagues of different medical colleges, where I have worked, i.e. Calcutta National Medical College, North Bengal Medical College and NRS Medical College, for their help and contribution in achieving diagnosis. I am thankful to my friend Professor Amitava Sengupta for two cases admitted in our college under his care. I earnestly thank Departments of Thoracic Surgery, Oncology, Radiology, Pathology, Microbiology and Biochemistry of medical colleges and private institutes for their contribution in those cases. I should be especially thankful to all my colleagues of college of Medicine and Sagore Dutta Hospital, for their cooperation in writing the book. I am thankful to my friend Dr S Guha, Thoracic Surgeon, for his help. I especially acknowlege my colleagues and staff of different private hospitals of Kolkata, for helping me to learn clinical medicine and for helping my patients with their knowledge, expertise and skills. I am thankful to my junior colleagues, junior doctors and postgraduate students, who have helped me to achieve diagnosis and thereby have enriched my knowledge. I am greatly thankful to my patients, who are the main contributors in the book. I express my gratitude for their love, respect and blessings. I specially acknowledge their help in my long teaching–learning process in clinical medicine.

Contents

1. Clinical Approach to Respiratory System

1

History Taking General Survey Examination of Respiratory System Examination of Other Systems

1 5 7 25

‰‰ ‰‰ ‰‰ ‰‰

2. Clinical Data Analysis ‰‰ ‰‰ ‰‰ ‰‰ ‰‰ ‰‰ ‰‰ ‰‰ ‰‰ ‰‰ ‰‰

First Step Second Step Third Step Fourth Step Fifth Step Sixth Step Seventh Step Eighth Step Ninth Step Tenth Step Some Fundamental Governing Theories of Clinical Medicine

26 26 27 27 28 28 29 29 30 30 30 31

3. Cases 33 Respiratory Diseases Discovered Accidentally by Chest Radiology ‰‰ Case 1: Asymptomatic Man with Hilar Calcification ‰‰ Case 2: Calcification in Solitary Pulmonary Nodule ‰‰ Case 3: Asymptomatic Spn in Smoker ‰‰ Case 4: Accidental Discovery of Irregular Calcification ‰‰ Case 5: An Adolescent Boy with Ill-defined Nodular Opacities ‰‰ Case 6: An Elderly Man Carrying Signs of Long-standing Pain ‰‰ Case 7: Accidental Detection of Bilateral Hilar Lymphadenopathy ‰‰ Case 8: A Young Lady with Chronic Nonspecific Symptoms ‰‰ Case 9: Accidental Discovery of Left-sided Air-fluid Level

33 33 35 38 41 43 46 48 51 55

Patients in Emergency with Respiratory Problem 59 Case 10: A Young Man had a Road Traffic Accident 59 ‰‰ Case 11: A Rare Cause of Shock 62 ‰‰ Case 12: Elderly Man with Acute over Chronic Respiratory Insults 66 ‰‰ Case 13: Pneumothorax with Reticulonodular Opacities 68 ‰‰ Case 14: A Case of Shock with Pleural Effusion 72 ‰‰ Case 15: Massive Hemoptysis with Past History of Pulmonary Tuberculosis 75 ‰‰ Case 16: Tuberculosis with Sudden Onset of Dyspnea 78 ‰‰ Case 17: A Young Girl with Sudden Onset Respiratory Distress 81 ‰‰ Case 18: Man with Fracture Neck Femur 83 ‰‰

xii

101 Cases in Respiratory Medicine Obstructive Airway Diseases ‰‰ Case 19: Middle-aged Housewife with Non-responding Dyspnea ‰‰ Case 20: Asthmatic Patient came with Hemoptysis ‰‰ Case 21: Unresponsive Chronic Obstructive Pulmonary Disease ‰‰ Case 22: Early Onset COPD or Late Onset Asthma ‰‰ Case 23: Frequent Exacerbations of COPD ‰‰ Case 24: A Simple Case Suddenly got Astonishing Attention ‰‰ Case 25: A Common Presentation of a Rare Disease in Adult ‰‰ Case 26: COPD with Rapid Change in Chest Radiology

87 87 90 95 98 102 105 108 112

Respiratory Problems in Intensive Care Unit Case 27: Elderly COPD Patient came with Leukocytosis ‰‰ Case 28: An Unusual Associate Disease in COPD ‰‰ Case 29: A COPD Patient with Type II Respiratory Failure ‰‰ Case 30: Ventilated Patient with Opaque Left Hemithorax ‰‰ Case 31: Masked Metabolic Acidosis in ICU ‰‰ Case 32: An Elderly Man with Metabolic Alkalosis ‰‰ Case 33: Young Asthmatic with Deteriorating ABG ‰‰ Case 34: Elderly Lady with Respiratory Acidosis ‰‰ Case 35: Obese Lady with Metabolic Syndrome ‰‰ Case 36: Guillain-Barré Syndrome with Ventilatory Failure ‰‰ Case 37: An Adolescent Girl with Hypoxic Respiratory Failure ‰‰ Case 38: Hypoxia with Bilateral Lung Infiltrates ‰‰ Case 39: Acute Onset of Dyspnea in SLE ‰‰ Case 40: Bilateral Consolidation in a Young Lady

117 117 119 123 126 129 131 134 137 140 144 148 151 156 159

Diffuse Parenchymal Lung Diseases Case 41: DPLD with Systemic Manifestation ‰‰ Case 42: A Classical Textbook Case of DPLD ‰‰ Case 43: DPLD with Changing Diagnosis ‰‰ Case 44: Importance of Untold Story ‰‰ Case 45: DPLD Presented Like Tuberculosis ‰‰ Case 46: Boy with Cervical Lymphadenopathy and Pneumothorax ‰‰ Case 47: Lady with Recurrent Pneumothorax ‰‰ Case 48: Lady with Peripheral Lung Opacities ‰‰ Case 49: Persistent Cough in Smoker ‰‰ Case 50: Persistent Cough in Rheumatoid Arthritis ‰‰ Case 51: Revisiting a Diagnosed Case ‰‰ Case 52: Sarcoidosis with Pleural Effusion

164 164 169 174 179 183 187 192 195 199 202 205 209

‰‰

‰‰

Tuberculosis 216 Case 53: A Boy with Progressing Pneumonia 216 ‰‰ Case 54: A Lady with Puerperal Fever 219 ‰‰ Case 55: A Middle-aged Lady with a Round Opacity 222 ‰‰ Case 56: A Young Man with Toxemia 225 ‰‰ Case 57: Life-threatening Episode Associated with Tuberculosis 229 ‰‰ Case 58: A Long Journey Seeking Treatment 236 ‰‰ Case 59: A Young Man with Non-responsive TB 239 ‰‰ Case 60: Man with Multiple Rib Erosions 242 ‰‰ Case 61: Persisting Cavity Despite Treatment 245 ‰‰ Case 62: Recurrent Right Paratracheal Lymphadenopathy 249 ‰‰ Case 63: Old Man with Pleuropulmonary Disease 253 ‰‰

xiii

Contents Pleural Diseases ‰‰ Case 64: A Classical Tuberculous Pleural Effusion ‰‰ Case 65: Hemorrhagic Pleural Effusion with Low Adenosine Deaminase ‰‰ Case 66: Changing Diagnosis ‰‰ Case 67: An Elderly Lady had a Concealed Truth ‰‰ Case 68: Right-Sided Loculated Empyema ‰‰ Case 69: Encysted Pleural Effusion with Liver Cyst ‰‰ Case 70: Thoracic Sequel of Past Sin ‰‰ Case 71: Bilateral Empyema Thoracic ‰‰ Case 72: Asymptomatic Lady wants to go Home ‰‰ Case 73: Boy with Hyperlucent Right Lung ‰‰ Case 74: A Case of Multiple Fluid-filled Cysts

257 257 260 263 266 269 274 278 281 285 287 291

Lung Infections Case 75: Typical Lobar Pneumonia ‰‰ Case 76: A Young Lady with Toxemia ‰‰ Case 77: Unresolved Pneumonia ‰‰ Case 78: Lady with Bilious Expectoration ‰‰ Case 79: A Case of Multiple Air-fluid Levels ‰‰ Case 80: Right Lower Lobe Lung Abscess ‰‰ Case 81: A Case of Gas under the Diaphragm ‰‰ Case 82: Old Man with Suspected Lung Cancer ‰‰ Case 83: Lady with Triangular Opacity Behind Heart

294 294 299 301 309 313 316 320 323 326

Thoracic Malignancy Case 84: Elderly Man with Right Lung Collapse ‰‰ Case 85: A Young Man Presented as Left-Sided Pleural Effusion ‰‰ Case 86: A Middle-aged Man Presented with Consolidation ‰‰ Case 87: Elderly Man with Left Upper Lobe Cavity ‰‰ Case 88: Localized Rhonchi in Adult ‰‰ Case 89: Man with Chest Wall Mass ‰‰ Case 90: Man with Amazing Chest Wall Swelling ‰‰ Case 91: Copd with Raised Left Dome of the Diaphragm ‰‰ Case 92: Diffuse Parenchymal Lung Disease with Severe Chest Pain ‰‰ Case 93: Lady Presented with Right-sided Fibrothorax ‰‰ Case 94: Man Diagnosed as Cervical Spondylitis ‰‰ Case 95: A Case of Superior Vena Cava Syndrome ‰‰ Case 96: A Middle-aged Man Presented with Recurrent Hemoptysis ‰‰ Case 97: Adolescent Girl with Unilateral Pleuroparenchymal Disease ‰‰ Case 98: Adolescent Girl with Anterior Mediastinal Mass ‰‰ Case 99: Young Man with Opaque Left Hemithorax ‰‰ Case 100: Atrial Septal Defect with Multiple Nodular Opacities ‰‰ Case 101: A Young Man with Nodules Behaving Differently

330 330 333 336 339 344 346 349 353 357 360 364 367 371 376 380 384 388 391

‰‰

‰‰

Index

399

Abbreviations

°C °F µg ABG ABPA ACTH ADA AECOPD AFB AFP AIDS AMI ANA anti-CCP anti-Scl 70 ARDS ART ATD AV malformation B BAL BiPAP BMI BOOP BP CAP CAT score CAT CBG CBNAAT CCAM CECT CMI COPD CPAP CT

-

Degree Centigrade Degree Fahrenheit Microgram Analysis Blood Gas Allergic Bronchopulmonary Aspergillosis Adrenocorticotropin Hormone Adenosine Deaminase Acute Exacerbation of COPD Acid Fast Bacilli Alpha Fetoprotein Acquired Immunodeficiency Syndrome Acute Myocardial Infarction Antinuclear Antibodies Anti-cyclic Citrullinated Peptides Anti-topoisomerase Antibody Acute Respiratory Distress Syndrome Antiretroviral Treatment Antitubercular Drugs Arteriovenous Malformation Basophils Bronchoalveolar Lavage Bi-level Positive Airway Pressure Body Mass Index Bronchiolitis Obliterans Organizing Pneumonia Blood Pressure Community-acquired Pneumonia COPD Assessment Test Score Category Capillary Blood Glucose Cartridge-based Nucleic Acid Amplification Test Congenital Cystic Adenomatoid Malformation Contrast-enhanced Computed Tomography Cell-mediated Immunity Chronic Obstructive Pulmonary Disease Continuous Positive Airway Pressure Computed Tomography

xvi

101 Cases in Respiratory Medicine

DAH DIP dL DOTs DPLD dsDNA DTH E EBUS ECG ELISA EPAP ESR ET tube ETB/E FDG-PET

-

FEF25-75%

-

FEV1 FEV1/FVC FiO2 FNAC FOB FVC GOLD HAART HAP Hb HCAP hCG HCO3HDL HIV HP HRCT I:E ratio IBW ICS ICU Ig ILD IMV INH/H

-

Diffuse Alveolar Hemorrhage Desquamative Interstitial Pneumonia Deciliter Directly Observed Treatment Short-course Diffuse Parenchymal Lung Disease Double-Stranded DNA Delayed-type Hypersensitivity Eosinophils Endobronchial Ultrasonography Electrocardiography Enzyme-linked Immunosorbent Assay Expiratory Positive Airway Pressure Erythrocyte Sedimentation Rate Endotracheal Tube Ethambutol 18 F-Fluoro-2-Deoxyglucose Positron Emission Tomography Forced Expiratory Flow Rate at 25–75% of the pulmonary volume Forced Expiratory Volume in 1st Second Ratio between FEV1 and FVC Fraction of Inspired Oxygen Fine Needle Aspiration Cytology Fiberoptic Bronchoscopy Forced Vital Capacity Global Initiative for Obstructive Lung Disease Highly Active Anti-retroviral Therapy Hospital-acquired Pneumonia Hemoglobin Health Care Associated Pneumonia Human Chorionic Gonadotropin Bicarbonate High Density Lipoprotein Human Immunodeficiency Virus Hypersensitivity Pneumonia High-resolution Computed Tomography Inspiratory-expiratory Ratio Ideal Body Weight Inhaled Corticosteroids Intensive Care Unit Immunoglobulin Interstitial Lung Disease Invasive Mechanical Ventilation Isoniazid

Abbreviations

IPAP IPF IRIS IV JVP K+ L L LABA LAM LAMA LDH LDL LFT LIP LVF LVH M M. MCTD MDCT MDR mEq Mg++ MGIT mL mmol mMRC MODS MRI MRSA N Na+ NIV NNRTI NRTI

-

NSAIDs NSCLC NSIP NT pro BNP O2 OAD OSA

-

Inspiratory Positive Airway Pressure Idiopathic Pulmonary Fibrosis Immune Reconstruction Syndrome Intravenous Jugular Venous Pressure Potassium Liter Lymphocytes Long-acting Beta-2 Agonist Lymphangioleiomyomatosis Long-acting Muscarinic Receptor Antagonist Lactate Dehydrogenase Low Density Lipoprotein Liver Function Test Lymphoid Interstitial Pneumonia Left Ventricular Failure Left Ventricular Hypertrophy Monocytes Mycobacterium Mixed Connective Tissue Disease Multi-detector CT Multi-drug Resistant Milli-equivalent Magnesium Mycobacterial Growth Indicator Tube Milliliter Millimol Modified Medical Research Council Multi-organ Dysfunction Syndrome Magnetic Resonance Imaging Methicillin Resistant Staphylococcus aureus Neutrophils Sodium Non-invasive Ventilation Non-Nucleoside Reverse Transcriptase Inhibitors  Nucleoside/Nucleotide Reverse Transcriptase Inhibitors  Nonsteroidal Anti-inflammatory Drugs Non-small Cell Lung Cancer Nonspecific Interstitial Pneumonia N-Terminal B-Type Natriuretic Peptide Oxygen Obstructive Airway Disease Obstructive Sleep Apnea

xvii

xviii

101 Cases in Respiratory Medicine

PA view PaCO2 PAH PaO2 PAS PCR PEEP PEFR PET- CT

-

pg PIE PLCH PMDT

-

PMF PO PPI Pplat PS PTE PZN/Z RA Factor RA RB RBC RB-ILD RICU RIF/R RNTCP RR SABA SABD SACE SAMA SaO2 SBT SCLC SGOT/AST SGPT/ALT SIRS SLE SM/S SPN

-

Posteroanterior View Partial Pressure of Arterial Carbon Dioxide Pulmonary Arterial Hypertension Partial Pressure of Arterial Oxygen Para-aminosalicylic Acid Polymerase Chain Reaction Positive End-expiratory Pressure Peak Expiratory Flow Rate Positron Emitting Tomography-Computed Tomography Picogram Pulmonary Infiltrates with Eosinophilia Pulmonary Langerhans Cell Histiocytosis Programmatic Management of Drug Resistant Tuberculosis Progressive Massive Fibrosis Per Oral Proton Pump Inhibitor Plateau Pressure Pressure Support Pulmonary Thromboembolism Pyrazinamide Rheumatoid Arthritis Factor Rheumatoid Arthritis Respiratory Bronchiolitis Red Blood Corpuscle Respiratory Bronchiolitis–Interstitial Lung Disease Respiratory Intensive Care Unit Rifampicin Revised National Tuberculosis Control Programme Respiratory Rate Short-acting Beta-2 Agonist Short-acting Bronchodilators Serum Angiotensin-converting Enzyme Short-acting Muscarinic Receptor Antagonist Oxygen Saturation in Arterial Blood Spontaneous Breathing Trial Small-cell Lung Cancer Aspartate Aminotransferase Alanine Aminotransferase Systemic Inflammatory Response Syndrome Systemic Lupus Erythematosus Streptomycin Solitary Pulmonary Nodule

Abbreviations

SpO2 SVC TB TBLB TBNA TNM TR grading TU Tv U/L UIP USG VAP VILI WBC XDR-TB ZN stain

-

Oxygen Saturation in Peripheral Blood Superior Vena Cava Tuberculosis Transbronchial Lung Biopsy Transbronchial Needle Aspiration Tumor-Lymph Node-Metastasis Tricuspid Regurgitation Grading Tuberculin Unit Tidal Volume Unit/Liter Usual Interstitial Pneumonia Ultrasonography Ventilator-associated Pneumonia Ventilator-associated Lung Injury White Blood Corpuscle Extensively Drug Resistant Tuberculosis Ziehl-Neelsen stain

xix

Introduction

“Pustakastha tu ja vidya para hastha gatam dhanam Karjya kale samutpanne na sa bidya na tat dhanam“ The shloka means bookish knowledge, like occupied wealth, is of no use at the time of needs. It is not possible to learn medicine from book/internet. For learning medicine, you have to go to the hospital wards and connect with patients. Only patients can teach you medicine and in return they demand a little care and empathy. Here teachers’ job is to facilitate the teaching–learning process. You have to listen to her story and you have to understand his clinical setting and then only you can learn medicine. Young doctors are often confused whether they should adapt the shortcut, easy and glamorous way of modern technologies, or they should take the long, difficult and cumbersome way of clinical medicine. My feeling is that those who can converge clinical medicine with modern technologies will be a master in the field of medicine. It is often wrongly perceived that clinical medicine will automatically die in the era of modern technology and instrumentalization. I beg to defer and I think clinical medicine is now becoming more relevant. Your every decisionmaking step will be challenged by modern technologies. There will always be a plethora of causes for a particular manifestation. You cannot ask for a battery of investigations due to socioeconomic constrain and other reasons. Any delay or unnecessary investigation will be challenged by patients. In the era of information technology, information is available everywhere. Anybody can just download them and learn them. So, at a particular time, on a particular disease, patient/patients’ relative may have gather more knowledge than the treating physician. Only your hold on clinical medicine and your interpretation skill will establish your supremacy. I truly feel, clinical medicine needs to be reinvented. Now, clinical medicine cannot be confined to history-taking and physical examination with inspection, palpation, percussion and auscultation. Clinical medicine should include interpretation of history, clinical findings and investigation results (chest X-ray, CT scan, ABG, spirometry, sputum report, pleural fluid reports and other relevant tests). Deep-seated lesions surrounded by normal lung parenchyma may not have any sign and only chest X-ray can unveil them. HRCT-thorax has revolutionized the outlook on DPLD. Obstructive airway diseases cannot be diagnosed without spirometry. The diagnosis of tuberculosis needs sputum examination. Analysis of pleural fluid reports is an essential step in pleural diseases.

xxii

101 Cases in Respiratory Medicine

My book has started with a small chapter on clinical approach to respiratory medicine followed by a chapter on clinical data analysis and finally by hundredone examples of cases, taken from life. These cases, I am presenting, are not mere case reports, but the pains and sufferings of some of my patients. I have gone step-by-step as these cases are explored along with my impression and background knowledge. I have also depicted my mistakes what, did and what I have learned from these mistakes. I request readers to be empathic to my patients and have an illuminating journey with them. I will be a proud facilitator in your learning process. Reading is of two types. Before examination, we read with aims to commit to memory and to vomit in the examination hall. The process is cumbersome, strenuous and boring. I believe that after getting degrees, the true examination begins where every patient will be our examiner. Secondly, reading is for learning and enjoying the process of getting a rim of knowledge. The process is a nice, charming and smooth journey. I want my readers to be in the second category. Please enjoy the world of clinical medicine. In my book, I am sharing my learning process with you, helps you in your day-to-day clinical practice, my endeavor would be successful.

Chapter

1

Clinical Approach to Respiratory System

HISTORY TAKING Introduction In respiratory system, history taking (the questionnaire part) is the most important part of clinical approach. It is often more helpful than clinical examination. Without proper history taking, clinical examination will be fruit less. As for example, uncomplicated pulmonary tuberculosis may not have any physical finding. Similarly a lung pathology surrounded by normal lung parenchyma like lung abscess, lung mass, etc. may not have any sign. Physical examination findings and investigation results should be interpreted on the basis of history. Clinical examination may suggest pleural effusion or highresolution computed tomography (HRCT) findings may suggest ILD but that means nothing. Proper history taking will help you in reaching a final complete diagnosis. History taking, the questionnaire part, should be done at least thrice and it is not a one–time job. The purpose of first time questionnaire is to have an idea about the disease, which system is involved, whether it is chronic or acute disease, what structure is predominantly involved, etc. The other purpose of first time history is to reduce the number of possibilities to a reasonable extent. Second time history is to be taken after completion of physical examination. As for example, if you get left-sided pleural effusion then specific questions should be asked to find out the probable causes of left-sided pleural effusion like pancreatitis. The points you have not asked previously. The second time history taking is to explore the possibilities that could explain the physical signs. A third time history is to be taken after completion of investigations. As for example, after investigation you get usual interstitial pneumonia, and then detail history is to be taken to exclude other causes of usual interstitial pneumonia (UIP) pattern before diagnose the case as idiopathic pulmonary fibrosis. The purpose of third time history is to reach a final diagnosis by interpreting clinical manifestations and investigation results. Investigation results may open up a new window and that may demand further clarification. History taking is an art and it depends on your communication skill. Sometimes you may have to ask very personal questions and that demands

2

101 Cases in Respiratory Medicine

a good rapport and trust. You must make patient comfortable before asking private questions. A good history taking requires knowledge and experience. Everybody cannot be a master of an art as it comes from within. Though history taking is an art, the process can be improved by a methodical approach. To ensure proper history taking a format has been designed and followed, worldwide.

Chief Complaints Patient may have many complaints or may have no complaint (patient may come with an abnormal chest X-ray done for routine check-up). Chief complaints are one or two (rarely three) important complaints among them and those will be determined by doctor not by patient. As for example, many elderly patients will complain of constipation and that cannot be their chief complaint. But in intestinal obstruction, constipation is the chief complaint.

History of Present Illness Onset of disease: History should start with how the disease has been started, the onset. Diseases may have insidious onset (when patient cannot point out when the disease has been started) and that indicates a chronic diseases like tuberculosis (TB), chronic obstructive pulmonary disease (COPD), interstitial lung disease (ILD), etc. Acute onset (patient can say when the disease has started) indicates acute disease occurring over days as for example, fever for 10 days. Sudden onset means patient can pin point the exact onset by time or by event (while reading newspaper I develop chest pain). Sudden onset means before a particular moment he was healthy. Sudden onset diseases are mainly vascular diseases like acute myocardial infarction, stroke, pulmonary thromboembolism (PTE), and pneumothorax, foreign body inhalation, etc. Progression: Secondly, history should include the progress of illness like disease progressed to reach a plateau and then decreases in severity (closed pneumothorax, collapse); continuous deterioration of disease (COPD, ILD), intermittent or episodic with intervening symptoms-free period (asthma), having sudden deterioration (acute exacerbation of COPD, ILD), or change in character of previous symptoms or addition of symptoms (bronchogenic carcinoma over COPD). Sequence of events: Thirdly, the sequence of events should be clearly noted even they are of same duration. Sudden onset pleuritic chest pain after a bout of cough and that is followed by dyspnea indicates pneumothorax. Duration of illness: Chronic diseases may persist for months or years (usually more than 6 weeks) like COPD, asthma, some ILD. Acute diseases usually last for days to weeks. In our country, TB should be suspected in all patients with cough for more than 2 weeks. Duration of illness more than 1 year usually excludes lung cancer except bronchioloalveolar cell carcinoma. Analysis of symptoms: There are six respiratory symptoms (cough, expectoration, hemoptysis, dyspnea, wheeze and chest pain) and few general

Clinical Approach to Respiratory System

symptoms (fever, swelling, weight loss, weakness, etc.) those are important in respiratory medicine. Permutation and combination of those six respiratory symptoms will give an idea about the disease. Dyspnea with wheeze usually indicates airway disease but sometimes the combination may be found in left ventricular failure and even in interstitial lung disease. Cough: Cough is the most common respiratory symptom. Cough may indicate infection, irritation, inflammation of airways, lung parenchyma, upper airways and pleura. Cough is a reflex with receptors present in the respiratory tract from pharynx to smaller bronchi and pleura. Center is in the medulla. During coughing respiratory muscles constrict vigorously with closed glottis; as a result intrathoracic pressure is increased; then glottis is suddenly opened up; and there is sudden expulsion of content of airways. Cough arising from larynx is harsh, barking and may be painful. Tracheal cough is harsh, dry and painful. Cough arising from airways (bronchitis) is usually associated with wheeze. It is important to know the onset, progression of cough, diurnal and seasonal variation of cough, and whether cough is associated with expectoration. Children, elderly, sick patients and often female patients cannot expectorate; then it is important to note the sound of cough to be sure whether it is actually dry cough. Expectoration: When expectoration is present then the amount (scanty, copious); color (whitish, pinkish, gray, yellow or green), character (serous, mucoid, mucopurulent or purulent), and odor (foul smelling) are to be noted. It is also important to know whether expectoration has a postural relation or diurnal variation. Macroscopic examination of expectoration often gives important clues: mucoid/mucopurulent sputum in COPD, bronchiectasis, lung abscess; scanty, blackish tenacious sputum in asthma, pink frothy sputum in left ventricular function (LVF), rusty sputum in pneumococcal pneumonia, etc. Hemoptysis: Hemoptysis may be frank hemoptysis (bronchiectasis, tuberculosis), blood streaks expectoration (tuberculosis, bronchitis), bloodstained expectoration (red currant jelly sputum in Klebsiella infection) and rusty sputum (pneumococcal pneumonia). Severity of hemoptysis is to be assessed by asking whether hemoptysis causes hemodynamic instability, demands hospitalization or blood transfusion. Chest pain: Pain arising from pleura is sharp, pin-pricking or stabbing in character, found in the lateral side of thorax that increased with deep inspiration and coughing. Mediastinal pain is usually dull-aching, poorly localized central chest pain. Myocardial pain is compressive or oppressive in the retrosternal area that increased with exertion and decreased with rest. Pericardial pain is due to inflammation of adjacent pleura (pericardium has no pain fiber) and has same characteristic of pleural pain. Lung has no pain fiber and pain in lung diseases is due to involvement of pleura, chest wall or mediastinum. The following points should be noted for chest pain: character of pain (pleuritic, dull aching); location of pain (central, lateral or back of chest wall), pattern of pain (built over minutes or immediately, fleeting) provoking

3

4

101 Cases in Respiratory Medicine

and alleviating factors, referred pain or radiation of pain, severity of pain, frequency or periodicity of pain, and associated symptoms. Dyspnea: Dyspnea is an unusually uncomfortable awareness of breathing. Dyspnea may be produced by disproportionate increase in work of breathing, or stimulation of receptors (J receptor) in the lung. It is said to be arising from signals in distorted muscle spindle. It is very important to differentiating dyspnea of cardiac origin from that of respiratory origin though it is often not possible. Cardiac dyspnea may be associated with exertion, palpitation, paroxysmal nocturnal dyspnea, orthopnea, etc. Dyspnea with wheeze suggests airway disease. Dyspnea with serous, mucoid or mucopurulent, scanty tenacious expectoration usually suggests lung disease. Whereas, pinkish and frothy expectoration suggests LVF. It is important to note the onset, duration, progression, aggravating and relieving factors and associated symptoms of dyspnea. Wheeze: Wheeze is a noisy sound occurring with respiration and usually occurs in obstructive airway diseases. It should be differentiated from stridor a predominantly inspiratory sound due to localized obstruction of upper air ways. Fever: Maximum normal oral temperatures are 37.2°C (98.9°F) at 6 AM and 37.7°C (99.9°F) at 4PM. Temperature above that should be considered as fever. Fever is classically described as continuous, remittent and intermittent. But in today’s era of antibiotics and anti-pyretic, those classical typing is rarely possible. Acute onset of high fever with cough suggests pyogenic respiratory infection. In high fever, due to liberation of exogenous pyrogens or endogenous cytokines thermal regulation center is set at a higher level; to achieve that goal body first preserves heat by peripheral vasoconstriction causing a sensation of chill, and then body produces heat by muscle constriction causing rigor. Fever with chill and rigor suggest organisms have entered bloodstream like viremia, bacteremia or parasitemia. Insidious onset of low-grade fever with cough suggests chronic infection due to tuberculosis, fungal disease. Evening rise of temperature is typically described for tuberculosis but it is not specific and it can be found in any chronic inflammation. It represents normal diurnal variation of temperature (minimum in the morning and highest in the evening). In low-grade fever, temperature crossed the threshold limit in the evening and thereby it is felt by patients. In chronic infection patient may be habituated with temperature and may not complain it. On the other hand, patient may have feverishness (sometimes an expression of weakness) not actual fever. It is always important to record temperature sublingually. Other symptoms: Swelling of face may occur in renal diseases, hypothyroidism, corticosteroid therapy (moon face) or superior vena cava (SVC) syndrome. In renal disease swelling usually starts from face; whereas in heart failure it starts from feet. In SVC syndrome swelling is associated with congestion whereas in other conditions swelling is associated with pallor. Pedal edema may be found in malabsorption in abdominal tuberculosis. Weight loss may be indirectly evident from loosening of garments. Weight loss is found in tuberculosis, lung

Clinical Approach to Respiratory System

cancer, acquired immunodeficiency syndrome (AIDS), anorexia and anxiety. Weight loss with normal or increased appetite may be found in diabetes mellitus or hypothyroidism.

Past History When present illness is not associated with or is a sequel of previous illness, the previous illness should be put in past history. When present illness is same or a continuation of previous illness then previous illness should be put in present history. Past history of major medical, surgical and gynecological illness should be taken. Special emphasis is to be given on past history of tuberculosis. It should be remembered that in our country TB is often over diagnosed. So, it is important to note how the tuberculosis was diagnosed; whether TB was diagnosed by sputum examination or by chest X-ray; how TB was treated, regime, drugs, doses, duration, intermittent or daily regime and from government or private set-up. It is utmost important to assess the regularity of ATD intake. History of other illnesses like measles, whooping cough, pneumonia, influenza, major trauma/accident, hospital admission, blood transfusion, etc. are to be taken. Some specific history should be asked for specific conditions/ diseases. As for example, history of malabsorption in cystic fibrosis, symptoms of meningeal irritation in miliary TB, etc.

Personal History Personal history should include the following things: 1. Smoking history including type of smoking (cigarette, biri, etc.); amount of smoking and duration of smoking (measured as smoke index or pack-year); duration of smoking cessation for ex-smokers. 2. Other addiction including alcohol, intravenous drugs, chewing tobacco 3. Marital history including offspring 4. Menstrual history 5. Present or previous occupation 6. Socioeconomic history 7. Exposure history and lifestyle 8. Habits, travels, pets, etc.

Family History Family history includes contact history with cases of tuberculosis; history of atopy or allergic diathesis in family; liver diseases for suspected alpha1 antitrypsine deficiency.

General Survey In general survey, some points are more important for respiratory medicine and they need special emphasis.

5

6

101 Cases in Respiratory Medicine

General look, signs of dyspnea, gross deformity: When patient enters your chamber the first look is very important. Following points are to be noted like patient coming walking without support, with support, by wheel-chair or by stretcher. Patient’s level of consciousness and orientation should be assessed while taking history. Signs of dyspnea should be looked for and those include tachypnea, activity of accessory muscles of respiration, suprasternal suction and intercostal suction. Any gross deformity and abnormalities should be noted. As for example, an amputation of limb may be due to osteosarcoma and present lung problem may be secondary to that. Facies, decubitus, built, nutrition, body mass index (BMI): Anxious facies is the usual presentation. A toxic and seek look indicates serious illness. Swelling of face and neck may be suggestive of SVC syndrome. Patient may be orthopnic or prefers lying down in bed or prefers a particular lateral decubitus position. It is always important to note why he preferred that particular position. It is important to calculate BMI (weight × height2) measured in Kg/m2. Normal BMI is between 19 to 26 and more than 30 is obesity. Nutrition is generally assessed by measuring BMI, muscle bulk by mid-arm circumference, subcutaneous fat by skin fold thickness over triceps or below the scapula and by looking for signs of vitamin deficiencies. Blood pressure (BP), pulse, respiratory rate, temperature: BP is important to assess whether patient is hypertensive (note history of antihypertensive medications) or hypotension (systolic 400 Hz, is found in obstruction in smaller airways. -- Low-pitched (sonorous) rhonchi with frequency < 200 Hz, is found in obstruction in large airways. Rhonchi/wheeze is now classified as: -- Monophonic wheezing consists of a single musical notes starting and ending at different times. It can be: ŠŠ Fixed monophonic: Wheeze has a constant frequency and a long duration; it is found in intrabronchial tumor, foreign body, bronchostenosis and mucus accumulation. ŠŠ Random monophonic: Wheeze has a varying frequency and duration presenting in both phases of respiration. It is seen in asthma. -- Polyphonic wheezing consists of multiple musical notes starting and ending at the same time and is typically produced by the dynamic compression of the large, more central airways. Polyphonic wheeze confined to the expiration only. It is found in COPD. −− Crepitation: Crepitation derived from Latin word ‘crepitare’, meaning crak or rattle. It is unambiguously used for nonmusical, short, explosive crackling sound that lasts less than 20 milliseconds. It is also described as moist-nonmusical, lathery sound mainly audible during inspiration. Some prefers to use crackles over crepitation. Fine crackles are produced within small airways, medium crackles are caused by air bubbling through mucus in small bronchi and coarse crackles arise from large bronchi or the bronchiectatic segments. -- Fine crackles are due to opening up of closed airways. Fine crepitation is softer, shorter in duration and higher in pitch than coarse crackles. Fine crackles are heard on mid to late inspiration and occasionally on expiration, unaffected by cough, gravity dependent and not transmitted to mouth. Late inspiratory crepitation previously called Velcro crepitation (sounds like opening of Velcro straps) is typically found in diffuse parenchymal lung diseases (interstitial lung diseases). It is also seen in pulmonary edema and pneumonia. -- Coarse crackles are heard on early inspiration and throughout expiration, affected by cough and transmitted to mouth. It is caused by bubbling of air through secretions. It is found in bronchiectasis, chronic bronchitis, fibrosis and cavities.

Clinical Approach to Respiratory System



The timing of crackles is also important and accordingly crackles are classified as: -- Early inspiratory crackles: Crackles start early in inspiration and usually end before the midpoint of inspiration. They are usually due to secretions within large airways and disappear on coughing. They are found in chronic bronchitis. -- Late inspiratory crackles: Crackles appear any time after the beginning of inspiration and last till the end of expiration. They are found in diffuse parenchymal lung diseases, pneumonia and pulmonary edema. -- Biphasic crackles: Crackles are predominantly inspiratory and continued during expiration. They are found in COPD, bronchiectasis and idiopathic pulmonary fibrosis. -- Crackles of bronchiectasis are found and present in both phases of respiration. Crackles start early in inspiration, continued to mid-inspiration and fade by the end of inspiration. In pneumonia, crackles are mid-inspiratory and fairly coarse. However, during resolution phase crackles are more late inspiratory and shorter in duration. -- Post-tussive crackles: They are not present normally on auscultation but appear after coughing as cough dislodges the thick secretions. They were classically described for tubercular cavities. They are also found in early pneumonia, lung abscess and lung parenchymal fibrosis. -- Post-tussive suction: It is a hissing sound heard over a collapsible cavity after a bout of coughing. −− Pleural rub: It is a friction sound, biphasic occurring at the end of inspiration and just after starting of expiration. The main difference between pleural rub and coarse crepitation is that it does not change in quality or quantity after coughing. −− Succussion splash: A splashing sound heard over the chest directly or by stethoscope at the air-fluid level with sudden sharp movement of thorax. This sign should not be elicited as it may push fluid from pleural space to lung in presence of bronchopleural fistula. −− d’Espine sign: It is described as bronchial breath sound and whispering pectoriloques heard over the spinous processes below T3 vertebrae in adult. It implies continuity between the main stem bronchus and the vertebrae by mass, usually by lymphadenopathy. The sign suggests malignancy, lymphoma, metastatic cancer, tuberculosis, sarcoidosis and other causes of lymphadenopathy. Reverse d’Espine sign: It is also described as anterior d’Espine sign. It is described as bronchial breath sound and bronchophony heard over supracardiac area due to large anterior mediastinal mass/ lymphadenopathy between trachea and sternum. Some important manifestations should be described as they are important in some diseases but not applicable in all types of cases.

23

24

101 Cases in Respiratory Medicine

Stidor: It is a loud high-pitched musical sound produced by the turbulent flow in the upper airways. It is louder over the neck than over chest wall. Stridor is mainly inspiratory sound. It is heard without stethoscope and may be heard from a distance. Stridor is inspiratory when it is associated with extrathoracic lesions (laryngomalacia, laryngeal and vocal cord diseases including tumors, tracheal stenosis including post-extubation lesions, etc.). Stridor may be heard over-expiration when it is associated with intrathoracic lesions (tracheomalacia, bronchomalacia, external compression of trachea and main bronchus). Coin test: It was described as a sign for tension pneumothorax. To elicit the sign—place a metallic coin flat against the chest wall just below the mid clavicle, strikes the coin with the edge of another metal coin with the help of an assistant or by patient himself, place the diaphragm of stethoscope at the opposite corresponding point in the posterior wall of chest of the affected side. Coin test is positive if high-pitched metallic and bell-like sounds are heard. It is occasionally found in pneumothorax, large cavity and large bulla. Scratch sign: The test can be done with patient in either sitting or supine position, place the diaphragm of stethoscope at the midpoint over the sternum, scratch with finger or a blunt object over lateral wall of both side of chest at equidistant points and sounds of two sides are compared. A positive sign consists of a considerably louder and harder sound on the side of pneumothorax. Hamman’s sign: The sign was described by Louis Hamman in 1939. The sign is found in pneumomediastinum and left-sided pneumothorax. The presence of free air between heart and chest wall produces a crunching, cracking sounds that are synchronous with each cardiac cycle and best heard over the precordium from 3rd to 5th intercostal spaces. The contraction of heart within the mediastinum leads to displacement of air bubbles and produces this classic raspy sound. Subcutaneous crepitation: Subcutaneous emphysema produces an unusual cracking sensation under the skin when pressed with fingers due to presence of air in the subcutaneous tissue. Subcutaneous/surgical emphysema over chest wall, neck and face is found in chest trauma, pneumothorax, rupture esophagus and commonly due to complication of intercostal tube drainage. Forced expiratory time (FET): It is a simple, inexpensive and sensitive test to detect airflow obstruction at bed side. FET is defined as the time taken for an individual to complete forceful exhalation after maximum inspiration. It is assessed by placing the bell of stethoscope in the suprasternal notch and measuring the duration of audible expiration. If the duration exceeds 6 seconds, it indicates obstructive airway diseases. A FET less than 5 seconds indicates FEV1/FVC ratio more than 60%, a FET more than 6 seconds indicates FEV1/ FVC ratio less than 50%.

Clinical Approach to Respiratory System

EXAMINATION OF OTHER SYSTEMS Examination of Cardiovascular System Special emphasis should be given on pulmonary component of second heart sound (P2) and its spitting. In pulmonary hypertension, P2 is loud, may be palpable and spitted but not fixed (the gap between aortic and pulmonary components of second heart sound varies with phases of respiration). Parasternal heave and epigastric pulsations may be found in cor pulmonle. Cardiac percussion is now obsolete as it has the risk of dislodging a clot. In respiratory medicine deep percussion over left and right parasternal line is sometimes required to assess shifting of heart when apical impulse is not palpable. Gallop heart beat (suggest heart failure) and murmur (suggestive of underlying heart disease) if present should be noted. Pulmonary ejection systolic murmur and tricuspid regurgitation murmur may be heard in corpulmonale.

Examination of Gastrointestinal, Lymphoreticular and Genital System Special emphasis should be given on venous prominence over abdomen that is suggesting vena caval obstruction or cirrhosis; hepatomegaly (lymphoma, disseminated TB, sarcoidosis, drug-induced hepatitis, liver metastasis); splenomegly (lymphoma, disseminated TB, sarcoidosis, cirrhosis); ascities (may cause right sided pleural effusion and abdominal TB may present with ascities); signs of pancreatitis (in left-sided pleural effusion) and any abdominal lump. Examination of lymphoreticular system includes examination of lymph nodes, liver, spleen, skin for signs of hemorrhage and sternal tenderness. Female genital tract including breast in female and testes in male should be examined.

Examination of Nervous System Level of consciousness and abnormal behavior should be assessed. Detailed neurological examination is not usually necessary and sometimes specific signs are to be elicited if they are indicated. As for example, signs of meningeal irritation for disseminated TB, signs of myasthenia gravis for anterior mediastinal syndrome, signs of metastasis and paraneoplastic syndrome for bronchogenic syndrome, and signs of paraplegia for spinal TB.

25

Chapter

2

Clinical Data Analysis

Introduction The art of medicine is a combination of medical knowledge, intuition, experience, and judgment. Medical decision-making is an important responsibility of physicians and it is essential at each stages in diagnosis and therapeutic process. Clinical data analysis is a systematic effort to achieve a provisional diagnosis and ultimately a final complete diagnosis. The process will help to narrow differential diagnosis to least number of possible probabilities and to plan least number of investigations. Clinical data analysis requires knowledge, experience, analytic skill and most importantly time to spend with patient. Clinical data analysis also requires clinical expertise, a combination of scientific knowledge and problem solving process. For better understanding of the complicated and ill-defined process, I am discussing clinical data analysis in steps. I am hopeful that those steps will help you in day-to-day clinical practice.

First step Collection of Clinical Information Collection of clinical data from patient and patient’s relatives (detail sleep event or daytime sleepiness) is the essential and most important step. It should be informative, complete and perfect; otherwise next steps will be ineffective. It is important to remember that we are dealing with a human being who has sentiment, personality, ego and self-respect. While taking history we may have to ask personal questions. We should not ask personal questions bluntly but make them comfortable by starting with some superficial questions. As patient become comfortable, we can proceed to ask personal questions after explaining the necessity of that information. Always be emphatic and remember that patients have come for help in their time of distress. A good history taking needs a clear knowledge and understanding of diseases. A gentle, emphatic, respectful and charismatic personality is essential for taking a good history. Similarly, during examination we must be careful and should not hurt patients’

Clinical Data Analysis

sentiment, privacy and ego. While examining private areas always explain him/ her about what, how and why this is done. For better elicitation of physical signs, it is essential that both patient and doctor are comfortable at the time of examination. As for example, important abdominal findings may not be elicited if patient makes his/her abdomen tough. Chest X-ray is now considered as a part of clinical examination. Many lung diseases, particularly in deep seated lung pathology surrounded by normal lung parenchyma, may not yield any clinical sign. In those situations chest X-ray can give an initial idea about the disease that can suggest future investigation protocol. It is important to remember that history (questionnaires) has to be taken at least thrice. Firstly, at the beginning usually history is taken to have an idea about the disease. Secondly, after completion of physical examination some question should be asked. As for example, manifestations of pancreatitis in left sided pleural effusion. Thirdly, after completion of investigations some questions or points need to be clarified. As for example, drug history in patients with usual interstitial pneumonia (UIP) pattern in high-resolution computed tomography (HRCT) thorax. Collection of relevant clinical information as much as possible is essential step before going to next step.

Second step Piercing through Individual Manifestation After collecting informations, next step is to critically analyze each symptom or sign. As for example, a chest pain may arise from chest wall, pleura, mediastinum, myocardium or pericardium. The probable origin of chest pain is to be determined by a detailed review of chest pain. Similarly each sign has to be assessed for its clinical significance. As for example, crepitation may appear from large airways, small airways, alveoli or interstitium, and every effort is to be made to determine the origin of crepitations with some degree of reasonable certainty.

Third step Grouping of Clinical Data After analyzing each manifestation, the next step is to group few symptoms and signs into a meaningful combination that will reflect some pathology. As for example, fever with chill, impaired percussion node without shift of mediastinum, and bronchial breath sound with crepitation indicate consolidation. A combination of cough without expectoration, dyspnea without wheeze and bilateral mainly basal end inspiratory crepitation suggest interstitial lung disease. Similarly a collection of signs and symptoms will indicate pleural effusion or superior vena caval obstruction. Consolidation, pleural effusion, superior vena caval obstruction or interstitial lung disease are not a disease but a pathological change. Identification of pathological change

27

28

101 Cases in Respiratory Medicine

is helpful in planning further investigations and it also helps in symptomatic management of patient.

Fourth step Understanding the Course of Clinical Manifestation Clinical course means onset, duration, progression, interval and chronology of manifestation. Onset may be acute or chronic. Acute onset when patient can tell when the disease has started like fever for 10 days, and that suggest acute inflammation like pneumonia. Insidious onset means when patient cannot tell when the manifestation started as for example, patient is telling that he has fever for 2–3 months (as if 2 or 3 months is equal), that may suggest chronic inflammation like tuberculosis. Sudden onset is a subset of acute onset when patients can pin-point the moment from where the disease has been started. Examples of sudden onset diseases are pneumothorax or any vascular event like pulmonary thrombo-embolism. Duration-wise, clinical manifestations are classified into acute, subacute and chronic. Cough with purulent expectoration and local crepitations may indicate lung abscess if history is of shorter duration but same manifestations persisting for longer duration will point towards bronchiectasis. Symptoms may be progressive without symptom-free interval with or without episodes of exacerbations as in case of chronic obstructive pulmonary disease (COPD), interstitial lung disease (ILD) cardiomyopathy, etc. On the other hand, disease may be episodic with symptom-free intervals as in case of asthma. Chronology of manifestation is also helpful as for example, a sudden onset pleuritic chest pain, sometimes preceeded by bouts of cough, that is followed by dyspnea is almost suggestive of pneumothorax.

Fifth step Dealing with Positive and Negative Clues Proper identification and analysis of every positive or negative clue is an important step. Identification of overt and covert risk factors is a distinctive step. As for example, for the diagnosis of COPD it is essential to take history of smoking (overt clue) and biomass fuel exposure (covert clue). Similarly, in a tuberculosis patient with persistent hypotension, finding of pigmentation of tongue or oral mucosa suggest Addison’s disease due to bilateral adrenal gland involvement. Negative clues are equally important. As for example, in a case of pleural effusion the absence of pleuritic chest pain, a sign of pleural inflammation, may point against tuberculosis and favors malignancy. Absence of crepitation including post-tussive crepitation practically rules out lung parenchymal fibrosis. Negative clues should be analyzed with caution as absence of obliteration of Traube’s space usually excludes left sided pleural effusion but that does not completely rule out encysted pleural effusion.

Clinical Data Analysis

Sixth step Pattern Identification When a pathologist examines a slide under microscope, he identifies certain pattern like changes in nucleus, nucleolus, cytoplasm, nuclear-cytoplasmic ratio, cell division, surrounding structures, etc. and arrives at a diagnosis. At that time he is visualizing a cytological/histological pattern where each finding is important but overall pattern is more important. Similarly clinicians also identify clinical pattern where each manifestation is important but overall combined clinical picture is more important. Acute onset of fever with chills and single rigor; pleuritic chest pain; scanty rusty expectoration in a toxic patient with signs of lobar consolidation suggest pneumococcal pneumonia. Scoring systems for diagnosis of certain disease, like allergic bronchopulmonary aspergillosis, may be considered as scientifically designed form of pattern identification. A male patient with positive diagnostic scoring for rheumatoid arthritis develops pleural effusion; we should consider rheumatoid pleural effusion along with other possibilities. But similar clinical picture in a female patient rarely indicate rheumatoid effusion as pleural involvement in rheumatoid arthritis is less common in female. Pattern identification needs knowledge as well as experience. Experience may be acquired directly by observing cases or may be acquired by reading books or case reports.

Seventh step Clinical Setting Every clinical manifestation should be analyzed on the background of clinical setting. It is not sufficient to identify a clinical pattern but we should interpose the pattern on the background of clinical setting. As for example, a 60 years smoker with superior vena caval obstruction, we should suspect lung malignancy. On the other hand, a 30 years nonsmoker with superior vena caval obstruction with cervical lymphadenopathy points to lymphoma. Hoarseness of voice in an 18 years asthmatic on steroid inhaler should point towards adverse reaction to inhaled steroid; hoarseness of voice in a political leader before election points towards voice abuse and a hoarseness of voice in a 60 years smoker points to recurrent laryngeal nerve paralysis in lung cancer.

Epidemiology of Disease Epidemiology is not a Para-clinical subject but all physicians are using the epidemiological data as an assisting tool in clinical medicine probably not appreciating its influence. Viral infections are often identified by their epidemiological pattern like influenza, swine flu, hepatitis, etc. A right upper lobe fibro-cavitary disease may suggest tuberculosis in India, melioidosis in Vietnam or histoplasmosis in northern America. Antibiotic selection depends on local epidemiological pattern of organisms and their sensitivity pattern.

29

30

101 Cases in Respiratory Medicine

EightH step Application of Clinical Expertise Clinical expertise is the most complicated thing to explain. Clinical expertise is a combination of scientific knowledge, experience, clinical prudence, analytic and problem solving capability. Clinical expertise requires not only cognitive proficiency, integration of verbal and visual informations but also complex fine motor skills necessary for invasive and non-invasive procedures. Clinical expertise can be acquired through experience. As you come across a disease repeatedly, you acquire experience on that particular disease and subsequent diagnosis will be easier. During a malaria epidemic in Kolkata I have diagnosed a case of ARDS in vivax malaria, an unusual condition. That experience makes me well conversant to diagnose subsequent cases of ARDS in vivax malaria easily and confidently. We learn more from our failures than our successes. Failures should not be ignored but critically analyzed. Successes occupy our conscious domain but failures remain in our unconscious memory. As for example, most of us at least once have wrongly diagnosed left ventricular failure as asthma. That can probably explain, why we are still using the term ‘cardiac asthma’, a terminology that has been removed from textbooks of medicine.

Ninth step Clinical Reasoning We know that during clinical examination some incidental findings may be uncovered that is completely unrelated to the clinical problem at hand. Clinical reasoning is the skill for deciding whether a clinical clue is worth pursuing or should be ignored in totality. Every clinical manifestation may not be explained but how much importance that deserves should be appraised properly. In some cases we cannot ignore an apparently insignificant finding and try to explain that finding. As for example, anemia in COPD is usually considered as its systemic manifestation. But in some cases anemia in COPD should be investigated as patient may has bleeding piles or associated hematological problems.

Tenth step Clinical Judgment It is a process that will help in selecting investigation or treatment protocol. Clinical judgment is a process of weighing whether a proposed test, management protocol or preventive measure entails a greater risk than the disease itself. Whether bronchoscopy will be helpful in a particular hypoxic patient? Whether patient can tolerate open lung biopsy? Whether patient can tolerate chemotherapy? Risk- benefit or cost-effective analysis is a part of clinical judgment. It is an essential skill that every clinician must possess and apply.

Clinical Data Analysis

It is important to follow up and keep surveillance over patients. This is another important step to learn medicine. Clinical medicine is a long journey, do not stop earlier.

Some fundamental governing theories of clinical medicine Clinical Bias Common diseases occur commonly and rare diseases occur rarely. Consider common disease first and you will be right in most occasions. There is a correlation between pre-test probability and post-test result. Clinical medicine is not an exeption from that rule. Suppose a patient has right upper lobe fibrosis, it can be anything but the chance of tuberculosis will be the highest in our country. A specialist has to deal with cases when others have failed. So, we cannot ignore less common probabilities. We should consider asthma in a 30 years, non-smoker female with cough, dyspnea and wheeze. We cannot ignore the possibility of vocal cord dysfunction, not an uncommon condition in adult female, that can mimic asthma or may be associated with asthma. In clinical medicine, never be biased and always start unbiased.

Explain by Single Pathology In clinical medicine the governing rule is try to explain all manifestations by a single disease. So, start with the hypothesis that patient is suffering from a single disease. When explanation of some manifestations is not possible by a single disease then think of associate disease or comorbidities. It is now a well-accepted fact that elderly person may have inflammatory involvement of multiple organs, a condition known as ‘inflam-aging’. So, an elderly man may have COPD, ischemic heart disease, diabetes, hypertension and lung cancer. IHD is a common comorbidity of COPD that can determine its outcome. It may be useful to start thinking of a single disease but never be dogmatic.

Diseases Flock Together Some diseases have inherent friendship and occur together. They may be linked together by same etiology or same pathological process. When one disease is identified then it is mandatory to search for other friendly diseases. A person can suffer from COPD and ischemic heart disease as both are related to smoking. Asthma patients usually have other allergic diseases like allergic rhinitis, allergic skin conditions like eczema/urticaria, etc.

Evidence-based Medicine Today is the era of evidence-based medicine. Though there are reservations on how and where evidences are generated, we cannot ignore evidence-based medicine in today’s world. The process of medical decision-making should be guided by evidences so that patient can be benefited from the available scientific knowledge. But despite the importance of evidence-based medicine,

31

32

101 Cases in Respiratory Medicine

some medical decision making relies on clinical judgment, an attribute that is difficult to quantify or even more difficult to assess qualitatively.

conclusion Clinical medicine is an art of possibilities and probabilities. It is an interesting and challenging job. Clinical medicine is a journey where surprise is waiting for you at every bends. Please enjoy the challenge and learn the art throughout your life as there is no end to the journey. Please remember, ‘student life is the best part of your life’. Please remain student throughout your life. When you can combine clinical medicine with modern technologies you will be a good clinician.

Chapter

3 Cases

Respiratory Diseases Discovered Accidentally by Chest Radiology Case 1: Asymptomatic Man with Hilar Calcification Case Report A 45-year-man had upper abdominal pain and ultrasonography (USG) detected gallstones. Surgeon planned cholecystectomy. For preoperative checkup a chest X-ray postero-anterior (PA) view was done and that showed some abnormalities. He was referred to chest department for opinion. He was a non-smoker and did not have any respiratory complain or past history of respiratory disease. His general survey and respiratory system examination did not reveal any abnormality. His chest X-ray PA view (Fig. 1)

Fig. 1: Chest X-ray PA view showing primary complex with large hilar (long arrow) and small parenchymal calcification (small arrow)

34

101 Cases in Respiratory Medicine

showed an irregular calcification at right hilum suggestive of calcification of right hilar lymph nodes. A careful search into the X-ray detected an irregular small calcified opacity in the right mid-zone. Lung parenchymal calcification was smaller than right hilar gland calcification.

Discussion Tuberculosis (TB) is caused by Mycobacterium tuberculosis and seven other closely related species of mycobacterium (M. bovis, M. africanum, M. microti, M. caprae, M. pinnipedii, M. canetti and M. mungi). TB bacilli spread as droplet nuclei (1–5 microns) generated during coughing, sneezing and talking of TB patients (pulmonary and laryngeal). Each aerosol droplet contains 1-400 bacilli. When a person inhales the droplet nuclei, bacilli reach alveoli of the lungs. In first time infection, human immune system is not ready to deal with them. After initial nonspecific immune response, alveolar macrophages come into action; they engulf bacilli and phagosomes are formed. Bacilli can survive within phagosomes either by preventing their fusion with lysosomes or by surviving within phagolysosomes. The capacity of bacilli to survive within adverse situation is due to the peculiarity and integrity of their cell wall. Cell wall of TB bacilli consists of a peptidoglycan layer covered by arabino-galactan layer, and they are covalently linked to an outward layer of mycolic acids and long chain fatty acids. This cell wall skeleton forms a waxy, non-fluid barrier that restricts penetration of both hydrophobic and hydrophilic molecules. TB bacilli multiply within macrophages and ultimately macrophages burst. Bacilli then spread to regional lymph node and reach systemic circulation via thoracic duct. Thereby bacilli spread to different organs of human body. In the meantime, antigen presenting cells (dendritic cells and alveolar macrophages) come into action; engulf and carry bacilli to hilar lymph nodes; and present them to TH1 lymphocytes. Ultimately cell mediated immunity (CMI) and delayed-type hypersensitivity (DTH) develops by 2 to 6 weeks time. There is a basic difference between CMI and DTH. CMI is due to cytotoxic T lymphocytes and natural killer cells capable of lysing macrophages infected with TB bacilli and killing individual bacteria. DTH kills bacilli en masse along with destruction of surrounding tissues and thereby forming caseous necrosis. Tubercular granuloma contains central caseous necrosis surrounded by epithelioid cells, Langerhan giant cells and T lymphocytes with an outer layer of fibroblasts. TB bacilli are contained within granuloma by acidic pH, poor oxygen level and toxic fatty acids. Primary complex consists of a larger glandular component and a smaller parenchymal component. Previous textbooks described an additional 3rd lymphatic component of primary complex. Fate of primary complex depends on the balance between virulence of bacilli on one side and CMI on the other side. More than 90% of primary complex heals spontaneously and they may be calcified. Within the focus bacilli remain dormant for long time. In favorable condition they may start multiplying causing post-primary TB. Primary complex may progress causing

Cases

progressive primary TB; it may rupture into surrounding structures causing pleural effusion, pericardial effusion; hilar lymph nodes may compress bronchus in children causing consolidation and collapse of distal lung (epituberculosis); in low immunity state it may progress to military TB; and focus in different organs may be activated causing TB of that organ or surrounding structures like meningitis. In our country, about 38% people are infected with TB and infection usually occurs in childhood. Primary TB may be totally asymptomatic or may cause symptoms like viral fever. In most of the cases, they heal spontaneously and remain asymptomatic. Primary TB can be identified by positive tuberculine test. Healed primary complex can be accidentally detected by chest X-ray as calcified hilar glandular and lung parenchymal lesions. Characteristically glandular components are much more prominent than lung parenchymal component. Approximately 10% My Learning of healed primary complexes are activated in individual’s lifetime. In persons living Radiological findings of primary with HIV, the rate of activation increased to complex, a common condition in our country, is typical but we are about 10% per year. In general no treatment less aware of. is required for healed primary complex.

Management We diagnosed the case as healed primary complex, reassured the patient and sent him back to surgeon for gallbladder surgery.

Case 2: Calcification in Solitary Pulmonary Nodule Case Report A 50-year-male smoker had right sided inguinal hernia. For preoperative checkup he had an X-ray chest (Fig. 2), and that showed a solitary pulmonary nodule (SPN) in the right mid-zone. He was referred to department of pulmonary medicine for preoperative checkup. On enquiry he told that he had cough with little mucoid expectoration for more than 2 years. He was a smoker and he smoked about 10 cigarettes per day for more than 20 years. He ignored cough as smoker cough. He denied any other respiratory symptom including dyspnea, wheeze, chest pain or hemoptysis. His general survey and examination of respiratory system did not reveal any abnormality. Examination of sputum was negative for acid fast bacillus (AFB). Spirometry was done as preoperative checkup as he was a smoker and spirometry findings were within normal limits.

Discussion Solitary pulmonary nodule is defined as a single nodule that is not more than 3 cm in diameter usually surrounded by lung parenchyma. Opacity more than

35

36

101 Cases in Respiratory Medicine

Fig. 2: Chest X-ray PA view showing solitary pulmonary nodule with popcorn calcification

3 cm in diameter is generally considered as mass lesion. Causes of SPN are malignant neoplasm, benign neoplasm, inflammatory granulomas and some less common causes like arteriovenous malformation, pulmonary infarct, lung cysts, etc. SPN is an important clinical problem and main issue here is to rule out malignancy. SPN needs a detailed evaluation as early detection of malignancy and surgical management will improve its prognosis. SPN should be carefully examined regarding its size, shape, border, density, homogeneity, calcification and rate of growth. Previous X-ray if available should be examined and compared with present X-ray to assess the rate of growth (tumor doubling time). Tumor doubling time is the time taken by the tumor to double its volume (measured as 1.25 time increase in diameter of tumor). Less than a month doubling time usually indicate inflammatory lesion whereas a doubling time more than 18 months indicates benign tumor. Doubling time in between should be evaluated for malignancy. No change in tumor size when compared with old X-ray done before 1 year or more, usually suggests benign tumor. Regarding size, incidentally detected SPNs of less than 6 mm diameter are usually benign and more than 3 cm goes in favor of malignant tumor. Lobulated structure indicates benign lesion. Distinctly smooth border goes in favor of benign lesion. Satellite lesions are also important as they may give some clue, and they are usually found in tubercular lesion. Calcification within SPN usually indicates benign lesion but does not rule out malignancy, particularly in eccentric calcification. Specific types of calcification as central, diffuse, stippled, laminar, concentric or popcorn calcification indicate benign lesion. High-resolution computed tomography (HRCT) thorax or sometimes contrast-enhanced computed tomography (CECT) thorax are useful in detailed evaluation of SPNs. Biopsies and

Cases

resections by thoracoscopy or thoracotomy should be considered for nodules more than 3 cm or lesions less than 3 cm without clear-cut evidence suggestive of benign lesion. Ultimately the probability of cancer to be estimated considering nodule size, location, growth rate (doubling time < 20 days or > 400 days), margin characteristics, age of the patient, smoking history, prevalence of malignancy in the community, and occult calcification on computed tomography (CT) densitometry. 18F-fluoro-2-deoxyglucose positron emission tomography (FDG-PET) has sensitivity of 97% and specificity of 78% for diagnosis of malignancy in nodules ≥ 10 mm in diameter. FDG-PET can have false negative result in carcinoid, bronchoalveolar cell carcinoma and small nodules less than 10 mm in diameter. On the other hand, false positive results may be found in inflammatory granulomas like tuberculosis, histoplasmosis and rheumatoid nodules. Four imaging observations reasonably exclude malignancy: (i) benign pattern of calcification, (ii) rate of growth too short or too long, (iii) specific shape indicating benign lesion and (iv) equivalent evidence suggesting of benign process like infarction or granuloma. Concentric calcification (laminated) is specific for tuberculosis and fungal granuloma. Punctate calcifications are caused by granuloma, hamartoma, amyloidoma, carcinoid and metastasis (osteosarcoma). Eccentric calcification may occur in bronchial carcinoma. Uniform calcification is virtually diagnostic of granuloma. Fat density within SPN is almost diagnostic of hamartoma and very rarely it can occur in lipoid pneumonia and metastatic liposarcoma. Contrast enhancement favors malignant neoplasm than benign tumor. Popcorn calcifications are randomly distributed and overlapping small rings of calcification suggest the presence of cartilage as in hamartoma and cartilage tumor. Hamartoma is a benign neoplasm probably derived from bronchial wall mesenchymal cell characterized by the presence of cartilage, loose fibroblastic tissue and adipose tissue. Some pathologists consider it as developmental anomaly. They are sharply defined smoothly marginated nodule with focal areas of fat density and multiple coarse foci of calcification or popcorn calcification. Though popcorn calcification is characteristic it is not a common finding. CT thorax can make a confident diagnosis in 50–60% cases by demonstrating focal areas of fat density in smooth marginated nodule. Definite diagnosis can be made by transthoracic core needle biopsy for peripheral hamartomas and bronchoscopic biopsy for endobronchial hamartomas. Management of hamartoma is not surgical but by giving assurance that it will not cause any harm. In intrabronchial hamartoma causing obstruction and collapse of lung, surgery may be required.

Management His chest X-ray PA view (Fig. 2) showed a SPN in the right mid-zone less than 3 cm in diameter with clear lobulated margin without satellite lesion and hilar enlargement. There was popcorn calcification within the nodule. We diagnosed

37

38

101 Cases in Respiratory Medicine

the case as hamartoma. We assured the patient that it was a benign tumor and it will not cause harm. We advised him to stop smoking and gave him symptomatic treatment for cough. We gave him fitness clearance for hernia operation after two weeks of smoking cessation.

My Learning It is the specific type of calcifi­ cation not merely the presence of calcification in SPN that exclude malignancy.

Case 3: Asymptomatic Spn in Smoker Case Report Step I A 55-year-old man was diagnosed as chronic cholecystisis. He was admitted in the surgery in-patient department for cholecystectomy. As preoperative check-up he was advised a chest X-ray. The chest X-ray came one day before surgery, and X-ray was reported as round opacity in right upper zone. The case was referred to us for opinion. He was a smoker about 5–10 cigarettes per day for about 25 years, and he stopped smoking for about 2 weeks as advised by anesthetist. He was a school teacher. He denied any respiratory symptoms except occasional cough. He had no other significant history. General survey and systemic examination failed to reveal any abnormality. His routine blood and biochemistry were within normal limits. His chest X-ray PA view (Fig. 3A) showed one faintly outlined, low density, small round opacity at the anterior end of right second rib. His spirometry was normal. Patient denied any past illness; he never gone to doctor before and he never had a chest X-ray previously.

Fig. 3A: Chest X-ray PA view showing a solitary pulmonary nodule in right upper zone

Cases

Discussion In a 55-year-old smoker without having any respiratory symptoms and signs, a SPN was accidentally detected during routine preoperative checkup. In persons aged more than 35 years with or without smoking history, SPN should not be ignored and must be investigated properly. If old X-ray was available we could have assessed the tumor doubling time. Roughly speaking if no change in tumor size was found over a period of 1 year or more, we could ignore that opacity as benign lesion. Lungs have no pain receptor and pain in lung cancer occurs when it invades mediastinum or chest wall. Lungs have a large physiological reserve, so manifestations of lung cancer occur late. Early detection and management is the key in success of lung cancer management. Lung cancer detected as SPN, stage –I cancer, has a good prognosis with 10 years survival more than 88%.

Step II I started with CECT-thorax, a non-invasive test. CECT-thorax (Fig. 3B) showed a round, heterogeneous, nodular lesion with smooth regular margin in right upper lobe that took little contrast. There was no satellite lesion or mediastinal lymphadenopathy. CT-guided fine-needle aspiration cytology (FNAC) suggested the diagnosis of adenocarcinoma of lung. There was no sign of metastasis, no pleural involvement or mediastinal involvement. Fiber-optic bronchoscope did not show any intraluminal lesion up to sub-segmental bronchi. TNM (tumor-node-metastasis) classification was T1b, N0, M0. We staged the tumor as stage I lung cancer. There was no history of myocardial infarction or cardiac disease. His forced expiratory volume in 1st second (FEV1) was 1.8 liter (L) and his arterial blood gas (ABG) analysis findings were within normal limits (without carbon-dioxide retention).

Fig. 3B: CECT-thorax mediastinal window and lung window showing heterogeneous SPN in right upper lobe

39

40

101 Cases in Respiratory Medicine

Discussion A SPN in elderly smoker should be investigated for evidence of malignancy. At that stage surgery is possible and prognosis will be good. SPN should be considered benign with smaller size, regular margin and denser lesion. Calcification if present usually indicate benign lesion but did not rule out malignancy. Some patterns of calcifications like egg cell, popcorn, central, laminated, etc. suggest benign tumor. Homogeneous nodule may be benign or malignant but the chance of malignancy increased with heterogeneous lesion. Non-calcified nodules are classified into pure ground-glass, partly solid and solid lesions. Possibility of malignancy in pure ground-glass nodules varies from 18 to 60%. Alveolar cell carcinoma or atypical adenomatous hyperplasia can present as ground-glass opacity. Chance of malignancy in partly solid nodule is very high. Solid nodules are usually benign but when occur in persons aged more than 35 years and in smokers should be evaluated. The risk of lung cancer in SPN depends on diameter (more the diameter more the risk); age of the patient (our patient aged 55 years had intermediate risk); smoking status (our patient smoking less than 20 cigarettes per day had intermediate risk); smoking cessation status (duration of smoking cessation above or below 7 years); and character of nodule margin (low risk for smooth, intermediate risk for scalloped and high risk for corona radiata or spiculated margin). In our case, all findings were nonspecific but they indicated an intermediate to high risk for malignancy. To confirm the diagnosis invasive procedures should be adopted. CT-guided FNAC/biopsy should be the first choice; and open thoracotomy or video-assisted thoracoscopy should be considered if CT-guided procedures failed.

Management We confirmed the diagnosis of adenocarcinoma by CT- guided FNAC. The next step should be anatomical staging of tumor to find out resectibility of tumor. We staged the tumor as T1b as the tumor was surrounded by lung parenchyma without invasion into proximal part of lobar bronchus, and the size of SPN was more than 2 cm but less than 3 cm in diameter. We classified the tumor as N0 and M0 as there was no evidence of lymph node involvement or metastasis. We searched for any contraindications for surgery like recent myocardial infarction. He was ambulatory with normal lung function. His ABG was within normal limits. As his anatomical staging, physiological staging and performance status were good we planned surgical resection. We referred the case to thoracic surgery department and they did right upper lobectomy. Staging at the time of surgery was not different and they took hilar lymph nodes for histopathological examination. Histopathological examination of tumor confirmed adenocarcinoma and that of hilar lymph nodes did not show any evidence of malignancy. We referred the case to oncology department, and they suggested to keep patient under surveillance. Patient was followed up with quarterly chest X-ray, and no clinical or radiological evidence of relapse was found during 1 year of surveillance.

41

Cases

Discussion I should have a PET scan before advising surgery but that facility was not available at that time. Moreover, at that time the opinion was against searching for metastasis if patients did not have manifestations suggestive of metastasis in non-small cell lung cancer. It is now well accepted that all non-small cell lung carcinoma should have whole body PET scan before surgery. As PET is not effective in detecting brain metastasis, additional CECT/magnetic resonance imaging (MRI) brain is suggested. The next step should be to assess the operability of patient including the contraindications for surgery and assessment of tolerability of lung resection. For resectable and operable tumor, surgery (lobectomy or pneumonectomy) should be done at the earliest. At the time of operation staging should be performed (surgical staging) for planning adjuvant chemo/radiotherapy. If patient’s My Learning tumor is found to be resectable but patient’s physiological staging does not permit operation, then curative chemo/radiotherapy should be considered.

Early detection and appropriate investigation of SPN is important for increasing survival time of lung cancer.

Case 4: Accidental Discovery of Irregular Calcification Case Report A 60-year man consulted his family physician for cough with expectoration and fever. He was treated with antibiotics (co-amoxiclav 625 mg 8 hourly for 7 days) and was advised to have a chest X-ray. He responded with treatment. His chest X-ray showed calcified opacities. He was referred to us for evaluation of X-ray abnormality. When we saw him, he had no symptom and he denied any past history of illness. He was nonsmoker, nondiabetic and nonhypertensive. General survey, examination of respiratory system and other systems were normal. His chest X-ray PA view (Fig. 4) showed opacity with calcification in right mid zone near paracardiac region. Calcifications were linear, beaded and vertically placed like candle wax or branches of banyan tree. The typical findings were suggestive of pleural calcification.

Discussion Pleural fibrosis may be focal or diffuse and may be associated with blunting of costophrenic angle. Diffuse pleural thickening is often associated with volume loss and it may have focal and extensive calcification. Diffuse pleural fibrosis is almost always preceded by exudative pleural effusion. It may be asymptomatic or may cause dyspnea as a result of entrapment of underlying lung. Pleural thickening may be caused by previous empyema, hemothorax,

42

101 Cases in Respiratory Medicine

Fig. 4: Chest X-ray PA view showing right sided pleural calcification and calcification of right diaphragmatic pleura (arrowhead)

thoracic surgery, thoracic irradiation, asbestos related pleural diseases, malignant mesothelioma, pleural metastasis, collagen vascular diseases and drugs (methysergide, methotrexate, bromocriptine, mitomycin etc.). In our country, possibly tubercular pleural effusion is the commonest cause of pleural fibrosis. Pleural plaque is a dense, acellular collagen deposit in the parietal pleura and often associated with dystrophic calcification. Calcifications are usually punctated, linear or coalescent. They present as focal, pleural based opacities with irregular margin, typically distributed in the posterolateral chest wall, lateral chest wall, dome of diaphragm and paravertebral pleura. They are usually bilateral and asymptomatic. It may take 20–30 years (latency time) after exposure of asbestos fibers to develop plaque. Asbestos fibers are carried to pleura by lymphatic channels or by direct penetration into pleura causing inflammation, fibrosis and ultimately calcification. Chest X-ray may be negative up to 50% cases and CT scan is superior in detecting pleural plaque. Isolated pleural plaque may be seen in tuberculosis, trauma and hemothorax. Pleural calcification can be easily identified by its peculiar appearance (candle wax pattern). It usually found posterolaterally. It is dense, linear and parallel to the inner chest wall. Sometimes they may produce ‘veillike opacity’ or ‘holly leaf opacity’. Causes of pleural calcification are empyema, hemothorax, tuberculous effusion and asbestos plaque. Empyema usually started as parapneumonic effusion. Empyema as a cause of pleural calcification can be diagnosed by long history of high fever, chest pain and signs of pleural effusion that needs treatment with prolong antibiotics and intercostal tube drainage. Pleural calcification in infection and hemorrhage are difficult to differentiate. They are unilateral and vary from minimal to massive. In massive calcification they are sheet-like, hazy veil-like opacity, closer look detect dense and linear opacity parallel to the inner chest wall.

43

Cases

Asbestos related pleural plaques usually involve parietal pleura whereas calcification in empyema can affect both layers of pleura. Asbestos plaques involve posterolateral and diaphragmatic pleura sparing apex, costophrenic sulci or mediastinal pleura. Now benign asbestos, chrysolite, is mostly used and carcinogenic asbestos like amphiboles, crocidolite and amosite are usually not used in industry. The use of chrysolite is thought to be the cause of pleural plaque without parenchymal fibrosis and intrathoracic malignancy. History of exposure may be absent in small and unilateral plaque as asbestos are widely used and individuals may get exposure unknowingly.

Management I diagnosed the case as asbestos plaque with calcification though the history of exposure to asbestos was not available. The points in favor were involvement of posterolateral pleura and diaphragmatic pleura without involvement of apex, costophrenic sulcus and mediastinal pleura. I assured the patient about the benign nature of opacity and referred him back to his physician.

My Learning Pleural calcification can be easily differentiated from lung parenchymal calcification by its characteristic radiological appearance. The presence of diaphragmatic pleural calcifi­ cation is strongly suggestive of asbestos induced pleural plaque.

Case 5: An Adolescent Boy with Ill-defined Nodular Opacities Case Report Step I A 16-year-old boy came to me with shortness of breath during exertion for 1 month. As per patient’s statement he had insidious onset dyspnea and that was increasing. Dyspnea occurred during strenuous exercise that restricted his activities and outdoor play. He had occasional cough without expectoration. There was no fever, wheeze, hemoptysis, paroxysmal nocturnal dyspnea or chest pain. General survey and systemic examination revealed no abnormalities. His routine blood and biochemistry were normal. His spirometry was within normal limits. His chest X-ray PA view (Fig. 5A) showed some non-specific paracardiac reticulonodular opacities. I gave a course of antibiotics (azithromycin 500 mg orally daily for 5 days) and salbutamol inhaler on as needed basis, but there was no response.

Discussion The boy had dyspnea of short duration as primary symptoms with no other clinical and laboratory findings. I thought there might be infection but that did not responded to antibiotics. Asthma should be considered as a possibility as normal spirometry does not rule out asthma. Miliary

44

101 Cases in Respiratory Medicine

Fig. 5A: Chest X-ray of younger brother showing ill-defined parahilar opacities

tuberculosis was considered but absence of fever, toxicity and normal general condition went against that diagnosis. I also considered hypersensitivity pneumonia as a possibility and took detail history regarding exposure to organic dust, particularly exposure to pigeon and other birds, but no such history was available. Cardiac disease like mitral stenosis was another possibility. Sometimes classical mid-diastolic rumbling murmur might not be audible due to rotation of heart. There was no clinical or radiological evidence of pulmonary hypertension. Atrial septal defect usually presented with respiratory tract infection and wide splitting of second heart sound, and that was clinically ruled out.

Step II His echocardiography was normal and IgE was within normal limits (525 mg/ dL). I took detailed history. There was no history suggestive of atopy: the boy did not have urticaria, rhinitis, eczema or family history of asthma or allergic diseases. But his elder brother was suffering from dyspnea. The elder brother was under treatment with a homoeopathic doctor. The younger brother himself came to me for medical help, probably alarmed by the fate of his elder brother. I asked him to bring his elder brother next day. Elder brother had progressive and persistent dyspnea and at that time he was dysphonic even at rest. Examination revealed RR—36/min, accessory neck muscles were working during respiration but examination of respiratory system revealed no abnormalities except reduced lung expansion suggesting bilateral decreased respiratory movement. Elder brother’s chest X-ray PA view (Fig. 5B) showed bilateral reticulonodular dense (more than rib density) opacities involving mainly lower and mid zones. The most astonishing finding

Cases

Fig. 5B: Chest X-ray of elder brother showing bilateral dense lung opacities with relatively lucent heart

Fig. 5C: CT-thorax of elder brother mediastinal window and lung window showing the dense nodules of alveolar microlithiasis involving fissure lines

was that lung opacities were denser than cardiac silhouette, and it appeared as if heart had been vanished. The radiological findings were consistent with the diagnosis of alveolar microlithiasis. The radiological picture of elder brother was confirmatory of alveolar microlithiasis. CT-thorax of elder brother (Fig. 5C) showed the typical dense nodules of alveolar microlithiasis those conglomerated in some areas. Nodules also invoved the fissure and septal lines.

Discussion Alveolar microlithiasis is a disease characterized by accumulation of numerous calcified bodies mainly in the alveoli. Microliths contains calcium phosphate with mean diameter 200 micrometer that may be surrounded by dystrophic calcification. Alveolar wall is not involved in early disease

45

46

101 Cases in Respiratory Medicine

but later the disease may involve interstitium. Sixty percent cases occur in families and the rest occur sporadically. The disease is usually detected in third and fourth decades. Dyspnea is the main symptom. Cough developed at a later stage. Hemoptysis, clubbing, pneumothorax and cor pulmonale may occur rarely. Chest X-ray characteristically showed enumerable, tiny, widely distributed (mainly basal) and pinpointed calcified nodules. Nodules may coalesce to form ground glass opacity or coarse nodules. In late stage heart may be relatively radiolucent than lungs (vanishing heart). Blebs and bullae are better seen in CT-thorax. The differential diagnosis includes metastatic calcification, renal failure, pneumoconiosis, hypersensitivity pneumonia and pulmonary infection like varicella and histoplasma. The disease may remain stable or progresses.

Management I diagnosed the younger brother as early alveolar microlithiasis as the disease has propensity to run in family. I decided to keep the younger brother under observation, assuring him that the disease might not progress as rapidly as his elder brother.

My Learning Family history is important and sometimes that may give vital clue leading to diagnosis.

Case 6: An Elderly Man Carrying Signs of LongStanding Pain Case Report A 74-year-old man came to our outpatient department with hemoptysis. Hemoptysis was described by him as streaks of blood with mucoid expectoration, particularly occurring in the morning. There was long history of cough with mucoid and mucopurulent expectoration, and progressive dyspnea. Patient was an exsmoker, he used to smoke biri 1 packet per day (1 packet contains 20 biries) for more than 30 years. He left smoking for about 10 years. He was diagnosed as chronic obstructive pulmonary disease (COPD) and was treated irregularly with oral bronchodilators and sometimes with antibiotics. He had pulmonary tuberculosis 15 years back and he was treated with daily antitubercular chemotherapy from private clinic. Old records were not available but as per his statement, he took medicines regularly for 1 year. General survey revealed no abnormality; his blood pressure (BP) was 110/70 mm of Hg; pulse was 96/min and respiratory rate (RR) was 22/min. His upper respiratory tract was normal. Examination of lower respiratory tract revealed vesicular breath sound with prolonged expiration. Other systems were essentially normal. His routine peripheral blood and blood biochemistry were within normal limits. His X-ray chest PA view showed (Fig. 6) heterogeneous opacities in right upper zone along with some small, round, discrete, dense and well-circumscribed opacities.

Cases

Fig. 6: Chest X-ray of elderly man showing pellets inside and outside lungs apart from some parenchymal opacities

Discussion As the patient came with hemoptysis, my first job was to treat hemoptysis and second job was to identify the cause of bleeding. In TB hemoptysis is usually less and can be easily controlled with conservative treatment. Massive hemoptysis in active TB can occur from cavity due to rupture of Rasmussen’s aneurysm, and as a result of post TB bronchiectasis or from aspergilloma in a cavity in patients with past history of TB. The old man had right upper zone opacities and past history of TB. I looked for the activity of TB. The patient had no symptoms suggestive of active TB like fever, and cough could be explained by associated COPD. Though his repeated negative sputum smears for AFB, I sent sputum for mycobacterial culture in solid media (BACTEC was not available at that time) and report came as negative after 6 weeks. On those days (CT scan was not available), we used to classify radiological opacities as hard and soft opacities where hard lesions indicated old tubercular lesion and soft lesions were considered as active disease. The differentiation of hard and soft lesions required long experience and was often subjective. Presence of cavity is usually indicative of activity. Any change of lesion (deterioration or even improvement) in the follow up X-rays was considered as signs of activity. The next question was about those tiny discrete round opacities. The opacities did not look like calcified parenchymal lesions found in pulmonary TB. The opacities were homogeneously dense, surprisingly round and remarkably discrete. The possibility of artifact was seriously considered and that was confirmed by detection of a similar opacity over right shoulder joint. The rule is to look outside lung border whenever there is a suspicion of artifact. If similar opacities are found outside lung border or the opacity can

47

48

101 Cases in Respiratory Medicine

be stretched outside thoracic cage, we can diagnose the opacity as artifact with confidence. Sometimes buttons of garments can produce artifact and that can be confirmed by taking another X-ray after putting off the garment. Sometimes nipple may produce artifact and that can be confirmed by taking X-ray putting a marker over the nipple. In that X-ray there were nipple shadows in both lower zones.

Final Diagnosis The opacities were pellets. I asked the old man, and he told me his story that he was eager to tell but nobody was listening. He was carrying those pellets for more than 50 years. There was a notorious riot in Calcutta, the great Calcutta killing, just before partition to take control of our beloved city. The riot was selectively deleted from the contemporary Indian history. During that riot the old man had a pellet-gunshot injury, and he was carrying those pains for more than 50 years.

Management I treated hemoptysis with rest, bronchodilators and etamsylate. I avoided sedations and cough suppressants as he had COPD. Hemoptysis stopped with conservative management. I gave him long acting theophyllin tablets for COPD. His sputum samples were negative for acid fast bacilli (AFB). I asked for sputum culture of mycobacterium. Subsequently the report came as negative. I My Learning did not start anti-TB treatment, and gave him Doctors are part of society, and a antibiotics (co-amoxiclav 625 mg 8 hourly for clear understanding of social and 7 days). I assured him that those dense round political events, without getting lesions were pellets and no treatment was involved into it, is essential. required for them.

Case 7: Accidental Detection of Bilateral Hilar Lymphadenopathy Case report Step I A 46-year-old housewife went to gynecology outpatient department with excessive vaginal bleeding and was diagnosed as fibroid uterus. As a routine preoperative check-up one X-ray chest was done, and that showed some abnormalities. She was referred to chest department. We took history and examined her. She told me that she had occasional cough. Dyspnea occurred on exertion and modified Medical Research Council (mMRC) grading was grade II. She had no fever, weight loss, night sweat but had weakness. Examination revealed no abnormality except pallor (probably related to blood loss). Routine blood and biochemistry were normal except hemoglobin (Hb) was 7 g/dL and erythrocyte sedimentation rate (ESR) was

Cases

Fig. 7: Chest X-ray showing bilateral symmetrical hilar lymphadenopathy with ill-defined parenchymal opacities

60 mm in 1st hour. Her chest X-ray PA view (Fig. 7) showed bilateral almost symmetrical hilar lymphadenopathy with parenchymal ill-defined reticulonodular opacity.

Discussion In a middle-aged lady with minimal respiratory symptoms, radiological abnormalities were detected by routine preoperative check-up. She had dyspnea, and that could be explained by anemia. The main radiological abnormality was bilateral hilar lymphadenopathy. In chest X-ray hilar opacity can be distinguished from mediastinal opacity by the presence of a radiolucent gap between opacity and mediastinal contour. Opacity that merges with mediastinum is usually considered as mediastinal opacity. Hilar lymphadenopathy can be differentiated from vascular hilar shadow by ‘hilar convergent sign’ (when a peripheral branch of pulmonary artery is traced centripetally it will merge with hilar vesicles in case of vascular enlargement not with hilar lymphadenopathy). The main causes of hilar lymphadenopathy are tuberculosis, sarcoidosis, lymphoma, bronchogenic carcinoma, etc. Tuberculosis and bronchogenic carcinoma usually have unequal hilar lymphadenopathy, while sarcoidosis and lymphoma may have equally enlarged hilar lymph nodes.

Step II Contrast enhanced computed tomography (CECT)-thorax showed bilateral hilar lymphadenopathy along with right paratracheal lymphadenopathy and few nodular opacity scattered over both lung field. Nodules could not be classified in CECT plates, and that would need high resolution CT (HRCT)-

49

50

101 Cases in Respiratory Medicine

thorax. Her Mantoux test was negative 5 x 6 mm with 10 TU. Her serum calcium was 9.6 mg/dl (normal value 8.7–10.2 mg/dl) and urine examination did not revealed calciuria. Her serum angiotensin-converting enzyme (SACE) level was 52 U/L (normal level 9–67 U/L). Fiber-optic bronchoscopy showed no intraluminal pathology. Bronchoalveolar lavage (BAL) fluid did not give any clue and BAL fluid for AFB was negative. We sent BAL fluid for BACTEC culture for tuberculosis. Transbronchial lung biopsy reports (TBLB) came as illformed granuloma without evidence of caseation. We planned endobronchial ultrasonography (EBUS) but patient could not afford that investigation.

Discussion Lung parenchymal nodules are classified into (i) centrilobular (a gap can be visible along subpleural areas and along fissures) found in hypersensitivity pneumonia; (ii) perilymphatic (nodules are visible over fissures) found in sarcoidosis and (iii) random nodules found in miliary tuberculosis and metastases. The classification of nodules requires good quality HRCT-thorax. Transbronchial lung biopsy has the problem of getting tiny tissue and the tissue available is usually crushed. Transbronchial lung biopsy is useful for detecting infective etiology particularly tuberculosis, disseminated malignancy and sarcoidosis (as the lesions are mainly peribronchovascular). The finding of ill-formed granuloma may be found in tuberculosis and sarcoidosis, but may not exclude lymphoma or carcinoma. In our case bronchogenic carcinoma was excluded by combination of factors like non-smoker, female sex, no chest symptoms, absence of lung mass in CECT thorax, symmetrical hilar lymphadenopathy and absence of intraluminal lesion in bronchoscopy. Absence of peripheral lymphadenopathy, hepatosplenomegaly went against lymphoma though pure intrathoracic lymphoma that confined within thorax is now considered as a definite entity. Considering clinico-radiological picture of that lady we restricted the differential diagnosis within tuberculosis and sarcoidosis. Absence of constitutional symptoms, negative Mantoux test, symmetrical lymphadenopathy, right paratracheal lymphadenopathy and absence of AFB in BAL fluid pointed towards sarcoidosis. Bilateral symmetrical hilar lymphadenopathy in an asymptomatic individual with negative tuberculin test favored the possibility of sarcoidosis.

Management We decided not to start treatment of sarcoidosis, because of lack of symptoms and lack of urgency of starting treatment. We also considerded that corticosteroid might increase perioperative complications. We sent back the patient to gynecologist for hysterectomy after correction of anemia with blood transfusion. She had undergone hysterectomy. In between BAL fluid BACTEC culture for mycobacterium report came as negative. After 1 month of surgery, we reviewed the case and decided to keep her under observation as at that time she did not have any respiratory symptoms.

Cases

Discussion Sarcoidosis is a multisystemic granulomatous disease of unknown etiology. Lungs are involved in more than 90% cases. The common radiological findings are bilateral hilar lymphadenopathy, paratracheal lymphadenopathy and reticulonodular opacities in lungs. Lung involvement can be staged as: (i) Stage 0—normal chest X-ray (found in 8–16% cases); (ii) Stage I—bilateral hilar lymphadenopathy with or without right paratracheal lymphadenopathy (25–65% cases); (iii) Stage II—bilateral hilar lymphadenopathy with lung parenchymal infiltration (14–49% cases); (iv) Stage III—parenchymal infiltration without hilar lymphadenopathy (10% cases); and Stage IV— parenchymal infiltration with overt pulmonary fibrosis (5% cases). In HRCTthorax, nodules in sarcoidosis are classically perilymphatic in distribution. Manifestations of other system involvement include erythema nodosum, peripheral lymphadenopathy, hepatosplenomegaly, keratoconjunctivitis, peripheral neuritis, pituitary gland involvement, polyarthralgia, lupus pernio, hypercalcemia and/or hypercalciuria and renal stone. SACE level may be helpful in diagnosis of sarcoidosis but it has low sensitivity and specificity. High SACE level is found in about 60% in acute and 20% in chronic sarcoidosis. SACE level may be increased in other conditions like disseminated tuberculosis, leprosy, hyperthyroidism and Gaucher’s disease, etc. The differentiation between sarcoidosis and tuberculosis is sometimes difficult. The presence of caseating granuloma, detection of tubercular bacilli by smear, culture or molecular methods and a positive tuberculin test should be considered as evidence for tuberculosis. The absence of evidence for tuberculosis and presence of multiorgan involvement with non-caseating granuloma can be taken as evidence for sarcoidosis. Treatment of sarcoidosis depends on the symptoms, presentations and organ involved. Treatment is mainly with oral corticosteroids; prednisolone My Learning 40–60 mg daily followed by maintenance Differentiation of sarcoidosis dose of 5–10 mg daily. Lung parenchymal from tuberculosis is difficult involvement usually requires treatment and sometimes impossible. A with corticosteroids. Our patient was pragmatic approach is the key asymptomatic and lung involvement was for decision making regarding treatment. trivial and doubtful, and we kept her under observation.

Case 8: A Young Lady with Chronic Nonspecific Symptoms Case report Step I A 30-year-old lady presented with central chest pain for more than 3 years. She was changing doctors as none of them gave her some relief. General physicians

51

52

101 Cases in Respiratory Medicine

gave her proton pump inhibitors (PPI) and she had continued with PPI for more than two years but there was no relief to pain. She was referred to a cardiologist and routine cardiological checkup was done. Cardiac investigations including electrocardiography (ECG) and echocardiography were with normal limits. She was referred to a psychiatrist. She did not go to psychiatrist probably due to social stigma and consulted a physician. The physician at last advised her a chest X-ray. He found abnormality in chest X-ray and referred her to me. The patient gave me history that she had insidious onset of chest pain in the front of central chest in the lower part. The pain was vague, dull aching, not increased with exertion or respiration. The pain was persisting but not progressive and that did not affect her normal day to day activity or sleep. She denied any history suggestive of acidity, reflux, heart-burn and chest trauma. Her general survey, examination of respiratory system and other systems were essentially normal. Her peripheral blood and routine biochemistry were within normal limits. Her chest X-ray PA view (Fig. 8A) showed homogeneous opacity at right lower zone obliterating right costophrenic angle. The opacity had sharp lateral and upper margins, but medial and lower borders were obscured. There was no mediastinal shifting. Her chest X-ray right lateral view (Fig. 8B) showed that the opacity was placed in the anterior mediastinum obliterating anterior costophrenic angle and anterior part of right dome of diaphragm.

Discussion Central chest pain is a common symptom in middle-aged female. The causes include acidity, heart burn, cardiac pain, costochondritis – Tietze syndrome (usually upper part of chest with tenderness of costochondral junction). But in most cases the cause of chest pain cannot be identified. Sometimes, it is considered as psychological in origin. The pain, when no definite cause is found, is treated with PPI, analgesics and sometimes antidepressants.

Fig. 8A: Chest X-ray PA view showing right lower zone opacity at the right cardiophrenic angle

Cases

Fig. 8B: Chest X-ray right lateral view showing opacity in anterior costophrenic angle

Any opacity that could not be separated from mediastinum should be considered as mediastinal mass. The differential diagnosis of opacity in anterior mediastinum can be described as ‘6 T’ (teratoma, thymoma, thyroid and parathyroid mass, terrible lymphoma, tortuous vessels and trauma). In our case the mediastinal mass was situated in the right cardiophrenic angle. Two conditions (i) Morgagni hernia and (ii) pleuropericardial cyst are specifically found in that particular area.

Step II I advised CT scan of thorax, and CT–thorax mediastinal window showed (Fig. 8C) a low density mass lesion with clear and sharp margin at the right cardiophrenic angle. The density of the lesion was similar to fat density. There was lobulation/septation within the mass lesion. We diagnosed the case as Morgagni hernia with herniation of omentum.

Discussion Morgagni hernia is a congenital hernia that occurs through the gap between sternal and costal origins of the diaphragm. It is usually filled with loose connective tissue and in some cases omentum may be herniated through the opening. Less commonly viscera may herniate and they include colon, liver, gallbladder, etc. Morgangi hernia is usually in the right side. Though it is congenital, presentation in childhood is very rare. In adult they usually present with vague symptoms like feelings of tightness, fullness or pain in the right anterior chest. Occasionally, the pain may be referred to shoulder tip. Strangulation of bowel within hernia may present with acute symptoms.

53

54

101 Cases in Respiratory Medicine

Fig. 8C: CT-thorax showing mass lesion in right cardiophrenic angle with fat density and loculation

Chest X-ray usually shows a rounded density in the cardiophrenic angle. Barium studies may be useful to detect the presence of intestine, and the hitched up transverse colon in omental herniation. Management is the surgical repair of the gap in the diaphragm. The lady was diagnosed as Morgagni hernia by vague symptoms, the characteristic site of lesion, low density in CT-thorax indicating fat/ omentum, and was confirmed during surgery. Fat density in mediastinum may be found in teratoma where fat density within a heterogeneous mass is typically found. As CT-thorax did not give any suggestion of gut herniation I avoided barium meal X-ray. Despite of milder symptoms repair of gap was done to avoid the risk of strangulation of hernia.

Management We sent the patient to thoracic surgeons and they repaired the gap after pulling down omentum into abdomen. She was discharged from hospital after 10 days of operation.

My Learning Morgagni hernia should be con­ sidered in differential diagnosis of opacity in right cardiophrenic angle.

Cases

Case 9: Accidental Discovery of Left-Sided Air-Fluid Level Case Report Step I A 50-year-old truck driver had fever and cough for 5 days. He went to a local physician. His physician prescribed him some medicines most likely an antibiotic and antipyretic, and he advised him X-ray chest. With medicines he became symptom-free and joined his duties. After one month, he did the X-ray and revisited his physician. Then his problems started. The physician referred him to a chest specialist and intercostal tube was introduced. As there was no drainage of fluid or air through the tube. The patient was referred to me. I found no fluid or air coming out through intercostal tube, there was no movement of air column, and I removed the intercostal tube. I started from the beginning and had a detail clinical examination. Patient was totally asymptomatic. General survey was within normal limits. Examination of thorax revealed decreased movement on left hemithorax. Trachea was central and apical impulse was not palpable on the left side but cardiac impulse was palpable on right parasternal area. Percussion notes were dull on left hemithorax from 5th intercostal space in midclavicular line, 6th intercostal space in mid-axillary line and 7th space downwards in scapular line with horizontal fluid level. Shifting dullness was demonstrable. Breath sounds were absent in all over left hemithorax except a small area over left suprascapular area. Breath sounds were normal in right side. Succussion splash was present on left side. The findings were consistent with the diagnosis of left sided hydropneumothorax. His peripheral blood and blood biochemistry were normal. I re-examined his chest X-rays. Chest X-ray PA view (Fig. 9A) showed an air-fluid level in the left midzone throughout left hemothorax with translucency above and homogeneous opacity below obliterating left cardiophrenic and costophrenic angles. Cardiac silhouette was shifted towards right; collapsed lung border was visible above and intercostal tube in situ. Chest X-ray left lateral view showed a horizontal fluid level from anterior to posterior border of chest wall; intercostal tube in situ and left dome of diaphragm was not visible.

Discussion Clinical manifestations specific for hydropneumothorax are that of pneumothorax with dullness in the lower areas with horizontal fluid level (meaning a line joining upper border of dullness in midclavicular, midaxillary, and scapular line was horizontal); presence of immediate shifting dullness; and presence of succusion splash. A delayed shifting

55

56

101 Cases in Respiratory Medicine

Fig. 9A: Chest X-ray PA view showing air-fluid in left hemithorax with mediastinal shift to opposite side

dullness may be found in free pleural effusion, and it signifies good prognosis as it indicate underlying healthy lung. Succusion splash may be found in full stomach and in large peripheral partly fluid-filled cyst. Our case appeared to be hydropneumothorax clinically and radiologically. So, the possibilities of blocked tube or misplaced tube were considered. Both possibilities were excluded and right placement of tube was confirmed radiologically. Then I considered differential diagnosis of air-fluid level in left hemithorax. A horizontal line may be produced by horizontal fissure and a consolidation of middle lobe can produce radiological shadow mimicking right sided horizontal air fluid level. The condition was totally different clinically. A large superficial partly filled cyst may produce signs of hydropneumothorax, but in that condition patient should be symptomatic. Clinical signs will be confined within a lobe and radiologically fluid level should not cross fissure line. The diaphragmatic hernia should come in the differential diagnosis of left sided hydropneumothorax.

Case report Step II I re-examined the chest X-rays carefully. I found lung was collapsed upwards towards the apex with concave lower border. I specifically asked and he disclosed the story he concealed previous. He had a road traffic accident 7 years back. He was admitted in a hospital at Amritsar and he was transfused 2 bottles of blood. Though previous records were not available he told us that he had chest X-ray at the time and that was conceivably normal. I searched for bowel sounds in thorax but I could not elicit the sign. I did chest X-ray PA

Cases

Fig. 9B: Barium meal X-ray showing stomach in the thorax

view after ingestion of barium meal. The barium meal X-ray (Fig. 9B) confirmed stomach was in the left hemithorax. My final diagnosis was late presentation of traumatic diaphragmatic hernia.

Discussion In pneumothorax lung will collapse towards hilum as hilum is fixed. Upwards collapse of lung is not a finding of hydropneumothorax except when lung is fixed by apical fibrosis. That gentleman had collapse lung border at the apex of thorax indicating lung was compressed from below. Audible bowel sounds in thorax may suggest diaphragmatic hernia but may occur in eventration of diaphragm and distended bowel. Road traffic accident is one of the important causes of DALY (disability adjusted life year). Blunt trauma in road traffic accident is the most important cause of diaphragmatic tear. More than 90% of traumatic diaphragmatic hernias are missed at the time of injury. A majority of them may present at a later time, even 15 years after initial trauma. Due to negative intrathoracic pressure abdominal organs are gradually dragged through the tear in diaphragm. Tears usually occur in left leaf of diaphragm and commonly occur

57

58

101 Cases in Respiratory Medicine

in the central and posterior portion of diaphragm. Hernias usually contain omentum, stomach, colon, or rarely kidney. Stomach thus herniated may be distended due to partial gastric outlet obstruction. A largely distended stomach can occupy almost whole left hemithorax, and produce physical or radiological signs mimicking hydropneumothorax. Diaphragmatic hernia may cause mild nonspecific pain in upper abdominal or lower chest. Intestinal obstruction and strangulation are the dreaded complications. Traumatic diaphragmatic hernias represent as high as 90% of strangulated hernias. Strangulated hernia produces sudden increase in pain, dyspnea and intestinal obstruction and features of shock. Though the patient was totally asymptomatic, I advised surgical repair of diaphragmatic tear to prevent that dreaded complication and to make left lung re-expanded.

Management I referred him to thoracic surgeon. They operated the case with thoracoabdominal approach. Stomach was pulled down to abdomen and diaphragmatic tear was repaired. Respiratory physiotherapy was started postoperatively with incentive spirometer. I followed him up for one year and left lung was partially reexpanded.

My Learning Traumatic diaphragmatic hernia may remain asymptomatic for long time, and may mimik hydro­ pneumothorax.

Cases

Patients in Emergency with Respiratory Problem Case 10: A Young Man had a Road Traffic Accident Case Report Step I A 30-year-old man had a road traffic accident and he was brought to hospital. He was in shock with unrecordable blood pressure. He was managed with intravenous fluid, inotropic agents and four bottles of blood transfusion. Peristaltic movements were sluggish and he was put on the usual management of intestinal obstruction with nasogastric tube suction and no feeding by mouth. There was no external injury except areas of bruises over chest wall and abdomen. Emergency doctors were suspecting internal visceral bleeding. After resuscitation, patient was reexamined quickly. Patient complained of chest pain in the left side and dyspnea. General examination revealed he was dyspneic; his BP was 100/70 mm of Hg; pulse was 110/min; RR was 32/min and SpO2 was 93% at room air. Examination of chest was done with difficulty as he had pain and tenderness over chest wall. There was diminished movement of lower part of left hemithorax; and there was no breath sound on left inframammary, infraaxillary and infrascapular areas. USG abdomen showed no visceral injury. His chest X-ray showed lucency over left hemithorax with mediastinal shifting towards right side. They diagnosed the case as left sided traumatic pneumothorax.

Discussion Traumatic (non-spontaneous) pneumothorax is common but often undetected. Trauma may be blunt injury, penetrating injury or iatrogenic. Pneumothorax occurs as a result of disruption of lung, bronchus or esophagus. Common iatrogenic causes include thoracentesis, central venous catheterization, surgery, esophagoscopic procedures (dilatation), chest tube malfunction and mechanical ventilation. Penetrating chest trauma can cause pneumothorax by allowing air entry through chest wall particularly if opening is more than 1.2 cm or by leak from tracheobronchial tree. Blunt trauma can cause pneumothorax by laceration of visceral pleura, by rib fracture or rib dislocation. Pneumothorax can occur without rib fracture in blast injury and high altitude falls into water where sudden increase in alveolar pressure causes disruption of alveolar membrane, and that may result in pneumothorax or pneumomediastinum. Traumatic pneumothorax may have coexisting injuries in tracheobronchial tree and esophagus. Fiberoptic bronchoscopy should be performed in presence of hemoptysis or persistent air leak. Most common site of injury is within 2.5 cm of carina usually in the right side at the interface of membranous and cartilaginous part. The main lobar bronchus and trachea are the next common sites of injury. Traumatic rupture of esophagus usually

59

60

101 Cases in Respiratory Medicine

presents as hydro/pyo-pneumothorax, dysphagia and pneumomediastinum. An elevated pleural fluid amylase level should be considered as an indicator of rupture esophagus. Pneumothorax in ventilated patient should be suspected by sudden decompensation of ventilator function manifested by increase in dyspnea, deterioration of patient’s condition or fall of oxygen saturation. The diagnosis is often missed (may be as high as in 30% cases) in chest X-ray done in supine or semisupine position, particularly with emergency portable X-ray.

Step II In the emergency ward, doctors introduced intercostal tube. With their utter surprise, they found no air was coming out through the tube. They repeated chest X-ray and referred the case to me for opinion. Chest X-ray (Fig. 10A) showed radiolucent left hemithorax, mediastinal shifting to right and a strange finding: the tip of nasogastric tube was spotted above the tip of intercostal tube. Position of stomach indicated by tip of nasogastric tube was noted in the thorax indicated by intercostal tube. The finding was highly suggestive of diaphragmatic hernia as that was an indirect evidence of stomach entered into thoracic space. We confirmed the diagnosis with chest X-ray after barium meal ingestion (Fig. 10B) that showed stomach was placed high in thorax.

Discussion Traumatic diaphragmatic hernia is usually caused by blunt non-penetrating trauma to the chest in road traffic accidents or falls from a height. It may be detected at the time of initial injury particularly if the tear is large. But a majority of cases, they may present at a later time, even 15 years after

Fig. 10A: X-ray showing tip of nasogastric tube (long arrow) above the tip of intercostal tube (short arrow)

61

Cases

Fig. 10B: Barium meal X-ray showing stomach in thorax

initial trauma. More than 90% of traumatic diaphragmatic hernias are missed at the time of injury. Tears usually occur in left leaf of diaphragm and commonly occur in the central and posterior portion. Hernias usually contain omentum, stomach or colon, and kidney may be herniated rarely. Only 5% of diaphragmatic hernias are traumatic but it represents as high as 90% of strangulated hernias. Traumatic diaphragmatic hernia may present with nonspecific upper abdominal or lower thoracic pain and dyspnea that may be aggravated after eating. Bowel sounds may be audible in the chest. With development of strangulation, there may be sudden increase of pain, dyspnea with manifestation of intestinal obstruction, abdominal guarding and shock. Radiological features are those of presence of viscera in the thorax like lucency due to fundal gas, single or multiple air fluid level. Barium study is usually necessary to demonstrate presence of abdominal viscera in thorax. As a result of partial obstruction of gastric outlet, stomach has been dilated and occupies whole hemithorax even displacing mediastinum to opposite side. CT-thorax may be helpful to demonstrate discontinuity of diaphragm. CT-thorax after ingestion of barium will be more helpful in confirming the diagnosis. Management should be surgical repair of diaphragmatic tear, repositioning of viscera and taking care of injured organs. That can be achieved by abdominal, thoracic or combined route but abdominal route is usually preferred as that can take care of abdominal viscera.

Management Patient was operated jointly be general surgeon and thoracic surgeon, and abdominal viscera were drawn back to abdomen and diaphragmatic tear was repaired. The left lung expanded gradually and patient was discharged after 2 weeks.

My Learning Diaphragmatic hernia should be considered in differential diagnosis of left sided pneumo­ thorax, particularly if there is a history of blunt trauma.

62

101 Cases in Respiratory Medicine

Case 11: A Rare Cause of Shock Case Report Step I A 32-year-old gentleman went to his office and at that time he was totally asymptomatic. He suddenly had a bout of cough and became unconscious. His colleagues hurriedly took him to a nearby hospital. He was in shock; his blood pressure was not recordable and pulses were feeble. Physical examination did not reveal any cardiovascular abnormality except peripheral circulatory failure and his heart rate was 120/min. His capillary blood glucose (CBG) was 98 mg/dL and oxygen saturation (SpO2) was not recordable. He was quickly resuscitated with jets of intravenous normal saline (2 liters in first hour); noradrenaline in syringe pump and intravenous hydrocortisone 200 mg immediately followed by 100 mg 6 hourly. His ECG was normal and bed side troponin test was negative. His condition improved dramatically and he was almost normal after 4 hour. He was investigated next day. His white blood corpuscle (WBC) count was 11000/mm3; neutrophil (N)—65%, lymphocyte (L)—28%, eosinophil (E)—5%, monocyte (M)—2%; hemoglobin (Hb)%—14.5 gm/dL; erythrocyte sedimentation rate (ESR)— 36 mm in 1st hour. His routine biochemistry showed sugar (fasting) was 110 mg/dL; urea—30 mg/dL; creatinine—0.8 mg/dL and liver function tests were normal. His chest X-ray PA view (Fig. 11A) showed a thin walled cyst in the right cardiophrenic angle. His echocardiography (ECG), 24 hour Holter monitoring, CT brain and MRI brain were normal. He was discharged after 2 days with advice to consult a pulmonologist for chest X-ray abnormality.

Fig. 11A: Chest X-ray showing empty cyst at right cardiophrenic angle

Cases

Discussion Shock is a clinical syndrome characterized by inadequate tissue perfusion and manifested by hypotension with a mean arterial pressure (MAP) 1000 mm 3), should include ABPA, tropical eosinophilia and Churg-Strauss syndrome. Tropical pulmonary eosinophilia, a reaction to microfilaria, found in topical countries with more symptoms at night and eosinophil count usually more than 3000/mm3. Churg-Strauss syndrome, also called allergic angiitis granulomatosis, is characterised by necrotizing eosinophilic vasculitis and eosinophilic infiltration of multiple organs.

93

94

101 Cases in Respiratory Medicine

Fig. 20B: HRCT-thorax showing bilateral central bronchiectasis and glove-finger pattern

ABPA is a disease caused by hypersensitivity reaction to Aspergillus fumigatus. Ideally, the disease should be called allergic bronchopulmonary mycosis as it may be caused by other fungus like Penicillium, Candida, Curvularia, or Helminthosporium. ABPA is found in about 1% cases of asthma. Episodic bronchial obstruction with mucus plugging leads to breathlessness and consolidation that may change position, flitting pneumonia. Patient may cough out brown sputum casts. The chest X-ray usually shows transient, recurrent infiltrates or may suggest the presence of proximal bronchiectasis. HRCT is a sensitive, noninvasive technique that can detect central bronchiectasis. The diagnostic criteria include major criteria and minor criteria. Major criteria includes; (i) asthma, (ii) pulmonary infiltrate, (iii) peripheral eosinophilia (>1000/L), (iv) immediate wheal-andflare response to Aspergillus fumigatus, (v) serum precipitins to Aspergillus fumigatus, (vi) elevated serum IgE, and (vii) central bronchiectasis. Minor criteria includes; (i) history of brownish plugs in sputum, (ii) culture of Aspergillus fumigatus from sputum, (iii) elevated IgE (and IgG) class antibodies specific for Aspergillus fumigatus. Treatment of ABPA is mainly with long term systemic glucocorticoids. Role of antifungal drugs in the treatment of ABPA is controversial.

Management The man presented as asthma, had peripheral eosinophilia, central bronchiectasis, raised serum IgE and wheal-and-flare skin reactivity to Aspergillus antigen. I confidently diagnosed the case as ABPA. Due to financial

Cases

problem I could not measure serum precipitins to aspergillus protein and aspergillus specific antibody. I treated the patient with oral prednisolone 40 mg per day for 6 weeks and then tapered the dose by 5 mg weekly along with symptomatic treatment with inhaled salbutamol. Tablet tanexamic My Learning acid 500 mg thrice daily was given for 5 days to control hemoptysis. Patient responded Borderline eosinophilia in well. I keep the patient under observation and asthmatics is not uncommon but significant eosinophilia (>1000/ continued with 5 mg prednisolone orally. Every mm3) in asthmatics should be attempt to withdraw oral corticosteroid leads investigated for ABPA. to exacerbation.

Case 21: Unresponsive Chronic Obstructive Pulmonary Disease Case Report Step I A 58-year-old man, farmer by occupation, presented with cough with mucoid expectoration for more than 2 years and progressive dyspnea for 6 months. As per patient he had cough that he initially ignored as smokers cough. The intensity of cough increased over time. He had expectoration less than half cup a day which was mostly mucoid but sometimes became mucopurulent. For last 6 months he observed that he had perceptible weakness and he had to restrict his physical activities. At the time of presentation he felt shortness of breath while performing minimum work. He had to stop while walking for 100 meters on ground level due to breathlessness. He also noticed occasional central chest pain and swelling of feet. He thought he had some cardiac problem. He went to a general physician. He was prescribed with aspirin 75 mg, losartan 25 mg and inhaled corticosteroid. Patient was not responding. An ECG was done, and that showed sinus tachycardia. He was referred to me for exploring respiratory cause of dyspnea. I took history and examined him in detail. He had dyspnea and his mMRC score was 3. He had cough most of the time; he felt phlegm in his lungs; he had no tightness of chest; he felt breathlessness while working up hill; he had limitations in normal day to day activities at home; he was not shaky in moving outside home; he had sound sleep and he had moderate loss of energy. His CAT score (COPD assessment test score) was 16. He had no history of hospitalization or history suggestive of exacerbation. He was a smoker and smoked more than 20 pack-years. He did not have any significant past or family history. General survey revealed his BP—120/70 mm of Hg; pulse—106/min (regular); RR—28/min; JVP—raised; no pedal edema and SpO2—92% at room air. Examination of respiratory system showed bilateral diminished chest movement (chest expansion 4 cm.); central mediastinum; resonant percussion notes; bilateral diminished vesicular breath sounds with prolong expiration, fine crepitations in both lung bases and few scattered rhonchi. Systemic examination showed accentuated pulmonary component of second heart sound.

95

96

101 Cases in Respiratory Medicine

His peripheral blood and routine blood biochemistry were within normal limits. His spirometry showed post-bronchodilator FEV 1/FVC was 62% and FEV1 was 42% of predicted value with 12% and 120 mL increase after bronchodilator use. Flow volume loop showed obstructive pattern with concavity upwards in expiratory part of loop.

Discussion As per Global Initiative for Obstructive Lung Disease (GOLD) guideline 2017, COPD is defined as a common, preventable and treatable disease that is characterized by persistent respiratory symptoms and airflow limitation that is due to airway and/or alveolar abnormalities usually caused by significant exposure to noxious particles and gases. Pathogenesis of COPD is divided into: (i) Initiation—inhalation of toxic and oxidative compounds (tobacco) producing an acute inflammatory response mediated by NF-k B and augmented by the host’s inflammatory response; (ii) Progression—degradation of the extracellular (alveolar) matrix by elastase, by the exposure to tobacco smoke and by self-amplifying injury loops leading to apoptosis and autophagy, and that ultimately leading to enhance fibrosis; and (iii) Consolidation—COPD may become an autoinflammatory disease due to persisting inflammatory response over times. As per guideline COPD should be diagnosed on the basis of respiratory symptoms (shortness of breath, chronic cough and sputum); identification of host risk factors (tobacco, occupation and indoor/outdoor pollution) and spirometry (required to establish diagnosis). After diagnosis, severity of COPD should be assessed. Dyspnea should be assessed by mMRC scoring and/or symptomatic assessment by CAT score. Exacerbation of COPD is defined as an acute worsening of respiratory symptoms that result in additional therapy. History of exacerbations and/or history of hospitalizations should be assessed as (i) low risk defined as one or no exacerbation and no history of hospitalization, and (ii) high-risk defined as two or more exacerbations and/or history of hospitalization. Then overall severity should be assessed by ‘ABCD protocol’ modified by GOLD 2017 guideline. Our patient had high symptoms (high symptoms mean mMRC score 2 or more and/or CAT score more than 10) but low risk of exacerbation. He was scored in ‘B group’. Group B COPD should be treated with one long acting bronchodilator; either a long acting muscarinic receptor antagonist (LAMA) or a long acting beta-2 agonist (LABA). LAMA is preferable than LABA as LAMA is more effective in preventing exacerbation. If monotherapy is not effective then LABA and LAMA combination should be used. The use of LABA and inhaled corticosteroids (ICS) combination is discouraged as it may increase the risk of pneumonia. The recommendation is based on a single study showing increased incidence of pneumonia in patients getting ICS but they have ignored the important finding that overall mortality was same in both groups.

Cases

Management I started indacaterol and glycopyrronium combination (110 + 50 µg) in a single dose dry-powder inhaler (DPI). Patient was not responding to treatment. I searched for complications and comorbidities. I did chest X-ray to rule out pneumothorax and HRCT-thorax to rule out any structural lung disease. His echocardiography showed moderate pulmonary arterial hypertension (45 mm of Hg) and low left ventricular ejection fraction (LVEF - 40%). His serum NT pro BNP (N-terminal B-type natriuretic peptide) was 750 pg/ml. I diagnosed him as COPD with LVF started carvedilol 3.125 mg twice daily to start with then increased the dose to 6.25 mg twice daily. Patient responded partially and I discharged him and asked him to come for follow up monthly.

Discussion Indacaterol is a novel inhaled, once-daily LABA with sustained 24-hour duration of bronchodilator action. It has superior improvements in FEV1 versus placebo and formoterol, fast onset of action, effective control of symptoms and acceptable safety and tolerability. Glycopyrronium is a selective, fast-acting, inhaled, once-daily LAMA with sustained 24hour duration of bronchodilator efficacy. It has superior improvement in FEV1 versus placebo, fast onset of action, effective control of symptoms and acceptable safety and tolerability. Combination of indacaterol and glycopyrronium is available as a single-dose DPI. Single-dose DPI is very easy to use but has two major disadvantages; (i) due to hygroscopic action particle size may be increased (particularly in humid atmosphere) and (ii) increased oropharyngeal deposit that eventually absorbed in blood and thereby increase side effects. When patient is not responding to usual management, we should think about complications and comorbidities of COPD. Complications like pneumothorax, infection, pulmonary thromboembolism, etc. should be ruled out first. Important comorbidities of COPD include cardiovascular diseases (LVF, ischemic heart diseases, arrhythmias, etc.); pulmonary and systemic hypertension; anemia, weight loss, obesity; diabetes, dyslipidemias and metabolic syndrome; skeletal muscle dysfunction; osteoporosis; psychosis (depression and anxiety) and malignancies (lungs and others). Comorbidities have negative repercussions on exercise tolerance, prognosis and survival. Study showed that causes of death in COPD were probably COPD in 40% cases, unrelated to COPD in 50% cases and of unknown cause in 9% cases. Cardiovascular disease and lung cancer are listed as the predominant causes of death in mild or moderate COPD, whereas respiratory failure is listed as the leading cause of death in patients with advanced COPD. Spillover of local inflammation to systemic circulation was thought to be the cause of comorbidities but no relationship between sputum values and blood values of proinflammatory cytokines have been found. Primary insults like smoking, oxidative stress, endothelial dysfunction, etc. were suggested

97

98

101 Cases in Respiratory Medicine

as another possible mechanism. A genetically determined proinflammatory phenotype is suggested as third mechanism. Multimorbidity, defined as the presence of two or more chronic condition, is associated with aging process “inflamm-aging” may be another possible mechanism. Cardiac diseases are important comorbidities that can influence overall mortality and morbidity. Even when the confounding factors are excluded, the cardiovascular mortality and morbidity are 2–3 times more in COPD patients. Smoking history, aging, hypoxia, increased blood viscosity as a result of polycythemia and leftward shifting of interventricular septum are the proposed mechanism of left ventricular dysfunction. The prevalence of LVF in COPD is around 20% and is probably related to increased intrathoracic pressure. LVF in COPD is often unrecognized as there is overlap of clinical manifestations. Measurement of NT-pro BNP (normal level < 125 pg/mL up to 75 years and < 450 pg/mL >75 years) may be helpful as the biomarker, as it is released from the failing heart. The biomarker is a sensitive indicator of LVF alongwith low ejection fraction. It is also increased in LVF with preserved ejection fraction. Pro BNP has a more negative prediction value than positive prediction value. A negative result almost excludes LVF but a positive result may be found in many other conditions including cor pulmonale. Pro BPN is also increased in elderly persons, renal failure, female gender, and may be falsely negative in obese persons. It is now increasingly recognized that increased morbidities and mortalities of COPD may be due to our lack of understanding, e.g. fear or reluctance to use β-blockers in COPD patients by pulmonologists or fear or reluctance to use inhaled β2 agonist by cardiologists. GOLD 2017 advocates to separate COPD from its comorbidities and treatment them separately as per their individual guidelines. The management of COPD should be done as per guideline My Learning irrespective of comorbidity. Similarly the Detection and management management of comorbid conditions like LVF of comorbid conditions is an should be done according to their guidelines essential step in the manage­ irrespective of presence or absence of COPD. ment of COPD.

Case 22: Early Onset COPD or Late Onset Asthma Case Report Step I A 50-year-old man came to our outpatient department with cough, expectoration for 7 years and dyspnea for 1 year. As per patient the disease started insidiously with occasional cough and little expectoration. Cough was intermittent and was increasing in frequency. Though cough was disturbing and more common at night, it did not interfere with his activities and sleep. Expectoration was little mucoid and particularly more in the morning. For last 1 year he developed shortness of breath during exertion and that was increasing.

Cases

Dyspnea interfered with his activities and business. He was comfortable at rest. He did not have wheeze, hemoptysis or chest pain. He visited a local homoeopathy doctor, and was given some powdered medicines that reduced his symptoms temporarily. He denied any history of swelling of feet. He was a businessman and he never smoked. He did not give history suggestive of allergic disorders like rhinitis, rhinorrhea, eczema, urticaria, etc. His father was a heavy smoker, he had dyspnea and was diagnosed as COPD. He did not have any significant past family or personal history. On examination, general survey revealed only tachypnea (RR—22/ min). Examination of respiratory system showed decreased lung expansion (3.5 cm difference between deep inspiration and deep expiration); mediastinum central; hyperresonant notes with liver dullness in right 6th intercostal space at midclavicular line and obliteration of cardiac dullness (no dullness in left parasternal percussion); diminished vesicular breath sounds with prolong expiration and few bilateral polyphonic rhonchi. No significant finding was observed in examination of other systems. His peripheral blood examination and routine blood biochemistry were within normal limits. His X-ray chest PA view showed bilateral low flat diaphragm, increased lucency, bilateral hilar prominence with peripheral pruning of vessels and obliteration of left costophrenic angle. Spirometry showed moderate obstructive pattern with post-bronchodilator FEV1/FVC was 68% and FEV1 was 46% of predicted values. There was 10% increase of FEV1 with absolute increase 150 mL, 20 minutes after salbutamol inhalation.

Discussion The disease started at the age of 43 years in a non-smoker man. The history and clinical findings were suggestive of obstructive airway disease. Vesicular breath sounds with prolong expiration and rhonchi were suggestive of airway obstruction whereas decreased lung expansion, hyperresonant notes, obliteration of dullnesses and diminished breath sound were suggestive of hyperinflation of lungs. The age was too early for COPD and too late for asthma. Patient was nonsmoker. Nocturnal cough was more suggestive of asthma; whereas morning expectoration was suggestive of COPD. Expectoration in asthma is typically described as scanty, thick and tenacious that is difficult to bring out and that cause some relieve to symptoms. Expectoration in COPD is usually mucoid or mucopurulent, and varies in quality and quantity. Dyspnea is the initial symptom in asthma; whereas cough with expectoration is the initial symptom in COPD. Wheeze and rhonchi are more common in asthma; on the other hand, hyperinflation of lungs is more common in COPD. Signs of cor pulmonale (epigastric pulsation, left parasternal heave, accentuated pulmonary component of second heart sound, murmur of tricuspid regurgitation, pulmonary ejection systolic murmur, etc.) and right ventricular failure (pedal swelling, raised JVP and hepatomegaly) are found in COPD. History of atopy (allergic manifestation in patient or family members with raised serum IgE level) is suggestive of asthma. History of atopy is uncommon in late onset asthma.

99

100

101 Cases in Respiratory Medicine

In chest X-ray low diaphragm (border of right dome of diaphragm in midclavicular line at or below anterior end of 7th rib) is a nonspecific sign of COPD as low diaphragm may be found in asthma. Low flat (less vertical height, less than 1.5 cm, of the arch of diaphragm) diaphragm is said to be more specific for COPD. Obliteration of costophrenic angles may be found in COPD patient due to low flat diaphragm. In spirometry asthma is classically described as reversible (more than 12% and 200 mL increase of FEV1 after bronchodilatation) obstructive airway disease and COPD as non-reversible airway obstruction. But many asthmatics, particularly late onset asthma may not show reversibility. On the other hand, some COPD patients may show reversibility. In most of the cases, asthma and COPD can be easily differentiated by physicians as asthma and COPD have striking differences. But in some cases the differentiation is not easy because of striking similarities between them. Asthma-COPD overlap is now considered as a definite entity. AsthmaCOPD overlap is diagnosed: (i) in COPD patients having some features of asthma like eosinophilic inflammation pattern, increased reversibility or better response to ICS; and (ii) asthmatic patients showing some features of COPD like history of smoking, nonreversible airway obstruction, etc. When the previous history of asthma is unknown, the diagnosis of asthmaCOPD overlap may be difficult due to the lack of specific biomarkers. The correct diagnosis of asthma-COPD overlap will require a comprehensive approach including clinical manifestations, spirometry with bronchodilator test, identification of eosinophilic inflammation, lung volumes and carbon monoxide diffusion capacity and CT scan. The diagnosis of asthma-COPD overlap should be made by a specialist not by a general physician.

Step II The patient had some findings suggestive of asthma and some features suggestive of COPD. But he did not fit into the diagnosis of asthma-COPD overlap. I advised a HRCT-thorax to see the structural changes in the lung. HRCT-thorax (Fig. 22A) showed numerous paraseptal bullae mostly at the lung bases giving ‘saw-teeth’ appearance.

Discussion Bulla, localized areas of overdistended emphysema, are definite radiological signs of COPD. Bulla are abnormal airspace more than 1 cm in diameter with characteristically thin and incomplete walls. They should be differentiated from cyst, cavity and pneumatocele. A cyst is a congenital or acquired abnormal airspace more than 1 cm in diameter. A cavity is an acquired cyst with wall thickness more than 3 mm that has nonepithelial lining. Pneumatocele is post-infective cyst usually but not specifically associated with Staphylococcus infection. Bulla produce avascular translucent area usually completely or sometimes partly separated from surrounding lung by thin curvilinear wall.

Cases

Fig. 22A: HRCT-thorax showing paraseptal emphysema (saw-teeth appearance)

HRCT is more effective in detecting bullae than plain chest X-ray. The main complications of bulla are pneumothorax, infection, hemorrhage in addition to respiratory insufficiency. Emphysema is pathologically divided into three types. Centriacinar emphysema is characterized by dilatation of central part of acini mainly respiratory bronchioles and surrounding alveoli. HRCT-thorax shows areas of low attenuation with ill-defined margin and without visible wall, the ‘moth eaten’ appearance. Panacinar emphysema affects entire acinus, is widespread and produces clinically significant disease. Early panacinar emphysema may be missed as they have diffuse involvement without juxtraposed contrast density between lung and emphysematous area. Paraseptal emphysema selectively affects alveoli adjacent to connective tissue septum and bronchovascular bundles. Paraseptal emphysema occurs in young smoker. Paraseptal emphysema is easily detected as subpleural well-defined low density areas with distinct hairline wall. Superior mediastinal borders, azygoesophageal recess, anterior junctional regions are usually affected giving ‘saw teeth’ appearance. The clinical manifestation of emphysema depends not on types of emphysema but on extent of disease. Smoking is the most important etiological factor in COPD. COPD may occur in nonsmoker or without environmental exposure to dust or biomass fuel. On the other hand, only 15% smokers develop COPD. There are many other factors, mostly unidentified, including genetic factors that will determine the susceptibility to develop COPD. Alpha 1 antitrypsin deficiency is one of those factors that is identified and widely recognized. Αlpha-1antitrypsin is a serum protein released by liver that inhibits lysosomal proinflammatory enzymes mainly elastases. Elastases are mainly responsible for destruction of alveolar wall in emphysema. Elastase is liberated from

101

102

101 Cases in Respiratory Medicine

neutrophils those are attracted to the area by inflammatory process resulting from smoking and infection. It is the balance between protease and antiprotease that will determine the destructive process. The level of α1-antitrypsin is dependent on two codominant alleles. PiMM is normal variety; PiZZ, PiSS, Pi—(silent allele) are homozygotic variety with lower serum level and PiMZ is the heterozygotic variety with intermediate level of α1-antitrypsin (around 60%). PiZZ variety is associated with 10–15% of serum level of α1-antitrypsin. Alpha-1 antitrypsin deficiency is linked with early onset (35–50 years) COPD and COPD in nonsmoker. In smokers the disease starts 10 years early than nonsmokers and the former has more severe course. The disease is predominantly panacinar and is predominantly involves lower zones. As α1-antitrypsin is liberated from liver, liver diseases/ jaundice in patient or patient’s first degree relatives are sometimes found. Treatment is by weekly injection of intravenous α1-antitrypsin.

Management Repeated questioning failed to disclose any history of jaundice or liver diseases in patient or his first degree relatives. Due to problems in availability and affordability we could not measure serum α1-antitrypsin level. My final diagnosis was COPD and I managed the case as per GOLD guidelines.

My Learning COPD can occur in nonsmoker and can present early in life. HRCT is an important tool in the diagnosis of structural diseases of lung.

Case 23: Frequent Exacerbations of COPD Case Report Step I A 56-year-old man had cough with expectoration and progressive dyspnea for more than 4 years. Cough was initially dry but later on it was associated with expectoration. Expectoration was less than half cup per day and was mostly mucoid and sometimes mucopurulent. Dyspnea was initially exertional and that was progressive without symptom-free periods. For last one and half years he noticed that he had recurrent attacks of increased dyspnea and purulent expectoration that required increased dose of bronchodilators and antibiotic use. The exacerbation of symptoms was increasing in frequency and severity. At the time of presentation he had dyspnea that restricted his normal dayto-day activity and sleep. He had more than one cup expectoration per day. He was an exsmoker, he smoked about 20–30 cigarettes per day for more than 15 years and he stopped smoking completely for 3 years as advised by his doctor. He had history of hospitalization for COPD exacerbation twice within last one year. He was put on invasive ventilation during his last hospital admission. At that time sputum was examined and that showed growth of Streptococcus pneumoniae and Pseudomonas aeruginosa. He received maximum COPD treatment. Yet his condition deteriorated rapidly.

Cases

On examination, general survey revealed BP—130/84 mm of Hg; pulse—98/ min; RR—36/min; SpO2—88% at room air. Respiratory system revealed signs of dyspnea; signs of hyperinflated lung and signs of airway obstruction. Most remarkable finding I noticed was the prominence of bilateral crepitation, mostly basal, over rhonchi. His peripheral blood and biochemistry were within normal limits except neutrophilic leukocytosis and borderline hyperglycemia. His chest X-ray showed hyperinflated lungs.

Discussion The problem with the case was increasing frequency and severity of exacerbations despite cessation of smoking for 3 years. He was not responding to most effective treatment of COPD with triple inhalers (LAMA, LABA and ICS) and theophylline. Crepitations mainly basal are not an uncommon finding in COPD but prominence of crackles over rhonchi was an unusual finding in our case. There was bacteriological evidence of infection/ colonization with potential pathogenic microorganisms (PPM). It is now clear that there is a wide variability in individual’s ability to maintain lung structure and promote repair. That results in different clinical phenotypes of COPD responsible for wide range of response to therapy and variation in exacerbations. Lower airway bacterial colonization is common in COPD and it is an independent stimulus to airway inflammation. Patients with frequent exacerbations have higher levels of induced sputum IL-6 and IL-8 in stable state, suggesting persisting airway inflammation. Isolation of pseudomonas from sputum is associated with more severe disease, greater degree of functional impairment and worse outcomes. Pseudomonas had high mutation rates, less motility and is more likely to form biofilms. PCR has demonstrated that pseudomonas may persist longer than previously thought. It is unclear whether persistent pseudomona colonization is related to structural changes or to more severe disease. It is claimed that the presence of PPM and chronic inflammation of the bronchial mucosa are responsible for triggering the mechanism that generates bronchiectasis. The presence of bacteria in the airway results in a breach of host defense and that fuels a vicious cycle of structural damage, loss of epithelial cell integrity, impaired mucociliary clearance and mucus hypersecretion. COPD could therefore be a risk factor for bronchiectasis, although longitudinal studies are needed to demonstrate this hypothesis. It is also unclear whether colonization of PPM is the cause or effect of associate bronchiectasis in COPD.

Step II His HRCT-thorax (Fig. 23) revealed bronchiectatic changes with non-tapering of bronchi and dilated bronchial rings with diameter more than that of adjacent arteries. Diagnostic criteria for bronchiectasis are mainly: (i) non-tapering bronchus with internal diameter ≥110% of the adjacent pulmonary artery; and (ii) presence of visible bronchi within 1 cm of the costal pleural surface

103

104

101 Cases in Respiratory Medicine

Fig. 23: HRCT-thorax showing bronchiectatic changes

or adjacent to the mediastinal pleural surface. HRCT-thorax is also helpful for scoring bronchiectasis as: (i) score 0— if no bronchiectasis was present; (ii) score 1— if 30/min; using accessory respiratory muscles; no change in mental status; hypoxemia improved with supplemental oxygen given via Venturi mask 25–30% FiO2; hypercarbia, i.e. PaCO2 increased compared with baseline or elevated 50–60 mm of Hg. • Acute respiratory failure life-threatening: RR >30/min; using accessory respiratory muscles; acute changes in mental status; hypoxemia not improved with supplemental oxygen given via Venturi mask or requiring FiO2 >40%; hypercarbia, i.e. PaCO2 increased compared with baseline or elevated >60 mm of Hg or the presence of acidosis (pH ≤7.25). In our patient COPD exacerbation was severe and he was admitted in respiratory intensive care unit. After hospitalization he was classified as acute respiratory failure—non-life-threatening as his RR was 36/min, accessory muscles were working, he was conscious and oriented, hypoxia

113

114

101 Cases in Respiratory Medicine

was corrected with O2, and there was hypercarbia (76 mm of Hg and PH was 7.32). He was treated with antibiotics having pseudomonas coverage (chance of pseudomonas infection is more in patients with FEV1 less than 35% of predicted value during stable condition). During exacerbations, short acting bronchodilators should be used while long acting bronchodilators are the drug of choice for COPD of stable condition. We used nebulization with combination of short acting β2 agonist and short acting muscarinic receptor antagonist. We used intravenous hydrocortisone, as oral or parenteral corticosteroids decreases the mortality as well as the length of hospital stays during exacerbation. BiPAP support has revolutionized the management of hypercapnic respiratory failure in COPD. We used BiPAP support as it was indicated and there was no contraindication of noninvasive bronchodilators.

Step II Two weeks after discharge the man was readmitted with acute increase of symptoms. He had increased dyspnea but there was no expectoration. Fever was irregular mostly low grade but sometimes above 100°F and there was no chill or rigor. His BP was 110/70 mm of Hg; RR was 36/min; accessory muscles were working; he was conscious and oriented; and SpO2 was 84% that was corrected by giving O2 at 2 L/min. His arterial blood gas (ABG) showed that he had hypercarbia (PaCO2—66 mm of Hg) and acidosis (pH—7.34). His chest X-ray (Fig. 26A) showed bilateral reticulonodular opacities (mainly nodular) distributed throughout lung fields but more prominent in lower zones. I compared it with his previous chest X-ray done at the time of discharge (Fig. 26B). I found that his previous chest X-ray done 2 weeks ago did not have any nodular or reticulonodular opacity. The unexplainable and remarkable

Fig. 26A: Chest X-ray showing bilateral reticulonodular opacities

Cases

Fig. 26B: Previous chest X-ray at the time of discharge showing absence of nodular opacity

Fig. 26C: HRCT-thorax showing ‘tree-in-bud’ opacities

changes surprised me. I advised HRCT-thorax to understand the problem. His HRCT-thorax (Fig. 26C) showed centrilobular nodules with branching pattern, classically described as tree-in-bud pattern. There were centriacinar emphysema and few bronchiectatic changes.

Discussion ‘Tree-in-bud’ was described by Japanese scientists for panbronchiolitis because of the similarity of the opacity with budding tree. In India and in countries where tuberculosis is endemic, tree-in-bud opacities are highly suggestive but not specific for tuberculosis. They may be found in any infection involving bronchioles and even may be found in intravascular

115

116

101 Cases in Respiratory Medicine

spread of malignancies. In our case, a rapid change in radiological opacities favored infective process. I did not considered tuberculosis as the radiological changes occurred too quickly for TB. I considered fungal infection as responsible possibility that could explain rapid appearance of nodular opacities/tree-in-bud pattern. Fungal infection is not a common cause of community acquired lung infection. Fungal infections of lungs usually occur when patients are immunosuppressed and when there is marked structural destruction of lungs. In advanced COPD there are structural changes in lung parenchyma. COPD patients are often treated with inhaled/systemic corticosteroids that depressed local and systemic immunity. Another factor, I considered as important factor was the poor hygiene of O2 delivery system and inhaler devices/nebulizer that resulted in contamination of devices with fungus.

Management I presumed fungal lower respiratory infection as a cause of exacerbation and rapid radiological changes. I took bronchoalveolar lavage (BAL) and sent the BAL fluid for fungal culture. I started itraconazole 200 mg twice daily orally along with the usual management of COPD. Patient responded well clinically and we were able to discontinue BiPAP on 5th day. X-ray chest after two weeks of treatment showed partial clearance of opacities. But the culture report came as My Learning negative though smear showed few fungal Fungal infection is not unusual hyphae. I discharged the patient with advice in advanced COPD particularly to continue itraconazole 100 mg twice daily for those on corticosteroids, and that can precipitate acute exa­ another 1 month. I also advised him to come cerbation. after 1 month for re-evaluation.

Cases

RESPIRATORY PROBLEMS IN INTENSIVE CARE UNIT CASE 27: ELDERLY COPD PATIENT CAME WITH LEUKOCYTOSIS CASE REPORT Step I An 80-year-old COPD patient came with high WBC count. The diagnosis of COPD was made 15 years back on the basis of long history of dyspnea, cough with mucoid expectoration, history of smoking > 30 cigarettes per day for more than 20 years and spirometry showing non-reversible obstructive airway disease. He was brought in stretcher with O2. According to his son, he was bed ridden for last 1 year. He was under treatment of a local physician, and was receiving home O2, nebulized bronchodilators, frequent antibiotics and oral corticosteroids. For last 10 days his condition deteriorated. The patient was referred to me for deterioration of his condition and neutrophilic leukocytosis. I admitted him to respiratory intensive care unit. At the time of presentation, he was dyspneic even at rest and he was bed ridden and he could not go to toilet. He was orthopneic, and he could not eat without assistance. He had cough but could not bring out expectoration. He did not have fever, chest pain, wheeze or hemoptysis. He was conscious, cooperative, oriented and answering questions coherently. His BP was 110/70 mm of Hg; pulse was 120/min regular; respiratory rate was 34/min and SpO2 was 92% with 2 L/min O2. There was no bed sore, venous line or catheterization. Examination of respiratory system revealed signs of dyspnea; breath sound bilaterally diminished with prolong expiration; and no rhonchi. Mediastinum was central and breath sounds were equal in both sides. His peripheral blood WBC count was 36500/mm3; neutrophil count was 90% with toxic granules and there was no abnormal cell. Routine blood biochemistry was normal. His chest X-ray showed signs of hyperinflation (low flat diaphragm) with increased lung markings at both lower zones. ECG showed sinus tachycardia and echocardiography showed moderate pulmonary hypertension and left ventricular ejection fraction was 68%. His ABG with 2 L oxygen showed pH—7.34; PaCO2—70 mm of Hg; PaO2—76 mm of Hg; HCO3–—36 mmol/L and SaO2 was 94%.

Discussion Cause of leukocytosis in this case was probably infection. Most likely site of infection was expected to be in the lungs. Other common sites like skin, urinary tract and venous line were excluded during examination. The patient had advanced stage of COPD, with expected FEV1 less than 30% of predicted value. At that advance stage with low lung function pseudomonas infection was expected and anti-pseudomonas antibiotics should be included. Elderly persons might not have the signs of infection like fever, toxicity or even leukocytosis. His ABG showed acidosis indicated by low PH, caused

117

118

101 Cases in Respiratory Medicine

by respiratory mechanism indicated by raised PaCO2, and that was chronic partially compensated indicated by raised HCO3–. Final ABG diagnosis was chronic respiratory acidosis.

Step II I diagnosed the case as sepsis on COPD. After sending blood for culture I started antibiotics (cefoperazone-sulbactam combination) along with other treatment. Patient was stable but no significant improvement was observed. His blood culture report came as negative. On the third day morning, suddenly patient’s condition deteriorated. He became unconscious with gasping breathing. His BP was 90/60 mm of Hg; pulse was 76/min, regular; RR was 40/min and SpO2 76% with 6 L/min. Examination of thorax showed breath sounds were bilaterally diminished but equal; no rhonchi; no mediastinal shift and no significant changes was observed in other systems. With 10 L/min O2 given through face mask and resolver bag, we could maintain his SpO2. ECG was normal and Troponin T kit test was negative. Portable chest X-ray did not revealed pneumothorax. His arterial blood gas (ABG) showed pH—6.964; PaCO2—130 mm of Hg; PaO2—58 mm of Hg; HCO3–—40 mmol/L and SaO2 was 88%.

Discussion Important causes of sudden deterioration of COPD are pneumothorax, acute myocardial infarction (AMI), pulmonary thromboembolism (PTE), etc. Identification of pneumothorax is quite difficult in patient of COPD with hyperinflated lungs with bulla causing hyperresonant percussion notes and diminished breath sound. Sudden deterioration of patient’s condition and shifting of mediastinum are important clues and should not be ignored. Bilateral pneumothorax is even more difficult to recognize clinically. So, all such patients should have a portable chest X-ray. Interpretation of supine (AP view) portable (usually poor quality) needs expertise. AMI is common in COPD patients because of same risk factors like age, smoking history, sedentary life style, etc. Moreover, other factors like hypoxia, polycythemia and shifting of interventricular septum towards left can impair left ventricular functions. Even when those confounding factors are excluded the risk of AMI is about 2 to 3 times more in COPD patients. AMI is difficult to identify in that situation as usual presentations like chest pain are difficult to recognize. Echocardiography has the problem of poor window as a result of hyperinflated lung. Cardiac enzymes may give important clues. Troponin T kit test, though not absolute, may be helpful as it is a quick bedside test. COPD patients are also susceptible for PTE because of prolong bed confinement, venous stasis due to right ventricular failure and probably due to change in coagulation mechanism. All patients with prolong recumbency, PTE should be considered seriously. Clinical clues may be sudden onset of deterioration, tachycardia, tachypnea, classical ECG changes, deep vein thrombosis identified by bedside Doppler study and D-dimer measurement. In our case we excluded pneumothorax by portable chest X-ray; AMI by ECG

Cases

and negative Troponin T kit test and PTE by excluding deep vein thrombosis by Doppler study.

Management I diagnosed the case as acute on chronic respiratory failure. I discussed the case with patient relatives they did not give consent for intubation. We gave intravenous sodium bicarbonate and bi-level positive airway pressure (BiPAP) support 18:4 cm of H2O. We continued the management of sepsis with antibiotics. We started low molecular weight heparin, though the chance of pulmonary thromboembolism was less as bed side Doppler study revealed no evidence of deep vein thrombosis. Patient’s condition deteriorated and his BP fell below recordable limit. We started IV fluid and noradrenaline in syringe pump (10 mL/sec). The patient died next day morning.

Discussion If we analyze his ABG result, we found marked rise of PaCO2 (from 70 to 130 mm of Hg) with marked fall of pH (from 7.34 to 6.94) indicating acute on chronic respiratory failure probably due to depression of respiratory center. Causes of sudden depression of respiratory center may be vascular, metabolic, infection and drugs. Patient had a tablet of alprazolam 0.25 mg on the previous night. I was not sure whether that small dose of an anxiolytic drug can cause respiratory center depression. Other probability included sepsis My Learning itself that might be responsible for respiratory Acute on chronic respiratory depression. A negative blood culture did not failure in COPD is usually due exclude the possibility of sepsis. Sepsis is to infec tion/sepsis. O ther causes should be kept in mind defined as systemic inflammatory response and searched for. Sedatives/ with documented or suspected infection. In anxiolytics should be avoided our case, neutrophilic leukocytosis with toxic in COPD patients. granules suggested infection.

CASE 28: AN UNUSUAL ASSOCIATE DISEASE IN COPD CASE REPORT Step I A 50-year-old man presented with cough and dyspnea for last 5 years. Cough was intermittent and particularly increased at night. Cough was usually dry but sometimes with little mucoid expectoration. Dyspnea was progressive and persistent and was associated with wheeze. For the last one month he noticed that dyspnea was intensified and at the time of presentation dyspnea was present even at rest. He also noticed that the intensity of wheeze was increased. He was farmer by occupation. He did not have any significant past, personal and family history except he was smoking about 20 biries per day for more than 35 years.

119

120

101 Cases in Respiratory Medicine

Fig. 28A: Chest X-ray PA view showing no obvious abnormality

On examination general examination revealed dyspnea with accessory muscles of respiration working; RR was 42/min; BP was 140/70 mm of Hg; pulse was 110/min and SpO2 was 82% at room air. Respiratory system examination revealed diminished chest wall expansion (2.5 cm); trachea central; apical impulse could not be localized; bilateral hyperresonant percussion notes, diminished vesicular breath sounds with prolong expiration on both sides and monophonic wheeze. His chest X-ray PA view (Fig. 28A) did not reveal any significant finding. His HRCT-thorax did not show any significant lung parenchymal abnormalities except centriacinar emphysema. I diagnosed him as acute exacerbation of COPD (AECOPD) and admitted him in the respiratory care unit. We started the management of AECOPD with O2 to maintain SpO2 > 88%, nebulization with combination of salbutamol and ipratropium bromide 6 hourly; oral prednisolone 40 mg/day and antibiotics piperacillin-tazobactam combination 4.5 g 8 hourly. Patient’s condition did not improve. His ABG with 3 L O2 through nasal cannula showed pH—7.3; PaCO2—70 mm of Hg; PaO2—62 mm of Hg and HCO3—32 mmol/L. Patient was conscious and cooperative and hemodynamically stable. We started non-invasive ventilation (NIV) with inspiratory positive airway pressure (IPAP)—14 cm of H2O and expiratory positive airway pressure (EPAP)—5 cm of H2O. I assessed him after 2 hour, and the patient did not show improvement or deterioration of his clinical or ABG parameters.

Discussion Noninvasive ventilation was started during the 1st half of the 20th century as ‘negative pressure ventilation’ where negative pressure was applied to the thorax. Negative pressure ventilation was used extensively during polio epidemics. They were Iron lung, Cuirass, Jacket ventilator, Hayek oscillator, etc. They were not popular due to frightening instruments and cumbersome procedure. Today, NIV means noninvasive positive pressure ventilation given through mask.

Cases

NIV refers to the delivery of mechanical ventilation to the lungs using techniques that do not require an invasive artificial airway (endotracheal tube or tracheostomy tube). NIV can be delivered by CPAP and by BiPAP. CPAP is the delivery of positive airway pressure that is maintain steadily throughout the respiratory cycle in a spontaneously breathing patient. CPAP is mainly used in obstructive sleep apnea, and may be used in cardiogenic pulmonary edema and respiratory insufficiency in neuromuscular diseases and chest wall deformity. BiPAP delivers different pressures during inspiration and expiration with a higher IPAP and a lower EPAP. The main four indications of BiPAP are COPD exacerbations, weaning COPD patients from invasive ventilation, cardiogenic pulmonary edema and immune-compromised patients. The use of NIV, especially in the early course of the disease, has revolutionized the management of patients with AECOPD. It is clearly a superior alternative to standard medical therapy. But it should be avoided in extremely sick, hemodynamically unstable and obtunded patients. Indications of NIV in COPD exacerbation include the following: (i) respiratory acidosis (pH 45 mm Hg), (ii) respiratory frequency >25 breaths per minute, (iii) severe dyspnea with clinical signs of respiratory muscle fatigue and increased work of breathing or both, (iv) use of respiratory accessory muscles, (v) paradoxical motion of abdomen (vi) intercostal retraction. NIV is contraindicated in respiratory arrest, cardiovascular instability (hypotension, arrhythmias and myocardial infarction), impaired mental status, uncooperative patient, high aspiration risk, gastrointestinal bleeding, viscous or copious secretions, extreme obesity, any local craniofacial abnormalities or injuries and clostrophobia. We should not try NIV if patient has PaCO2 >90 mm of Hg, pH 30 cm H2O, deliver FiO2 >45–50% and do not have alarm system. Portable machine have single tube for inspiration and expiration; and minimum EPAP should be ≥4 cm of H2O to avoid rebreathing. In general, IPAP is meant for alleviate dyspnea, increasing Tv and reducing PaCO2 or increasing PH. Whereas, EPAP acts as positive end-expiratory pressure (PEEP) to improve oxygenation. The initial NIV setting for COPD include a relatively low EPAP (5–8 cm H2O) and a relatively moderate IPAP 10–14 cm H2O. The intrinsic PEEP (PEEPi) cannot be measured by NIV machine; therefore a low EPAP that does not exceed PEEPi in patients with obstructive lung disease is given. IPAP should be increased by 2–5 cm increments at a rate of approximately 5 cm H2O every 10 minutes with a usual IPAP target of 20 cm H2O or until a therapeutic response is achieved or patient tolerability has crossed. There

121

122

101 Cases in Respiratory Medicine

should be at least 5 cm H2O differential between EPAP and IPAP that may be increased depending on ventilation requirements. O2 should be entrained into the circuit and the flow adjusted to maintain SpO2> 88–92%. NIV should be critically monitored by (i) subjective responses like patient’s comfort level, (ii) physiological response including decrease in RR and heart rate, BP, ECG monitoring, patient ventilator synchrony and mask leak, (iii) ABG and continuous SpO2 monitoring. ABG should be measured after 1 hour of NIV therapy; or 1 hour after every every subsequent change in settings; and 4 hours or earlier in patients who are not improving clinically. Most important factor in NIV use is the “Decision making” on when to switch over to invasive mechanical ventilation in a setting of failure of NIV. Criteria for terminating NIV and switching over to invasive ventilation include: (i) worsening pH and PaCO 2, (ii) tachypnea (over 30/min), (iii) hemodynamic instability, (iv) SpO 2 30/min; he was using his neck muscles including scalene group of muscles during respiration; SpO2—82% with 6 liter O2 given through face mask. His ABG showed pH—7.164; PaO2—66 mm of Hg; PaCO2—89 mm of Hg and HCO3–—36.2 mmol/L. He was put on invasive mechanical ventilation with assisted control mode and 100% oxygen. Ventilatory set up used were tidal volume (Tv)—550 mL; respiratory rate (RR)—24/min; inspiratory flow rate—60 L/min keeping inspiratory-expiratory ratio (I:E ratio) more than 1:3 and positive end expiratory pressure (PEEP)—3 cm of H2O. Fraction of inspiratory oxygen (FiO2) was gradually reduced by 10% every 2 hours up to 40% maintaining SpO2 ≥88%.

123

124

101 Cases in Respiratory Medicine

Discussion The assessment of COPD has been slightly but significantly modified in GOLD 2017 guidelines (Fig. 29). Clinical parameters are separated from airflow limitations. The airflow limitation in revised ABCD protocol will determine the prognosis and non-pharmacological management whereas the clinical parameters will determine the pharmacological management. During hospitalization, in clinics or in emergency room, patient should be assessed on the basis of clinical parameters (severity of symptoms and exacerbations history) independent of the spirometry for initiating pharmacological treatment. Individualized pharmacological intervention strategies like use of medicines, change of medicine or addition of medicine should be guided by clinical parameters. Hospitalized COPD patients are classified into 3 groups as defined as: (i) no respiratory failure, (ii) acute respiratory failure—non-life-threatening and (iii) acute respiratory failure—life-threatening. The criteria of lifethreatening acute respiratory failure includes RR > 30/min, using accessory respiratory muscles, acute changes in mental status, hypoxemia not improved with supplemental oxygen given via Venturi mask or requiring FiO2 >40%, hypercarbia, i.e. PaCO2 increased compared with baseline or elevated >60 mm of Hg or the presence of acidosis (pH ≤7.25). According to the guidelines, our patient should be included in the 3rd category, “acute respiratory failure—life-threatening”. In our patient, NIV is contraindicated and invasive mechanical ventilation (IMV) was started considering his clinical and ABG parameters and most importantly considering pH (7.164). Lungs in COPD are more compliant and distensible. The principles of ventilatory set up in COPD should be more Tv, less RR and increased I:E ratio (1:3 to 1:4) as patient needs more expiratory time to drive out air due to airway obstruction. Ventilatory set up in COPD should be with

Fig. 29: Table showing refined ABCD assessment tool

Cases

high tidal volume 8 mL/kg ideal body weight (IBW) then to increase as per requirement; RR should be low 10–14 breath/min to allow more expiratory time; flow rate should be high >80 L/min to make less inspiratory- expiratory ratio (1:3 to 1:4) that give more time to complete expiration; FiO 2—to maintain oxygen saturation ≥ 88% and PEEP 2–3 cm of H2O (physiological PEEP to maintain the physiological pressure caused by closure of glottis). Monitoring intrinsic PEEP (iPEEP) is essential and an additional PEEP up to 80% of iPEEP to decrease the work-load of ventilation may be applied. In general, plateau pressure (Pplat) should be maintained below 30 cm H2O and peak pressure to be maintained less than 50 cm H2O.

Step II After 24 hour, patient’s condition showed some improvement. ABG was repeated after 24 hour, and ABG parameters with 40% FiO2 were PH—7.61; PaO2—86 mm of Hg; PaCO2—46 mm of Hg and HCO3–—39.2 mmol/L.

Discussion If we analyze the ABG findings in isolation, PH was high (7.61) indicating alkalosis and bicarbonate was high (39.2 mmol/L) indicating metabolic disorder, high PaCO2 (46 mm of Hg) indicating compensation and our diagnosis would be metabolic alkalosis. In that condition, we should search for common causes of metabolic alkalosis like diuretic use, dehydration, hypokalemia, hypomagnesemia, etc. If we considered ABG findings in the background of previous ABG findings done 24 hours back then the dramatic changes were quite obvious. PH changed dramatically from 7.164 to 7.61 (from acidosis to alkalosis); HCO3– change was not marked (36.2 to 39.2 mmol/L) but PaCO2 changed markedly from 89 to 46 mm of Hg. The dramatic change of PH could not be explained by change of bicarbonate (metabolic disorder) but could be explained by change of carbon-dioxide level (respiratory disorder). So, our conclusion was respiratory alkalosis (ventilator induced) over partially compensated respiratory acidosis. Cause of respiratory alkalosis was increased rate, depth or both of respiration.

Management We reset ventilator set up to reduce minute ventilation. Minute ventilation could be reduced by lowering Tv or RR or both. RR could not be reduced as patients spontaneous RR was high and that was only possible by completely paralyzing respiratory muscles. We did not want to use sedatives and muscle relaxants as patient’s condition was improving and we were planning for weaning. We achieved that goal by reducing Tv from 550 mL to 450 mL. Next morning we gave him a spontaneous breathing trial (SBT). He could not tolerate SBT for more than 1 hour. We continued mechanical ventilation for another two days. Then we switch over to spontaneous pressure support (PS)

125

126

101 Cases in Respiratory Medicine

ventilation with PS 16 cm of H2O. We reduced PS gradually, by 1 cm of H2O every 2 hour, up to 6 cm of H2O. Next day morning we discontinued mechanical ventilation and NIV at 18:4 cm of H2O. Patient was totally weaned off on the 8th day and was discharged on 12th day.

Discussion Now, the term ‘weaning from ventilation’ is discouraged and the term ‘discontinuation from ventilation’ is used. The basic idea is that weaning is a long process and we cannot unnecessarily delay the process of withdrawing endotracheal tube as that will increase the risk of ventilator associated pneumonia (VAP). When patient is in mechanical ventilation, the ventilatory function is maintained by the machine. We must be sure before weaning that patient will be able to continue ventilatory function by himself. To assess that there are general parameters including maintaining hemodynamic parameters and electrolyte balance; ABG criteria and respiratory/ventilatory criteria. After satisfying with them it is prudent to carry on a SBT. After 2 hour ABG should be measured to assess whether there is carbon dioxide retention. General criteria including respiratory rate are to be reassessed. If criteria are favorable then extubation should be considered. In our patient SBT failed and we had to delay the process to gradually weaning through pressure support ventilation. The application of NIV as a post-extubation procedure is now widely recognized. NIV is effective in preventing ventilatory failure and reduces the chance of reintubation in post-extubated My Learning COPD patients. It should be remembered ABG should not be read in here that if ventilatory failure developed after isolation but should be read extubation then NIV will not be effective. on the background of previous For post-extubated COPD patients, NIV is ABG and clinical setting. NIV is a preventive measure and not a therapeutic a preventive measure in postextubated COPD patients. measure.

CASE 30: VENTILATED PATIENT WITH OPAQUE LEFT HEMITHORAX CASE REPORT Step I A 52-year-old man was admitted in intensive care unit of a private nursing home with acute exacerbations of chronic obstructive pulmonary disease (AECOPD). The patient had high fever, cough with purulent expectoration and increased respiratory distress. He was obtunded with low BP (100/50 mm of Hg); borderline pulse (98/min); high RR (42/min) and hypoxia (SpO2 was 78% at room air). He was intubated and put on mechanical ventilation. Ventilator set-up was as per guideline of COPD. Culture was sent from endotracheal

Cases

suction and that showed no growth. He was on intravenous meropenem 1 g 8 hourly and teicoplanin 200 mg twice daily. He was treated with nebulization with levosalbutamol 1.25 mg four times daily and hydrocortisone 1000 mg thrice daily. He did not respond to treatment. On examination, ICU doctors found dullness over left hemithorax with almost absent breath sound. They did chest X-ray portable and that showed homogenous opacity in left hemithorax. They diagnosed the case as massive pleural effusion. As patient was on invasive mechanical ventilation, they did not try aspiration and referred the case to me.

Discussion That gentle man was classified as having severe exacerbation with acute respiratory failure—life-threatening. Exacerbations should be managed with short acting bronchodilators, appropriate antibiotics, corticosteroids, etc. Respiratory failure should be managed with either NIV or IMV. The use of NIV, especially in the early course of the disease, has revolutionized the management of patients with AECOPD. NIV has a definite role as first line intervention in AECOPD. NIV is clearly a superior alternative to standard medical therapy, and it reduces the likelihood of endotracheal intubation, treatment failure and mortality. But the patients had contraindications of NIV like impaired mental status, hemodynamical unstability and high respiratory rate. They justifiably put the patient on IMV. On the other hand, IMV should be used in conditions like respiratory arrest; severe respiratory failure (RR >35/min, PH 150 mg/dL or on specific medication; (iii) low HDL cholesterol 60 mm of Hg and to reduce FiO2 to less than 60% within 24 hour. It should be remembered that the oxygen in high concentration itself can cause lung injury. In volume control mode, pressure is variable that means will be determined by the machine. During inspiration, pressure will rise sharply to reach the peak (peak pressure) and then it gradually settled to a plateau (plateau pressure).

149

150

101 Cases in Respiratory Medicine

Peak inspiratory pressure depends on inspiratory flow resistance property (combination of airway resistance and inspiratory flow rate) may reach >50 cm of H2O in COPD (target should be less than 50 cm of H2O). High peak pressure is an indicator of endotracheal tube blockage. Plateau pressure is more important in ARDS as it is an indicator of pressure required to inflate stiff lung. In ARDS plateau pressure should be kept under 30 cm of H2O (target 65 mm of Hg. Both inverse ratio ventilation and prone position ventilation improve oxygenation but their effect on survival and other important clinical outcomes remains uncertain. High-frequency ventilation (HFV) [i.e. ventilating at extremely high respiratory rates (5–20 cycles per second) and low Tv (1–2 mL/kg)]. Partial liquid ventilation (PLV) with perfluorocarbon, an inert, high-density liquid that easily solubilizes oxygen and carbon dioxide, has revealed promising preliminary data on pulmonary function in patients with ARDS but it also does not show any survival benefit.

Management We sent blood and urine for culture and drug sensitivity. We started meropenem 1 g intravenously 8 hourly with levofloxacin 750 mg intravenously daily. We intubated the patient and started mechanical ventilation in assisted control mode with TV 350 mL; RR—30/min; inspiratory flow 50 L/min maintaining I:E ratio 1:2, PPEP—8 cm of H 2O and FiO 2—100%. We could maintain SpO2 to above 90% and BP to above 90/60 mm of Hg. As she responded we

Cases

reduced FiO2 gradually to 60% on the 3rd day. My Learning We could reduce the PEEP to 5 cm of H2O by 7th day. Culture reports came as negative. We Early detection and proper ventilatory management can were able to extubate on 8th day. We stopped save life in ARDS. antibiotics on 10th days. We discharged her after 2 weeks. Urinary tract infection is common in female but that rarely leads to sepsis and ARDS. We did not found any factors like diabetes or any other immunosuppression that facilitated sepsis. Fortunately, we diagnosed ARDS early and the source of infection was identified from the beginning. With proper antibiotics to control infection and proper ventilatory care to maintain SpO2, we could save the patient.

CASE 38: HYPOXIA WITH BILATERAL LUNG INFILTRATES CASE REPORT Step I A 50-year-old man presented with acute onset of respiratory distress for last 5 days. Dyspnea was increasing rapidly and at the time of presentation patient was dyspneic even at rest. He could not go to toilet and even could not stand at bedside. He had cough without expectoration. He had irregular low grade fever without evening rise. There was no headache, body ache, malaise, lassitude or any constitutional symptoms. He had anorexia but there was no nausea or vomiting. Patient noted significant weight loss for last 6 months but weight was not measured. On enquiry he confessed that he had profound weakness for last 2 to 3 months. He was a businessman. He was married with two sons and all of them was healthy. He was non-smoker but he took alcohol regularly. He had no other addiction. He denied any significant past and family history. General survey revealed patient was dyspneic with accessory muscles of respiration working, BP was 100/60 mm of Hg, pulse was 104/min, RR was 46/min and SpO2 was 82% at room air. Examination of respiratory and other systems were essentially normal. He was admitted in respiratory care unit. He was resuscitated with oxygen at 10 L/min with face mask with reservoir bag. His SpO2 became 89%.

Discussion There are enumerable causes of acute onset of respiratory distress. Pneumothorax is one of the possibility but in pneumothorax the onset is usually sudden (patient can pin-point the onset). Acute onset that progress rapidly is unlikely for pneumothorax, cardiac disease or any vascular disease like pulmonary thromboembolism. Pleural effusion can cause rapidly progressive dyspnea but examination findings did not suggest that possibility. In alcoholic patients hepatopulmonary syndrome may be considered but patient had no manifestations of chronic/acute liver disease. ARDS was a definite possibility but no identifiable etiology was found. Acute DPLD may be considered but it is a rare disease. Pneumonia may have acute onset but it usually presents with fever and cough, and dyspnea as a presentation is

151

152

101 Cases in Respiratory Medicine

unusual. Low-grade fever may be suggestive of a chronic infective process like tuberculosis but the course of disease and dyspnea as a presentation were against tuberculosis or any other chronic infection. Oxygen delivered through facemask and reservoir bag increases FiO2. When we give oxygen through nasal prongs a part of oxygen comes from oxygen delivered from cylinder and rest comes from atmosphere. As for example, in a patient with 500 mL tidal volume and respiratory rate 20/min, oxygen is delivered at a rate of 6 L/min (100 mL/second). His inspiratory time will be 1 second (one-third of respiratory time). Of 500 mL tidal volume 100 mL from cylinder and 50 mL coming from anatomical reservoir with pure oxygen. Anatomical reservoir is the areas of upper respiratory tract (mainly nasopharynx) where pure oxygen is collected during expiration. Rest 350 mL comes from air containing 20% (70 mL) oxygen. So, FiO2 is calculated as (100 + 50 + 70)/500 = 0.44. Face mask and reservoir back acts as additional anatomical reservoir that collect pure oxygen during expiration and thereby increase FiO2.

Step II His peripheral blood and routine blood biochemistry were within normal limits. His ABG showed hypoxia with respiratory alkalosis (PH—7.486 and PaCO2—34 mm of Hg). His chest X-ray PA view (Fig. 38A) showed bilateral perihilar fairly homogeneous opacities (butterfly or bat wing pattern), obscuring adjacent blood vessels without loss of lung volume. His HRCT-thorax (Fig. 38B) showed bilateral perihilar consolidation with presence of air-bronchogram.

Discussion His chest X-ray findings were remarkable. The causes of bilateral perihilar opacities include cardiogenic pulmonary edema, non-cardiogenic pulmonary edema, alveolar hemorrhage, alveolar proteinosis, organizing pneumonia, pneumocystis pneumonia (PCP), etc. Bat wing pattern is classically described for pulmonary edema. The mechanism of the typical distribution is not clear. The proposed mechanism is the poor lymphatic drainage of central areas of lungs. Lymphatic drainage of lungs depends on lung movement/excursion. Naturally peripheral areas of lungs have better lymphatic drainage than central parts. HRCT findings in our patient were consistent with consolidations with air-bronchogram. The difference between ground-glass attenuation and consolidation is the identifiable lung marking underneath the opacities in the former. Air-bronchograms are typically found in consolidations; but contrary to our belief they may be found in pulmonary edema, alveolar proteinosis. A HRCT finding of alveolar proteinosis is classically described as crazy paving pattern (a combination of ground glass and reticular pattern). Alveolar proteinosis and organizing pneumonia usually have a insidious onset and chronic course. Organizing pneumonia may have a peribronchovascular distribution but symmetrical bat wing distribution

Cases

Fig. 38A: Chest X-ray showing bilateral parahilar opacity

Fig. 38B: HRCT-thorax showing bilateral parahilar opacities with air-bronchogram

is unusual. Pulmonary edema (cardiogenic or noncardiogenic), alveolar hemorrhage usually have acute onset but presentation as consolidation is unusual in those conditions.

153

154

101 Cases in Respiratory Medicine

The striking feature of our patient was disproportional hypoxia with bilateral perihilar consolidations. The degree of hypoxia could not be matched with that of radiological opacity. Disproportional hypoxia is a characteristic feature of Pneumocystis pneumonia (PCP). Pneumocystis (P.) jirovecii, an opportunistic fungus, usually infect immunocompromised human beings. The fungus typically infect persons with HIV infection, and is considered as criteria for AIDS. PCP usually occurs in persons with CD4+ T cell count less than 200/L. The infection may occur in persons receiving anti-cancer drugs and glucocorticoids for cancer or organ transplantation; patient receiving biologic agents (infliximab, etanercept); primary immunedeficient or malnourished children. Patients usually present with dyspnea, fever and non-productive cough for several weeks. Physical findings include tachypnea, tachycardia, cyanosis, fall of SpO2, respiratory alkalosis. Physical examination of lungs usually revealed no abnormality. Classical chest radiological finding is bilateral lung infiltration starting from perihilar areas; but nodular opacities, cavities are rarely found. At early stage chest X-ray may be normal. HRCT usually reveals ground glass opacities or evidence of consolidation. High index of suspicion in the background of specific causes of immunosuppression is important for diagnosis of PCP. Collection of biological sample is the essential step and can be done by inducing sputum. Sometimes fiberoptic bronchoscopy with bronchoalveolar lavage and transbronchial lung biopsy may be required for diagnosis. Pneumocystis needs special staining for diagnosis. Methenamine silver selectively stains the wall of Pneumocystis cysts, while Wright-Giemsa stains the nuclei of them. Immunofluorescence with monoclonal antibodies is more sensitive and specific. PCR is an emerging test.

Step III I asked him about his sexual life. He confessed that he had occasions of extramarital sex during his business tours. I sent his blood for HIV serology and CD4+ count. The reports came as HIV-1 positive and CD4+ T cell count was 150/µL. I asked for induction of expectoration with 3% saline nebulization. Patient developed bronchospasm, and we resuscitated him with nebulization. At that stage FOB was problematic and frankly speaking I was not willing to perform FOB as there was only one scope available.

Discussion The peculiarity of our case was that PCP was the first presentation of HIV infection. PCP was suspected in our case with disproportionate hypoxia (hypoxia more than that expected on the basis of radiological extent of disease), bilateral almost symmetrical perihilar consolidation

Cases

on the background of HIV positivity. The diagnosis was not confirmed microbiologically. Pulmonary complications are most frequent in HIV infected persons. Secondary infections due to immunosuppression are common and infections are important causes of mortality and morbidity. Infections like recurrent bacterial pneumonia, tuberculosis, and PCP are AIDS-defining illnesses. Pneumonia is commonly due to common respiratory pathogens like Streptococus pneumoniae, Haemophilus influenzae, etc. In our country, tuberculosis is most important infection occuring in HIV infected persons. Other mycobacterial infections, other fungal infections, lymphoid interstitial pneumonia, Kaposi sarcoma, and lymphoma are important complications of HIV infection.

Management I empirically started trimethoprim-sulfamethoxazole (TMP-SMX) double strength tablets at the dose of 1 tablets four times daily for 3 weeks. Fortunately patient tolerated the medicine. We continued other supportive measures. I consulted specialist on HIV medicine. As per their advice I started antiretroviral therapy with zidovudine (300 mg twice daily), lamivudine (150 mg twice daily) and efavirenz (600 mg daily). Patient responded and he could maintain SpO2 ≥88% without oxygen. His chest X-ray showed improvement. I discharged him after 2 weeks with advice to continue the medicine for another 1 week and to consult a HIV specialist.

Discussion TMP-SMX is the drug of choice and to be given in higher dose. TMPSMX in doses of 5 mg/kg and 25 mg/kg respectively 6 to 8 hourly orally or intravenously. Treatment should be continued for 21 days in HIV infected persons. The combination may cause toxicities like fever, rash, cytopenias, hepatitis, hyperkalemia, GI disturbances, etc. particularly in HIV infected patients. If patient cannot tolerate the combination then alternative regimen should be considered. Overall prognosis of PCP in HIV infected persons is better than in non-HIV infected persons. HIV infection should be treated with highly active antiretroviral therapy (HAART). HAART usually contains two nucleoside/neucleotide reverse transcriptase inhibitors (NRTI) like zidovudine or tenofovir plus lamivudine or emtricitabine; and one nonnucleoside reverse transcriptase inhibitors (NNRTI) like efavirenz or nevirapine. Protease inhibitors are usually kept reserved My Learning as second line drug. Antiretroviral therapy Pneumocystis jirovecii pneu­ improves survival in HIV-positive patients. monia should be suspected in In addition, it reduces TB rates by up to 90% patients with disproportionate hypoxia in HIV infected persons. and TB recurrence rates by 50%.

155

156

101 Cases in Respiratory Medicine

CASE 39: ACUTE ONSET OF DYSPNEA IN SLE CASE REPORT Step I A 28-year-old lady had multisystemic presentations. According to her, the disease started with nonspecific symptoms like fatigue, malaise, fever, anorexia, weight loss, arthralgia and myalgia. Later she developed painful swelling of hands and she went to a local doctor. She was treated with analgesic but pains worsened. She was referred to a medicine specialist. On examination the doctor found erythematous rashes over malar eminences. She gave a history of photosensitive rashes over-exposed areas. Her peripheral blood showed raised ESR (120 mm in 1st hour), leukopenia (WBC count was 3400/ mm3) and anemia (Hb% was 8.5 g/dL). Urine examination showed proteinuria 3+. Blood ANA and dsDNA were positive. X-ray hand showed no evidence of erosive arthritis. She was diagnosed as systemic lupus erythematous (SLE) on the basis of those findings. She was treated with oral prednisolone 40 mg/day for two weeks then tapered by 5 mg weekly and ultimately she was receiving 5 mg prednisolone on alternate day. She was also given hydroxychloroquine 200 mg twice daily and ibuprofen 200 mg thrice daily for 2 weeks.

Discussion Systemic lupus erythematous is an autoimmune disease caused by immune mediated damage of tissues as a result of tissue-binding autoantibodies and immune complexes. The disease is characterized by multisystemic involvement and heterogeneous clinical manifestations. Though people of all genders, ages, and ethnic groups are susceptible, 90% patients are women in the childbearing age. Prevalence is highest in black women and lowest in white men. The diagnosis of SLE is done with 95% specificity and 75% sensitivity when 4 or more of 11 criteria are present. Sometimes the criteria are accrued over a period of time making the sensitivity of scoring system less accurate. Criteria for diagnosis of SLE are (i) malar rash, (ii) discoid rash, (iii) photosensitivity, (iv) oral ulcers, (v) arthritis vi) serositis (involving pleura or pericardium), (vii) renal disorder (proteinuria or cellular cast), (viii) neurologic disorder (seizures or psychosis), (ix) hematologic disorder (hemolytic anemia, leukopenia, lymphopenia, thrombocytopenia), (x) immunologic manifestations (positive anti-dsDNA, anti-SM and antiphospholipid antibodies) and (xi) positive test for antinuclear antibodies (ANA). ANA is positive in about 98% cases. ANA has more negative prediction value (negative results usually exclude SLE) but it has less positive prediction value (as several diseases including old age may have positive results). Hightiter antibodies to double-stranded DNA and Sm antigen are specific for SLE. Treatment of SLE depends on organ involved, life-threatening conditions, damage to vital organs and chance of reversibility of particular manifestations. It is also important to prevent and treat complications of

Cases

SLE. Another important factor is the toxicity and tolerability of medicines (glucocorticosteroids or immunosuppressant drugs). Conservative management in absence of vital organ damage and lifethreatening condition includes NSAIDs (non-steroidal anti-inflammatory drugs) and antimalarial drugs (hydroxychloroquine, chloroquine, and quinacrine). Antimalarials are used for reducing dermatitis, arthritis, and fatigue. Hydroxychloroquine is useful in reducing tissue damage and joint manifestations. Dermatitis may be managed with topical sunscreens, antimalarial, and topical glucocorticoids. In some cases (serositis, renal, hematological or neurological involvement) corticosteroids are indicated. Azathioprine or methotrexate may be considered as steroid spearing agent or as an additive agent.

Step II Six months after diagnosis of SLE, she developed acute onset of dyspnea. She had episodes of hemoptysis (blood-stained expectoration). She attended emergency and was admitted in casualty ward. She was resuscitated and was referred to me. As I examined her, I found she was severely dyspneic with accessory muscles of respiration working. General survey showed presence of anemia; BP was 90/50 mm of Hg; pulse was 116/min; RR was 42/min and SpO2 was 94% with 6 L O2 per minute. Examination of lungs revealed few scattered fine crackles. Her peripheral blood showed Hb—6.8 g/dL; WBC—5600/mm3, N—74%, L—24%, M—2%, E—2% and ESR—86 mm in 1st hour. Her routine blood biochemistry results were within normal limits. Her chest X-ray PA view (Fig. 39A) showed bilateral patchy heterogeneous opacities distributed over all zones of both lungs with areas of confluent opacities looking like consolidation.

Fig. 39A: Chest X-ray showing bilateral diffuse nonhomogeneous opacities

157

158

101 Cases in Respiratory Medicine

Fig. 39B: HRCT-thorax showing bilateral dense centrilobular nodules with confluent areas

I transferred her to our respiratory care unit and started primary care with intravenous fluid, antibiotics, tranexamic acid, corticosteroids and moist O2 inhalation. After stabilizing her condition I sent her for HRCT thorax. Her HRCT thorax (Fig. 39B) showed bilateral dense centrilobular opacities, patchy ground-glass opacities and areas of consolidations. Opacities typically spear fissure lines and subpleural space. I diagnosed the case as diffuse alveolar hemorrhage (DAH) secondary to SLE.

Discussion Respiratory system involvement in SLE is quite common. Pleuritis with or without pleural effusion is the most common pulmonary complication. The differential diagnosis of diffuse pulmonary infiltrations in SLE includes infection, interstitial lung fibrosis, and alveolar hemorrhage. DAH and lupus pneumonia are two life-threatening lung conditions found in SLE. DAH should be suspected in presence of (i) hemoptysis; (ii) bilateral airspace opacities more prominent in perihilar areas, mid and lower zones of lung; (iii) patchy or diffuse bilateral ground-glass opacities or consolidations over the clinical setting of SLE, microscopic polyangiitis, Goodpasture syndrome, Wegener granulomatosis, etc. DAH is caused by widespread damage of small pulmonary vessels (arterioles, capillaries, and venules) due to immunological mechanism. The causes of DAH are either immune complex mediated like SLE, HenochSchönlein purpura or microscopic polyangiitis; or due to an antibody against the alveolar basement membrane like Goodpasture’s syndrome; or due to direct effects of autoantibodies on the alveolar capillary endothelium like Wegener’s granulomatosis; or idiopathic pulmonary hemosiderosis. Cough, dyspnea, chest pain and hemoptysis are the usual presentation. However, hemoptysis may not be present, and in those cases diagnosis is usually difficult. DAH should be clinically suspected when a patient had (i) acute onset of dyspnea with or without hemoptysis; (ii) unexplainable fall

Cases

of hematocrit; and (iii) acute diffuse pulmonary infiltrate in chest radiology in the setting of above mentioned clinical conditions. FOB will be helpful by demonstrating bloody lavage fluid or hemosiderin-containing macrophage. The chest radiograph in DAH demonstrates diffuse or focal patchy alveolar infiltration, and HRCT thorax reveals ground-glass abnormality, confirming the presence of air space-filling disease. Blood examination usually reveals iron-deficiency anemia. An unexpected increase in the diffusion capacity may give an important clue.

Management I started intravenous pulse doses of methylprednisolone 1000 mg daily for 3 days. That was followed by oral prednisolone 40 mg daily. I performed fiberoptic bronchoscopy after 5 days and took bronchoalveolar lavage. The lavage fluid showed hemosiderin laden macrophages. I discharged the patient after 10 days in stable condition.

Discussion Management of life-threatening SLE is systemic glucocorticoids (0.5–1 mg/kg per day orally or 1000 mg of methylprednisolone sodium succinate intravenously daily for 3 days followed by prednisone 0.5–1 mg/kg per day. Oral My Learning prednisolone should be continued for 4–6 Early diagnosis and immediate weeks and thereafter should be tapered as treatment with pulse doses of rapidly possible depending upon clinical methylprednisolone in DAH is situation. The maintenance daily dose varies life-saving. from 5–10 mg of prednisone orally.

CASE 40: BILATERAL CONSOLIDATION IN A YOUNG LADY CASE REPORT Step I A 23-year-old lady presented with high rise of temperature with chill and rigor and went to a local doctor and was treated with oral antibiotics along with antipyretics and other supportive medications. As she was not responding and her condition deteriorated she was transferred to Kolkata. She was admitted in our inpatient department. She had continuous high fever for about ten days that temporarily and incompletely respond to paracetamol. She did not have headache, body ache, myalgia, arthralgia or dysuria. She had cough with little expectoration for ten days and shortness of breath that progressed to dyspnea even at rest at the time of admission. She developed bilateral sharp chest pain for last 3 days over lateral sides of chest that worsened during breathing and coughing. Two years back, she had joint pain affecting small joints of hands without swelling. She was diagnosed as rheumatoid arthritis on the basis of rheumatoid arthritis factor (RA factor) positivity and was treated with oral prednisolone

159

160

101 Cases in Respiratory Medicine

and hydroxychloroquine. She stopped medicines by herself after 2 weeks and she had no problem afterwards till recent illness. General survey revealed that she was conscious, toxic, prostrated, temperature 103°F, BP—100/50 mm of Hg, pulse—140/min, respiratory rate—40/min, pallor was present and O 2 saturation was 92% with 2 L/ min O2. Chest examination revealed impaired percussion notes in both infrascapular areas with decreased vesicular breath sound on those areas and few bilateral fine basal crepitations. Her trachea and apical impulse were not shifted. Her liver and spleen were not palpable. Cardiovascular system was normal except tachycardia and there was no neurological abnormality. She had no joint deformity, oral ulcer, alopecia, skin rash or bleeding point. Peripheral blood showed Hb%—9.1 g/dL; WBC—1800/mm3; N—83, L—17; platelet—225000/mm 3 and ESR—110 mm in 1st hour. Blood biochemistry showed fasting sugar—78 mg/dL ; urea—22 mg/dL ; creatinine—0.9 mg/dL and liver function tests were normal. Sputum examination revealed epithelial cells >10/high power field (HPF) and neutrophil 2-times the upper limit of serum level. Pleural effusion in SLE commonly occur in women; usually small and bilateral; cells are polymorphonuclear leukocytes in early phase and mononuclear cell in later phase; and it is frequently associated with enlarged cardiac silhouette. Pleural fluid glucose is usually more than 80 mg/dL; pH is more than 7.2 and LDH is less than 2-times the upper limit of serum level. Pleural fluid ANA, LE cell and dsDNA are nonspecific in SLE pleural effusion. Except high LDH all parameters could be explained by SLE pleural effusion.

Management The case was provisionally diagnosed as lupus pneumonia. The diagnosis of SLE was suggested by pancytopenia, polyserositis, pulmonary parenchymal involvement and past history of arthralgia. As the condition is a potentially life-threatening, we started pulse dose of methylprednisolone (intravenous 1000 mg/day for 3 days) after sending blood for serology. Patient responded and we switch over to oral prednisolone (40 mg/day) and after 7 days she was discharged. Her report came and antinuclear antibody (ANA) was strongly positive (3+). My final diagnosis was lupus pneumonia.

Discussion We seriously considered the possibility of SLE in the differential diagnosis as patient had pancytopenia, polyserositis, bilateral pneumonia and past history of joint pain without deformity. Lupus pneumonia is an unusual (50% cases). In emergency management, it is But sometimes it responds dramatically to the clinical prudence and clinical steroids with complete or partial clearance. judgement, not the academic In some cases cyclophosphamide is required. correctness that saves live.

163

164

101 Cases in Respiratory Medicine

DIFFUSE PARENCHYMAL LUNG DISEASES CASE 41: DPLD WITH SYSTEMIC MANIFESTATION CASE REPORT Step I A 32-year-old lady had cough and dyspnea for 3 months. She came to me with hemoptysis for 10 days. When she entered my chamber my eyes fell on her hands (Fig. 41A). Her fingers were sclerosed, erythematous and there were erosion of distal phalanges of both index fingers and complete erosion of distal phalange of left ring finger. When I asked her how those changes occurred, she answered that those changes were due to exposure to cold. On enquiry, she told me that with exposure to cold her fingers became blue and painful. She also confessed that she had dysphagia; she could not open her mouth totally; and she had arthralgia involving large joins of both sides. I looked for signs of systemic sclerosis. She could not open her mouth completely and I could not drag down her lower eyelids due to stiffness of her face. She had mild pallor, BP—130/86 mm of Hg; pulse—124/min; RR—36/ min; she was dyspneic and her SpO2 was 74% at room air. Examination of her hands revealed skin tightness and erythema over fingers. Skin over fingers could not be pinched up. There was no telangiectasia. I observed erosions of tips of fingers and distal phalanges (autoamputation). I diagnosed the case as systemic sclerosis.

Discussion Systemic sclerosis (SS) is a disorder of connective tissue with diffuse cutaneous and organ (lungs, gastrointestinal tract, heart, and kidneys)

Fig. 41A: Picture of hands showing sclerodactyly and autoamputation of distal phalanges of both index fingers and left ring finger

Cases

involvement of unknown etiology. Skin involvements include progressive skin indurations, starting in the fingers and ascending from distal to proximal part of extremities, the face, and the trunk. Characteristically, there are thickening of the skin (scleroderma), sclerosis of fingers (sclerodactyly), Raynaud’s phenomenon, calcinosis cutis and telangiectasia. Raynaud’s phenomenon is characterized by typical sequential color changes of fingers in response to cold. Raynaud’s phenomenon is due to vascular involvement of fingers and classical color changes in sequence are white due to vasoconstriction, blue due to ischemia and red due to reperfusion. Raynaud’s phenomenon may be primary or secondary, caused by systemic sclerosis or other connective tissue diseases, hematologic disorders, endocrine conditions, occupational disorders and induced by drug (beta-blocker atenolol and anticancer drugs such as cisplatin and bleomycin). Chronic ulcerations at fingertips may occur due to lack of vascular supply leading to breakdown of atrophic skin. Ulcers are usually painful and may get infected. Osteomyelitis of distal phalanges is not uncommon. The finding of fixed digital pits may occur as a result of healing of ulcers. Loss of soft tissue at the fingertips due to ischemia is a frequent finding and it may be associated with resorption of the terminal phalanges, known as acro-osteolysis. Our patient was suffering from system sclerosis and the diagnosis was made confidently from history and general survey. Examination of face and fingers is very important for diagnosis of systemic sclerosis. In our case, the disease was in advance stage and the diagnosis was quite obvious. In early stages, the disease may be missed and it needs careful general survey to detect early signs.

Step II I took history regarding her respiratory problems. She told me that she had cough of insidious onset that occurred mainly during talking and during exertion. Cough was mainly dry but sometimes she had small mucoid/purulent expectoration. She had occasional blood streaked expectoration but for last 10 days she had frank hemoptysis. Dyspnea was also insidious onset, progressive, without wheeze and at the time of presentation she had dyspnea on minimum exertion. She had low grade irregular fever with evening rise of temperature. General survey was performed with difficulty. She had mild pallor, BP—130/86 mm of Hg; pulse—124/min; RR—36/min; she was dyspneic with accessory muscles were working and her SpO2 was 74% at room air. Examination of respiratory system revealed coarse crackles scattered over both lungs. There were epigastric pulsation; left parasternal heave and loud pulmonary component of second heart sound. Peripheral blood examination revealed Hb%—7 g/dL; WBC—12600/mm3 with neutrophil 92%; platelet count—4 lacs/mm3 and ESR—90 mm in 1st hour. Routine blood biochemistry was within normal level. Her chest X-ray PA view (Fig. 41B) showed bilateral patchy consolidations along with reticular opacities. High-resolution computed tomography (HRCT) thorax (Fig. 41C)

165

166

101 Cases in Respiratory Medicine

Fig. 41B: Chest X-ray showing bilateral reticulonodular opacities with areas of patchy consolidations

Fig. 41C: HRCT-thorax showing bilateral ground-glass opacities with cystic spaces; reticular opacities and dilated esophagus

showed bilateral almost symmetrical ground glass opacities with areas of consolidations and cystic spaces of different sizes mainly distributed at the periphery, but there was no honeycombing or traction bronchiectasis. The esophagus was dilated. HRCT finding of diffuse parenchymal lung disease (DPLD) with esophageal dilatation is strongly suggestive of systemic sclerosis. Echocardiography showed right ventricular enlargement and pulmonary arterial systolic pressure was about 50 mm of Hg. Urine examination revealed no abnormality and her creatinine was 0.9 mg/dL. Her specific blood examination showed that anti-Scl 70 (topoisomerase antibody) was positive; ANA was positive; rheumatoid factor was positive; anti-centromere antibody IgG was negative and anti-U1RNP antibody was negative.

Cases

Discussion Mixed connective tissue disease (MCTD) comes in the differential diagnosis of systemic sclerosis. MCTD is characterized by features of systemic sclerosis with coexisting features of SLE, polymyositis, and rheumatoid arthritis. Typically, there is high titer of autoantibodies to U1RNP. The disease initially presents with Raynaud’s phenomenon, puffy fingers and myalgia. Gradually other features of systemic sclerosis (sclerodactyly, calcinosis, and cutaneous telangiectasia); features of SLE (malar rash, photosensitivity) or dermatomyositis (heliotrope rash on the eyelids, erythematous rash on the knuckles) and arthralgia (sometimes erosive polyarthritis) develop. Respiratory manifestations include pulmonary fibrosis and isolated or secondary pulmonary arterial hypertension (PAH). Laboratory tests showed elevated ESR, hypergammaglobulinemia and high titers of anti-U1RNP antibodies. MCTD has a good response to treatment with glucocorticoids, and a better prognosis than systemic sclerosis. I excluded MCTD as there was no clinical manifestation of other collagen vascular disease as well as low titer of anti-U1RNP antibody. Pulmonary involvement in systemic sclerosis is characterized by pulmonary fibrosis and pulmonary arterial hypertension. Lungs may also be involved by aspiration pneumonia (due to esophageal dysmotility) and malignancy (bronchioloalveolar carcinoma). In our patient, chest X-ray showed opacities sparing lower zones that was inconsistent with aspiration pneumonia. HRCT findings were consistent with nonspecific interstitial pneumonia (NSIP), a common type of diffuse parenchymal lung disease (DPLD) found in systemic sclerosis. NSIP pattern should be differentiated from usual interstitial pneumonia (UIP) pattern. The findings of more ground glass opacity and less fibrosis favor NSIP pattern, whereas more fibrosis, basal involvements, honeycombing and traction bronchiectasis are suggestive of UIP pattern. DPLD could explain cough and progressive dyspnea in our case. Streaks of blood and purulent expectoration might be explained by superadded infection. But frank hemoptysis could not be explained by that mechanism. Pulmonary arterial hypertension (PAH), defined as a mean pulmonary arterial pressure more than 25 mm of Hg at rest, is found in around 15% cases of systemic sclerosis. PAH in systemic sclerosis may occur in association with DPLD or may occur in isolation. Patients with PAH may be asymptomatic or may have exertional dyspnea, angina, syncope, and manifestations of right-sided heart failure. Doppler echocardiography is widely used for the diagnosis of PAH.

Step III At that point, my diagnosis was systemic sclerosis with pulmonary fibrosis (NSIP pattern) and PAH. I could not find out the cause of frank hemoptysis. A carefully look at the HRCT plates gave an important clue. There was a fungal ball within a cystic space with air crescent sign (Fig. 41D). The air crescent moved

167

168

101 Cases in Respiratory Medicine

Fig. 41D: HRCT-thorax showing cysts filled-up with solid material and air-crescent sign (arrow head)

with supine and prone HRCT-thorax. The cause of hemoptysis was identified as secondary colonization of cystic space, and the diagnosis was aspergilloma.

Management I treated aspergilloma with oral itraconazole 200 mg twice daily along with conservative medical management of hemoptysis. Surgical options were not considered as patient had multiple cystic spaces and fungal infection might be more extensive than visible in HRCT thorax. Usually ground glass opacities indicate more inflammation and as a result, they are usually responsive to corticosteroids. I did not consider corticosteroids in our case as ground glass opacities in systemic sclerosis was considered to be due to microfibrosis and not by granuloma or inflammation. Moreover, corticosteroid can precipitate renal failure in systemic sclerosis. I started mycophenolate mofetil orally in dose of 500 mg twice daily. Cyclophosphamide is found to be effective in reducing the progression of DPLD in systemic sclerosis, but it is very toxic and usually it is not well-tolerated. Toxicities of cyclophosphamide include bone marrow suppression, opportunistic infections, hemorrhagic cystitis, bladder cancer, ovarian failure and secondary malignancies. I started sildenafil citrate for PAH orally 25 My Learning mg twice daily. For Raynaud’s phenomenon, G e n e r a l s u r ve y m ay gi ve I advised her to dress warmly, wear gloves, important clue in DPLD. Clinical minimize cold exposure or stress, and avoid journey should continue till you drugs particularly beta blockers, and started are totally satisfied and be able to explain all symptoms. nifedepine 10 mg thrice daily orally.

Cases

CASE 42: A CLASSICAL TEXTBOOK CASE OF DPLD CASE REPORT Step I A 57-year-old man presented with cough and dyspnea for 3 months. As per patient cough was present for more than 3 months, more frequent at night and sometimes sleep was disturbed. There was no expectoration. Dyspnea was insidious in onset and progressive. At the beginning, dyspnea was with strenuous work but at the time of presentation he felt dyspneic with his day-today activities. There was no wheeze, hemoptysis, chest pain or fever. He was a school teacher and he smoked on average 5 cigarettes per day for more than 30 years. There was no other significant present, past, personal or family history. On examination, general survey revealed BP—130/80 mm of Hg; pulse 110/ min regular; respiratory rate—32/min at rest and 2° clubbing. In respiratory system positive findings were active accessory muscles of respiration and end inspiratory crackles in both lung bases. His peripheral blood count and blood biochemistry were within normal limits. His chest X-ray PA view (Fig. 42A) showed reticulonodular opacities both lower zones with fuzzy cardiac and diaphragmatic borders. Pulmonary vessels were also indistinctly visible.

Discussion The 57-year-old man, a smoker, had cough without expectoration, dyspnea without wheeze and bilateral basal end inspiratory crackles. All findings were suggestive of diffuse parenchymal lung disease (DPLD). The terms interstitial lung disease (ILD) and DPLD are interchangeable. The term DPLD is preferable than ILD as alveoli are also affected along with interstitium. In some conditions, the disease actually starts from alveoli causing alveolitis.

Fig. 42A: Chest X-ray showing bilateral reticular patterns with fuzzy cardiac and diaphragmatic borders

169

170

101 Cases in Respiratory Medicine

However, ILD is a widely accepted term. For that reason both terms are equally accepted. End inspiratory crackles are due to sudden opening of closed airways. As lungs become stiff more negative pressure is required to open peripheral airways. At later part of inspiration, the required negative pressure is generated and the peripheral airways opened up causing end inspiratory crackles. The end inspiratory crackles are also described as velcro crackles because of its similarity with sound produced by velcro. As our patient was a smoker chronic obstructive pulmonary disease (COPD) should come in the differential diagnosis. The shorter duration, no expectoration, absence of rhonchi and classical crackles went against the diagnosis of COPD. Dry cough and basal crackles may be found in chronic left heart failure, cardiomyopathy, post-infarction left ventricular dysfunction, hypertensive heart disease, etc. Sometimes, detail cardiological examination and echocardiography may be required to differentiating those conditions from DPLD. In DPLD patients, chest X-ray shows signs of volume loss that will be more demonstrable with serial X-rays. Opacities are typically described as nodular, reticulonodular, ground-glass, consolidation and cystic lesions or a combination of those shadows. In chest X-ray, gross radiological patterns can be identified. In early stages, the borders of heart or the diaphragm will be fuzzy. Indistinct borders may be the early radiological sign. The classification of DPLD needs HRCT-scan of thorax. HRCT-thorax is important to identify the patterns of attenuation. Then by analyzing the pattern or combination of patterns along with clinical manifestations the diagnosis is confirmed. Some studies in Japan suggested that diagnosis based on HRCT findings is as good as that of lung biopsy.

Step II We advised HRCT-thorax. His HRCT-thorax (Fig. 42B) showed reticular opacities mainly in the lower zones but also involving other zones. There were few areas of ground-glass opacities. There were subpleural honeycombing and traction bronchiectasis. The findings were consistent with the diagnosis of usual interstitial pneumonia (UIP).

Discussion Radiologically UIP pattern is diagnosed on the basis of four criteria: (i) subpleural, basal predominance; (ii) reticular abnormality; (iii) honeycombing with/without traction bronchiectasis; and (iv) absence of features listed as inconsistent with UIP pattern. Features listed as inconsistent with UIP pattern include: (i) upper or mid lung predominance; (ii) peribronchovascular predominance; (iii) extensive ground glass abnormality (extent > reticular abnormality); (iv) profuse micronodules (bilateral, predominantly upper lobes); (v) discrete cysts (multiple, bilateral, away from areas of honeycombing); (vi) diffuse mosaic attenuation/air trapping

Cases

Fig. 42B: HRCT-thorax showing predominantly reticular pattern with subpleural honeycombing (arrow head) and traction bronchiectasis (arrow)

(bilateral, in 3 or more lobes); and (vii) consolidation in bronchopulmonary segment (s)/lobe(s). Presence of any of the seven features should be taken as inconsistent with UIP pattern, and thereby the diagnosis of UIP is excluded. When all four criteria including bilateral, basal, subpleural predominantly reticular pattern with honeycombing and without any sign inconsistent with UIP pattern are present, our diagnosis should be ‘consistent with UIP pattern’. In that situation lung biopsy is usually not required. When the third criterion (honeycombing with/without traction bronchiectasis) is absent the diagnosis will be ‘probable of UIP pattern’. In probable UIP, biopsy (open lung biopsy) should be considered for confirmation of diagnosis. UIP pattern is usually idiopathic and the disease is named as idiopathic pulmonary fibrosis (IPF). The disease was previously called cryptogenic fibrosing alveolitis. UIP pattern may be caused by many conditions including, connective tissue diseases (rheumatoid arthritis), occupational lung diseases (asbestosis), drug-induced lung diseases (nitrofurantoin), radiation-induced lung diseases, chronic aspiration, sarcoidosis, chronic hypersensitivity pneumonia, organized chronic eosinophilic pneumonia and pulmonary langerhans cell histiocytosis (PLCH). IPF is a diagnosis by exclusion of other causes of UIP pattern. The diagnosis of UIP should be made with caution as the disease is potentially a dangerous condition. At that point, our job was to exclude other causes of UIP pattern by taking detail history. In some cases, the manifestations of interstitial lung disease may precede the development of collagen vascular diseases, and collagen screening tests are mandatory.

Step III We took detailed history and excluded drug-induced lung diseases, occupational lung diseases and hypersensitivity pneumonia. There was no

171

172

101 Cases in Respiratory Medicine

history of joint pain or any manifestations of systemic involvement. His serum rheumatoid factor, antinuclear factor and serum angiotensin converting enzyme was negative. Finally, we diagnosed the case as IPF.

Discussion Idiopathic pulmonary fibrosis is a specific form of chronic, progressive fibrosing interstitial pneumonia of unknown etiology. IPF occurs in adults and it is limited to the lungs. Histological and/or radiological pattern of IPF is that of UIP. Essential step for the diagnosis of IPF is to exclude other forms of idiopathic interstitial pneumonia and interstitial lung diseases associated with environmental exposure, medication, or systemic disease. Separating IPF from other interstitial lung diseases is a vital step as IPF, like malignancies, has a bad prognosis and has a markedly poor response to therapy. The clinical manifestations of IPF include exertional dyspnea, a nonproductive cough, and end inspiratory crackles predominantly at lung bases. Smoking history and digital clubbing may/may not be present. Highresolution computed tomography (HRCT) lung scans shows usual interstitial pneumonia (UIP) pattern. Spirometry shows a restrictive pattern, a reduced diffusion capacity, arterial hypoxemia and respiratory alkalosis. Exerciseinduced hypoxia is considered as an early sign. Confirmation sometimes requires open lung biopsy. The histological hallmark is heterogeneous appearance at low magnification with alternating areas of normal lung, interstitial inflammation, foci of proliferating fibroblasts, dense collagen fibrosis, and honeycombing. The histological pattern is opposite to the typical spatially and temporally uniform pattern found in non-specific interstitial pneumonia. The interstitial inflammation is patchy with infiltration of lymphoplasmacytes in the alveolar septa and hyperplasia of type 2 pneumocytes. Fibrotic zones are composed of dense collagen with scattered proliferation of fibroblasts. Honeycombing is characterized by cystic fibrotic air spaces usually lined by bronchiolar epithelium that contains mucin. There are areas of smooth-muscle hyperplasia. The natural history of IPF is progressive relentless deterioration with exacerbations (infective and immunogenic). The differentiation between infective and non-infective exacerbation is important as former requires antibiotics and later requires pulse doses of methyl prednisolone. Other causes of exacerbations are pulmonary embolism, pneumothorax, heart failure and ischemic heart disease. Acute exacerbation of IPF is a phase of rapid clinical deterioration, and is recognized by worsening of dyspnea within days to weeks; newly developing diffuse ground-glass abnormality and/or consolidation superimposed on UIP pattern; worsening hypoxemia; and absence of other causes of exacerbations.

Management We treat him with omeprazole 20 mg in the morning in empty stomach. We gave him nocturnal oxygen through nasal prongs. We started pirfenidone

Cases

1800 mg orally daily. Patient developed gastrointestinal side effects and we had to stop medicine. His conditions deteriorated and we gave 1,000 mg methyl prednisolone intravenously for 3 days. Patient did not responded and we lost the patient about 6 months after diagnosis.

Discussion General treatment of IPF consists of (i) proton-pump inhibitors, should be given to neutralize gastroesophageal reflux that is considered as an etiological or precipitating factor; (ii) oxygen therapy for patients with hypoxia at rest or patients with exercise-induced hypoxia; (iii) pulmonary rehabilitation with exercise training, nutritional counseling, energy-conserving techniques, breathing strategies, psychological counseling and patient education; and (iv) lung transplantation. IPF with coexisting emphysema have a worse prognosis. Those patients usually develop pulmonary hypertension and they may require long-term oxygen therapy. Two drugs have been introduced and approved for the treatment of IPF, and they are pirfenidone and nintedanib. Neither drug is a cure for IPF but possibly they inhibit important pathways and help to prevent scarring, and thereby slowing the progression of the disease. Apart from the cost, the drugs are less tolerated at the higher doses required for their effectiveness. Pirfenidone, a novel compound with combined anti-inflammatory, antioxidant, and antifibrotic effects, had potential therapeutic benefits for idiopathic pulmonary fibrosis. The precise mechanism of pirfenidone is not clear but it may act by inhibiting transforming growth factor (TGF)-beta, a chemical mediator that controls many cell functions including proliferation and differentiation of fibroblast, and thereby prevent pulmonary fibrosis. It also inhibits the synthesis of TNF-alpha, and thereby prevents inflammation. A higher dose (2403 mg/day) of pirfenidone prolonged progression-free survival by 26% compared with placebo and significantly reduce the decline of FVC. Pirfenidone causes gastrointestinal side effects (dyspepsia, nausea, gastritis, gastroesophageal reflux disease and vomiting); dermatological side effects (photosensitivity reactions, rash, pruritus and dry skin); increase hepatic enzyme levels; dizziness and fatigue and weight loss. Nintedanib inhibits tyrosine kinases and targets growth factors like vascular endothelial growth factor receptor, fibroblast growth factor receptor, and platelet-derived growth factor receptor. It binds with adenosine triphosphate binding pockets of these receptors and blocks the intracellular My Learning signaling, crucial for the proliferation, IPF is diagnosed by excluding migration, and transformation of fibroblasts. other DPLD patterns and by Nintedanib has the side effects like diarrhea, excluding other causes of UIP pattern. nausea and vomiting, and those sometimes lead to discontinuation of the drug.

173

174

101 Cases in Respiratory Medicine

CASE 43: DPLD WITH CHANGING DIAGNOSIS CASE REPORT Step I A 65-year-old lady had insidious onset of shortness of breath and cough for more than 3 years. As per patient’s statement, she had dyspnea for long time and she could not definitely say the exact duration of illness. Dyspnea was initially occurring during strenuous work. The degree of dyspnea increased over years and now she felt dyspneic even at rest. Gradually her normal activities like going to toilet, taking bath, etc. were affected. She could not lie down and had to go to sleep with two to three pillows below her head. However, her sleep was not disturbed. She had intermittent attacks of cough and that increased with talking and exertion. She did not have expectoration or wheeze. She felt weakness, but she never had fever. She consulted local physician and she was diagnosed as asthma. She was treated with meter dose inhalers with spacer and nebulization. She was receiving salmeterol and fluticasone combination 250 µg 2 puffs twice daily regularly and salbutamol 200 µg two puffs thrice daily. She was asked to take nebulization with salbutamol solution when necessary. She was frequently treated with short courses of oral prednisolone and oral antibiotics. As her condition was deteriorating she consulted a medical specialist. A series of investigations were done. Peripheral blood and routine blood biochemistry were within normal limits. Her ANA and rheumatoid factor were negative. Chest X-ray (Fig. 43A) showed reticular opacities in both lower zones. There were scattered opacities in other zones bilaterally. Spirometry, done 6 months back, showed restrictive pattern with sharp, short and small flow volume loop, and FEV1/FVC was 76%. FEV1, FVC, FEF 25–75% and PEFR were 36%, 28%, 44% and 28% of predicted values respectively. HRCT-thorax (Fig. 43B) showed bilateral predominantly reticular attenuations and scattered areas of ground-glass opacities. There were few cystic spaces. The radiologist reported the case as UIP pattern. The case was referred to me for opinion.

Discussion The elderly lady had progressive dyspnea without wheeze and cough without expectoration. She was not sure about the duration and it was presumed that her illness was of more than 3 years duration. The disease had progressed over years without symptom free interval or variability. The combinations of symptoms were suggestive of interstitial lung disease. Asthma may not have wheeze or expectoration, and the diagnosis of chronic asthma was not completely ruled out. Asthma should have some degree of variability and reversibility. Our patient did not show any improvement and the disease was relentlessly progressive. Moreover, spirometry showed restrictive pattern. Chronic heart failure may have same presentation and should come in the differential diagnosis. Her investigations, chest X-ray, spirometry and HRCT-thorax, findings were suggestive of DPLD. DPLD is not a complete diagnosis. The next step

Cases

Fig. 43A: Chest X-ray showing reticular opacities in both lower zones with scattered opacities in other areas

Fig. 43B: HRCT-thorax showing bilateral predominantly reticular opacities

175

176

101 Cases in Respiratory Medicine

should be to classify the disease. The gold standard for classification of DPLD is lung biopsy. Lung biopsy may be transbronchial or open lung biopsy. Transbronchial route is simpler with minimum complications (except bleeding and pneumothorax) in expert hands. The main problem is the availability of tiny tissue and even that tissue is crushed making histological diagnosis and classification practically impossible. Open lung biopsy is a cumbersome job and needs general anesthesia. Surgeons are usually reluctant to perform open lung biopsy, and they always prefer lingular segment by small thoracotomy. Most of the pathologists lack the expertise to classify DPLD as they are not getting enough cases of lung biopsy. HRCT-thorax is usually used as an effective tool to classify DPLD. Several studies have demonstrated almost equal efficacy of HRCT-thorax in classifying DPLD comparing with open lung biopsy. Conventional CT has problems of quality and needs longer time. Dyspneic patient cannot hold breath and thereby producing motion artifact. Helical CT technology has the advantage of faster scan, better contrast enhancement and thinner collimation. As imaging is obtained within a single breath-holding maneuver, there is less chance of motion artifact. In addition, helical CT scanning allows imaging reconstruction into coronal, or sagittal planes. Newer detector technologies include additional detectors along the scanning axis (z-axis). The multidetector CT (MDCT) scanners can obtain multiple thinner slices in a single rotation within a shorter period of time. MDCT has enhanced resolution capability to produce clearer final images and increased image reconstruction ability. MDCT requires shorter breath holds, and is very helpful especially in children, elderly, and critically-ill patients. In HRCT, UIP is diagnosed by bilateral basal, subpleural, mainly reticular attenuations with honeycombing/traction bronchiectasis and without any features inconsistent with UIP pattern. The elderly lady was diagnosed as UIP by the radiologist. UIP pattern is associated with worst prognosis, almost similar to that of cancer, and the scope for treatment is minimum. So, we must be very careful before diagnosing a case as UIP/idiopathic pulmonary fibrosis (IPF).

Step II The stricking point in the case was the longer duration of disease and slower progression. I took history in detail. She had no joint pain or any manifestations of system involvement. She was nondiabetic, nonhypertensive and she was HIV negative. She was not using any long-term medicines except those prescribed for her lung condition. She was a housewife and she had no history of exposure with organic or inorganic dust. General survey did not give any clue except signs of dyspnea, tachycardia (heart rate—104/min), tachypnea (RR—42/min) and hypoxia (SpO2—86% at room air). Respiratory system examination revealed few end inspiratory crackles scattered over both lung fields. I examined her HRCT-thorax meticulously. I could find out some inconsistency for the diagnosis of UIP. Firstly, there was no honeycombing

Cases

Fig. 43C: HRCT-thorax in normal cuts and sagittal cut showing relatively clear lung bases

or traction bronchiectasis, and that meant the case should be classified as ‘probable UIP pattern’ in HRCT. Secondly, as I looked downwards I found comparably clearer lung bases (Fig. 43C) at lower cuts as well as in sagittal section. The feature was not consistent with the diagnosis of UIP pattern. I diagnosed the HRCT pattern as fibrotic nonspecific interstitial pneumonia (NSIP).

Discussion Most important point in the diagnosis of UIP pattern is the absence of any features those are inconsistent with UIP pattern like more ground-glass attenuation, nodular attenuation, cystic lesions other than honeycombing, mosaic attenuation, consolidation, and upper/mid zone or peribronchovascular predominance. As lung bases were spared I was totally confused. I thought it would be prudent to classify the pattern as fibrotic NSIP. NSIP, as per my understanding at that time, should include all patterns inconsistent with other specific diagnosis. Actually in 1994, NSIP was first described as histological patterns which did not fit with any of the existing descriptions of interstitial pneumonia. Later on, it was established that NSIP include some specific radiological and histological pattern. Radiologically, NSIP is characterized by (i) lower lung predominance; (ii) peripheral and/ or peribronchovascular distribution; (iii) ground-glass abnormality; (iv) lower lobe volume loss; and (v) minimum reticular abnormality, sparse or no honeycombing, or consolidation. Histologically, NSIP is characterized by spatially and temporally uniformity in all sections that differentiate NSIP from UIP. There are three histological types of NSIP depending on amount of inflammation and fibrosis. Type I—predominantly inflammatory; type II—mixed; and type III—predominantly fibrotic. NSIP includes a spectrum of diseases varying from the predominantly ground-glass pattern that has better respond with treatment to predominantly reticular pattern with poor response to treatment. Survival of fibrotic NSIP is poorer as low as 45% for 5 years. The mean age of diagnosis is 45–55 years, and the average duration of illness is usually 6–12 months. CT features of NSIP include predominantly ground-glass abnormalities and minimum reticular abnormalities.

177

178

101 Cases in Respiratory Medicine

Traction bronchiectasis and honeycombing are either sparse or absent. Ground-glass abnormality is the most common abnormality and is found in more than 90% cases. About half of the cases of NSIP are associated with collagen vascular diseases, hypersensitivity pneumonia, inhalation exposure and drug exposure. The treatment of NSIP is mainly by corticosteroids and immunosuppressive drugs (azathioprine). There is no special mention of treatment of fibrotic NSIP.

Step III I was not sure and I was not satisfied with my diagnosis. I tried again to find out the etiology of NSIP pattern. Finally, I was successful. Her daughter-in-law told me that the lady used to feed pigeons religiously twice daily in their terrace till last 6 months. Patient did not disclose that history and I did not asked that specifically. I made the final diagnosis as chronic hypersensitivity pneumonia (HP). Then I was able to explain predominantly upper zone involvement, areas of air tapping and little loss of lung volume, the findings I had ignored previously.

Discussion Hypersensitivity pneumonia is due to exposure of organic dusts. It is probably the most common type of DPLD in our country. Sometimes, patient are too habituated with a practice to consider it special. Patient often cannot correlate his disease with his habits. The diagnosis is often missed unless the specific history is specifically asked. Among non-Bengali community, particularly the Marwari community, there is a religious practice to feed pigeons. Pigeons are considered as a sign of prosperity and wealth. In Bengalis also, there is a ‘babu culture’ of keeping and feeding pigeons. Pigeon’s feather and excreta are notorious to cause HP in susceptible individuals. Radiologically, HP presents with ill-defined centrilobular nodules without lung volume loss. As the disease progress, there is fibrosis that makes the disease non-reversible. The differentiation between NSIP and hypersensitivity pneumonia is sometimes blunt. Later should be suspected in presence of nodules or air tapping. I should confess here that I missed the HRCT findings like spearing lung bases, no lung volume lose and areas of air tapping (Fig. 43B), and thereby I had missed the diagnosis. Sometimes, I wonder why farmers’ lung is so uncommon in India though there is no scarcity of moldy hay in rural India. Previously, I thought that the condition may be under reported. Now with the availability of HRCT-thorax, we are advising HRCT in some of our COPD patients with rural background. I am now convinced that farmers’ lung is really less common. That made me thinking and the possible explanation may be that farmers are habituated with smoking biries. They told me that their profession demands smoking. The idea was probably generated from practical experience and percolated though generations. The smoking habits may protect them from farmers’ lung.

179

Cases

Management

My Learning

I started prednisolone 40 mg daily after breakfast along with a proton pump inhibitor (pantoprazole 40 mg in the morning in empty stomach). The response to treatment was poor as expected.

In respiratory medicine, it is the history taking that will say the last word. Hypersensitivity pneumonia can be easily missed if specific questions are not specifically asked.

CASE 44: IMPORTANCE OF UNTOLD STORY CASE REPORT Step I A 32-year-old man presented with cough and dyspnea for 3 weeks. He had cough with little expectoration, and that was more prominent at night and during exertion. Dyspnea was insidious in onset and progressive. He had low grade irregular fever without evening rise of temperature and night sweat. He was initially treated with a broad spectrum antibiotic (co-amoxyclav 625 mg 8 hourly). As he was not responding a chest X-ray was done. Anti-tubercular chemotherapy with four drugs was started on the basis of X-ray findings. He did not show any improvement after 2 months of chemotherapy, he was advised to consult a pulmonologist. At that time, he had dyspnea even at rest. He was a non-smoker and did not have any other addiction. He had no significant past, personal or family history. He was married having one 2 years healthy boy. He denied any history of exposure or contact with tuberculosis. As per his statement, he took medicines regularly and that was confirmed by his wife. General survey and systemic examination revealed signs of dyspnea and few crepitations in both infraclavicular areas. His routine blood and blood biochemistry were within normal limits except ESR was 50 mm in 1st hour. His sputum samples were negative for AFB. He was not diabetic and he was HIV negative. His chest X-ray PA view (Fig. 44A) showed bilateral patchy opacities mainly in the mid zones extending to upper and lower zones spearing parts of the lung close to diaphragm.

Discussion Peculiarity of the case was dyspnea that was present from the beginning and that progressed to dyspnea at rest. Dyspnea as a presentation of pulmonary TB is most unlikely and dyspnea can occur with pleural or pericardial involvements. Dyspnea may occur lately in extensive bilateral lungs involvements and in miliary tuberculosis. Progressive dyspnea as an initial presentation in pulmonary TB could not be explained in our case. Secondly, TB was diagnosed on the basis of radiology. There is no radiological finding that confirms TB. Bilateral involvement, involvement of upper lung fields, non-homogenous opacities, presence of cavity, etc. are suggestive of tuberculosis. In our case, patient’s sputum was negative for AFB. In bilateral extensive involvements, sputum smear negativity goes

180

101 Cases in Respiratory Medicine

Fig. 44A: Chest X-ray showing bilateral patchy opacities spearing lung bases

against the diagnosis of TB. Sometimes sputum may be negative in extensive TB when patient is not able to expectorate as a result of weakness, in diabetic patients and in person living with HIV. Thirdly, patient had not responded to ATD. We confirmed drug compliance. Non-compliance is the most important cause of non-response to ATD. It should be remembered that non-compliance is a global and universal phenomenon; it does not depend on age, sex, socioeconomic, educational and marital status or geography. At that point we thought to re-evaluate the diagnosis of TB. In our country, TB is often over diagnosed and sometimes considered as a dustbin diagnosis.

Step II We were planning for fiberoptic bronchoscopy for taking samples by lavage and transbronchial lung biopsy. Before going for invasive procedure, we advised him to have a HRCT-thorax. His HRCT-thorax (Fig. 44B) showed bilateral extensive dense centrilobular nodular opacities with areas of confluent opacities typically spearing subpleural areas and along the fissure lines. There was no lung volume loss. Coronal section of HRCT-thorax (Fig. 44C) clearly showed lung bases were typically speared. On the basis of HRCT findings, we diagnosed the case provisionally as hypersensitivity pneumonia. We asked specific questions and he disclosed the interesting part of his story. At the age of 16 years, he left home, took a ship and went to Andaman. He worked there as laborer with different types of corals; collecting, processing and selling them. As he got rich, he shifted his work from manufacturing to marketing part of business. He also confessed that he had exertional shortness of breath for many years.

Cases

Fig. 44B: HRCT-thorax showing bilateral dense centrilobular opacities with areas of confluent opacities typically spearing subpleural and along the fissures

Fig. 44C: Coronal section HRCT-thorax showing lung bases were typically speared

Discussion Interpretation of HRCT-thorax depends on identification of radiological patterns like nodule, reticular shadow, ground glass opacity, consolidation, low attenuation, etc. Nodular opacities are of three types: centrilobular,

181

182

101 Cases in Respiratory Medicine

perilymphatic and random. Centrilobular nodules classically spear the subpleural areas and areas around fissures. In HP, the HRCT findings include poorly defined centrilobular nodules of low density, ground-glass attenuation and mosaic attenuation due to air tapping. Lung bases are typically speared and there is classically no lung volume loss. Reticular opacities are found in chronic hypersensitivity pneumonia and it may looks like UIP pattern in late stage. HP, previously called extrinsic allergic alveolitis, is caused by repeated inhalation of variety of organic particles in a susceptible host. HP includes farmer’s lung (due to thermophilic bacteria and fungal spores); bird fancier’s lung (due to exposure with excreta and other proteins of birds like pigeon, parrots, chicken), bagassosis (moldy sugar cane), humidifier lung and many other. Interestingly HP is less common in smokers. HP was thought to be an immune complex–mediated disease but now it is increasingly recognized that T cell–mediated hypersensitivity reaction plays a vital role in the pathogenesis of HP. BAL in patients with HP characteristically shows T lymphocytes but polymorphonuclear leukocytes are found in patients with recent or continual exposure to antigen. Clinical presentations of HP may be acute, subacute or chronic. The acute form usually presents with cough, fever, chills, malaise and dyspnea after 6 to 8 hours of exposure. The subacute form is usually insidious in onset with cough and dyspnea, and symptoms may persist if there is continued exposure to antigen. Clinical manifestations of acute or subacute form usually disappear within days, weeks, or months of cessation of exposure. The chronic form is related to continuous low-level antigen exposure or repeated episodes of antigens exposure. Symptoms include cough, weight loss, malaise and progressive dyspnea. Signs include inspiratory crackles and clubbing. Chronic form is sometimes clinically indistinguishable from pulmonary TB. Characteristically, bird fancier’s lung is associated with more fibrosis while farmer’s lung is associated with more emphysema. Diagnosis of HP depends on characteristic clinical manifestation, typical radiological picture, the detection of serum precipitins against suspected antigens and most importantly history of exposure to specific antigen. Sometimes visit to site of exposure, the home or working place is required. HRCT-thorax findings may vary from ground-glass infiltrates, reticulonodular pattern, confluent alveolar opacification, centrilobular nodules, air trapping or mosaic attenuation, patchy emphysema and interstitial fibrosis. Classically opacities spear basal areas of lungs. Lack of lung volume reduction in HP is probably due to air trapping. Other causes of DPLD without lung volume loss are sarcoidosis, lymphangiomyomatosis and most importantly DPLD in smokers. Spirometry usually shows restrictive pattern but may show obstructive pattern. Histopathological findings in lung biopsy include peribronchial noncaseating granulomas and interstitial alveolar infiltrate with plasma cells, lymphocytes, and occasional eosinophils and neutrophils.

Cases

Management We started oral prednisolone 40 mg daily for 6 weeks then tapered it by 5 mg weekly along with other symptomatic treatment. Patient responded clinically but there were little change in follow-up chest X-rays. There was slight but non-significant improvement in lung function in subsequent spirometries.

Discussion Treatment of HP includes withdrawal of exposure to antigen and glucocorticoids. Prednisone is given at the dose of 1 mg/kg per day for 7 to 14 days for acute/subacute form and a longer course (2 to 6 weeks) is required for chronic HP. We diagnosed the case as HP depending upon typical clinical manifestation, HRCT findings and history of exposure to organic agent (choral). HRCT findings those were present in our patient included centrilobular nodule, spearing lung bases, and without lung volume loss. Classically the centrilobular nodules of HP are ill-defined and of low density. Peculiarity My Learning of our case was that the centrilobular nodules Histor y taking is the most were well-defined and dense. This type of important step in diagnosis of nodules is more common in exposure to hypersensitivity pneumonia. inorganic dust exposure (pneumoconiosis). Occupational history should include not only the present job Our probable explanation was the biology of but also the past job. choral as chorals represent organic as well as inorganic agent.

CASE 45: DPLD PRESENTED LIKE TUBERCULOSIS CASE REPORT Step I A 37-year-old man presented with cough, dyspnea and fever for 1 month. Cough was his main symptoms and was insidious in onset. There was no diurnal variation of coughing and that was associated with little mucoid expectoration. Patient observed shortness of breath during exertional activity like climbing stairs, carrying weight, etc. Dyspnea was not progressive and was not associated with wheeze. Patient had low grade irregular fever without evening rise of temperature. Patient felt weakness and malaise. Patient also noted 2 kg weight loss during last 1 month. There was one episode of small blood streak expectoration. There was no chest pain. His appetite, sleep, bowel and bladder functions were good. He was working in private financial company. He had no addiction. He was married with two healthy daughters. There was no other significant history. Hemoptysis made him anxious and he consulted a local doctor. He was treated with oral antibiotic (clarithromycin 500 mg twice daily for 5 days), tranexamic

183

184

101 Cases in Respiratory Medicine

Fig. 45A: Chest X-ray showing bilateral mainly reticular opacities affecting all zones with some areas of conglomerated opacities

acid 500 mg thrice daily for 3 days, salbutamol inhaler and a cough suppressant syrup. Patient’s symptoms remained unchanged but hemoptysis stopped. As patient was not responding the physician asked for routine blood tests, sputum examination and chest X-ray. His peripheral blood showed Hb—14.2 g/dL; WBC—7800/mm3, N-72%, L—24%, E—2%, M—2%; and ESR—66 mm in 1st hour. His blood biochemistry showed fasting glucose—94 mg/dL; urea—32 mg/dL; creatinine—0.8 mg/dL and normal liver function test. His sputum samples (mainly serous) were negative for AFB. Chest X-ray (Fig. 45A) revealed bilateral reticular opacities along with patchy and non-homogeneous opacities. The physician started anti-tubercular chemotherapy with a widely available kit preparation containing INH—300 mg, RIF—450 mg, PZN—1500 mg and ETB—800 mg. As patient was not responding with antitubercular chemotherapy, he attended our outpatient department.

Discussion The gentleman presented with cough along with exertional dyspnea, fever, weakness, weight loss and negligible hemoptysis. All manifestations pointed towards respiratory tract infection, particularly tuberculosis in our country. Exertional dyspnea could not be explained by tuberculosis unless it was extensive or complicated. The differential diagnosis at that state should include infective pneumonia, lung abscess, bronchiectasis, asthma and heart disease like mitral stenosis. Delayed resolution of pneumonia and necrotizing pneumonia may persist for longer time but there should be high rise of temperature at least initially. Bronchiectasis and chronic lung abscess should have copious expectoration. The shorter duration of illness was against the diagnosis of bronchiectasis. Asthma should have wheeze and variability. Heart disease (mitral stenosis) may have hemoptysis, cough

Cases

and dyspnea. Weight loss was a remarkable feature in our case. Weight loss could occur in tuberculosis or any chronic suppurative lung pathology. Another feature of the case was non-response to treatment with antibiotics and antitubercular drugs. In this gentleman, the clinical manifestations favored tuberculosis and his symptoms could easily explained by the disease. The first line tests for the diagnosis of TB, as per guideline, are sputum examination and chest X-ray. His sputum samples were negative for AFB could be explained by nonrepresentative sample (serous expectoration). Chest X-ray was confusing and may be suggestive of TB but predominantly reticular opacities did not match with the diagnosis of TB. Molecular methods should be the next step but the problem was the nonavailability of biological samples (there was little expectoration). Proper biological samples can be collected by inducing expectoration or by bronchoscopic lavage. HRCT-thorax can detect the more detailed view of lung opacities and can identify the typical pattern suggestive of tuberculosis. As patient was not responding to antitubercular chemotherapy, our job was to confirm the diagnosis as well as to explore other possibilities.

Step II We admitted the patient for investigation. We advised HRCT-lung scan and his HRCT (Fig. 45B) showed bilateral reticular opacities and multiple patchy areas of consolidations. Consolidations are mainly peribronchovascular in distribution and there were dilated bronchus within consolidations. There were few areas of ground-glass opacities. We performed fiberoptic bronchoscopy and no intraluminal pathology was seen up to subsegmental bronchi. We took bronchoalveolar lavage (BAL) and transbronchial lung biopsy (TBLB). BAL fluid was examined by Gram stain and ZN stain, and no organism was detected. BAL fluid cartridge based nucleic acid amplification test (CBNAAT) was negative. Bacteriological and fungal culture showed no growth. Histopathology of TBLB was reported as chronic inflammation with fibrotic granuloma within air spaces. We finally diagnosed the case as organizing pneumonia.

Discussion Organizing pneumonia is also named as bronchiolitis obliterans-organizing pneumonia (BOOP). The term BOOP is popularized as there is a tendency to plugs and occludes smaller bronchioles. But bronchiolitis is not an essential component of the syndrome and the term organizing pneumonia is now preferred. Organizing pneumonia is a diagnosis of exclusion and the diagnosis depends on exclusion of other causes of consolidation mainly infections. Organizing pneumonia is a nonspecific reaction to lung injury. It may be secondary to collagen vascular disease (polymyositis, dermatomyositis and rheumatoid arthritis), drugs, lung or bone marrow transplantation,

185

186

101 Cases in Respiratory Medicine

Fig. 45B: HRCT-thorax showing bilateral reticular opacities and areas of consolidations with dilated bronchus within consolidation (arrow head)

cryptococcosis, granulomatosis with polyangiitis, lymphoma, hypersensitivity pneumonitis and eosinophilic pneumonia. Cryptogenic organizing pneumonia (COP) is a clinic-pathological syndrome of unknown etiology. The mean age of presentation is 58 years (20–80) and there is no gender predilection. Influenza-like prodrome is found in about 25–50% cases. Clinical manifestations include nonproductive cough and exertional dyspnea occurring for weeks to months. Other symptoms are fever, malaise, fatigue and weight loss. Pleuritic chest pain and hemoptysis are rarely found. Physical examination can detect occasional inspiratory crackles. Lung function tests show restrictive lung defect and hypoxia. More than half of the cases are initially diagnosed as infective pneumonia. Common radiological presentation is multifocal consolidation, and that is found in about 80% cases. Atypical findings include multiple large nodules, 1–2 cm ill-defined nodules, smaller or irregular nodules of 1–10 mm diameter, basal reticulonodular or irregular reticular pattern and rarely unilateral or bilateral pleural effusion. HRCT-thorax shows bilateral, patchy, airspace opacity, consolidation or ground-glass opacity. Consolidations are nonsegmental of about 2–6 cm in diameter; with no craniocaudal predilection or lower zone involvement. Lesions are usually peribronchovascular in 2/3rd cases or subpleural in distribution. Presence of air bronchograms with dilated airways within consolidations is characteristic.

Cases

In 10% cases, opacity may be unilateral or focal. Focal lesions are usually detected accidentally, are non-progressive and having good prognosis. Consolidation may disappear even without treatment and may relapse. Migrating consolidation is another important feature. Causes of flitting pneumonia includes organizing pneumonia, eosinophilic lung disease, Loffler syndrome, drug and parasite hypersensitivity, ABPA, diffuse interstitial hemorrhage, recurrent pulmonary infarction, recurrent aspiration, vasculitis, Churg-Strauss syndrome, etc. Lung biopsy shows granulation tissue with intraluminal organizing fibrosis within small airways, alveolar ducts, and airspaces and chronic inflammation in the surrounding alveoli and interstitium.

Management We started oral prednisolone 40 mg daily for 4 weeks and then tapper the dose gradually over weeks. Patient responded symptomatically and radiologically. HRCT-thorax after 1 month of starting treatment showed disappearance of consolidations and ground-glass opacities but My Learning reticular opacities persisted. Glucocorticoid therapy induces clinical Organizing pneumonia usually recovery in two-thirds of patients and total presents with multifocal peri­ radiological clearance occurs in 30–60% cases. bronchovascular or peripheral consolidations. The diagnosis Relapse occurs in about 15% cases. A small d e p e n d s o n e xc l u s i o n o f fraction of cases have a rapidly progressive other causes of consolidation, fulminant course with fatal outcomes despite particularly infective pneumonia. glucocorticoid therapy.

CASE 46: BOY WITH CERVICAL LYMPHADENOPATHY AND PNEUMOTHORAX CASE REPORT Step I A 15-year-old boy from lower socioeconomic condition was referred to us with left-sided chest pain, shortness of breath and fever for 2 weeks. History was taken from both patient and his mother. Fever was high-grade, continuous, without chill and rigor. Fever was associated with cough with scanty mucoid expectoration. Later patient developed left-sided chest pain. Chest pain was sharp, pinpricking in character occurred in the lateral part of left side of chest that increased with respiration and coughing. Patient had dyspnea that was increased for last 2 weeks and at the time of presentation, he was dyspneic even at rest. The dyspnea was not associated with wheeze. On enquiry patient’s mother gave two important informations. The boy was not well for last 1 years and he could not compete with his friends in physical activities. Patient developed multiple swellings over neck areas for which he had ATDs for more than 1 year. Despite treatment swellings ruptured producing sinuses and scars.

187

188

101 Cases in Respiratory Medicine

On examination, we found that the boy was toxic and prostrated. He was alert, conscious and cooperative. General survey revealed pallor; BMI was 15.4; BP was 106/72 mm of Hg; pulse was 104/min; RR was 30/min, temperature was 100.8°F and SpO2 was 92% with room air. The boy had bilateral cervical lymphadenopathy, and lymph nodes were multiple, firm and size varied from 1 to 5 cm in diameter. There were scars in the cervical areas extending towards upper chest wall with multiple discharging sinuses (Fig. 46A). Examination of respiratory system revealed diminished movement on left side of chest; central trachea; hyperresonant notes on left hemithorax; and diminished vesicular breath sounds in the left hemithorax with decreased vocal resonance. There were bilateral, scattered, fine crepitations, and those were more in the lower part of chest. Breath sounds on right side were normal vesicular and crepitations were more prominent in right side. His liver and spleen were not palpable, and there was no evidence of bleeding anywhere. Examination of other systems was normal. His peripheral blood showed Hb%—9.9 g/dL; total WBC count—9200/ mm3 with N—82%, L—15%, E—2%, M—1%; platelet count—3 lac/mm3; and ESR—90 mm in 1st minute. Blood biochemistry showed fasting sugar—86 mg/ dL; urea—27 mg/dL; creatinine—1.0 mg/dL; total bilirubin—0.4 mg/dL; total protein—5.5 g/dL; albumin—3.5 g/dL and liver enzymes were within normal levels. HIV test was non-reactive. Sputum for AFB was negative. His chest X-ray PA view (Fig. 46B) showed bilateral reticulonodular opacities and opacities were more prominent in both lower zones. There was left-sided loculated hydropneumothorax. Pleural aspirate was opaque. We introduced intercostal tube in the left side. Pleural fluid culture showed growth of Staphylococcus aureus and ZN stain was negative. We started intravenous vancomycin 1 g twice daily for 10 days along with symptomatic treatment.

Fig. 46A: Boy showing cervical lymphadenitis with extensive scarring and sinus formations

Cases

Fig. 46B: Chest X-ray showing bilateral reticulonodular opacities with left-sided hydropneumothorax

Discussion This 15 years boy had bilateral cervical lymphadenopathy with sinus formation and local scarring. It looked like scrofuloderma. Patient had ATDs for about 1 year, but the lesions did not regress. The possibilities considered were atypical mycobacterial infection and drug resistant TB. The patient came to us with left-sided pyopneumothorax, and that could be explained by TB, and that was secondarily infected with Staphylococcus. But the chest radiology, the finding of bilateral reticulonodular opacities, was inconsistent with TB. Differential diagnosis at that stage included TB, atypical mycobacterial infection, drug resistant TB, lymphoma, leukemia, pulmonary complications in persons living with HIV and DPLD. Sinus formations and scarring were against the diagnosis of lymphoma, leukemia and DPLD. We extended the differential diagnosis and brought in chronic infections like actinomycosis, nocardiosis, etc.

Step II To investigate the case, we choose one non-invasive method, HRCT-thorax for anatomical classification of lung opacities; and one invasive test, lymph node FNAC for pathological diagnosis. HRCT-thorax (Fig. 46C) showed (i) hydropneumothorax of left side; (ii) well-defined but irregular-shaped cystic lesions of different sizes with linear opacities mainly in the mid and lower zones; (iii) ill-defined nodular opacities mainly in the upper and mid zones; and (iv) no sign of lung volume loss. FNAC of lymph nodes (Fig. 46D) showed atypical histiocytic cells with convoluted nuclei and with few longitudinal grooves, some binucleated

189

190

101 Cases in Respiratory Medicine

Fig. 46C: HRCT-thorax showing cystic spaces of bizarre shaped and different sizes with left sided hydropneumothorax

Fig. 46D: Lymph node FNAC showing atypical histiocytes

forms were also seen along with it, there was admixture of small lymphocytes, neutrophils and eosinophils. Imprint smear showed mostly matured squamous cells along with few histiocytic cells with grooves. The features were suggestive of Langerhans cell histiocytosis (LCH).

Cases

Discussion Langerhans cell histiocytosis, previously called histiocytosis X, is an idiopathic disorder characterized by abnormal proliferation of histiocytes and mononuclear cells centers around bronchioles with associate involvement of interstitium and vasculatures. Langerhans cells (LCs) and bone marrowderived dendritic cells infiltrate tissues like bone, lymph nodes, liver, spleen, CNS, etc. Pulmonary Langerhans cell histiocytosis (PLCH) is associated with smoking in 90–100% cases, and most patient are under the age of 40 years (usually 20–40 years of age. The common presenting symptoms are cough, dyspnea, chest pain, weight loss, and fever. The disease may be asymptomatic at presentation and accidentally deteced by chest radiology done for other purpose. The disease may presents with pneumothorax in about 25% cases. Other manifestations include skeletal defect, diabetes insipidus, etc. In children, the disease may be multisystemic involving lungs, bones, pituitary and skin. Radiologically, the disease is characterized by a combination of nodules and cysts. They usually involved upper and mid zones spearing costophrenic sulci, and there is usually no lung volume loss. HRCT-thorax usually shows a combination of ill-defined nodules that may cavitate and thin/thick-walled bizarre shaped, unequal sized and diffusely distributed cysts. As disease progresses nodules tend to regress while cysts and linear opacities tend to remain or increases. Pulmonary artery enlarges and pulmonary hypertension occurs in advanced stages. Histologically, PLCH is characterized by the presence of nodular fibrotic lesions with mixed cellular infiltrates containing Langerhans cells. Nodules are poorly defined and bronchiolocentric in distribution. As diseases progresses, there are development of pericicatricial air space enlargements forming cystic spaces. LCH is classified into three distint types: 1. Acute disseminated LCH (Letterer-Siwe disease) present with fever, anemia, thrombocytopenia; pulmonary infiltrate; skin lesions; enlargement of lymph nodes, spleen and liver; and involvement of nervous system. This type usually has cystic changes in lung and pneumothorax. 2. Classic multifocal LCH (Hand-Schüller-Christian disease) includes diabetes insipidus, exophthalmos and bony defects particularly of cranium. It may affect a variety of systems like liver, spleen, lymph nodes and rarely lung (in one-third patients). Mucocutaneous lesions are mostly infiltrated nodules and ulcerated plaques. 3. Chronic unifocal LCH (eosinophilic granuloma of bone) presents as a solitary calvarial lesion involving vertebra, rib, mandible, femur, ilium and scapula in young adults.

191

192

101 Cases in Respiratory Medicine

Management Treatment of PLCH includes smoking cessation, corticosteroids, cytotoxic drugs (vinblastin, cyclophosphamide, methotrexate, etoposide) and lung transplantation. We diagnosed the case as acute disseminated form of LCH. We referred the case to hematology and oncology department. After a long discussion, we decided to start chemotherapy with vinblastine 6 mg on day 1 and etoposide 80 mg on day 1 to day 3. After receiving 2 cycles of chemotherapy patient improved, all the neck glands disappeared but My Learning scar persisted. Radiological lesions remained as before. We discharged him in stable condition. Pneumothorax, pulmonar y Patient was kept under surveillance and was infiltrates and scrofuloderma like skin lesions usually suggest given total six cycles of chemotherapy. Patient TB but do not be biased. Some­ was responding well, patient did not had any times, a rare disease may mimic further attacks of pneumothorax but had a common condition. recurrent attacks of respiratory tract infections The combination of nodules and those were treated with broad spectrum and cysts in HRCT is suggestive of PLCH. antibiotics.

CASE 47: LADY WITH RECURRENT PNEUMOTHORAX CASE REPORT Step I A 45-year-old lady came to emergency with sudden onset chest pain and dyspnea. She told the attending doctor that while cooking in kitchen she developed left sided, sharp, pinpicking chest pain that prevented her from taking deep breath (a catch of breath). She then developed shortness of breath. At emergency, she had a chest X-ray PA view that showed left-sided pneumothorax. She was treated with intercostal tube drainage, O2 and antibiotics. She improved and she was referred to me. I transferred her to chest inpatient department. On enquiry, she confessed that she had two similar attacks. First one was 2 years back when she had sudden onset of chest pain (site she could not remember) and dyspnea. She could not stand and took breath. She was compeled to go to bed and took rest. By two hours her symptoms improved and she did not consult any doctor or told that event to her family members. Second attacks occurred four months back and she had right-sided chest pain and dyspnea. That time she went to a local hospital, she was admitted and treated conservatively with O2, and medicines. She was discharged after 7 days and she was advised to take rest. Old records and X-rays were not available. She was a housewife having two children. She was non-smoker or having no other addiction. She had no other significant personal and family history. On examination, general survey and other system examination were within normal limits. Respiratory system examination revealed intercostal tube in

Cases

situ, and breath sounds almost equal on both sides. Chest X-ray showed expansion of lung. We did pleurodesis with betadine solution and removed intercostal tube.

Discussion Spontaneous pneumothoraxes are classified into primary and secondary. Primary spontaneous pneumothoraxes are caused by rupture of apical pleural blebs or small subpleural cystic spaces. Chance of recurrence in pneumothorax is high and the chances increased rapidly with subsequent pneumothoraxes. The chance of first relapse is about 16% whereas it increases to 64% for second relapse and 80% for third time relapse. Secondary pneumothoraxes are caused by underlying lung diseases most commonly obstructive lung diseases. Secondary pneumothoraxes are more lifethreatening, and need treatment with tube thoracostomy. Pleurodesis is recommended for any pneumothorax treated with tube thoracostomy. Pleurodesis is usually done by talc powder insufflations, but can be done with chemicals like tetracycline. Recently betadine solution, a readily available solution, is used for pleurodesis. It is very important to differentiate between primary and secondary spontaneous pneumothorax. It should be noted here that pneumothorax is a manifestation, not a disease. All cases of pneumothorax should be investigated properly to find out the underlying mechanism or lung disease.

Step II After withdrawal of intercostal tube, we advised a digital chest X-ray PA view as previous X-rays done at emergency was not of satisfactory quality. Her chest X-ray (Fig. 47A), showed opacities and some cystic lesions in both lower zones along with left-sided volume loss. We diagnosed the case as recurrent secondary pneumothorax. We advised HRCT-thorax to delineate the underlying lung disease. Her HRCT thorax (Fig. 47B) showed bilateral multiple thin-walled round cysts of different sizes throughout both lung fields; normal lung parenchyma in between cysts and left-sided pleural fibrosis (as a result of pleurodesis). The HRCT findings were consistent with the diagnosis of pulmonary lymphangioleiomyomatosis (LAM).

Discussion Pulmonary LAM is a rare condition and founds in premenopausal women. It is more common in white race. LAM usually presents as emphysema, recurrent pneumothorax and chylous pleural effusion in young women. It is often misdiagnosed as asthma or chronic obstructive pulmonary disease. LAM is pathologically characterized by the proliferation of atypical pulmonary interstitial smooth muscle cells. Smooth muscle proliferations occur in lymphatics, airway vessels, alveolar septa, mediastinum and retroperitoneum. LAM is also characterized by cyst formation. It causes recurrent pneumothorax,

193

194

101 Cases in Respiratory Medicine

Fig. 47A: Chest X-ray showing bilateral lower zone opacities and heart shifted towards left

Fig. 47B: HRCT-thorax showing bilateral thin-walled round cysts with left-sided pleural thickening

Cases

chylous effusions, hemoptysis and eventually respiratory failure. Air-filled cysts are lined by fattened epithelial and ciliary bronchial elements. Cysts are scattered over both lungs with no zonal predilection. Cysts are thought to be due to air trapping produced by proliferation of smooth muscles in small airways. The etiology of LAM is not clear. LAM typically occurs in female in child bearing age, it exacerbates during pregnancy, parturition and exogenous estrogen administration. LAM is characterized by the presence of estrogen receptor in abnormal smooth muscle. All those factors indicate towards female sex hormone as etiological mechanism. Radiologically, the disease is characterized by thin-walled cysts and normal lung parenchyma in between with no zonal predilection. At the initial stage, radiological findings include fine nodular, reticular or reticulonodular opacities distributed bilaterally and symmetrically or with lower zones predilection. Cysts are the pathognomic feature. Cysts are multiple, thin-walled (1–2 mm in thickness), usually round but sometimes polygonal or bizarrely, and distributed uniformly throughout lungs. Blood vessels are characteristically seen at the margin of the cysts, and that point is very important in differentiating LAM from centriacinar emphysema. In centriacinar emphysema blood vessels are found at the center of cysts. Cysts are found in juxtaphrenic lung and costophrenic sulci and that will differentiate LAM from PLCH, where sulci are typically speared. Ground glass opacity and consolidation are rarely seen due to alveolar hemorrhage.

Management Pregnancy increases and oophorectomy decreases symptoms in LAM. Oophorectomy is not recommended and the disease is treated with progesterone (10 mg/day) or luteinizing hormone–releasing hormone analogues. The disease is progressive and mean time of survival is 8–10 years from the time of diagnosis. We treat her with progesteron 10 mg orally My Learning per day and advised her to avoid strenuous Pneumothorax is notorious exercise like weight lifting, weight carrying, for recurrence, but recurrent climbing stairs, etc. I counseled her and pneumothorax in ladies at her husband about the disease, chance of child bearing age, should be investigated for LAM. recurrence and prognosis before discharge.

CASE 48: LADY WITH PERIPHERAL LUNG OPACITIES CASE REPORT Step I A 50-year-old lady presented with progressive dyspnea for last 6 weeks. As per patient she developed insidious onset of dyspnea that was progressive and at the time of presentation she could perform her day-to-day activities with distress. She had cough with little expectoration, and sometimes she noticed

195

196

101 Cases in Respiratory Medicine

wheeze while coughing. She had occasional low-grade fever. She also felt intense weakness, malaise, myalgia and nonspecific joint pain. She went to a local doctor, she was diagnosed as tuberculosis on the basis of chest X-ray, and ATD with INH, RIF, PZN and ETB was started. After a month of treatment there was no improvement, and actually patient’s symptoms deteriorated. She came to us and we admitted her in our in-patient department.

Discussion Some of the clinical manifestations like low-grade fever, cough, weakness might simulate TB. But dyspnea from the beginning and that was progressive was unexplainable in pulmonary TB without complication. On the other hand, some symptoms favored systemic nature of illness. Moreover, no radiological opacity is pathognomonic of TB. Sputum examination should be done in all suspect of TB. Sputum examination was not done probably because of lack of expectoration. But surprisingly routine peripheral blood examination and blood biochemistry were not done before starting ATD.

Step II We took detail history but that did not give any clue. She was a house-wife having three children and all of them were healthy. She was non-hypertensive, non-diabetic, and she was not on long-term medications. She had joint pain but did not have joint swelling. She had no significant past history, personal history or family history. General survey and systemic examination revealed nothing except few rhonchi scattered over both lung fields. A close look at her previous chest X-ray PA view (Fig. 48A) revealed there were bilateral nodular and patchy opacities distributed over the periphery of lung fields. There was hump (partial eventration) in the right dome of the diaphragm. Right hilum was appeared to be enlarged.

Fig. 48A: Chest X-ray showing bilateral peripheral opacities

Cases

Fig. 48B: HRCT-thorax showing bilateral peripheral patchy consolidations

Her peripheral blood showed Hb%—9 g/dL, WBC count—18000/mm3 with N—46%, L—14%, E—37%, M—2%, B—1%; and ESR—86 mm in 1st hour. Absolute eosinophil count was 6660/mm3. Routine blood biochemistry showed fasting sugar—90 mg/dL; creatinine—0.7 mg/dL; urea—26 mg/dL; bilirubin—1.2 mg/dL; SGPT—50 U/L; SGOT—52 U/L; alkaline phosphatase—310 U/L. Her spirometry was normal with FEV1/FVC ratio was 78%; FEV1, FVC and PEFR were 84%, 90% and 74% of predicted values, respectively; but there was indications of early airway obstruction, FEF 25–75% was 54% of predicted value. We advised HRCT-thorax to delineate lung opacities and to make a possible diagnosis. HRCT-thorax (Fig. 48B) showed centrilobular nodules, ground-glass opacities and patchy consolidations bilaterally distributed in the peripheral areas of lung fields. We diagnosed the case as chronic eosinophilic pneumonia on the basis of peripheral opacities and peripheral blood eosinophilia. Patchy consolidation/ground-glass opacities distributed bilaterally over peripheral lung fields with normal clear lung in the central (perihilar) areas is characteristically found in eosinophilic pneumonia. Distribution of alveolar opacities occurs in distribution opposite to bats’ wing distribution of pulmonary edema. The distribution is called negative bat’s wing pattern.

Discussion Eosinophilic pneumonia or pulmonary infiltrates with eosinophilia (PIE) is a syndrome characterized by eosinophilic pulmonary infiltrates that is usually associated with peripheral blood eosinophilia. PIE may be idiopathic or may be caused by drugs, fungal infection or parasitic infestation. Clinically, it may present as Loeffler’s syndrome; acute eosinophilic pneumonia; chronic eosinophilic pneumonia; tropical pulmonary eosinophilia; Allergic bronchopulmonary aspergillosis; allergic granulomatosis of Churg-Strauss; and hypereosinophilic syndrome.

197

198

101 Cases in Respiratory Medicine

Loeffler’s syndrome is characterized by transient/migratory pulmonary infiltrations usually associates with drugs or parasitic infestation. Drugs causing PIE include nitrofurantoin, sulfonamides, penicillin, chlorpropamide, thiazides, tricyclic antidepressants, hydralazine, gold salts, isoniazid, indomethacin, etc. Allergic bronchopulmonary aspergillosis/mycosis (ABPA) is an IgE-mediated hypersensitivity to Aspergillus/other fungal antigens. ABPA is clinically manifested with atopic asthma; peripheral eosinophilia more than 1000/mm3; elevated IgE; central bronchiectasis and wheal-andflare skin reactivity to aspergillus antigens. Tropical eosinophilia occurs in tropical countries caused by microfilaria, and the disease presents like asthma with nocturnal exacerbations and peripheral eosinophilia (>3000/ mm3). Churg-Strauss syndrome, or allergic angiitis granulomatosis, presents with asthma like symptoms and eosinophilia, and it is pathologically characterized by necrotizing eosinophilic vasculitis and eosinophilic infiltration of multiple organs, including the lung. The hypereosinophilic syndrome is characterized by idiopathic eosinophilia >1500/mm3 for 6 months or longer; manifestations of multisystem organ dysfunction due to tissue infiltration by relatively mature eosinophils. Organs involved include heart, lungs, liver, spleen, skin and nervous system. It is a fatal condition, and treatment includes glucocorticoids and/or hydroxyurea. Acute eosinophilic pneumonia is an idiopathic eosinophilia presents with fever for 0.5 g/d or cellular casts), viii) neurologic disorder (like seizures or psychosis), (ix) hematologic disorder (hemolytic anemia or leukopenia (250 U/L + symptoms. It is also important to rule out other possibilities like viral hepatitis or non-ATD (azathioprine) induced hepatitis. Hepatitis occurs in about 0.6%, 1.6%, 2.7% and 2% with INH alone, INH plus other drugs (except RIF), INH plus RIF and in persons aged >50 years respectively. INH, RIF, and PZN have hepatotoxic side effects. INH

Cases

hepatotoxicity is due to its metabolites acetyl hydrazines. Population can be divided into rapid and slow acetylators group. It was hypothesized that hepatotoxicity is more common in rapid acetylators and neuropathy is more common in slow acetylators. But there was no clinical data to prove that theory. RIF may rarely cause hepatotoxicity by idiosyncratic reaction but the main effect of RIF is on INH metabolism. RIF induces cytochromal enzyme P-450 and there by increases the metabolism of INH to acetyl hydrazine and precipitate hepatotoxicity. The question may come, why there is no increased hepatotoxicity in rapid acetylators. It was postulated that in rapid acetylators acetyl hydrazine is rapidly converted into bi-acetyl hydrazine and thereby hepatotoxic effect was neutralized. Management of drug-induced hepatitis depends on severity of liver disease, severity of TB and phase of treatment. In continuation phase we can continue with nonhepatoxic drugs or with single hepatotoxic drugs. In less severe disease we can discontinue ATD and restart treatment after normalization of liver function. Rifampicin is the preferred first drug to reintroduce followed by INH and PZN. In patients who have experienced jaundice PZN is better to be avoided. Fortunately and surprisingly reintroduced ATDs are well-tolerated. In severe disease, a non-hepatotoxic regimen, consisting of streptomycin, ethambutol and fluoroquinolone may be started. ATD regimens advocated for patient with impaired liver function are (i) regime containing two hepatotoxic drugs, 9HRE; 2HRE/6HR and 6-9RZE; (ii) regime containing one hepatotoxic drug, 2SHE/10HE and (iii) regime containing no hepatotoxic drug, 18-24SE + fluoroquinolones. Liver function tests should be monitored weekly for 1–2 weeks and every 2 weeks in those pre-existing liver diseases till LFT reversed back to normal and clinical symptoms (nausea, vomiting, appetite, abdominal pain) resolved.

Management In that gentleman, as there were clinical and radiological evidences of reduction of pleural fluid, I thought that chest pain might be due to pleural friction as pleural layers came closer with reduction of fluid. Pleuritic chest pain sometimes persists for longer time even after remission of pleural effusion. His chest pain was increasing and that did not response to analgesics. I advised a CECT thorax and that showed a lucent area in the upper part of liver (Fig. 52C). His USG upper abdomen showed an abscess in the right upper quadrant of liver. USG-guided aspiration was done and the report came as caseating granuloma with presence of few acid fast bacilli. I continued treatment. I followed him with liver function tests every fort-night for first two months and then every month, and USG upper abdomen monthly. I observed clinical response after 1 month of chemotherapy. The abscess disappeared completely by the end of 8 months treatment. I continued medicines for another 1 month (total 9 months with 4 drugs in first 3 months and then stopped PZN). My final diagnosis was right-sided pleural effusion and tubercular liver abscess in a case of sarcoidosis.

213

214

101 Cases in Respiratory Medicine

Fig. 52C: CT-thorax at lower cut showing a large liver abscess

Discussion Tubercular liver abscess is a rare form of extrapulmonary tuberculosis. In our case the isolation of TB bacilli from liver abscess ultimately confirmed and justified antitubercular chemotherapy. Our patient had both tuberculosis and sarcoidosis. The diagnosis of sarcoidosis had been established previously and the diagnosis of tuberculosis was confirmed later on. The disease remained asymptomatic for more than 3 years confirmed sarcoidosis as a background disease. Liver involvement in TB may be in focal or nodular and those may present as tuberculoma or liver abscess. Liver is usually involved as a part of disseminated TB. Isolated hepatic tuberculosis is even rarer. Liver abscess is quite common in developing countries. The usual causes are amebic and pyogenic liver abscesses. Tubercular liver abscess though rare should be considered in cases of unexplained liver abscess. Pus collected under USGguidance should be examined for AFB and BACTEC culture. Tubercular liver abscess may be single or multiple. The presentation is usually subacute with pain upper abdomen and raised serum alkaline phosphatase level, raised ESR and low hemoglobin. Three forms of hepatic tuberculosis have been described: (i) diffuse liver involvement as a part of disseminated/miliary TB, (ii) diffuse involvement without lung disease, and (iii) focal liver involvement like tuberculoma and liver abscess. TB lesions are usually more than 2 mm in diameter and they presents as macronodule or pseudotumor. Radiological imaging cannot differentiate them from hepatocellular carcinoma, lymphoma or metastasis. The association between sarcoidosis and TB is a complex and undecided subject. TB has been described as preceding or coexisting condition with

Cases

sarcoidosis. On the other hand, TB may occur as a result of secondary infection or steroid therapy. In sarcoidosis due to depression of cellular immunity there is risk of increased infection due to fungi, Mycobacterium and nocardia. Due to similar pathogenesis and histology, there was an old school of thought that both conditions represent different spectrum of same disease. Later on, the hypothesis was challenged by the fact that sarcoidosis is rare in endemic areas of TB. Sarcoidosis was thought to be rare in India but increasing number of sarcoidosis cases has been reported recently. There is a hypothesis that sarcoidosis is due to a hypersensitivity reaction to some tubercular proteins. Some scientists have demonstrated the presence of a mycobacterial protein, catalase-peroxidase, in sarcoid granulomas. This protein may act My Learning as a persisting antigen in sarcoidosis. An Exudative pleural effusion in immune response to mycobacterial protein our country is commonly due to has been documented in sarcoidosis. There tuberculosis and that possibility should be kept in mind even is another hypothesis that Mycobacterium when another possibility is similar to M. tuberculosis is responsible for obvious. sarcoidosis.

215

216

101 Cases in Respiratory Medicine

TUBERCULOSIS CASE 53: A BOY WITH PROGRESSING PNEUMONIA CASE REPORT Step I An adolescent boy presented with cough and fever for 2 weeks, and chest pain for 5 days. Patient stated that he was quite well before 2 weeks. He developed cough which was initially dry. Later on cough was associated with scanty mucoid expectoration. Fever was high grade (around 102°F) with feeling of chill but without rigor. He had chest pain in the lateral aspect of right side of chest. Chest pain was sharp, pin pricking and increased with respiration and coughing. There was no significant past history, personal history and family history. General survey showed BP—100/60 mm of Hg, pulse—116/min, RR—32/ min, temperature—102.4°F and SpO2—98% at room air. Positive findings in examination of respiratory system were diminished movement of right hemithorax and pleural rub in right infraaxillary area. Examination of other systems was essentially normal. Peripheral blood showed WBC—14500/mm 3 with N—78%, L—16%, E—4% and M—2%; Hb%—12.8 g/dL and ESR—70 mm in 1st hour. Routine blood biochemistry was normal. His chest X-ray PA view (Fig. 53A) showed consolidation in the right midzone with surrounded areas of patchy opacities. Sputum, induced by nebulization with hypertonic saline, was negative for gram stain and ZN stain. Sputum was sent to the laboratory for culture. We diagnosed the case as community acquired pneumonia and started oral antibiotic (amoxicillin 500 mg 8 hourly for 7 days with clarithromycin 500 mg

Fig. 53A: Chest X-ray showing consolidation right midzone with ill-defined medial margin

Cases

Fig. 53B: Follow-up chest X-ray showing deteriorating radiological opacities

twice daily for 5 days), paracetamol 500 mg thrice daily and other symptomatic medicines. Chest pain decreased, fever responded partly and pleural rub disappeared but cough persisted. Chest X-ray after 7 days (Fig. 53B) showed deterioration of lung opacities.

Discussion The clinical presentations, acute onset, high fever, pleuritic chest pain, neutrophilic leukocytosis and radiological findings were consistent with community acquired pneumonia. Sputum examination is an essential part of diagnosis and sputum induction should be attempted in all case of no/scanty expectoration. We started treatment of community acquired pneumonia with both pneumococcal and atypical bacterial coverage. Mycoplasma is a common organism causing community acquired pneumonia in college growing age group. Pneumonia patient should demonstrate some clinical response by third day. As there was little clinical response, we considered the case as non-responding pneumonia. We decided to investigate the case and we repeated chest X-ray. Normally pneumonia requires more time to show radiological improvement than clinical improvement. The second chest X-ray showed radiological deterioration. We considered the case as progressing pneumonia and searched for the causes of failure to treatment. We considered common causes and excluded noncompliance, inadequate antibiotics and inadequate doses. At that stage, we considered wrong diagnosis, complicated pneumonia like parapneumonic effusion, empyema, necrotizing pneumonia, etc. and associated factor like diabetes, immunosuppression, etc. as possibilities.

217

218

101 Cases in Respiratory Medicine

Step II The boy was not diabetic and his HIV serology was negative. His chest X-ray did not suggest pleural effusion, empyema or necrotizing pneumonia. We decided to review our diagnosis. We advised HRCT-thorax. His HRCTthorax (Fig. 53C) showed area of consolidation in the posterior segment of right upper lobe with areas of low attenuation within consolidation. Surrounding the area of consolidation, there were satellite lesions. The nodules surrounding consolidation were centrilobular nodules with branching pattern (tree in bud appearance). There was radiological sign of lung volume loss evident by shifting of mediastinum towards right. We advised CT-guided FNAC from the area of consolidation, and the report came as epithelioid granuloma with caseating necrosis and there was presence of few acid fast bacilli.

Discussion CT-thorax is not a routine investigation in community acquired pneumonia. All cases of non-responding, progressive or unresolved pneumonia should have CT-thorax before going for invasive procedures. Depending on suspected possibility, type of CT-scan should be selected; HRCT to visualize detailed lung parenchyma and airway pathology, and contrast-enhanced computed tomography (CECT) to visualize mediastinum. Consolidation with areas of necrosis and signs of volume loss can occur in tuberculosis and other chronic infection like necrotizing pneumonia and nocardiosis. Both conditions usually have a longer history. The ‘tree in bud sign’ is described as well-defined centrilobular nodule with branching opacities. In our country, post-primary tuberculosis is the most important cause of tree in bud attenuation, and it represents endobronchial spread of tuberculosis. It usually occurs in mid to lower zones

Fig. 53C: HRCT-thorax showing consolidation in posterior segment of right upper lobe with areas of breakdown and satellite lesions with tree in bud pattern

219

Cases

but the sign is not specific. Tree in bud pattern was described by Japanese scientist for panbronchiolitis. In panbronchiolitis, the nodules are distributed throughout both lung fields mainly lower lobes and they are associated with air tapping and bronchiolectasis. Other causes include infectious bronchiolitis, fungal infection, mucoid impaction distal to bronchiectasis, bronchial obstruction or in association with ABPA, aspiration bronchiolitis and even intravascular metastasis. Tuberculosis should be confirmed by isolation of mycobacterial tuberculosis by smear, culture or by molecular techniques. Caseating necrosis is almost specific for tuberculosis. Non-caseating epithelioid granuloma is classically described for sarcoidosis, but it could not exclude tuberculosis completely. It depends on the area from where FNAC was taken, and treated tuberculosis may not show caseation necrosis. On the other hand, necrosis within consolidation may looks like caseating necrosis. FNAC material should be stained by ZN staining for demonstrating AFB in all suspected cases of tuberculosis, or in all unexplained lesions. Tuberculosis is the most important cause of non-resolving pneumonia in our country. TB was found to be accountable for about 35.7% of non-resolving pneumonia. Sputum negative pulmonary tuberculosis presenting as lobar or segmental consolidation may be confused as non-resolving pneumonia. Differential diagnosis of community acquired pneumonia should include tuberculosis and sputum for AFB smear is an essential investigation. Patients with findings like indolent course, poor response to appropriate antibiotics, sign of volume loss, consolidation crossing fissure line, areas of necrosis, etc. should be investigated to exclude tuberculosis.

Management We started antitubercular chemotherapy with category I directly observed treatment short course (DOTS) from Revised National Tuberculosis Control Program (RNTCP). Patient responded well and he became totally symptom-free by 1 month. After 6 months of therapy, there was complete clearance of radiological opacities.

My Learning Tuberculosis is an important cause of non-responding, progressive or unresolving pneumonia. Clinical material should be examined for AFB in all cases of non-responding pneumonia.

CASE 54: A LADY WITH PUERPERAL FEVER CASE REPORT A 28-year-old lady presented with fever and cough for 2 months. Patient stated that after delivery of her younger son, she had insidious onset of low-grade fever with evening rise of temperature. She also had cough with little mucoid expectoration that responded partially with cough syrup. She noticed loss of weight that was not recorded but observed by loosening of her garments (blouse). She had no addiction or past history of illness. She had two sons and

220

101 Cases in Respiratory Medicine

Fig. 54: Chest X-ray showing superior mediastinal widening with patchy opacities in left upper and midzones and right-sided cervical rib

the younger one was on breastfeeding. Her menstruation had not started after delivery and there was no abnormal vaginal discharge. General survey revealed mild pallor, BP—100/60 mm of Hg, pulse—110/ min, RR—36/min and temperature—100.2°F. Examination of respiratory and other systems revealed no abnormality. Her peripheral blood showed Hb% —8.5 g/dL; WBC—12000/mm3; N—65%, L—30%, M—3% and E—2%; and ESR—60 mm in 1st hour. Routine blood biochemistry was normal except borderline fasting blood sugar—120 mg/dL. Urine culture showed no growth of organism. Her chest X-ray PA view (Fig. 54) showed superior mediastinal widening along with heterogeneous opacity in the left upper and mid zones. There was a cervical rib on right side. Even after induction of sputum by nebulization with 3% saline, there was little expectoration and those samples were negative for AFB. Tuberculin test (Mantoux test) was positive 22 mm × 20 mm with 5 TU.

Discussion There are numerous causes of fever after delivery those should include puerperal sepsis. The history of abnormal, often foul smelling, vaginal discharge is important. Absence of abnormal vaginal discharge pointed to other causes. The differential diagnosis should include causes not directly related to pregnancy and causes that may flare up after delivery. The presence of cough and fever naturally pointed towards respiratory infection. Naturally, the next step should be an X-ray chest. She had superior mediastinal widening. There are many causes of superior mediastinal widening like thymic mass, germ cell tumor, retrosternal goiter and lymphadenopathy. Her X-ray showed convex lateral border with

Cases

lobulation and notching of superior mediastinal opacity. Lobulation and notching are strongly suggestive of lymphadenopathy. Thymic mass usually has concave lateral margins. Retrosternal goiter was excluded by absence of thyroid swelling on clinical examination. Mediastinal opacity should be investigated with contrast-enhanced CT-scan thorax. At that time, CT-scan was not available and we were totally dependent on X-ray findings. Our patient had left upper and midzone patchy opacities. The combination of mediastinal lymphadenopathy with lung parenchymal lesion may be caused by tuberculosis, lymphoma, sarcoidosis and lung malignancies. In a nonsmoker young female presented with fever, we were not comfortable with the diagnosis of lung cancer. Sarcoidosis usually presents with bilateral symmetrical hilar lymphadenopathy with/without right paratracheal lymphadenopathy. Superior mediastinal lymphadenopathy is unusual for sarcoidosis. Lymphomas should have peripheral lymphadenopathy, though pure intrathoracic lymphoma is a definite entity. In our country, tuberculosis should be considered first. FNAC/biopsy of lymph node/mass should be the best option for confirmation of diagnosis. Unguided FNAC/biopsy (before the era of CT scan and USG) from superior mediastinum containing many vascular structures is unsafe. So we relied upon tuberculin test. Positive tuberculin test, though not absolute, ruled out the possibility of sarcoidosis and lymphoma. Tuberculin test positivity suggests infection not disease. On the other hand, tuberculin test may be negative in active tuberculosis, and the causes include miliary/disseminated TB and TB in immune suppressed conditions like HIV infection, patient on corticosteroid or other immunosuppressive agents, malnourished individuals and after viral infection. Though not absolute, a strongly positive tuberculin test (induration >20 mm) favors active tuberculosis. Cervical ribs are sometimes missed in radiological examination of chest X-ray. Cervical ribs may compress nerves causing pain radiating to the arm. Otherwise cervical rib has no other clinical significance. During examination, examinee should be careful and should not miss cervical rib in chest X-ray.

Management We diagnosed the case as progressive primary tuberculosis. We started antitubercular treatment with daily INH—300 mg; RIF—450 mg; PZN—1000 mg and ETB—800 mg. Patient tolerated antitubercular medicines, and within 2 weeks she showed clinical improvement with remission of fever and reduction of cough. We followed her by chest X-ray. After second month of treatment, there was marked radiological improvement and we stopped PZN. At the end of 6 months treatment, her chest X-ray became normal and we stopped medicines.

Discussion We diagnosed the case as pulmonary TB (progressive primary TB) on the basis of typical clinical manifestation, Mantoux positivity and suggestive chest X-ray. At that time, modern imaging techniques and invasive methods

221

222

101 Cases in Respiratory Medicine

like FOB and EBUS were not available. Rigid bronchoscope and rigid mediastinoscope were available in limited centers. Unfortunately even today, the diagnosis of TB remains elusive and often we cannot confirm the diagnosis bacteriologically. I am deadly against therapeutic trial in TB. When clinical and radiological findings are reasonably suggestive of TB (clinically diagnosed tuberculosis), we should start treatment of TB instead of a therapeutic trial. In therapeutic trial, one should not use drugs that are effective against other organisms like rifampicin and streptomycin. Legionella pneumonia may mimic TB, and rifampicin is a very effective drug in that condition. It is always preferable to confirm the diagnosis. We should definitely ask for EBUS for mediastinal lymphadenopathy and BAL from lung parenchymal lesion. Academically, we must classify the type of TB. In our case, the glandular component was larger than lung parenchymal component, and that was suggestive of primary TB. But patient was symptomatic with progressive symptoms and parenchymal component was quite large involving left upper and midzones. We classified our case as progressive primary TB. Another important point I should mention here is the relation of TB with pregnancy. During later stages of pregnancy, the diaphragm is pushed upwards, and that causes a splinting effect on lung similar to that of pneumothorax My Learning or pneumoperitoneum. After delivery as We should move towards micro­ the diaphragm returns back to its normal bio­logically confirmed TB. Clinical position, the chance of activation of TB diagnosed of TB requires clinical prudence and reasoning. increases.

CASE 55: A MIDDLE-AGED LADY WITH A ROUND OPACITY CASE REPORT A 45-year-old Punjabi lady came to me crying. She was coming from Asansol, West Bengal. She had cough and recurrent attacks of hemoptysis for last 1 month. Cough was insidious in onset and progressive. Cough was associated with mild mucoid expectoration and cough was more in the morning. She noticed recurrent episodes of blood-streak sputum. On enquiry, she confessed that she felt feverish, particularly in the evening but she never measured temperature. She also told us that she had weight loss, evident from loosening of garments. She went to a local doctor who treated her with antibiotics. Her sputum samples were examined for AFB, and they were negative. An X-ray chest was done and she was referred to a specialist doctor. As advised by specialist doctor a CT-thorax was done. On the basis of CT-thorax, the possibility of malignancy was discussed and fiberoptic bronchoscopy was advised. Patient and her husband were frightened and they came to Kolkata for help. They insisted

Cases

that she would not undergo any invasive investigation. I admitted her in chest inpatient department. Additional information we get from her were, (i) she was completely healthy one month back and (ii) she used to drink alcohol regularly but never smoked. She denied any history of dyspnea or wheeze or any other symptoms. She did not have any other significant past, personal or family history. General survey and systemic examination including respiratory system examination were absolutely normal. Her peripheral blood and routine blood biochemistry were within normal limits. Her chest X-ray PA and right lateral views (Fig. 55A) showed a circular well defined mass in the right midzone close to heart border. Within the mass lesion, there was a small area of central lucency. Lateral view suggested that the lesion was located posteriorly. Her CT-thorax (Fig. 55B) showed circular

Fig. 55A: Chest X-rays PA and right lateral view showing a round opacity with central lucency

Fig. 55B: CT-thorax mediastinal and lung windows showing round opacity with central lucency in posterior segment of right upper lobe

223

224

101 Cases in Respiratory Medicine

lesion with central area of low attenuation in the posterior segment of right upper lobe without any lymphadenopathy or satellite lesion.

Discussion Our patient had insidious onset cough, streaks of blood with sputum, low grade fever and weight loss for 1 month. All symptoms were suggestive of tuberculosis. The general physician rightly asked for chest X-ray and sputum examination. The general physician was perhaps confused by peculiarity of radiological finding and negative sputum reports. The specialist doctor wisely asked for a noninvasive test, CT scan of thorax. He correctly described the lesion as mass lesion with central area of necrosis and considered the possibility of lung cancer and justifiably asked for an invasive test fiberoptic bronchoscopy. But I felt that there was a communication problem and the patient was terrified. I asked my postgraduate student to describe the CT-scan finding. One of them describe the lesion as reversed hallo sign (ground glass opacity surrounded by consolidation), a sign just reversed to hallo sign (consolidation surrounded by ground glass opacity). Hallo sign is classically found in invasive aspergillosis, whereas reverse hallo sign is found in organizing pneumonia. His diagnosis was organizing pneumonia. Focal organizing pneumonia represents 10 to 15% cases of organizing pneumonia.

Management I admitted her, assured her that she was not having malignancy, and gave her an anxiolytic drug (alprazolam 0.25 mg twice daily). Next day I asked one postgraduate student to collect induced sputum (by nebulization with 3% sodium chloride) under his direct supervision. The induced sputum was examined under fluorescent microscopy with rhodamine stain. The report came as presence of scanty acid fast bacilli. We started daily regime with INH 300 mg, RIF 600 mg and ETB 1000 mg for 6 months, and PZN 1250 mg for 2 months. She responded well to treatment. Chest X-ray PA view done after completion of treatment showed complete disappearance of radiological opacity. She went back home laughingly.

Discussion The specialist described the radiological opacity as mass lesion with central necrosis, probably ignoring her clinical setting (a non-smoker young lady with clinical manifestations like chronic inflammation). The postgraduate student described the lesion as reversed hallo sign, probably overcharged with his theoretical knowledge of interstitial lung disease. Considering her clinical setting I described the lesion as consolidation with central area of break-down, the description I learnt from my teacher late Professor Dr S Kargupta. My description was based on clinical presentation of the case. So, my diagnosis was pulmonary tuberculosis. Two points went against my diagnosis; i) absence of satellite lesion around main lesion and ii) negative results of previous sputum tests for AFB.

Cases

Interpretation of radiological findings or any laboratory findings should be based on clinical setting of patient. Her clinical manifestations, insidious onset of disease, cough, hemoptysis and weight loss, were suggestive of tuberculosis. Our job was to prove the diagnosis of tuberculosis by properly collect sputum samples and properly examining sputum. As there was no expectoration at that time we induced sputum to have representative sample. Getting representative sample (from lower respiratory tract) was the key to diagnosis. We examined sputum under fluorescence microscopy using auraminerhodamine staining and that is more sensitive than examination under light microscopy using Ziehl-Neelsen basic fuchsin dye. Possibly previous sputum examination was negative because of non-representative samples and/or examination under light microscopy that has lesser diagnostic yield. For interpretation of sputum report, it is essential to notice the nature of sample, whether it is sputum or saliva. It is an essential step to educate patient how to bring out My Learning sputum. Proper sample, properly collected, Description and interpretation properly transported and properly examined of radiological findings depends on clinical settings of patients. are the basic steps for any investigation.

CASE 56: A YOUNG MAN WITH TOXEMIA CASE REPORT Step I A 24-year-young man presented with fever and cough for 3 weeks. Fever was initially low-grade irregular but gradually increased in intensity. At the time of presentation, fever was high grade with chill but there was no rigor. Fever was associated with weakness, malaise, headache, anorexia, nausea and weight loss. There was no history of sore throat, headache or dysuria. Cough was dry, irritating and disturbing. Cough was not associated with expectoration. Cough was temporarily relieved by cough syrup. He was first seen by a general physician and he was treated with oral antibiotics (cefixime 200 mg twice daily) along with symptomatic treatment and vitamins. Some preliminary investigations were done by previous doctor. He was referred to our outpatient department. On examination, patient was toxic with tachycardia (HR—124/min), tachypnea (RR—32/min), BP—100/60 mm of Hg and fever (temperature 102°F). Examination of respiratory (both upper and lower) did not reveal any abnormality. Examination of other systems was essentially normal. His peripheral blood examination showed mild anemia (Hg—10 g/dL), raised ESR (70 mm in first hour) and normal leukocyte count (9800/mm 3 with 68% neutrophils, 28% lymphocytes and 4% eosinophils). His routine blood biochemistry was normal. He was HIV negative. His X-ray PA view (Fig. 56A) showed right hilar prominence. Careful examination of the chest X-ray detected small, ill-defined, low density opacity in the right mid-zone below the anterior end of right second rib (arrow head).

225

226

101 Cases in Respiratory Medicine

Fig. 56A: Chest X-ray showing right hilar prominence with a ill-defined opacity in right mid zone (arrow head)

Discussion Initial presentation of the young man was like low intensity infection that subsequently changed to severe infection with toxemia. The changing scenario could be explained by viral infection followed by severe bacterial infection like staphylococcal pneumonia following influenza. Presumably, severe bacterial infection (lobar or bronchopneumonia) should have some physical signs in thoracic examination and leukocytosis. Chest X-ray findings were not consistent with severe infection. The most striking feature of the case was profound symptoms and marked toxicity with no respiratory findings. Chest X-ray showed a soft parenchymal opacity with ipsilateral hilar prominence. Small lung parenchymal lesion with ipsilateral hilar lymphadenopathy would suggest primary complex. But the clinical picture did not match with that of primary tuberculosis. In primary TB, manifestations are usually negligible and it is usually ignored as viral fever. The differential diagnosis includes lymphoma, bronchogenic carcinoma and chronic infections.

Step II I advised a fresh chest X-ray PA view (Fig. 56B), and that showed bilateral extensive miliary opacities. In retrospect when I examined his previous chest X-ray, I could identify some tiny nodular opacities dispersed bilaterally over midzones. HRCT-thorax (Fig. 56C) showed randomly distributed nodular opacity of uniform size with some conglomerated opacities. The patient did not have any sign of meningitis. On the basis of clinical manifestations, X-ray and CT-scan findings and absence of manifestations suggestive of other causes of miliary opacities, I diagnose the case as miliary TB.

Cases

Fig. 56B: Chest X-ray showing bilateral miliary opacities with right hilar prominence

Fig. 56C: HRCT-thorax showing randomly distributed miliary nodules with few areas of conglomerated lesions

227

228

101 Cases in Respiratory Medicine

Discussion I should have diagnosed the case after seeing the first X-ray. I missed the diagnosis due to my casual approach. It should be remembered that early miliary opacities appear in mid zones. Early miliary opacities can be easily missed unless you specifically search for them. High fever, cough, toxemia and absence of thoracic signs were indicative of miliary TB. Yet I had not searched specifically for miliary opacities. Fortunately, I advised another chest X-ray and thereby the diagnosis was established. The difference between two X-rays was noticeable. First X-ray was probably expiratory. The most important cause of miliary (millet seed like opacity 70 U/L is considered to be specific to tuberculosis provided other causes like rheumatoid pleuritis or empyema are excluded. Empyema fluid usually shows a high ADA level and measurement of ADA level is a useless investigation in frank empyema.

Cases

Fig. 63: CT-thorax showing left-sided pleural effusion with heterogeneous opacity in the underlying lung

Step II His induced sputum samples, by nebulization with salbutamol and normal saline, were examined under fluorescent microscopy. Samples were positive for tubercle bacilli. His HRCT-thorax (Fig. 63) showed left-sided pleural effusion with patchy opacities, mainly ground-glass attenuation, in the underlying lung. We advised him HIV serology and that was positive for HIV-1. On enquiry, patient told us that for business purpose he had to travel throughout country, and he confessed exposure history.

Discussion Tuberculous pleural effusion is usually a hypersensitivity reaction to tubercular protein as a result the chance of isolation of bacilli from pleural fluid is grim. Pleural biopsy is a better choice for confirmation of diagnosis. Tubercular empyema is due to rupture of tubercular cavity into pleura and as a result it is easier to isolate bacilli from the fluid/pus. In contrary, in people living with HIV isolation of bacilli in pleural fluid is possible in up to 20% of cases by smear and up to 40% cases by cultures. Sputum induction and sputum microscopy should be done in all cases of pleural effusion. Underlying lung may have active TB and that may be concealed by the opacity of pleural effusion. Induced sputum in tubercular pleural effusion may be positive for tubercle bacilli in up to 12% by smear and in up to 52% cases by culture. CT scan thorax after aspiration is an important investigation to visualize the underlying lung. In RNTCP, all TB patients are screened for HIV infection; and on the other hand, all HIV positive patients are screened for TB. But at that time HIV screening was not mandatory. People living with HIV are more likely to have extrapulmonary and sputum smear-negative TB. Prevalence of MDR-TB is more in HIV positive patients. Among treated TB patients, death

255

256

101 Cases in Respiratory Medicine

rates are higher in HIV-positive than in HIV-negative patients. Case-fatality rate is higher in people living with HIV with smear-negative pulmonary and extrapulmonary TB, as these patients are generally more immunesuppressed than those with smear-positive TB. About 10% patients with HIV and TB coinfection chest X-ray may be normal. High-mortality rates have been reported among people living with HIV who have drug resistant-TB, and the rates may exceed 90% in patients with XDR-TB and HIV coinfection.

Management We started daily regimen with 4 drugs (INH, RIF, PZN and ETB) and sent him to HIV clinic for management of HIV infection. As per their advice, we started ART with two NRTI and one NNRTI. We also started prophylactic cotrimoxazole. Fortunately, our patient tolerated drugs well with some minor problems. We continued ATD for 8 months. We advised him to continue ART.

Discussion Management of TB in HIV positive patient should be daily regime both in intensive as well as continuation phase, and there is no scope for intermittent treatment in HIV positive persons. The optimum duration of treatment is same as patient without HIV infection. However, the expert opinion after a systematic review is in favor of extending duration of ATD for 8 months or more. The first priority for HIV-positive TB patients is to initiate TB treatment, and that should be followed by antiretroviral treatment (ART) and cotrimoxazole therapy. In all HIV-positive TB patients, preventive therapy with cotrimoxazole should be initiated as soon as possible and should continue throughout TB treatment. It has been found that preventive cotrimoxazole substantially reduces mortality in HIV-positive TB patients. ART should be initiated for all people living with HIV with active TB, irrespective of CD4 cell count. ART improves survival in HIV-positive patients. In addition, it reduces TB recurrence rates by 50%. One randomized controlled trial provides evidence for early initiation of ART, and that reduces mortality, improved TB outcomes and reduced incidence of immune reconstruction syndrome (IRIS). The rationale for starting ART soon after TB diagnosis is that case-fatality among HIV-TB patients occurs mainly in the first 2 months of TB treatment. ART should contain 2 NRTIs (zidovudine or tenofovir and lamivudine or emtricitabine) and 1 NNRTI (efavirenz or nevirapine). Protease inhibitors are kept reserved for second-line regimens. A rifabutin-based TB treatment is recommended for individuals who My Learning require an ART regimen containing protease inhibitor. When efavirenz is not tolerate or Examination of induced sputum contraindications, an alternative nevirapine and HIV serology are two essential investigations for all based regime or a triple NRTI regimen is suspected cases of tubercular recommended. pleural effusion.

Cases

Pleural Diseases Case 64: A Classical Tuberculous Pleural Effusion Case Report A 21-year-college student developed in sequence fever, chest pain and shortness of breath for two weeks. Fever was low grade with evening rise of temperature. Chest pain was pleuritic in nature (sharp, stabbing, and increased with respiration and coughing) involving left lower hemithorax. On enquiry, he could remember that at the beginning, the pain started in the left upper abdomen. Patient had shortness of breath only during excertion and more than his usual activities. He was totally well before two weeks. He was non-smoker and did not have any addiction. There was no significant past history or family history. His general survey and examination of upper respiratory tract did not reveal any abnormalities. Examination of lower respiratory system showed movement of left lower hemithorax was diminished and rightwards shifting of trachea and apical impulse could not be localized. Percussion notes were stony dull from 4th intercostal space downward in left midclavicular line, throughout left midaxillary line and in left lower interscapular area and along left scapular line. Traube’s space resonance was obliterated. There was no shifting dullness. Breath sounds were absent over the areas of dull notes and diminished over other areas in left side. Examination of other systems was essentially normal. His routine blood and biochemistry were normal. He was HIV negative. X-ray chest PA view showed classical left-sided moderate pleural effusion. USG abdomen showed no abnormality. Nine hundred ml of straw color fluid was aspirated from left pleural space. Pleural fluid examination revealed cell count—300/mm3, N—10%, L—88% and mesothelial cell—2%, protein—3.5 g/ dl; sugar—46 mg/dl, and ADA—74 U/L.

Discussion Our case was a plain and simple case of left-sided exudative, lymphocytic and high ADA pleural effusion. Pleural effusion is not a diseases but manifestation of diseases. Accumulation of fluid is the pathological change and the etiology should be searched. Plethora of diseases may manifest as pleural effusion. Our reasonable next step is to find out the cause of effusion. So, let us start with individual finding and analyze them. Low-grade fever suggest chronic inflammation. Evening rise of temperature is due to upward movement of normal diurnal variation of normal temperature curve and that crosses threshold limits in the evening. This type of fever is not specific for tuberculosis but may occur in any chronic inflammatory disease. Pleuritic pain suggests inflammation of pleura and shortness of breath can be explained by accumulation of fluid. Some patient may describe it as heaviness of chest. In our patient, pain started in the abdomen. Many abdominal diseases particularly pancreatitis can cause left-sided pleural effusion. On the other

257

258

101 Cases in Respiratory Medicine

hand, pain of diaphragmatic pleurisy may be referred to abdomen due to same dermatome nerve supply. So, it is not unusual for pleuritic pain to start in the upper abdominal. Abdominal pathologies must be considered and excluded under that condition. In our case, we excluded abdominal cause of pleural effusion by abdominal examination as well as USG abdomen. Free fluid in pleural space physically should have a horizontal upper border. Radiologically, we get highest point in the axilla due to visual elusion as more X-ray beams have to pass through fluid in the axillary region. Clinically also we get highest point in the axilla as lung collapses toward hilum resulting in maximum distance between air-filled lung (causing resonant nodes) and chest wall in the axillary area. In pleural disease mediastinum is shifted towards opposite side. Previously, it was thought that the shifting is due to pushing effect of pleural fluid/air. Now, the concept has been changed. Mediastinal shifting is now considered not due to push but due to pull by the negative pleural pressure of the opposite side. Normally, negative pleural pressure keeps the lung inflated by neutralizing the collapsing tendency of lung due to elastic recoil. A positive pleural pressure, due to accumulation of air, fluid or both, cannot hold the lung as a result underlying lung collapses, and this type of collapse is called relaxation collapse. The first step in pleural fluid examination must be directed to differentiate between exudative and transudative effusion. In our case, history suggestive of pleural inflammation and pleural fluid protein >3 g/dl confirmed exudative effusion. If there is any doubt, Light‘s criteria will be helpful. Any one of the three criteria will suggest exudative effusion (i) pleural fluid protein/serum protein >0.5, (ii) pleural fluid LDH/serum LDH >0.6, and (iii) pleural fluid LDH > 2/3rd of the upper limit of normal serum LDH. Second step should be to analyze cell types. In acute inflammation neutrophil and in chronic inflammation lymphocytes are expected to increase. Lymphocyte neutrophil ratio is very helpful; if the ratio is more than 75% it is suggestive of chronic inflammation, particularly tuberculosis in our country. Mesothelial cell increase is suggestive of malignancy and more than 5% count practically rules out tuberculosis. In lymphocytic exudative effusion ADA plays an important role and ADA is more than 90% sensitive and specific for diagnosis of tuberculous pleural effusion with cutoff value more than 70 U/L. A value between 40 and 60 U/L is considered as borderline and value less than 40 U/L practically exclude tuberculous pleural effusion. ADA may be more than 70 U/L in empyema, rheumatoid arthritis or some cases of lymphoma. So, ADA has more negative prediction value than positive prediction value. Tubercular pleural effusion can be diagnosed with confidence in exudative pleural effusion with lymphocyte neutrophil ration more than 75% and ADA more than 70 U/L; and when there is no evidence suggestive of other disease. Confirmation of diagnosis of tuberculous pleural effusion requires demonstration of tubercle bacilli or their DNA fragments in the sputum, pleural fluid, or pleural biopsy specimen.

Cases

Fig. 64: Chest X-ray showing left-sided pleural effusion with left-sided transverse fissure (arrow head)

Management Our case was an adolescent boy with low-grade fever, evening rise of temperature, pleuritic chest pain, straw color, lymphocytic, exudative fluid with lymphocyte neutrophil ration—88% and ADA—74 U/L, and there was no manifestation suggestive of other disease. We diagnosed the case as tuberculous pleural effusion and started category-III DOTS. At that time less severe tuberculosis was treated with Cat-III regimen with INH, RIF and PZN for first 2 months followed by INH and RIF for 4 months. Patient responded with ATD.

Uniqueness There was a twist in this story. His post-aspirated X-ray (Fig. 64) showed a horizontal line in left mid zone starting from pleural border up to junction between lateral My Learning 2/3rd and medial 1/3rd of left hemithorax. The The diagnosis of tubercular line was a replica of transverse fissure, with or pleural effusion can be made without fluid collection, except it was found in confidently in lymphocytic, left side. We diagnosed the horizontal line as exudative, high ADA effusion without any manifestations of left sided transverse fissure and the condition other diseases. as trilobed left lung, a developmental anomaly that do not has any clinical significance.

259

260

101 Cases in Respiratory Medicine

Case 65: Hemorrhagic Pleural Effusion with Low Adenosine Deaminase Case Report Step I A 25-year-old lady, college student and daughter of a renowned dental surgeon, presented with irregular high-grade fever without chill or rigor and dry cough for 7 days. She also had left-sided pleuritic chest pain at the beginning that was replaced by heaviness of chest. She had no significant past history; she had no addiction and no menstrual abnormality. She did not have joint pain or any manifestation of collagen vascular disease. General survey revealed fever (temperature—100.6°F), tachycardia (heart rate—110/min), tachypnea (RR— 34/min) and signs of dyspnea. Examination of thorax revealed diminished movement of left hemithorax, trachea shifted to right, apical impulse not palpable, stony dullness of all areas of left hemithorax except over suprascapular area and absent of breath sounds over left hemithorax except at lung apex. Other systems were normal. Her routine blood and biochemistry including amylase and lipase were normal. She was HIV negative. Her X-ray chest PA view showed opaque left hemithorax obliterating both cardiophrenic and costophrenic angles with mediastinal shift toward right side. I admitted her in a private hospital. I aspirated 1200 ml hemorrhagic fluid on first day and 1000 ml on second day and 600 ml on third day. Pleural fluid showed plenty of RBC, WBC—1800/mm3 with N—6%, L—82% and E—12%. Pleural fluid protein was 5.4 g/dl; sugar was 46 mg/dl and ADA was 24 U/L. Pleural fluid was negative for AFB. Pleural fluid PAP stain was negative for malignant cells on three occasions. Induced sputum sample was collected and examined for AFB. Report came as negative.

Discussion Lady was suffering from left-sided massive, lymphocytic, exudative, hemorrhagic pleural effusion with low ADA. From the beginning it appeared that pleural effusion was caused by inflammation supported by fever and pleuritic chest pain. But the findings like massive and hemorrhagic effusion were suggested malignancy. When there is a diagnostic difficulty the golden rule is to start from the beginning. She had high fever of acute onset. The picture might suggest pyogenic infection but absence of chill and rigor and over all clinical setting went against that possibility. Lymphocytic pleural fluid possibly indicated chronic inflammation. Causes of lymphocytic effusion include tuberculosis, viral effusion, fungal infection, collagen vascular diseases, malignancies and lymphoma. High fever and hemorrhagic effusion did not exclude TB

Cases

completely. Pleuritic chest pain was suggestive of inflammatory nature of pleural involvement. High lymphocyte neutrophil ratio (>75) was suggestive of TB but low ADA rejected the diagnosis of TB. Pleural fluid eosinophilia is arbitrarily defined as pleural fluid eosinophil count more than 10%. High pleural fluid eosinophil count is found in hemorrhagic effusion and in presence of air in pleura; other causes include asbestos related, drug induced and parasite induced pleural effusion. ADA has more negative prediction value than positive prediction value. A high ADA value is nonspecific but a low ADA value practically excludes TB. ADA is of two forms: a small form and a large form. Tubercular effusion has only the large forms whereas other forms have the prominence of small form. The typing of ADA is not popularized as due to their nonspecificity they do not help in diagnosis. In that situation we should explore other possibilities. There was no clinical manifestation suggestive of collagen vascular disease. Rarely, respiratory manifestations may precede the development of rheumatic manifestations of collagen vascular diseases. SLE may present as hemorrhagic pleural effusion in middle aged female. Viral pleural effusion is usually small and is usually associated with constitutional manifestations like myalgia, lassitude, weakness, headache, etc. We excluded pancreatitis by absence of suggestive clinical manifestation, normal serum amylase and lipase levels, and absence of suggestive USG and CECT findings of pancreatitis. We seriously considered malignancies and particularly lymphoma. Fever might be explained by solid tumor. Pleuritic chest pain usually suggests inflammation of pleura and is unexpected in malignant pleural effusion. Malignant pleural effusion is commonly secondary to primary tumor in lung, breast, ovary, abdominal organs, thyroid and sarcomas. Primary malignant pleural effusion is due to mesothelioma occurs as a result of asbestos exposure. Lymphoma can cause hemorrhagic pleural effusion, and is usually associated with peripheral lymphadenopathy, hepatosplenomegaly and other manifestations. Pure intrathoracic lymphoma though rare is a distinct possibility.

Step II We started with an open mind. We took detail history again but we did not get any clue. Breast examination and gynecological check-up were negative. We sent blood for collagen profile and we found rheumatoid factor, ANA, dsDNA and antiCCP were negative. Post aspiration CECT thorax did not reveal pleural nodule, lymphadenopathy, lung parenchymal lesion, mass lesion or intraluminal pathology. USG abdomen and CECT abdomen were completely normal. Mammography was normal. Under that circumstance we were planning invasive investigation like FOB, thoracoscopy and pleura biopsy. Her father did not give permission for any invasive procedure.

261

262

101 Cases in Respiratory Medicine

Discussion In our case, history, clinical examination and noninvasive investigations failed to point to a diagnosis. We were considering invasive procedures. We planned FOB to rule out proximal intraluminal growth and to take bronchoalveolar lavage. In nonsmoking young lady the possibility of lung cancer was least; moreover, her CECT did not show any underlying lung pathology. We planned blunt pleural biopsy. The yield of blunt pleural biopsy in tuberculous pleural effusion is quite high; the chance of getting granuloma is 80%; the chance of isolation of AFB in the biopsy material is 25.8%; the chance of culture positivity for Mycobacterium from biopsy material is around 56%; and at least one of the three tests positivity is found in 91% cases. In malignancies, the yield of pleural fluid cytology would be more than blunt pleural biopsy as lesions are expectedly scattered in distribution. The yield of blunt pleural biopsy varies between 39% and 75% in different studies. On the other hands, the diagnostic yield of pleural fluid cytology is around 80% when 3 separate pleural fluid specimens are examined. So, we seriously considered thoracoscopic pleural biopsy that would give additional informations regarding pleural pathology. But we could not proceed in that direction as necessary informed consent was not available even after repeated persuasion.

Management We tried induction of sputum but the specimen was AFB negative. She was non-diabetic and HIV negative. After discussion with the patient and her father, we started daily antitubercular chemotherapy. Patient responded well and we continued ATDs for 6 months. After the end of treatment she was totally healthy and her follow-up chest X-ray showed complete clearance of radiological opacity. Two years follow-up did not revealed recurrence of effusion.

Discussion In that lady the clinical manifestations and pleural fluid reports were in favor of tuberculosis except low ADA. There were no manifestations suggestive of other diseases clinically as well as by screening investigations. Low ADA, massive effusion and hemorrhagic effusion were against the diagnosis of tuberculosis. Both hemorrhagic effusion and massive effusion were rarely found in TB but they do not exclude tubercular effusion completely. Their presence needs further investigation. I investigated the case in detail and failed to get any positive clue. Lower ADA in tubercular pleural effusion may be found in immunesuppressed conditions. Another interesting aspect is that a much lower

Cases

level of ADA has been reported in Asian populations. In doubtful cases it is preferable to have blunt pleural biopsy or thoracoscopic biopsy. After total investigation, I had excluded My Learning most of the possibilities. At that point I was seriously thinking of three possibilities: Massive and hemorrhagic plural tuberculosis, intrathoracic lymphoma and effusion does not exclude tubercular pleural effusion. unknown primary malignancy. As later two Measurement of ADA level is a conditions are conspicuously uncommon useful investigation in pleural and tuberculosis is particularly common in effusion, but it is not absolute. our country, I decided to start ATD.

Case 66: Changing Diagnosis Case Report Step I A 27-year-old man, farmer by occupation, had insidious onset of cough and low grade irregular fever for one and half months. The disease started insidiously with cough, and that was increasing in intensity. Cough was associated with mild mucoid expectoration and one episode of hemoptysis. He had also irregular low-grade fever that was associated with nonspecific general symptoms. He was treated by a local physician, and hemoptysis was controlled with conservative treatment but cough was persisting. Then he noticed progressive dyspnea and heaviness of right side of chest. He was advised an X-ray chest PA view. X-ray showed right-sided massive pleural effusion. He was referred to our medical college. We admitted him in our in-patient department. Additional information we got was that he was a smoker and he smoked about 1 packet, containing 20 biris for more than 10 years. General examination revealed patient was toxic with tachycardia, tachypnea and signs of dyspnea. Examination of thorax revealed signs of right-sided massive pleural effusion with diminished movement on right hemithorax; trachea shifted towards left, apical impulse on left 5th intercostal space just outside midclavicular line; stony dull percussion notes; and absent breath sounds on lower part and diminished vesicular breath sounds in the upper part of right hemithorax. Routine peripheral blood and blood biochemistry were within normal limits. Chest X-ray showed right sided massive pleural effusion with homogeneous opacity involving whole right hemithorax except a small area at the top of lung, obliterating right cardiophrenic and costophrenic angles with mediastinal shifting toward left. We were planning intercostal tube drainage. Before tube thoracostomy, one junior doctor proceeded for aspiration of pleural fluid. A peculiar incidence occurred during aspiration. All ambulatory patients fled away from the ward because of the extraordinary strong foul smell. The poor junior doctor could not comprehend what he should do. We put intercostal tube and started antibiotics

263

264

101 Cases in Respiratory Medicine

and given anaerobic coverage with injection clindamycin 600 mg 8 hourly. His pleural fluid report came as plenty of Gram-positive cocci and Gram-negative bacilli. Pleural fluid aerobic culture showed no growth. Pleural fluid cytology for malignant cells and AFB smear were negative.

Discussion Those who have aspirated anaerobic empyema know the bewildering experience. The strong foul smell is the characteristic finding and practically diagnostic of anaerobic empyema. Presence of multiple organisms, including Gram-positive and -negative cocci and bacilli with no growth in aerobic culture is another feature of anaerobic infection. Anaerobic culture is the investigation of choice for establishing the diagnosis. Precaution must be taken during sending any biological sample to laboratory for anaerobic culture by avoiding contact with air during transport or by directly putting the material to culture bottle. Pleural aspirate is sent in a capped syringe for anaerobic culture. In our case, we established anaerobic empyema by peculiar foul smell of pleural fluid, presence of multiple bacteria in smear and absence of growth in aerobic culture. Due to non-availability of anaerobic culture facility, we could not confirmed the diagnosis. We treated the case with intercostal tube drainage and antibiotic with anaerobic coverage.

Step II Patient was responding with treatment, fever and toxicity disappeared, and pleural fluid lost its foul odor. But pleural fluid drainage was continuously more than 250 ml fluid per day even after 10 days of therapy. We again sent pleural fluid for Gram stain and culture. The report came as Gram-negative bacilli and growth of Pseudomonas aeruginosa in smear and culture, respectively. We started intravenous piperacillin-tazobactum combination 4.5 g 8 hourly. Patient responded and pleural fluid became culture negative after 7 days. There was qualitative (became serous) and quantitative (reduced to 100 ml/ day) improvement of pleural fluid drainage.

Discussion Superinfections are secondary bacterial infections that usually develop over a viral infection like common cold or flu. While our immunity is disturbed, bacteria take the opportunity to invade lungs or sinuses and cause a superinfection. Bacterial sinusitis is probably the most common superinfections. The most dangerous and lethal superinfections are secondary pneumonias. Superinfection is responsible for most cases of deaths in influenza epidemics, worldwide. People with respiratory problems, such as asthma, COPD,

Cases

emphysema, smokers and those with compromised immune systems like diabetics, renal failure, malignancy, immunesuppressed condition and pregnant women have higher risks of superinfection. The usual organisms responsible for superinfections are Streptococcus pneumonia, Haemophilus influenzae, and Group A Streptococcus, and Staphylococcus. In our patient, community acquired anaerobic empyema was secondarily infected by hospital acquired aerobic organism. Pseudomonas aeruginosa is usually a hospital-acquired infection. Advanced stages of COPD with structural lung diseases may have community-acquired Pseudomonas infection, and even them past history of hospitalization is obtainable. Pseudomonas aeruginosa infection usually occurs during use of medical instruments and in patient with history of previous antibiotics use. When patient is not responding with appropriate antibiotics, the possibility of superinfection should be considered. In our patient the diagnosis of superinfection was proved by the finding of new growth of Pseudomonas aeruginosa in pleural fluid that was culture negative in earlier aerobic culture. Fortunately, patient responded to antipseudononal antibiotics.

Step III As pleural fluid collection persisted we advised CECT-thorax. His CECTthorax showed pleural effusion with consolidation of right lower lobe without mediastinal lymphadenopathy. A careful look showed a small nodule at the right wall of trachea just at and above the level of carina (Fig. 66). The finding surprised us. As a next logical step we performed fiberoptic bronchoscopy. We found an intraluminal infiltrative growth in the right main bronchus extending to trachea just above carina. We took biopsy from the lesion. The report came as adenocarcinoma of the lung.

Fig. 66: CECT-thorax showing a nodule in the trachea at the level of carina

265

266

101 Cases in Respiratory Medicine

Discussion The case started with right-sided moderate to massive pleural effusion. During pleural aspiration it was found to be anaerobic empyema. Subsequently, it was proved that there was superinfection by Pseudomonas aeruginosa; And finally, the case was diagnosed as adenocarcinoma of the lung. Empyema was secondary to postobstructive pneumonia. Histologically, adenocarinoma is the most common lung malignancy and that has replaced squamous cell carcinoma. Adenocarcinoma is the expected lung malignancy in nonsmokers. Contrary to our perception, adenocarcinoma is increasing found in smokers. Increasing frequency of adenocarcinoma in smokers in perhaps related to changes in smoking habits. As, for example, use of filters gives a false sense of security results in deep inhalation of smoke.

Management We took oncologist opinion and started chemotherapy with cysplatin and etoposide. Patient responded well and we were able to remove the intercostal tube after two weeks and discharge him. We advised him to continue next cycles of chemotherapy from oncology department at day-care service.

My Learning There are many turning points in the long journey of patient’s suffering, and never stop before you reach the final destination.

Case 67: An Elderly Lady Had a Concealed Truth Case Report Step I A 64-year-old lady presented with dyspnea for last 1 month. As per her statement she was well and could perform her usual activities including cooking before one month. She developed progressive shortness of breath along with discomfort in left side of chest. At the time of presentation, she had dyspnea at rest and that disturbed her normal activities and sleep. She had cough without expectoration. She did not have fever, chest pain, hemoptysis or any significant personal, family or past history. Her clinical findings and chest X-ray were suggestive of left sided massive pleural effusion. She was admitted in a private hospital. Pleural fluid was aspirated. Pleural fluid was hemorrhagic, protein—4.5 g/dl, sugar—60 mg/ dl, ADA—46 U/L, RBC plenty, WBC—1200/mm3 with lymphocyte—64%, eosinophil—18%, mesothelial cell—10% and neutrophil—08%, and PAP stain was negative for malignant cell. She was aspirated 1,000 ml, 1,200 ml and 800 ml of hemorrhagic fluid within 5 days. There was temporary relieve of her symptoms following aspiration. Her CT thorax after aspiration of fluid on the

Cases

5th day showed moderate pleural effusion on the left side with some ill-defined heterogeneous opacity in the underlying lung but there were no evidence of mass lesion or metastasis. The case was referred to me for opinion.

Discussion Clinically, malignant pleural effusion should be suspected in the presence of following manifestations: elderly person, positive smoking history, absence of pleuritic chest pain, massive effusion, hemorrhagic effusion, recurrent accumulation of pleural fluid, effusion without mediastinal shifting, presence of clubbing, peripheral lymphadenopathy, manifestations of mediastinal involvement like hoarseness of voice/SVC syndrome, manifestations of metastasis, etc. Most patients complain of dyspnea, which is frequently out of proportion to the amount of the effusion. The pleural fluid is usually exudative and lymphocytic, and its glucose level may be reduced that is proportional to tumor burden in the pleural space. In our case the elderly lady, who was previous healthy, developed massive hemorrhagic pleural effusion pointing to malignant pleural effusion. Raised pleural fluid eosinophil is a common finding in pneumothorax and hemorrhagic pleural effusion. Mesothelial cells more than 5% practically exclude tuberculous effusion and empyema and are suggestive of malignant pleural effusion. There may be confusion between malignant pleural effusion and pleural effusion in malignancy. Malignant pleural effusion is due to direct involvement of pleura by malignant process that can be demonstrated by pleural biopsy or the presence of malignant cells proved by pleural fluid cytology. Malignant pleural effusions are usually secondary. Primary pleural malignancy (mesothelioma) is rare. In female, common primary sites of malignant pleural effusion are lungs, breasts, gastrointestinal tract, uterus, ovaries, thyroid, lymphoreticular system. Pleural effusion in malignancy may be due to different mechanisms and they include pleural metastases, obstruction to lymphatic drainage of pleura, hypoproteinemia, collapse of lung, superior vena caval obstruction, parapneumonic effusion from post obstructive pneumonia, etc. The main object in this case was to confirm malignancy. Pleural fluid cytology done thrice failed to show malignant cell; postaspiration CT thorax failed to give any clue; and there was no manifestation suggestive of any other primary site. Pleural biopsy was an option but blind pleural biopsy is not superior to pleural fluid cytology in the diagnosis of malignant pleural effusion. Due to immaturity of the cells (anaplasia) and lack of adhesive molecules malignant cells are exfoliated into pleural fluid, and thereby increase the chance of getting them in pleural fluid cytology. On the other hand, pleural metastases are scatterly distributed so the possibility of getting them by blind biopsy is less. The diagnostic yield of pleural fluid cytology varies from 40–87%. The less yield may be due to effusion resulting from

267

268

101 Cases in Respiratory Medicine

other mechanisms (effusion occurs as a result of obstruction in bronchus or lymphatic drainage) and types of malignancy (squamous cell carcinoma has less yield whereas adenocarcinoma has more yield). The initial pleural fluid is positive in about 60% cases and that increased to about 80% if three separate samples are tested. The accuracy increased when both smear and cell blocks are prepared. The diagnostic yield of blunt pleural biopsy is less and varies from 35–75%, where as it is about 50–80% effective in detecting tuberculous granuloma. When malignant pleural effusion is suspected, thoracoscopic pleural biopsy is preferable than blind pleural biopsy. When pleural cytology is negative, I usually prefer thoracoscopic pleural biopsy than blind pleural biopsy. Moreover, plerodesis, a palliative procedure for malignant pleural effusion, can be done through thoracoscopy. In suspected tuberculous effusion I prefer blind pleural biopsy. An alternative to thoracoscopy is CT- or ultrasound-guided needle biopsy of thickened pleura or pleural nodules.

Step II I started with detail history taking and meticulous clinical examination. I got no clue in history except a short duration of illness and before 1 month she was totally healthy. She was a nonsmoker. There was no pleuritic chest pain, and that exclude pleural inflammation. There was no joint pain or any manifestations suggestive of collagen vascular disease. There was no manifestation suggestive of pancreatitis or any other abdominal pathology. There was no history of post-menopausal bleeding or bleeding from any sites. She denied history of breast lump, or any manifestations of primary tumor elsewhere. General survey showed signs of dyspnea with tachypnea, tachycardia and active accessory muscles of respiration. Examination of thoracic cage revealed signs of left sided massive effusion with trachea central and nonpalpable apical impulse. There was no peripheral lymphadenopathy. Abdomen was soft without organomegaly, mass lesion or ascites. Examinations of other systems were within normal limits. There was no thyroid swelling. I reviewed investigations and I failed to get any clue. Her chest X-ray showed left sided massive pleural effusion with minimum shifting of mediastinum. USG whole abdomen was within normal limits. Post-aspiration CT thorax showed nonspecific heterogeneous opacity in right lower lobe that may be suggestive of infective pathology. Then I asked sister to remove her upper garment after explaining and getting her permission. I was astounded that her left breast was puckered. I asked her why she had not told me that before. She insisted that the lesion was nothing abnormal but was due to insect bite one and half years back. Biopsy from breast lump showed scirrhus carcinoma of breast.

Cases

Discussion Malignant pleural effusion is the second most common cause of exudative effusion. Overall three malignancies lung carcinoma, breast carcinoma and lymphoma contributed about three-fourth of malignant pleural effusion. The most common primary sites for malignant pleural effusion in female is lung cancer in smokers and breast cancer in nonsmokers.

Management We put on intercostal tube and drained pleural fluid. We did FOB to exclude intrabronchial lesions. Intrabronchial metastasis though rare may be found in breast cancer (it is mostly seen in hypernephroma). We performed pleurodesis with betadine solution as a palliative measure. Patient was started with chemotherapy as per advice of oncologist. We lost the patient by sixth months.

Discussion Patients with a malignant pleural effusion are treated symptomatically since the presence of the effusion indicates disseminated disease. Malignant pleural effusion is generally considered as advanced stage disease and treatment with palliative intent is only possible. Malignant pleural effusion causes dyspnea due to recurrent accumulation of fluid. The symptomatic treatments available are (i) repeated thoracentesis, (ii) tube thoracostomy, and (iii) pleurodesis. Repeated aspiration or tube thoracotomy cause loss of protein and that augment fluid collection. My Learning Pleurodesis is preferred if total drainage of fluid can be achieved and there is no Meticulous physical examination intrabronchial growth. Pleurodesis can be has no substitute. Examination done by talc insufflations, or instillation of of respiratory system is not complete without examination talc emulsion, tetracycline, doxycycline, of breast in female. beomycin and providone iodine.

Case 68: Right-Sided Loculated Empyema Case Report Step I A 36-year-man presented with pain in the right upper abdomen, fever, cough and shortness of breath for 3 months. The disease started insidiously with dull aching pain in right upper abdomen, anorexia and low-grade fever. The pain changed it character to stabbing in nature that increased with respiration and coughing. Fever was initially low-grade and later it increased to high grade intermittent fever with chill but without rigor. Cough was intermittent with little

269

270

101 Cases in Respiratory Medicine

mucoid expectoration. He noticed shortness of breath with his normal activity. He had anorexia, nausea, malaise and extreme weakness. He was a farmer; he was married with two children. He took 10–15 biris per day for more than 20 years. He used to take country-made alcohol regularly. He was totally healthy before the present illness. There was no significant personal or family history. General survey revealed patient was toxic, prostrated and cachectic. His BP was 110/60 mm of Hg; pulse was 120/min; RR was 34/min and SpO2 was 96% at room air. Examination of respiratory system revealed decreased movement in right hemithorax, and mediastinum was central. Percussion notes were dull in inframammary, infraaxillary and part of axillary area, and infrascapular areas. Breath sounds were decreased over those areas of dullness. There was tenderness over right lower intercostal spaces. Shifting dullness and succussion splash was absent. Abdominal examination revealed liver was just palpable and tender. There were no other abnormalities in abdomen and examination of other systems.

Discussion Clinical findings were suggestive of right sided moderate pleural effusion, probably empyema. The findings in favor of empyema were high-grade fever, toxicity and intercostal tenderness. Sometimes, pain of basal pleurisy can be felt in upper abdomen because of same dermatome supply. In our case, the pain was initially dull aching and not pleuritic. The finding cannot be explained by referred pain of basal pleurisy. Moreover, there was palpable and tender liver suggestive of abdominal pathology. In right sided pleural effusion, liver may be pushed down and palpable but that could not explain tender liver. Abdominal causes of pleural effusion are liver abscess, subphrenic abscess, pancreatitis (usually left-sided or bilateral and rarely may involve right side), peritonitis and any causes of ascities. Transudative right-sided or bilateral pleural effusion with enlarged tender liver may be found in congestive cardiac failure. Absences of congested neck veins, pedal edema and manifestations of cardiac disease excluded that possibility. In cirrhosis of liver, transudative effusion may occur in right side or bilaterally but liver is usually not palpable and nontender. Absence of manifestations of cirrhosis, ascities, venous prominence over abdomen and splenomegaly ruled out cirrhosis. Moreover, presence of pleuritic chest pain suggested exudative pleural effusion. We suspected right sided empyema might be due to some liver or subdiaphragmatic pathology. Anorexia, nausea, extreme weakness and low grade fever might point toward liver pathology.

Step II Routine peripheral blood showed neutrophilic leukocytosis (neutrophils—92% of 14000/mm3 leukocytes), anemia (Hb% 8.6 g/dl) and raised ESR (80 mm in 1st hour). Blood biochemistry revealed only slightly raised liver enzyme (SGPT —62 U/L). His chest X-ray PA view showed (Fig. 68A) right-sided multiloculated

Cases

Fig. 68A: Chest X-ray showing right-sided multiloculated hydropneumothorax

Fig. 68B: Reddish-brown fluid aspirated from pleural space

hydropneumothorax with some heterogeneous opacity in right lower zone. We aspirated fluid and that was typically chocolate color or reddish-brown and thick fluid like ‘anchovy sauce’ (Fig. 68B). We put on intercostal tube in right lower chest in the infraaxillary line. His USG upper abdomen (Fig. 68C) showed a large liver abscess in the right upper quadrant. Our final diagnosis was amebic liver abscess that ruptured into right pleura causing multiloculated hydropneumothorax and pneumonia of right lower lobe.

271

272

101 Cases in Respiratory Medicine

Fig. 68C: USG abdomen showing a large abscess in the right upper quadrant of liver

Discussion Amebic liver abscess is caused by protozoa, Entamoeba histolytica. Trophozoite form of parasites can remain in the large-bowel lumen without causing disease or they can invade the mucosa causing amebic colitis. In some cases, parasites invade mucosa, enter bloodstream and can reach liver through the portal circulation causing amebic liver abscesses. Sometimes, liver abscess burst into pleura or lung causing amebic lung diseases. Amebic lung disease may be (i) pleural effusion, (ii) pleuropulmonary amebiasis, (iii) amebic lung abscess/isolated amebic abscess, and (iv) hepatobilliary fistula. Pleural effusion is a common complication of amebic liver abscess virtually universal. Serous pleural effusion is usually sterile and needs no extra treatment. Empyema can occur due to rupture of amebic liver abscess into pleura that needs aspiration or intercostal tube drainage. In about 10% cases liver abscess may rupture through the diaphragm causing pleuropulmonary amebiasis. Pleuropulmonary amebiasis is a significant complication of amebic liver abscess. It can rarely occur through hematogenous spread. Pleuropulmonary amebiasis may present with sudden-onset cough, pleuritic chest pain, and shortness of breath. It is more common in men (10:1) and is associated with chronic alcohol consumption. In more than 80% cases right lung is involved. It is pathologically characterized by necrosis consisting of trophozoites surrounded by granular eosinophilic material but no leukocytes except at the periphery. Clinical features includes point tenderness over liver; atelectasis at right lung base, empyema and hepatomegaly. Aspirate from pleura or abscess is colorless, orderless, or

Cases

yellow to brown material (anchovy paste). Pleural fluid may demonstrate trophozoites but is usually sterile in culture. Pleuropulmonary amebiasis should be differentiated form bacterial pneumonia and empyema. Hepatobronchial fistula is a rare complication characterized by coughing up copious brown sputum that may contain E. histolytica trophozoites. The diagnosis of amebic liver abscess is based on the detection of a single and large abscess usually located in the right lobe of the liver, in the endemic area. The finding of multiple abscesses is not uncommon. Serological test to detect antibody to E. histolytica is highly sensitive (> 94%) and highly specific (>95%) for the diagnosis of amebic liver abscess. The diagnosis has been traditionally based on demonstration of cysts in stool or demonstration of trophozoites in colonic mucosa, aspirate from liver abscess, pleural fluid or from sputum in hepatobronchial fistula.

Management We treated him with intravenous metronidazole 400 mg 8 hourly for 5 days and then switched over to oral ornidazole 500 mg twice daily for another 2 weeks. We drained pleural fluid by introducing intercostal tube into the pleural space under USG guidance. We gave him symptomatic treatment with antiemetics drug, domperidone 10 mg thrice daily Patient showed clinical response by 5 days and we removed intercostal tube on 14th day. We followed him with chest X-ray, USG upper abdomen and liver function tests. His liver abscess took about a month to regress completely. Follow up X-ray at 2 month showed right-sided pleural thickening.

Discussion Treatment of amebic liver abscess is with metronidazole (750 mg thrice daily orally or intravenously for 5–10 days) or tinidazole (2 g orally per day for 3 days), followed by luminal cysticidal agent. Metronidazol in dose of 750 mg is not tolerated by our patient and we usually give lower doses for longer duration. Tinidazole is usually well-tolerated. We used ornidazole as it has least gastrointestinal side effect. Liver abscesses usually response well without aspiration. Aspiration is needed for secondary bacterial infection, uncertain My Learning diagnosis, left lobe abscess, to prevent It is mandatory to search for liver imminent rupture and patients who failed pathology in right-sided pleural to response to medicines. Pleuropulmonary disease. abscess always needs drainage by aspiration or intercostal tube drainage.

273

274

101 Cases in Respiratory Medicine

Case 69: Encysted Pleural Effusion with Liver Cyst Case Report Step I A 62-year-man presented with chest pain and dyspnea for 20 days. He had sharp chest pain in the right side of chest that increased with respiration and coughing. Subsequently, he developed dyspnea that was progressive. Patient also had dry cough particularly while talking. He was a smoker and smoked about 15 cigarattes per day for more than 40 years. He was a retired government employee. He had no past history of medical or surgical illness. General survey revealed BP—100/86 mm of Hg, pulse—98/min, RR—38/ min, SpO2—90% at room air and patient had signs of dyspnea. Positive findings in respiratory system include bulging of right hemithorax, diminished movement in the right hemithorax, trachea central, apical impulse on left 5th intercostal space 2 cm lateral to mid clavicular line; percussion notes stony dull throughout right hemithorax and absent breath sounds in right hemithorax except diminished vesicular breathsounds in right infraclavicular and suprascapular areas. Breath sounds in left hemithorax was vesicular with prolonged expiration with polyphonic rhonchi. His peripheral blood showed Hb—7.8 g/dl, WBC—6800/mm3, N—76%, L—18%, E—2%,M—3%, B—1% and ESR—96 mm in 1st hour. Blood biochemistry showed fasting sugar—110 mg/dl, urea—46 mg/dl, LDH—136 mg/dl HDL 32—mg/dl, triglyseride 150—mg/dl; bilirubin—1.2 mg/dl, SGPT—50 U/L; SGOT—48U/L and alkaline phosphatase—250 U/L. His X-ray chest PA view (Fig. 69A) showed homogeneous opacity almost covering right

Fig. 69A: Chest X-ray showing right-sided encysted pleural effusion

Cases

whole hemithorax with obliteration of both cardiophrenic and costophrenic angles. There was minimum mediastinum shifting, and the upper border of opacity was convex upwards and medially. Chest X-ray picture was suggestive of loculated pleural effusion. Pleural fluid was aspirated thrice within a week amounting 1000 ml, 800 ml and 900 ml. Pleural fluid was hemorrhagic (plenty of RBC), exudative (protein—4.5 g/dl), lymphocytic (cell count—450/mm3 and lymphocyte—80%) with few atypical cells with hyperchromatic nucleus. He was provisionally diagnosed the case as malignant pleural effussion.

Discussion A 62-year-old man, smoker 30 pack-year presented with massive hemorrhagic right- sided pleural effusion that needed repeated aspiration indicated malignant pleural effusion. Pleural fluid was hemorrhagic, exudative, lymphocytic and containing few atypical cells. All findings were suggestive of malignant pleural effusion. As patient was a smoker, it was predictably secondary to bronchogenic carcinoma. There was no manifestation of lung disease like hemoptysis, expectoration or signs of underlying lung collapse (documented by no or same sided mediastinal shifting), or manifestation of mediastinal involvement. Pleuritic chest pain, as noticed in our case, could not be explained by malignant process. Pleuritic chest pain usually suggested pleural inflammation and could be found in postobstructive pneumonia in malignancy. Dry cough could be explained by pleural irritation or by lung compression. Only finding that was suggestive of benign nature of illness was the encystment of effusion. Next step should to confirm the diagnosis of malignant pleural effusion by confirming the atypical cells as malignant cells. The yeild of blind pleural biopsy is not superior to that of pleural fluid cytology. Malignant pleural effusion can be diagnosed based on identification of primary tumor.

Step II He was referred to a cancer hospital. They reviewed the slides and diagnosed the atypical cells as reactive mesothelial cells. CECT-throax was done. CECT thorax (Fig. 69B) showed a large encysted pleural effusion with pleural fibrosis without mediastinum shifting and with out mediastinal lymphadenopathy. There was a circular filled-up cyst with calcified wall in upper part of right lobe of liver. They sent him to us for confirmation of diagnosis.

Discussion Cytology needs more expertize than histology, as it involves examination of individual cells. Moreover, the differentiation between malignant cell and reactive mesothelial cell is a challenging task, as the latter may have hyperchromatic nucleus. As a second opinion regarding atypical cells was contrary to previous report, they decided to investigate the case further. A

275

276

101 Cases in Respiratory Medicine

Fig. 69B: Coronal section of CECT thorax showing a massive encysted pleural effusion with pleural fibrosis, and a circular calcified cyst in upper part of right lobe of liver

CECT should be the next step; contrast enhancement would be helpful to identify mediastinal lymph nodes or mass by differentiating it from vascular structures, underlying lung mass and metastatic pleural nodules. Fortunately, none of them were found. Moreover, they got pleural fibrosis and encystment that went against malignant effusion. In addition they got a filled-up cyst with clacified margin, that was suggestive of hydatid cyst. Hydatid cysts of liver are usually calcified whereas hydatid cysts of lung are usually not calcified, probably due to continuous movement of lungs. They naturally correlated liver cyst with pleural effusion and thought pleural effusion was nonmalignant.

Step III As we were examining the case we found patient was dyspneic. We put intercostal tube in right 6th intercostal space in anterior axillary line. We sent pleural fluid for malignant cell and examination for scolices. Both of them came as negative. Cytology report again came as presence of reactive mesothelial cells. We decided to start from the begining. Detailed history taking did not give any additional information. We found one small (about 1 cm diameter), firm to hard in consistency, nontender lymph nodes in the medial group of right axillary region. The lymph node was pea-seed in size that can be palpable with difficulty by rolling the skin over chest wall. We convined the

Cases

surgeon to take biopsy and he agreed relunctantly. We repeated CECT thorax after pleural fluid drainage, and we got nothing additional except expected collapse of the underlying lung. We did FOB, and we did not find any pathology up to subsegmental bronchi. Lymph node biopsy report came as poorly differentiated metastatic lymph node.

Discussion Significance of lymph nodes depends on the size, location, consistency and most importantly the clinical setting. A lymph node less than 2 cm diameter are usually considered as nonsignificant. Even more than 2 cm lymph nodes in inguinal region may be found in normal individuals, and that may be significant in cervical region. Any palpable lymph node in axillary area should be considered significant. In our case where clinical manifestation was suggestive of malignant, any palpable lymph nodes should be considered significant and should be investigated. Second question is when to do a CT-thorax. One school suggested before aspiration as pleura and pleural-based nodules will be better visualized. On the other hand after drainage of fluid underlying lungs will be better seen. Third point was the presence of reactive mesothelial cells. It should be remembered that mesothelial cells more than 5% practically excluded TB and perhaps suggested malignancy. Always explore the possibility of malignancies when pleural fluid mesothelial cell count is more than 5%. Moreover, it is often very difficult to differentiate between reactive mesothelial cells from exfolliated malignant cells. Lastly, the question remained where is the primary. In elderly smoker and with no evidence of malignancy elsewhere, we thought the possibility of lung cancer should be considered. FOB has the limitation that it could not reached distal part of lung. Drainage areas of axillary lymph node include upper limb, lung, breast, chest wall and lower part of neck. Clinical examination excluded other possibility and pointed towards lung. Peripheral lung tumor that involve parietal pleura and chest wall can spread to axillary lymph node. The chest pain could be explained by that mechanism though radiologically we did not get any findings suggestive of parietal involvement. Encysted effusion did not exclude malignancy and the liver cyst (most probably hydatid cyst) was an incident finding.

Management Our final diagnosis was poorly differentiated My Learning bronchogenic carcinoma with pleural effusion A meticulous physical examina­ and axillary lymph node metastasis. We tion has no substitute. Axillary referred the case to onchology department of lymphadenopathy is always our hospital and chemotherapy was started. significant. The peculiary of the case was the presence of a liver cyst that confused the scenerio. We referred the case to a general surgeon

277

278

101 Cases in Respiratory Medicine

and he opined against surgical intervention. As there was continuous collection of fluid we could not remove the intercostal tube. Patient did not respond to chemotherapy and died within 2 months.

Case 70: Thoracic Sequel of Past Sin Case Report Step I An 18-year-adolescent boy went to a red light area of Patna and subsequently developed urethral stricture. He came to Kolkata and he was treated with repeated urethral dilatation. He developed fever, cough and right-sided chest pain. Fever was acute in onset, high-grade with chill and rigor. Cough was intermittent and more at night and in lying down position. Cough was not associated with expectoration. Pain was severe and increased with respiration or coughing but not stabbing in character. He was a bachelor, had no addiction, and did not have any significant past or family history. General survey findings were normal except temperature was 104°F. Examination of thorax revealed diminished movement of left hemithorax; trachea shifted to left and apical impulse on left midclavicular line; percussion notes were dull in the inframammary, infra-axillary, infrascapular areas and lower part of interscapular area; and diminished vesicular breath sound over those areas. Shifting dullness and succussion splash were absent. No abnormality was detected in peripheral blood and routine blood biochemistry. His chest X-ray PA view (Fig. 70A) showed homogeneous opacity

Fig. 70A: Chest X-ray showing homogeneous opacity in right mid and lower zones with horizontal fluid level

Cases

occupying right mid and lower zones and an air fluid level. Right costophrenic and cardiophrenic angles were obliterated but there was a clear area just above the right costophrenic angle.

Discussion The boy presented with high fever, cough and chest pain and that indicated that there was acute infection in the thorax. His chest X-ray PA view was suggestive of a large lung abscess with pleural involvement. His clinical manifestations like diminished movement, dull percussion notes, decreased breath sound, mediastinal shifting towards opposite side and most importantly absence of expectoration favored pleural disease. We considered lung abscess with parapneumonic effusion, as that can explain clinical signs as well as his radiological picture. But the diagnosis could not explain the absence of expectoration. An alternate possibility might be encysted pyopneumothorax as the physical findings were in favor of pleural effusion/empyema. Chest X-ray PA view was suggested lung abscess as there was a clear circular thick line particularly the medial margin, with parapneumonic effusion. In pre-CT scan era, it was difficult to differentiated pleural disease from lung parenchymal disease. Sometimes, even today, it is not an easy task to differentiate between lung abscess with parapneumonic effusion from encysted pyopneumothorax with or without bronchopleural fistula. The differentiation is important as the managements of lung abscess and empyema are different. The angles between opacity and pleura are important. In lung parenchymal disease the angles with chest wall will be acute whereas they will be obtuse in pleural disease. When CT scan was not available, we used to rely on lateral view to differentiate between pleural and parenchymal disease. The differentiation is based on the fact that lung abscess is confined within a lobe as a result it cannot cross fissure line. When opacity crosses fissure line (visible or artificially drawn), then it is considered as pleural pathology. Oblique fissure can be drawn artificially by drawing a line connecting 4th intervertebral disk with the junction between anterior third and posterior two-third of dome of the diaphragm.

Step II Chest X-ray right lateral view (Fig. 70B) showed homogeneous opacity posteriorly with sharp convex anterior margin. The right dome of diaphragm was obscured posteriorly with obliteration of posterior costophrenic angle and clear anterior costophrenic angle. The oblique fissure was clearly visible below the homogeneous opacity (arrow in Fig. 70B). The air-fluid level crossed above and below the oblique fissure. We excluded lung abscess and diagnosed the case as encysted pyopneumothorax.

279

280

101 Cases in Respiratory Medicine

Fig. 70B: Chest X-ray right lateral view showing opacity with air-fluid level posteriorly that crossed the oblique fissure line (arrow head)

Discussion I have discussed the issue with many radiologists and they opined that right lateral and left lateral view could not be differentiated by seeing the film. Contrary to our belief the differentiation should be done by seeing the marker put by technician. Identification of domes of diaphragm is important in interpretation of lateral view. Right dome is normally totally visible and anterior part of left dome is not usually visible because of silhouetting with heart. Indistinct posterior part of right dome may be due to collection of fluid or due to consolidation of posterior segment of right lower lobe. Fortunately, we could see the lower part of oblique fissure due to collection of fluid. Oblique fissure is not normally visible. As air fluid level crossed oblique fissure we diagnosed pleural pathology.

Management We put on intercostal tube in the infrascapular area, 950 ml pus came out and we sent pus for culture and drug sensitivity. We started intravenous cefoperazone-sulbactam combination 2 g 12 hourly and amikacin 500 mg twice daily. Pus culture report came as growth of Klebsiella pneumoniae sensitive to cefoperazone. We removed intercostal tube after 10 days and withdrawn antibiotics after 14 days. Follow-up chest X-ray showed the lung was completely expanded, and the only abnormality was obliteration of right costophrenic angle. We referred the boy to urology department and reconstruction of urethra was done. The boy went home completely cured.

Cases

Discussion Treatment of collection of pus (lung abscess, bronchiectasis or empyema) is mainly by drainage and antibiotics. Localization of pus is important for planning drainage methodology. In lung abscess or bronchiectasis drainage can be achieved by postural drainage; whereas empyema needs intercostal tube drainage. Antibiotics should be started empirically as culture report takes 2–3 day time and more importantly the report may come as negative in more than half of the cases. Antibiotic selection should be based on a calculated guess. Assessment of clinical setting is important to determine the expected organism involved. The boy had repeated urethral manipulation and organisms was expectedly come from urinary tract through hematogenous route. We expected Gram-negative bacilli and therefore we started antibiotics that had Gram negative coverage. Our prediction was vindicated latter on by culture report. A direct inoculation of bacteria within pleural space does not cause empyema but can cause septicemia. The pathogenesis of pyogenic empyema thoracic initiates from pneumonia as simple parapneumonic effusion (without invasion of microorganisms). Aspiration of pleural fluid along with antibiotic is the treatment for this condition. Then there was invasion of organism into the fluid causing complicated parapneumonic effusion. Complicated parapneumonic effusion is characterized by low PH 1, patients should be considered for admission. Another scoring system, Pneumonia Severity Index (PSI), is not frequently used due to its complexity. Our patient’s CRB 65 scoring was 2 and we admitted him in chest ward.

Cases

Investigation His total WBC count was 13600/mm3 with neutrophil 78%; fasting blood sugar was 98 mg/dL; urea—36 mg/dL; creatinine—0.8 mg/dL and liver function tests were normal. His chest X-ray PA view (Fig. 75A) showed homogenous opacity in left lower zone and part of mid zone. The left border of heart and left dome of diaphragm were clearly visible. We interpreted the chest X-ray as consolidation involved left lower lobe spearing basal segments. Chest X-ray, left lateral view (Fig. 75B) demonstrated consolidation involved apical segment of left lower

Fig. 75A: Chest X-ray showing homogeneous opacity in left lower and mid zone with clear left border of heart and left dome of the diaphragm

Fig. 75B: Chest X-ray left lateral view showed consolidation with air-bronchogram in apical segment of left lower lobe

295

296

101 Cases in Respiratory Medicine

lobe. Air bronchogram (air in bronchus visible as a result of surrounding consolidation), a characteristic sign of consolidation, was clearly visible in the lateral view. His SpO2 was 89% with room air.

Discussion The clinical pictures were classical of typical pneumonia, usually caused by Streptococcus pneumoniae. A single chill strongly suggest pneumococcal pneumonia. The classical picture of typical pneumonia is now rarely found due to overuse of antibiotics. On the other hand, organisms usually responsible for atypical pneumonia like Mycoplasma, Ligeonella, etc. may present as typical pneumonia. Atypical pneumonia is characterized by less severe course, more extrapulmonary manifestations and less pulmonary manifestations. Atypical pneumonia is usually caused by Mycoplasma pneumoniae, Coxiella burnetii, Chlamydia psittaci and Legionella pneumophilia. The terminologies typical and atypical pneumonia have lost their clinical relevance. Pneumonia is now classified as communityacquired pneumonia (CAP), hospital-acquired pneumonia (HAP) and healthcare-associated pneumonia (HCAP). CAP is defined as pneumonia occurring in patients at community or within 24 hours of admission in hospital. HAP is defined as pneumonia occurring after 24-hour of hospitalization. HCAP is a bacterial infection that occurs in people who live in long-term care facilities or who receive care in outpatient clinics, including dialysis centers. Like HAP, HCAP can be caused by bacteria that are resistant to antibiotics. The causes of CAP are Streptococcus pneumoniae, Haemophilus influenzae, Moraxella catarrhalis and Mycoplasma pneumonia. Less common organisms are Staphylococcus aureus, Klebsiella pneumoniae, Pseudomonas aeruginosa, Legionella species, Chlamydia pneumoniae, etc. In the management of pneumonia, focus should be to identify unusual microorganisms, unexpected resistance to antimicrobials, or host immune disorders, general debility, poor immune status and poor social support. In history taking, following things are to be given emphasis like smoking, diabetes, COPD, malignancy, HIV, alcohol and other addiction, immunosuppressant therapy. Important epidemiologic clues (illicit drug use, environmental factors, vulnarable profession, leisure, pets, foreign travel, etc.) must be searched. Physical examination should be directed to assess initial severity of pneumonia and particular emphasis should be given to find out the presence of complications and comorbidities. In admitted cases investigation should be directed to identify the causative organism by sputum examination, including Gram stain, Ziehl-Neelsen stain and immunofluorescence stains for Legionella. Cultures for conventional bacteria (quantitative/semiquantitative bacterial cultures) should be done from representing sputum. Sputum that represents lower respiratory tract can be identified by the presence of more than 25 neutrophils and less than 10 squamous epithelial cells per low power field. Two sets of blood culture to be sent in all admitted patients. In suspected cases, urine for Legionella

Cases

antigen should be send. Routine blood examination should include total and differential count of WBC, blood sugar, urea, creatinine, electrolytes, liver function test, C-reactive protein and HIV serology. Pulse oximetry is used to identify hypoxia, and arterial blood gas analysis is reserved for patients with room-air hypoxia. If pleural effusion is present, pleural fluid should be aspirated and examined.

Management His sputum demonstrated Gram-positive cocci and sputum culture showed no growth. Our patient was suffering from CAP without comorbidity and complication. He had no past history of antibiotic therapy and he was treated in non-ICU set-up. We started with amoxicillin–clavulanic acid combination 1.2 g thrice daily intravenously along with oral clarithromycin 500 mg twice daily. On the third day, patient became clinically stable and co-amoxyclav was switch over to oral route and continued for 7 days. Patient was discharged on the 8th day in healthy state.

Discussion Management of CAP includes management of comorbidities and complications, and antibiotics. Antibiotics should be started immediately after sending samples to the laboratory and should not be delayed beyond 2 hours of admission. Antibiotic selection is an art of calculated guess. In general, choice of antibiotics depends on: (i) probable responsible organism (suspected on the basis of clinical setting and on the basis of local epidemiological data), (ii) probable drug resistance pattern (based on local data and history of previous antibiotics use), (iii) associate factors like diabetes, renal disease, liver disease, etc., (iv) switch over therapy (same molecule that can be given by both parenteral route and oral route is preferable), and lastly, (v) the cost of medicine (it should be a two way calculation considering not only the cost of antibiotics but also the cost of prolonged morbidity and hospital stay). Antibiotics selection should depend on local epidemiology and local drug sensitivity pattern. Fortunately, Indian guideline for management of pneumonia is now available.

Management of Pneumonia as per Indian Guideline Antibiotics for outpatient treatment: In patients with no comorbidities, previously healthy with no risk factors for drug-resistance: we can use oral macrolides e.g. azithromycin 500 mg per oral (PO)/clarithromycin 500 mg PO twice daily or oral β-lactams (amoxicillin 500–1000 mg PO thrice daily).There is insufficient evidence to recommend tetracyclines. In patients with history of receiving prior antibiotic within 3 months: azithromycin/clarithromycin plus  amoxicillin 0.5–1 g PO thrice daily/amoxicillin-clavulanate 625 mg PO thrice a day/respiratory fluoroquinolone (e.g. levofloxacin 750 mg PO daily or moxifloxacin 400 mg PO daily). Fluoroquinolones should not be used for

297

298

101 Cases in Respiratory Medicine

empiric treatment particularly in our country and may be used only if it is absolutely necessary and after excluding tuberculosis. Moxifloxacin should be kept reserved for extensively drug resistant tuberculosis (XDR-TB). When comorbidities are present (e.g. alcoholism, bronchiectasis/cystic fibrosis, COPD, intravenous (IV) drug user, post-influenza, asplenia, diabetes mellitus, lung/liver/renal diseases), we can give levofloxacin/moxifloxacin; or combination of a beta-lactam (amoxicillin/amoxicillin-clavulanate/ cefuroxime 500 mg PO twice daily) plus  a macrolide (azithromycin/ clarithromycin). In patient admitted in non-ICU ward: The recommended regimen is a combination of a β-lactam plus a macrolide (preferred β-lactams include cefotaxime1 g thrice daily IV, ceftriaxone 1–2 g twice daily IV and amoxicillin– clavulanic acid 1.2 g thrice daily IV. In patients where there is a risk of aspiration pneumonia/anaerobic lung infection/lung abscess, we can start with clindamycin 300–600 mg PO 8 hourly/ampicillin-sulbactam 3 g IV 6 hourly/ ertapenem 1 g IV daily or ceftriaxone 1–2 g twice daily IV plus metronidazole 500 mg IV 6 hrly. When methicillin resistant Staphylococcus aureus (MRSA) is suspected, we can start with vancomycin 15 mg/kg IV 12 hourly/linezolid 600 mg IV/PO 12 hourly (Linezolid should be avoided as a first line antibiotic). In the uncommon scenario of hypersensitivity to β-lactams, respiratory fluoroquinolones (e.g. levofloxacin 750 mg daily) may be used if tuberculosis is not a diagnostic consideration at admission. Route of administration (oral or parenteral) should be decided based upon the clinical condition of the patient and the treating physician’s judgment regarding tolerance and efficacy of the chosen antibiotics. Switch to oral from intravenous therapy is safe after clinical improvement in moderate-to-severe CAP. In patient admitted in ICU ward: The recommended regimen is a β-lactam (cefotaxime, ceftriaxone, or amoxicillin–clavulanic acid) plus a macrolide for patients without risk factors for Pseudomonas aeruginosa. When P. aeruginosa is an etiological consideration, an antipseudomonal antibiotic (e.g. cefepime 1–2 g IV two to three times a day, ceftazidime 2 g IV thrice daily, piperacillin– tazobactam 4.5 g IV four times a day, cefoperazone–sulbactam 1–2 g IV twice daily, imipenem 0.5–1 g IV three to four times a day or meropenem 1 g IV thrice daily) should be given. Combination therapy may be considered with addition of an aminoglycosides/antipseudomonal fluoroquinolones (e.g. ciprofloxacin). Duration of therapy: Antibiotics should be given for a minimum of 5 days for outpatient and 7 days for inpatient treatment. Objectives of treatment should be to achieve clinical improvement within 48–72 hours and hemodynamic stability within a period of at least 48 hours. Longer duration of therapy may be needed if patients have delayed response or if there are complications like lung abscess, empyema, extrapulmonary spread of infections, etc. Outcome of treatment: The median time to clinical stability (temperature ≤37.2°C, heart rate 90 mm of Hg, respiratory

299

Cases

rate ≤24/min and SPO2 > 90%) in treated CAP patients is 3–5 days and maximum 6 days. But radiological clearance takes longer time, 50% resolution of radiographic opacities usually occurs in 2–4 weeks, and total resolution occurs within 4–8 weeks. Radiographic resolution may take up to 12 weeks in patients with extensive disease or with underlying lung disease.

My Learning In todays era of antibiotics, the classical typical pneumonia is a rarity; it is useless to differentiate between typical and atypical pneumonia, and it is prudent to follow Indian guideline for management of pneumonia.

CASE 76: A YOUNG LADY WITH TOXEMIA CASE REPORT Step I An 18-year-old lady presented with fever, cough and chest pain for 10 days. She was too sick to tell her story. History was taking from patient and her in-laws. As per her in-laws, she had high-grade intermittent fever that came with chilling sensation and rigor that required covering her with blankets. Fever remitted with drenching sweating. Fever responded temporarily with antipyretic medicine. She had frequent cough throughout days and night, and that was disturbing her sleep. There was little serous expectoration. She had nonspecific dull aching chest pain. Her condition deteriorated rapidly. She became dyspneic and became bed-ridden. She stopped taking food for last two days. She had no known past history. She had no addiction. She had two children and younger one was one year old. She was treated by a local quack. As her condition was deteriorating, she was brought to our hospital by her in-laws. She was very toxic, prostrated and cachectic. She was conscious but confused and disoriented. She had BMI less than 20 and she had severe anemia with puffiness of face. Her BP was not recordable; pulse was 130/min and feeble; RR was 40/min and temperature was 104°F. Respiratory system revealed fine crepitations scattered over both lung fields. Her abdomen was diffusely tender without hepatosplenomegaly. Her peripheral blood showed WBC 34000/mm3 with neutrophils 96% containing toxic granules; Hb% was 5 g/dL and ESR was 140 mm in first hour. Malaria parasite was not found in thick and thin blood film. We sent blood and urine for culture and sensitivity. We started resuscitation measures with moist oxygen inhalation, intravenous normal saline 500 mL in jet followed by 500 mL 6 hourly; and injection hydrocortisone 100 mg 8 hourly. We started parenteral antibiotics (coamoxyclav 1.2 g 8 hourly and gentamicin 500 mg 12 hourly).

Discussion I saw the case in 1988, when I was a senior resident. At that time, there was no respiratory intensive care unit and the concept of critical care was emerging. Patient was very toxic and frankly speaking she was in a terminal stage. Her in-laws took her to hospital as a last resort. Fever with chill and rigor usually signifies that the organism (virus, bacteria or parasite) has

300

101 Cases in Respiratory Medicine

entered into bloodsteam. The common causes of fever with chill and rigor in our country are malaria and urinary tract infection. Severe respiratory tract infection with or without sepsis can cause fever with chill and rigor. Low BP in infection is suggestive of septic shock. Fall of BP is due to shifting of fluid from intravascular compartment to extravascular compartment. In our case, physical findings were suggestive of respiratory tract infection. Patient was in shock as a result of severe infection, and we diagnosed the case a septic shock. We managed her with hospital available medicines and antibiotics. Patient’s relatives were too poor to purchase medicines from outside. I should confess that the antibiotics and treatment given to her were not adequate.

Step II Her BP stabilized and rose to a recordable level 90/50 mm of Hg. We gave her two units of whole blood transfusion. We sent her to radiology department. A chest X-ray was done. The chest X-ray (Fig. 76) showed multiple findings, including opacity in right lower zone with obliteration of right costophrenic angle; cavities with fluid level in right mid zone and left mid zone. With careful examination, we could detect a thin wall cyst in left lung at the border of upper and mid zone. X-ray findings were suggestive of bilateral consolidation, multiple lung abscesses, right-sided pleural effusion and a pneumatocele in the left upper zone.

Discussion Her X-ray showed plenty of findings involving both lungs suggestive of hematogenous spread of infection. Bilateral lung involvement with

Fig. 76: Chest X-ray showing bilateral consolidations, multiple cavities with fluid level, right-sided pleural effusion and a pneumatocele in the left upper zone (arrow heads)

Cases

consolidation and cavities; pleural involvement and pneumatocele were suggestive of staphylococcal infection. Contrary to Gram-negative septicemia, Gram-positive septicemia has a lesser rapid course and its response to treatment is better. In general, Gram-positive septicemia has lesser worse prognosis than Gram-negative septicemia.

Final Outcome We lost the patient on the third day after admission. The reason was obviously inadequate treatment. As expected, her blood culture report came as growth of Staphylococcus aureus and urine culture report came as no growth. Just before her dead, I unearth the most shocking fact about the case that made me remember My Learning the case even today. She, at the age of 18 Criminal negligence by patient’s years, had a nonmedical abortion of her third relative is a social reality, and pregnancy. Probably that was the cause of her that can be concealed till the last severe anemia and pneumonia. Her in-laws movement. suppressed the fact till the eleventh hour.

CASE 77: UNRESOLVED PNEUMONIA CASE REPORT Step I A 15-year-old village boy (student) presented with high rise of temperature with chill and rigor, cough with scanty expectoration, nonspecific chest pain, and weakness. He consulted a local physician and was treated with co-amoxyclav 625 mg 8 hourly along with supportive medicines. As he was not improving, levofloxacillin 500 mg daily was added. After 10 days of local treatment, his father brought him to Kolkata for better treatment, as patient’s condition deteriorated.

Examination He was very toxic; his temperature was 100.6°F; BP was 100/60 mm of Hg; pulse was 120/min (regular); respiratory rate was 36/min (abdomino/thoracic) and SpO2 was 88 with room air. Examination of thorax revealed crepitations with harse vesicular breath sound in right mammary and inframammary areas. Other systems were essentially normal.

Investigation His WBC count was 12000/mm3 with 94% neutrophil, blood sugar (fasting) was 86 mg/dL, urea—32 mg/dL, creatinine—0.8 mg/dL, bilirubin—0.96 mg/ dL, SGPT—64 U/L, SGOT—58 U/L and alkaline phosphatage was 238 U/L. Sputum was negative for Gram stain and Z-N stain. Urine routine and culture was normal. Blood culture showed no growth. Mantoux test with 5TU was 6 × 6 mm induration. HIV serology was negative. His chest X-ray PA view (Fig. 77A)

301

302

101 Cases in Respiratory Medicine

Fig. 77A: Chest X-ray showing consolidation in middle lobe

showed homogeneous opacity in the right paracardiac area that obliterated right heart border suggestive of consolidation of middle lobe.

Treatment We admitted him in our in-patient department. We started cefoperazonesulbactum intravenously 1 g 8 hourly with clarithromycin 500 mg twice daily orally along with moist O2 inhalation 2 L/min, intravenous fluid and other supportive treatment. Patient was not responding to treatment and his condition was deteriorated further. We categorized the case as nonresponding/progressive pneumonia.

Discussion In CAP, the median time to clinical stability (temperature ≤37.2°C, heart rate 90 mm of Hg, respiratory rate ≤24/min and SPO2 >90%) is 3–5 days and maximum 6 days. Radiological clearance takes longer time, 50% resolution of radiographic changes usually occurs in 2–4 weeks, and total resolution occurs within 4–8 weeks. Radiographic resolution may take up to 12 weeks in patients with extensive disease or with underlying lung disease. Unresolving pneumonia is defined as pmeumonia with a slow resolution of radiological infiltrates and clinical symptoms despite adequate antibiotics (10–14 days) treatment. Slowly resolving pneumonia is defined as either less than 50% clearing at 2 weeks or less than complete clearance at 4 weeks in a patient who has symptomatic improvement with antibiotic therapy. Non-responding pneumonia is defined as absence of clinical response to antibiotics after 3–5 days of treatment. Progressive pneumonia is defined

Cases

as an increase in radiographic abnormalities and clinical deterioration during the first 72 hours of treatment. Approximately 10–15% of hospitalized patients with community acquired pneumonia do not adequately respond to empirical treatment, and another 6% of nonresponders have progressive pneumonia.

Approach to Nonresponding/Progressive Pneumonia When patient does not respond to treatment we should consider following issues step by step: 1. We should review the diagnosis of pneumonia and should exclude pneumonia mimics. It is noteworthy to mention here that in our country tuberculosis is the most common cause of unresolved pneumonia, accounts for approximately 35.7% cases. Malignancies are the second important cause (11–27% cases). Pneumonia mimics (organizing pneumonia, lupus pneumonia, chemical pneumonia, etc.) are important cause, and they should be excluded. 2. Noncompliance and wrong drug prescription are second important factor. Whether proper antibiotics in proper doses are given as per local epidemiological and sensitivity pattern? 3. We should search for the complications that may delay response to antibiotics like necrotizing pneumonia, lung abscess, empyema, local or metastatic spread of infection and superinfection (infection by another organism commonly hospital acquired over community acquired pneumonia). 4. We should exclude associate factors those may delay clinical response like advanced age, smoking, diabetes, HIV and other immunosuppressive conditions, COPD, malignancy, alcoholism and multilobar pneumonia. Response to antibiotics depends on lobes involved, pneumonia severity index, complications and associate conditions. 5. Next step would be to consider uncommon micoorganisms like Pseudomonas, Klebsiella and enteric Gram-negative bacilli; Mycobacteria, Nocrardia species, anaerobes, fungi, P. jiroveci, etc. Psudomonas was found to be responsible for about 10% of cases of nonresponding CAPs as a result of either persistent infection or subsequent nosocomial superinfection. Antibiotic adjustment should be the next step with coverage to P. aeruginosa, MRSA, and the endemic flora. Coverage to Aspergillus is necessary in immunosuppressive conditions or in patients with structural lung damage.

Step II In our patient chest X-ray after 1 week (Fig. 77B) showed radiological deterioration with increase in the area of consolidation. There was no complication, no associate factors and patient was compliant to medications. There was single lobe involvement, pneumonia severity index was low and

303

304

101 Cases in Respiratory Medicine

Fig. 77B: Chest X-ray after 1 week showing radiological deterioration

he was not immunosuppressed. Under that condition we should suspect pneumonia caused by uncommon organisms. So, we adjusted antibiotic by adding vancomycin 750 mg twice daily intravenously for MRSA coverage.

Chest Radiology Chest imaging is most important step in the evaluation of nonresponding pneumonia. Chest X-ray is useful for detecting pleural effusion, cavitation, mediastinum involvement and new or progressive infiltrates. Chest computed tomography (CT) can provide a more detailed study of the parenchyma, interstitium, pleura and mediastinum. It can also demonstrate specific sign and nonspecific signs like halo sign, ground-glass opacity, nodular opacity, tree in bud opacity, multiple masses with or without cavitation, diffuse or mixed interstitial infiltrates and lymphadenopathy.

Step III Chest X-ray done after 2 weeks (Fig. 77C) showed right-sided pleural effusion and right paratracheal opacity. USG right hemithorax confirmed the presence of pleural effusion. Pleural fluid was lymphocytic exudative with borderline ADA (36 U/L). CECT thorax (Fig. 77D) showed subcarinal and paratracheal lymphadenopathy with areas of central low attenuation. Consolidation of middle lobe had spreaded to right lower lobe, and there was small pleural effusion on right side. Mediastinal lymphadenopathy may be seen in malignancies, lymphoma, sarcoidosis and many other conditions but central necrosis is not a feature of those conditions, and in our country, the usual cause is tuberculosis. Our provisional diagnosis was tuberculosis and we planned transbronchial fine needle aspiration (TBNA) for confirmation of diagnosis. Patient’s father

Cases

Fig. 77C: Chest X-ray after 2 weeks showed appearance of right-sided pleural effusion with right paratracheal opacity

Fig. 77D: CECT-thorax showing right paratracheal and subcarinal lymphadenopathy

refused invasive procedures. We started antitubercular chemotherapy with isoniazid 300 mg, rifampicin 450 mg, ethambutol 800 mg and pyrazinamide 1000 mg daily. He was discharged with advice to come after 2 weeks for followup. Patient came earlier after 7 days as patient’s condition deteriorated. We measured weight and he lost 10 kg body weight from his weight at first visit. We admitted him and convinced them for invasive procedures. At that time, we got both his liver and spleen just palpable.

Invasive Procedures Invasive techniques are recommended when simpler procedures do not provide a diagnosis in unresponding pneumonia. Fiberoptic bronchoscopy (FOB) is usually done to exclude proximal obstruction by growth, foreign

305

306

101 Cases in Respiratory Medicine

body, stricture or external compression. Brochoalveolar lavage should be sent for Gram stain, Z-N stain, malignant cell, culture and other investigations. Protected sheath brushing (PSB) can help in identifying causative organism as it excludes upper airway commensal organism. Bronchoscopic lung biopsies are seldom useful for establishing a bacteriological diagnosis but they are sometimes useful in the diagnosis of TB, fungal infection, sarcoidosis and pneumonia mimics. Open lung biopsy is rarely done and are best indicated when other diagnostic methods have failed and in immune-suppressed patients where a specific diagnosis may be lifesaving.

Step IV We did FOB and TBNA from subcarinal lymph nodes. FOB (Fig. 77E) showed whitish cheesy material in the posterior wall of trachea just above carina and a wide carinal angle. Histopathological examination of both biopsy from cheesy material and TBNA (Fig. 77F) showed intracellular cysts suggestive of histoplasmosis. USG abdomen showed hepatosplenomegaly without ascites or adrenal gland involvement. Adrenal gland involvement is a common finding in dissiminated histoplasmosis. As there were involvement of lung parenchyma, pleura, mediastinum and hepatosplenomegaly we diagnose the case as progressive disseminated histoplasmosis (PDH).

Discussion Histoplasmosis is caused by a fungus Histoplasma capsulatum. The disease is usually found in North America and rare in India. The disease has many clinical and radiological similarities with tuberculosis. Human beings

Fig. 77E: Fiberoptic bronchoscopy showing ulceration with whitish material in the posterior wall of trachea

Cases

Fig. 77F: High-power view of histopathology of bronchoscopic biopsy showing intracellular cysts suggestive of histoplasmosis

acquire the disease following inhalation of microconidia. Microconidia are engulfed by alveolar macrophages. Within macrophase they are transformed into budding yeasts. They spread to local draining lymph nodes and then they spread hematogenously throughout the reticuloendothelial system. Cell-mediated immunity developed by 2 weeks and T cells produce gamma interferon, interleukin 12, tumor-necrosis factor alpha, etc. Ultimately there is granuloma formation those ultimately fibrosed and calcified. Calcified mediastinal lymph nodes and hepatosplenic foci are frequently found in healthy individuals from endemic areas. In patient with impaired cellular immunity, the infection disseminates forming PDH with involvement of bone marrow, spleen, liver, adrenal glands, and mucocutaneous membranes.

Clinical Manifestation The clinical spectrum of histoplasmosis depends upon amount of fungal exposure and immune response of host. Patient may be asymptomatic; or may have flulike illness; rheumatologic symptoms, including erythema nodosum and pulmonary manifestations (pneumonitis, pulmonary infiltrates, hilar or mediastinal adenopathy, large mediastinal masses). Mediastinal mass may rupture into bronchus causing fistula. Chronic cavitary histoplasmosis occurs in smokers and patients with structural lung diseases and they present with productive cough, dyspnea, low-grade fever, night sweats and weight loss. Chest X-ray usually simulates tuberculosis (upper-lobe infiltrates, cavitation, and pleural thickening). Fibrosing mediastinitis may be unilateral or bilateral that can cause superior vena cava syndrome, obstruction of pulmonary vessels, and airways.

307

308

101 Cases in Respiratory Medicine

Progressive Disseminated Histoplasmosis Progressive disseminated histoplasmosis (PDH) is typically seen in immunocompromised individuals (AIDS with CD4+ T cell count 3 cm diameter or tumor of any size with any of the following: (i) visceral pleural invasion, (ii) atelectasis of less than entire lung, and (iii) proximal extent at least 2 cm

Cases

from carina. 7th Edition defined T2: as tumor size >3 cm but 3 cm but 5 cm but left). The lung window of CT-thorax (Fig. 88B) confirmed hyperlucency in right upper lobe and right hilar enlargement. I detected few nodules in the left upper lobe. Mediastinal window showed right hilar lymphadenopathy. FOB was done and it showed a mass lesion in right upper lobe bronchus occluding almost two-third of its lumen extending to right main bronchus. Tumor was more than 2 cm away from carina. I took biopsy from that lesion, and the report came as adenocarcinoma of lung.

Cases

Fig. 88A: Chest X-ray showing hyperlucency in right upper zone with prominence of right hilum

Fig. 88B: HRCT-thorax showing air-tapping in right upper lobe with right hilar mass and few nodules in left upper lobe

345

346

101 Cases in Respiratory Medicine

Discussion Interpretation of chest X-ray depends on the clinical setting. When examination findings suggest the possibility of malignancy, we must be very careful in interpretation of X-ray. In our case, localized rhonchi in right infraclavicular area suggested partial bronchial obstruction of right upper lobe. So, I carefully looked at right upper lobe and found increased lucency. That finding again pointed toward right hilum and I got right hilar heaviness that anybody could have missed easily. CT-thorax findings corroborated with my X-ray interpretation. In addition, I got few nodules in left lung field suggesting metastases. Pulmonary nodules should be differentiated from round opacities produced by end-on view of pulmonary artery. Pulmonary artery is always accompanied by a branch of bronchus and arterial opacity always have an accompanying airway shadow (ring like). Lung nodules are diagnosed by absence of adjacent ring opacity caused by accompanied bronchus.

Management FOB confirmed adenocarcinoma of lung. As there were metastases of the opposite lung the disease was in stage IV and no further investigation for staging was necessary. We had no choice but to treat her with palliative chemotherapy.

My Learning Meticulous clinical examination is an essential step and the finding of localized monophonic rhonchi in adult is an ominous sign.

CASE 89: MAN WITH CHEST WALL MASS CASE REPORT Step I A 46-year-old man presented with left-sided chest pain for 20 days. Patient stated that he had pain in the upper part of left side of chest. The severity of pain increased rapidly and now he was unable to perform his normal activities. Chest pain was disturbing his sleep. The pain was persistent and only temporarily relieved by analgesics. Chest pain used to increase with respiration, movement and coughing. He noticed bulging of upper part of left side of chest for last 1 week. Patient had cough over years with mucoid expectoration. Patient also noticed shortness of breath during exertion for about 6 months. There was no history of hemoptysis, fever or wheeze. He denied any past medical or surgical illness. He was a farmer. He smoked more than 20 biries per day for more than 20 years. He had 3 sons and 2 daughters and all of them were healthy. He did not have significant family history. General survey revealed BP—160/90 mm of Hg, pulse—86/min, RR— 24/min, 2° clubbing and most strikingly he was screaming with pain. There was no peripheral lymphadenopathy. When he put off his shirt, we noticed

Cases

a diffused swelling in the left infraclavicular area that was very tender, and patient was not allowing examination over the swelling. Respiratory system examination showed decreased movement of left hemithorax and trachea was shifted towards right. We got diminished breath sounds with no added sound over left infraclavicular area. Examination of other areas of lungs revealed normal percussion notes, vesicular with prolong expiration and no added sound. Examination of other system was normal.

Discussion The patient had clinical evidence of COPD, including long history of cough, mucoid expectoration, shortness of breath, smoking history, and bilateral vesicular breath sound with prolong expiration. Over that background he developed persistent, progressive and severe chest pain suggestive of malignancy. The chest pain was localized to infraclavicular area that increased with movement, coughing and respiration indicating chest wall pain. Painful swelling over the chest wall was suggestive of tumor either arising from chest wall or arising from lung protruding though chest wall or metastasis to ribs from malignancy of lung or other sources. There was no symptom suggestive of tumor arising from airways or lung parenchyma. The presence of clubbing and history of smoking were in favor of bronchogenic carcinoma. Shifting of trachea to opposite side, in absence of pathology in same side, was suggestive of large tumor in left upper lobe pushing trachea.

Step II We admitted the patient. We started symptomatic treatment with pain killers (tramadol and paracetamol tablet), alprazolam 0.25 mg at night along with management of COPD. We started investigating the case. Peripheral blood and routine blood biochemistry were within normal limits. Chest X-ray PA view (Fig. 89A) showed homogenous opacity with sharp inferolateral margin and ill-defined upper margin. The medial border of the opacity could not be delineated. There was left hilar prominence. Trachea was shifted to right with ill-defined right paratracheal opacity; finding could be explained by rotation. CECT-thorax (Fig. 89B) showed a large well defined mass lesion with central areas of less attenuation that protruded into anterior thoracic wall causing destruction of adjacent ribs. The mass also pushed and compressed mediastinum toward right side. Left hilar lymphadenopathy was observed in hilar level cuts. Both FNAC and true cut biopsy was taken from the chest wall mass. FNAC report came on the next day as non-small cell lung cancer.

Discussion Chest X-ray of the patient was suggestive of left lung mass that shifted upper mediastinum to the right with left hilar prominence suggestive of lymphadenopathy. Erosions of the anterior ends of ribs are very difficult

347

348

101 Cases in Respiratory Medicine

Fig. 89A: Chest X-ray showing a homogeneous opacity in left midzone with sharp inferiolateral border

Fig. 89B: CECT-thorax showing a heterogeneous mass in left hemithorax protruding through chest wall

to detect due to presence of cartilage. CECT thorax delineated the extent and inhomogeneity of tumor, and hilar lymphadenopathy. CECT thorax is essential to identify mediastinal involvement. The tumor was classified as T3

Cases

as tumor size was more than 3 cm and it invaded the chest wall; N1 as there was enlargement of left hilar lymph node without involvement of mediastinal lymph nodes; and M0 as clinically there was no evidence of local or distant metastasis. FNAC needs more expertise as only cells are examined but it has the advantage of early reporting. Biopsy has the advantage of availability of tissue there by giving more concrete diagnosis, but it takes longer time. Moreover, sometimes we need immunohistochemistry and that needs tissue samples. It is always better to have tissue for histopathological examination. Squamous cell carcinoma are usually central in location, but there are some squamous cell carcinoma of lung those are essentially peripheral like pancoast tumor.

Management We referred the case to oncology department for palliative radiotherapy. Biopsy report came on 7th day as squamous cell carcinoma of lung. The anatomical staging was T3, N1, M0; and that was staged as stage IIIA. We started chemotherapy 6 cycles with cisplatin and paclitaxel along with local palliative radiotherapy.

Discussion Staging of lung cancer is rapidly changing. In our case with T3N1M0 tumor the staging was IIIA as per TNM 6th edition but it would be stage IIIB as per TNM 7th edition. In stage IIIA, there is a scope for surgery followed by adjuvant chemo/radiotherapy. But as there was extensive chest wall invasion, we decided to start chemotherapy along with palliative radiotherapy to address My Learning his severe local chest pain. Other indications Peripheral bronchogenic carci­ of palliative radiotherapy are superior vena noma may invade chest wall and caval syndrome, uncontrolled hemoptysis, presents as a parietal mass. unmanageable dyspnea, etc.

CASE 90: MAN WITH AMAZING CHEST WALL SWELLING CASE REPORT Step I One day in my busy outpatient department a 45-year-man came with severe chest pain. I had little time to spend and I started from his chest X-ray. His chest X-ray PA view (Fig. 90A) showed a homogeneous opacity in the right midzone and a heterogeneous opacity in the left lower zone. Right lung opacity had sharp medial margin, lateral margin merged with chest wall and the angles between opacity and chest wall were obtuse. I was sure that the opacity was arising from chest wall. I asked the patient to put off his shirt and I found a swelling over right scapula. I was satisfied with my radiological expertise and admitted him in our in-patient department.

349

350

101 Cases in Respiratory Medicine

Fig. 90A: Chest X-ray showing homogeneous opacity in right mid zone with obtuse angles with chest wall and rib erosions; and a heterogeneous opacity in left lower zone with left hilar enlargement

In the afternoon, I went to ward and gathered a small assemble of junior doctors and started discussion on the significance of ‘D’ sign in chest X-ray. After finishing my lecture, I asked the patient to put off his shirt. To my utter surprise, I found no swelling and I was bewildered. I enquired and found that one junior doctor had aspirated the mass and it was a collection of bloody material. I asked the junior doctor to send the material to pathology department for cytological examination.

Discussion The differentiation between peripheral lung mass and parietal (pleural and chest wall) mass is an important step in respiratory medicine. Chest wall mass can easily be seen by physical examination after total exposure of chest. The differentiation between pleural mass and peripheral lung mass is always challenging. Clinical features like dullness and absent/diminished breath sounds may be found in both conditions. Radiologically, lung mass usually has acute angles with parietal wall and sometimes the margin of mass may be undelineated from lung parenchyma. Pleural mass usually has a sharp medial margin and obscured lateral broader with obtuse angles at the junction with parietal wall. In crowed outpatient department, I did not care to examine the chest wall swelling probably I was captivated with my radiological interpretation skill. The energetic junior doctor examined the swelling and found it as a fluctuating mass and aspirated the mass and found it as a blood filled collection. It is always prudent to send the aspirate to pathology for cytological examination.

Cases

Step II The junior doctor had taken detailed history of the case. As per patient, he had severe chest pain in the right side for more than 2 weeks that disturbed his sleep and normal activities. Chest pain did not respond to analgesics. Patient had cough for about 1 month that he ignored as smoker’s cough. He denied any history of dyspnea but he stopped his work completely for last 2 weeks due to chest pain. There was no history of hemoptysis. He was an agricultural labor and he took more than 15 biris per day for more than 25 years. There was no other significant present, past, personal and family history. General survey did not show any significant positive findings, and there was no peripheral lymphadenopathy. Respiratory system examination revealed bilateral vesicular breath sounds with prolonged expiration with few rhonchi and tenderness over right side of chest in the back. Detailed examination of chest X-ray showed erosions of right 6th and 7th ribs; and left hilar prominence with heterogeneous opacities in left lower lobe. Left dome of the diaphragm was elevated. The fluctuating swelling in right chest wall reappeared on the third day.

Discussion Persistent, progressive and severe right-sided chest pain was suggestive of malignant pain. Chest X-ray showing multiple rib erosions were confirmatory of malignancy. A parietal mass presented as blood filled collection was suggestive of chest wall metastasis. We searched for the primary. Then I focused on the opposite lung and found ill-defined heterogeneous opacities and a definite left hilar enlargement. In same X-ray, as I examined it at different times I got different findings. In fact, I got findings as I was searching. Eyes cannot see what mind does not know. At first I was besieged with a mass lesion and I concentrated on the location of the lesion. Then I got history of severe chest pain and I searched for rib erosions. Lastly, I was searching for the primary site and got lesions in the opposite lung. The method of examining chest X-ray may be global, as I was doing, and systemic view. Naturally systemic view is preferable where all structures are systematically examined. There is no definite rule. I usually start from parietal soft tissues and extrathoracic areas as abnormalities in those areas are commonly missed. Then one-by-one I examine chest wall including ribs and vertebrae, trachea, heart (size, position and chamber enlargement), the diaphragm with angles. Then I look at lung parenchyma, examining and comparing each zones separately. Lastly, I search for abnormalities in hila as hila are the most difficult area.

Step III We advised CECT-thorax that revealed left hilar lymphadenopathy. To our surprise we found a cavity with thick irregularly undulated inner wall in

351

352

101 Cases in Respiratory Medicine

Fig. 90B: CT-thorax showing a thick-walled cavity with undulated inner margin near left hilum

left lower lobe (Fig. 90B). There were few ipsilateral lung nodules in the left lung. Report of cytology of bloody material from parietal mass came as clusters of malignant cells. We did fiberoptic bronchoscopy and detected an irregular mass in the left lower lobe bronchus. We took biopsy and that confirmed squamous cell carcinoma. Our final diagnosis was left lower lobe cavitary non-small-cell lung cancer with left hilar lymphadenopathy, ipsilateral lung metastases and contralateral chest wall metastasis.

Discussion Lung has no pain fiber. Chest pain in lung cancer occurs when there is parietal or mediastinal involvement. Lung has enormous physiological reserve as a result dyspnea appear late due to either pleural effusion or lymphangitic spread. About two-thirds of NSCLC patients are in advanced stages with pleural effusion or with metastases at the time of diagnosis. Their expected median survival is around 4–5 months. Only 10% of them have 1-year survival. The mechanism of chest wall swelling was noteworthy. It was a confirmed fact that there were metastases in right 6th and 7th ribs. Pocket of blood collection in the chest wall was probably due to bleeding at the metastatic site as a result of vascular invasion. Rib involvement in lung cancer may be due to metastasis or direct invasion. The most common chest wall tumors are metastasis and multiple myeloma. Most common malignant chest wall bone tumor is chondrosarcoma. Most common benign rib lesion is fibrous dysplasia. Ribs may be involved in osteomyelitis caused by actinomycosis or tuberculosis resulting in rib erosions.

Cases

Management As there were metastases involving ipsilateral lung and contralateral chest wall. Further investigation for staging was unnecessary. We started chemotherapy with palliative intent along with right sided focal palliative radiotherapy to relieve chest pain. We gave him supportive management.

Discussion Advanced stage lung cancer should be managed symptomatically with standard medical management, the judicious use of pain medications, nutritional support and psychological support. Appropriate use of radiotherapy and chemotherapy with palliative intent should be aimed to relieve symptoms and improve quality of life. Therapy (chemotherapy or radiotherapy) may be with curative intent like stage I and II lung cancer where surgery is not possible due to some other reasons. Therapy may be of palliative intent to reduce patient suffering and improve quality of life for stage III and IV lung cancer. Therapy may be adjuvant along with surgery. Postoperative adjuvant therapy is given for stage IIIA disease to deal with nonresectable part of malignancy and to irradiate the remnant cells. Preoperative adjuvant (neoadjuvant) therapy is given for Pancoast tumor to reduce the size of mass so that surgery can be made possible. The scope of prophylactic therapy (prophylactic cranial radiation or intrathecal My Learning chemotherapy) is limited in patient with Global viewing of chest X-ray small-cell lung cancer who has achieved should be discouraged as complete remission. Palliative radiotherapy obvious lesion may draw full attention and obscure other is often indicated for SVC syndrome, important findings. uncontrolled hemoptysis, unrelieved chest pain, distressing dyspnea, etc.

CASE 91: COPD WITH RAISED LEFT DOME OF THE DIAPHRAGM CASE REPORT Step I A 56-year-man presented with cough, hemoptysis, chest pain and hoarseness of voice for last 20 days. As per patient, he had insidious onset cough that was intermittent usually more profound at night. Cough was associated with little mucoid expectoration. Patient had three episodes of blood stained expectoration within that period. He had dull aching poorly localized central chest pain that was persistent and progressive. On repeated enquiry, patient confessed that cough was presented for more than 10 years and that was

353

354

101 Cases in Respiratory Medicine

associated with mucoid expectoration. He noticed that he could not perform as his earlier days and had to restrict his activities. He ignored those symptoms and now he came to doctor for hemoptysis. He was a farmer, married and had five offsprings. He was smoking for last 35 years about 20–30 biries per day. He had no other significant history.

Discussion The man, smoker, had long history of cough and dyspnea suggested that he was probably suffering from COPD. It is important to recognize dyspnea, as dyspnea is a subjective manifestations. Sometimes, patient may adjust physical disability by changing lifestyle. The peculiarity of natural history of COPD must be understood. The disease usually starts with cough that patient usually ignores as smokers’ cough. Then he develops dyspnea that he thinks due to aging process and accommodates it by reducing physical activities. When the severity of symptoms increases or a complication develops he consults physician. COPD is mostly underreported. Even then COPD is now the 4th leading cause of death and is projected to be the 3rd leading cause of death by 2020, worldwide. Our patient had some changes in his symptoms like appearance of hemoptysis, voice change and central chest pain. Hemoptysis might be explained by COPD itself and it usually occurs at the time of infection; such history was not available in our case. Dull aching poorly localized chest pain in the central area of chest usually indicates mediastinal involvement or it may be found in cor pulmonale. Changes in manifestations of a chronic diseases or appearance of new symptoms should not be ignored. Those might suggest appearance of new diseases, complication or comorbidity over the chronic disease process. Even a change in the nature of cough should not be ignored.

Step II General survey was within normal limits except the presence of 3° clubbing. Respiratory system examination revealed bilateral diminished thoracic expansion; central mediastinum, bilateral hyperresonant notes on percussion; liver dullness on right 6th intercostal space in right midclavicular line, Traube’s space was noted in left 4th intercostal space; breath sounds were bilaterally diminished vesicular with prolong expiration and there were presence of bilateral polyphonic rhonchi. Tidal percussion did not revealed any difference of lower border of resonant notes during inspiration and expiration on both sides. Examination of other systems was within normal limits except a loud pulmonary component of second heart sound.

Discussion Examination findings (diminished chest expansion, central mediastinum, hyperresonant percussion notes, diminished breath sounds and rhonchi) were consistent of COPD. Bilateral diminished thoracic expansion; bilateral

Cases

hyperresonant nodes on percussion; liver dullness on right 6th intercostal space in midclavicular line; and bilateral diminished vesicular breath sounds were suggestive of hyper-inflated lungs. On the other hand, prolong expiration and bilateral polyphonic rhonchi were signs of airway obstruction. Clubbing and upward shifting of Traube’s space were two important findings that should be explained. Clubbing was rarely reported in COPD but was usually associated with recurrent infection. Upward shifting of Traube’s space/left dome of diaphragm could be explained by volume loss of left lung (fibrosis or collapse); abdominal pathology (distension of stomach, pyloric obstruction) and diseases of the diaphragm (eventration and phrenic nerve palsy).

Step III His peripheral blood examination and routine blood biochemistry were normal. Sputum examination did not show AFB or Gram-positive/negative organism. Spirometry showed obstructive type flow volume loop with post bronchodilator FEV1/FVC ratio was 56% and FEV1 was 45% of predicted value. His chest X-ray PA view (Fig. 91A) showed left parahilar opacity with raised left dome of the diaphragm. An artifact was also present in left upper zone. X-ray chest left lateral view (Fig. 91B) showed raised left dome of diaphragm. Sniff test done under fluoroscopy revealed paradoxical movement of left dome of diaphragm. Fiberoptic bronchoscopy showed a growth in the left upper lobe bronchus partially obstructing it. Biopsy was taken from the growth and histopathology was suggestive of poorly differentiated squamous cell carcinoma.

Fig. 91A: Chest X-ray PA view showing raised left dome of the diaphragm with heterogeneous opacity near left hilum and an artifact

355

356

101 Cases in Respiratory Medicine

Fig. 91B: Chest X-ray left lateral view showing raised left dome of the diaphragm

Discussion In our patient spirometry confirmed the diagnosis of COPD. Additional manifestations could not be explained by exacerbation of COPD. Chest X-ray showed left parahilar opacity that was produced by lung mass, lymph nodes or both. Elevated left dome of diaphragm, in absence of left lung fibrosis or collapse, suggested left phrenic nerve palsy. Unilateral phrenic nerve palsy can be diagnosed by sniff test under fluoroscopy. Deep inspiration during sniffing produces a negative pressure in thorax that pull paralyzed dome upward while normal lung is contracting downward, the paradoxical movement of diaphragm. In bilateral diaphragmatic palsy, paradoxical movement cannot be appreciated radiologically but paradoxical movement of abdominal wall may be observed. Bilateral diaphragmatic paralysis can be diagnosed by simultaneous measurement of esophageal and gastric pressure. Lung malignancies in COPD were previously thought to be related to the same etiological factor (smoking). The prevalence of lung cancer is more in COPD patients even when the confounding factors like age, smoking, etc. are excluded. It is now well appreciated that lung cancer is a systemic manifestation of COPD itself. The mechanism of systemic manifestations include (i) systemic spillage of inflammatory cytokines, (ii) common etiological factor that causes COPD can also cause systemic manifestations, and (iii) same genetic phenotype that is responsive for COPD (only 15% smoker developed COPD), is also responsive for its systemic manifestations.

Cases

Management FOB and bronchoscopic biopsy confirmed the diagnosis of non-small cell lung cancer. There was involvement of left phrenic nerve. As surgery was not possible, further investigations for staging were useless. Treatment (radiotherapy/chemotherapy) with palliative intent was only possible at that stage. We My Learning treated him with palliative chemotherapy. Any change in character of We treated his COPD component with symptoms or appearance of tiotropium bromide metered dose inhaler new symptoms over a chronic two puffs daily; salmeterol-fluticonazole disease should not be ignored. Lung cancer is an important inhaler two puffs twice daily with spacer; comorbidity of COPD. parenteral antibiotics and short course of oral corticosteroids.

CASE 92: DIFFUSE PARENCHYMAL LUNG DISEASE WITH SEVERE CHEST PAIN CASE REPORT Step I A 46-year-lady presented with dry cough and exertional dyspnea for last 1 month. She had pain in the neck for 2 weeks that radiated to the left hand. She was a housewife from Jalpaiguri district of North Bengal. She went to a big hospital of South India. She was investigated thoroughly. Her chest X-ray (Fig. 92A) showed reticular opacities in both lower zones with honey-combing in both lower zones. There were flattening of both domes of the diaphragm. Her CT-thorax (Fig. 92B) confirmed honeycombing at both lung bases. FOB

Fig. 92A: Chest X-ray showing bilateral basal reticular opacities with honey-combing

357

358

101 Cases in Respiratory Medicine

Fig. 92B: CT-thorax showing honey combing at both lung bases

was done; BAL was taken and the results were inconclusive. Her collagen profile was within normal limits. She was treated with glucocorticoids and azathioprine. She was referred to orthopedic surgeon for neck pain. She was diagnosed as cervical spondylitis. She was advised exercise and pain killers. She was discharged with the advice to continue medicines and to revisit for follow-up after two months. Her pain increased and that disturb her sleep and day-to-day activities. Her brother, a pharmacist of our hospital, brought her to our hospital for opinion.

Discussion The patient was diagnosed as having two diseases: cervical spondylitis and interstitial lung disease (ILD). Cervical spondylitis was diagnosed by history and by X-ray cervical spine that showed some degenerative changes in 4th and 5th cervical vertebra and was treated by an orthopedic surgeon. They had investigated for ILD with collagen profile to exclude ILD secondary to collagen vascular disease. They did FOB and BAL fluid examination. At that time, the concept of ILD was at the emerging stage and diagnosis of ILD was thought to be sufficient. Identification of HRCT patterns and classification of ILD was beyond imagination. Then corticosteroids and immunosuppressant medicines were used for all types of ILD.

Step II I took detail history and examined her in outpatient department. As she came to me with pain, I evaluated her chest pain first. The pain was persistent, progressive and severe (suggested by disturbance of sleep and normal activity). The type of pain was characteristically suggestive of malignancy. I asked her to point with a

Cases

single finger to the site of maximum pain and she showed suprascapular area. She had insidious onset of progressive dyspnea without wheeze, and cough without expectoration. The history should be taken as an indicator of ILD. She was a nonsmoker and she had two children. She had no significant past, personal or family history. General survey revealed pallor and 1° clubbing. Respiratory system examination revealed tenderness over left suprascapular area and end inspiratory crepitations audible over both lung bases. I decided to revisit all previous investigation before prescribing new one. I found erosion of left 2nd and 3rd ribs in her previous chest X-ray after putting the chest X-ray with left hemithorax above in the view box (arrow of Fig. 92C). Upper cut of CT-thorax (Fig. 92D) showed ill-defined thin rim of opacity surrounding the apical part of left lung. I advised CT-guided FNAC and the report came as non-small-cell lung cancer.

Fig. 92C: Same chest X-ray viewing at different angle showing erosions of left 2nd and 3rd ribs

Fig. 92D: CT-thorax (upper cut) showing opacity encircling left apex of lung

359

360

101 Cases in Respiratory Medicine

Discussion A careful examination and interpretation of previous investigations are more important than carrying on further investigation. The patient was pointing her fingers in the left upper chest but that was ignored. The pain was treated as neck pain. It is always important to ask patient to point at the site of maximum intensity of pain by one finger. That clinical clue is of tremendous importance, though sometimes it may be misleading due to referred pain. Identification of site of pain is important and that will guide physician to search for rib erosion in the chest X-ray. Another method, I learned from my teacher Late Professor S Kargupta, is to place X-ray in the view box with disease side upward. Following those two methods had enabled me to detect rib erosions in chest X-ray. As upper part of hemithorax is overlapped by ribs and clavicle it is sometimes very difficult to identify erosion of 1st and 2nd ribs. Rim of opacity at the lung apex surrounding lung parenchyma is not an uncommon finding. It is called apical cap, and that is usually caused by nonspecific infection. Tuberculosis is one important cause of apical cap in our country. But apical cap should be investigated to rule out malignancy, particularly the superior sulcus tumor. ILD with lungs malignancy could be explained by exposure to asbestos fiber. ILD associated with exposure to asbestos fiber is called asbestosis. Contrary to our believe bronchogenic carcinoma is more common than mesothelioma in asbestos exposed persons. Asbestos exposure may occur in industries using asbestos. Housewife may get exposed to the fibers while washing husband’s clothes. Another controversial source of exposure may be leaving in house with asbestos roof.

Management I asked for any history suggestive of asbestos My Learning exposure. Her husband was a school teacher. She told us that her kitchen roof was made of asbestos. A radiologist can miss rib erosion, I referred her to oncology department, and but after clinical examination a they started palliative radiotherapy to alleviate physician cannot afford to miss it. her chest pain. I continued corticosteroids, analgesics and sedatives. She expired by 6 months after diagnosis.

CASE 93: LADY PRESENTED WITH RIGHT-SIDED FIBROTHORAX CASE REPORT Step I A 40-year-old housewife from Agartala presented with cough and chest pain for 6 months. Both cough and chest pain was insidious in onset and progressive. Cough was more at night and associated with little expectoration.

Cases

Chest pain was poorly localized over retrosternal area and right side of chest. Chest pain was persisting and causing discomfort to the patient. For last one month she noticed right side of her chest was gradually contracting and that caused noticeable deformity of chest. She had dyspnea on normal activities that she was observing for last 2 weeks. She was a nonsmoker and had no history of exposure to passive smoking or biomass fuel. She had no significant past history. She was married having two healthy sons. She was aspirated by local doctor and her pleural fluid showed lymphocytic (90% cells); exudative (protein 3.5 g/dL); high ADA (62 U/L); and malignant cell was not found in pleural fluid. She was receiving Category I DOTS from local hospital. Even after 3 months of antitubercular therapy, she was not responding and she came to Kolkata for opinion. On examination, general survey findings were within normal limits. Examination of respiratory system revealed gross flattening of right hemithorax with dropping of right shoulder, winging of scapula and scoliosis with concavity towards right side. On palpation movement of right hemithorax was remarkably decreased; trachea was shifted towards right; apical impulse was palpable on left 5th intercostal space 2 cm medial to midclavicular line; and rib crowding on right side of chest. Percussion notes were dull throughout right hemithorax; and breath sounds were diminished vesicular in the right hemithorax. There was no peripheral lymphadenopathy, no hepatosplenomegaly and examination of breast revealed no abnormalities. She was taking medicines regularly, and that was confirmed by repeated questioning her and family members. Routine investigations including peripheral blood, blood biochemistry and sputum examination did not reveal any clue. Her chest X-ray (Fig. 93A) showed opacity throughout right hemithorax obliterating right costophrenic

Fig. 93A: Chest X-ray showing less dense opacity in upper part and more dense homogeneous opacity in lower part of right hemithorax with rib crowding and trachea shifted to right

361

362

101 Cases in Respiratory Medicine

and cardiophrenic angles; trachea was shifted towards right; heart was not shifted; and there was rib crowding in the right hemithorax. A careful look on the opacity revealed that the opacity was homogeneous and denser in the lower part of hemithorax.

Discussion The lady had right-sided pleural fibrosis as suggested by history of pleural fluid aspiration, right-sided thoracic volume loss with gross chest wall deformity, diminished movement of right hemithorax, dull percussion notes and decreased vesicular breath sounds in the right hemithorax. Trachea shifting towards right might be due to apical fibrosis but there was no crepitation over the area to suggest that. Normal position of apical impulse might be explained by combined factor of pleural fibrosis and effusion. Presence of effusion in patient having pleural fibrosis is difficult to assess, and can only be clinically recognized when stony dullness (dullness with feeling of resistance) is elicited. Her chest X-ray confirmed the presence of pleural fibrosis by evidences of same sided thoracic volume loss. Presence of pleural effusion was suggested by denser homogeneous opacity in lower part with no shifting of cardiac silhouette. Pleural fibrosis could be explained by tuberculosis though the degree of fibrosis might be questioned. The problem we considered important in our case was not only lake of clinical improvement but also worsening of clinical manifestations. Clinical and routine investigation did not give any clue. In our case, the main pathology was in pleura and we were interested to visualize underlying lung and mediastinum. For clearer visualization of pleura, contrastenhanced CT-scan is preferable. Contrast is also important for differentiating mediastinal lymphadenopathy/mass from vascular structures and in shifted mediastinum the differentiation is more imperative where all structures are distorted.

Step II Her CECT thorax showed gross pleural fibrosis with narrowing of right side of thoracic cage and small collection of fluid (Fig. 93B). There was no mediastinal lymphadenopathy. Lung window (Fig. 93C) showed irregular interlobular septal thickening on right upper lobe and random nodules in the left hemithorax. Fiberoptic bronchoscopy did not revealed intraluminal growth/pathology and revealed gross anatomical distortion of tracheobronchial tree. I searched carefully in the mediastinal window of CECT thorax and found a mass lesion at the level of carina (Fig. 93D) that I had missed earlier. CT-guided FNAC from that mass lesion confirmed the diagnosis of adenocarcinoma of lung.

Discussion As expected from clinical and chest X-ray findings, CECT-thorax revealed pleural fibrosis and encysted pleural effusion. Underlying lung particularly

Cases

Fig. 93B: CECT-thorax at a lower level showing encysted pleural effusion with irregularly thickened pleura

Fig. 93C: Lung window of CT-thorax showing marked loss of right lung volume with pleural thickening; irregular reticular pattern in right upper lobe; and random nodules in left lung field

in right upper lobe showed reticular pattern. Reticular pattern is classified as interlobular (polygonal lines along the borders of lung lobules) and intralobular (fine reticular lines within lung lobules). Interlobular lines may be smooth or irregular. In our patient, the reticular pattern was interlobular and irregular. Considering the clinical setting we thought of the possibility of lymphangitic spread of lung malignancy. Opposite lung showed multiple nodular opacities. Nodular opacities are classified into centrilobular (absence of nodules in subpleural areas and along fissure lines); perilymphatic (presence of nodules along fissure lines and septum) and random (with no definite pattern). In our patients, nodules were random in distribution. Random nodules are mainly found

363

364

101 Cases in Respiratory Medicine

Fig. 93D: CECT-thorax showing a lung mass (arrow) at the level of carina

in miliary tuberculosis and metastases. In our case considering the clinical setting and non-uniformity of size of nodules, we considered them as metastases. Our next duty was to find out the primary site. We excluded possible primary sites like bronchogenic carcinoma by FOB; breast carcinoma by breast examination; abdominal malignancies by USG abdomen; genital tract malignancies by gynecological examination. A relook into the CECT thorax revealed a mass lesion, and ultimately the diagnosis was confirmed. The peculiarity of the case that astounded us was gross pleural fibrosis in adenocarcinoma. Mesothelioma with pleural infiltrations and underlying lung atelectasis may cause thoracic volume loss. We were astonished by the diagnosis of adenocarcinoma of lung.

Management

My Learning

She was treated with chemotherapy with palliative intent along with supportive care. She went back to Agartala. We lost her 3 months after diagnosis.

Do not be prejudiced with the myth, “Pleural fibrosis is always benign and rules out malignancy”.

CASE 94: MAN DIAGNOSED AS CERVICAL SPONDYLITIS CASE REPORT Step I A 65-year-old man presented with neck pain radiating towards medial side of right upper limb. He went to local doctor. Routine investigations and

Cases

Fig. 94: MRI scan of cervical spine showing a mass at right lung apex invading lower cord of brachial plexus and upper ribs of right side

X-ray cervical spine were done. He was diagnosed as cervical spondylitis. He was treated with pain killers and neck collar. His pain deteriorated and it became severe disturbing his normal activity and sleep. He was referred to an orthopedic surgeon and he was advised MRI of cervical spine. MRI cervical spine (Fig. 94) incidentally detected a right lung apical mass with erosion of 1st and 2nd ribs and compression of lower cord of brachial plexus. He was referred to me. On enquiry patient clarified that he hand pain in the neck as well as on the right shoulder that radiated to medial side of arm including right little and ring fingers. There was cough and history of hemoptysis once 2 days back. He was a smoker and smoked around 10–15 cigarettes per day for more than 30 years. He was a retired government officer. There was no other significant history. As I examined him, I found him screaming with pain. I noticed drooping of upper lid of right eye. Examination of eye confirmed partial ptosis of right side. There was enophthalmus, meiosis and hypohidrosis of right upper part of face. Findings were suggestive of Horner’s syndrome. There was atrophy of hypothenar muscles of right hand. One lymph node in the medial group of right supraclavicular lymph nodes was palpable in between two heads of sternomastoid muscle. The gland was about 2 cm in diameter, hard in consistency, fixed to deeper structure and free from skin. Examination of respiratory system showed no abnormality except dullness and tenderness over apex of right lung.

365

366

101 Cases in Respiratory Medicine

Discussion An elderly smoker presented with neck pain and naturally he was diagnosed as cervical spondylosis. Orthopedic surgeon advised MRI as he was not responding. Unfortunately specialists are focused with their own subject and fortunately patient was advised MRI scan of cervical spine. Plethora of findings were missed due to a prefixed bias. Pain in neck radiating to arms is not an uncommon finding in cervical spondylitis. Apical lung tumor was accidentally detected by MRI scan. A malignant tumor at the apex of the lung is called Pancoast tumor or superior sulcus tumor as the tumor is above the groove produced by subclavian artery. Pancoast’s syndrome includes apical lung tumor; erosions of 1st and 2nd ribs; Horner’s syndrome and involvement of lower cord of brachial plexus causing pain in the shoulder radiating to ulnar distribution of upper limb. Horner’s syndrome is the triad of ptosis, miosis, and facial anhidrosis although anhidrosis is not a constant feature. Enophthalmos and loss of ciliospinal reflex are considered as additional features. Ptosis is due to involvement of Müller’s muscles supplied by cervical sympathetic nerves. Ptosis is called pseudoptosis as patient can raise his upper eyelid. In contrast, in ptosis due to 3rd nerve palsy patient cannot raise upper eyelid and ocular movements will be affected. Horner’s syndrome is caused by involvement of sympathetic nerve chain supplying face including cervical sympathetic ganglions. Causes of Horner’s syndrome include a wide range of diseases like brainstem stroke, carotid dissection, neoplastic invasion of sympathetic chain and rarely it may be idiopathic. The clinical manifestations of invasion of lower cord of brachial plexus by neoplastic lesion include pain in the distribution of ulnar nerve, weakness and atrophy of hypothenar muscles of hand. Pancoast tumor is sometimes confused with apical fibrosis. Asymmetry and parenchymal opacity is useful to differentiate it from apical cap. MRI is helpful in early detection of apical lung tumor. Pancoast syndrome results from local extension of a tumor growing in the apex of the lung that involve thoracic wall, lower cord of brachial plexus and sympathetic nerves. Shoulder pain in Pancoast tumor characteristically radiates in the ulnar distribution of the arm.

Step II We did CT-guided FNAC from lung mass and direct FNAC from cervical lymphadenopathy. From both sides, the report came as moderately differentiated squamous cell carcinoma. CECT-thorax showed right hilar and ipsilateral mediastinal lymphadenopathy. There was no clinical evidence of distant metastasis. Clinically, I staged the case as T3N3M0.

Discussion Superior sulcus tumor/Pancoast tumor invades adjacent structures and produce Pancoast’s syndrome including Horner’s syndrome, shoulder and/ or arm pain, and weakness with/without atrophy of hypothenar muscles of

Cases

the hand. Pancoast tumors are usually of squamous cell histological type and occur in smoker. Histologically, it is characterized by infiltrating nest of tumor cells that lack intercellular bridges and often associated with keratinosis. Pancoast tumor should be staged as staging of other non-small cell lung cancer. TNM classification depends on size and extends of tumor, lymph node involvement and metastasis. T3 tumor is defined as (i) tumor >7 cm or directly invades any of the following: (i) chest wall (including superior sulcus tumors), phrenic nerve, mediastinal pleura, parietal pericardium; (ii) tumor 75% of cells and most importantly by absence of manifestations suggestive of other diseases. The diagnosis can be confirmed by pleural biopsy showing caseating epithelioid granuloma or culture of pleural tissue showing growth of Mycobacterium tuberculosis complex. Tuberculous pleural effusion is common in adolescent age group and is due to hypersensitivity reaction to tuberculous protein. The disease usually starts insidiously with constitutional symptoms (low grade fever, weight loss and night sweat) and local symptoms like cough, pleuritic chest pain and heaviness of chest/dyspnea. Pleuritic chest pain suggests pleural inflammation and is usually associated with TB pleural effusion. In our patient, pleuritic chest pain was absent, but its presence is not mandatory for diagnosis of tubercular pleural effusion. There were no constitutional symptoms and that might be explained by localization of inflammation to pleural compartment. For same reason, tuberculin test may be negative in TB pleural effusion. ADA is an enzyme liberated by mononuclear cells and is a sensitive and nonspecific test. ADA has less positive prediction value and more negative prediction value. ADA level more than 60 U/L is highly suggestive of TB pleural effusion. ADA between 40–60 U/L is considered as borderline. As pleural effusion is due to hypersensitivity reaction, the chance of isolating TB bacilli from fluid is less. Pleural fluid is a suboptimal sample for Cartridge Based Nucleic Acid Amplication Test (CBNAAT) and the sensitivity is as less as 40–50%. Pleural biopsy is more sensitive in TB effusion, but usually done when clinical and biochemical parameters are inconclusive. Caseating granuloma

Cases

is diagnostic of tuberculosis but noncaseating granuloma may be found in some cases, particularly treated cases. On the other hand, tissue necrosis may be confused with caseating necrosis. The confirmation of tubercular pleural effusion is established by demonstrating bacilli or DNA fragments by pleural tissue culture or by CBNAAT.

Step II On examination, general survey revealed only signs of dyspnea (RR was 36/ min and accessory muscles of neck were working). Examination of thorax revealed signs of moderate right-sided pleural effusion (decreased movement, dullness and decreased breath sound in the right lower chest). There was no peripheral lymphadenopathy, no hepatosplenomegaly and other systems were essentially normal. Her chest X-ray showed right-sided pleural effusion without mediastinal shift. We did blind pleural biopsy and aspirated 800 mL pleural fluid. Pleural fluid was sent to laboratory for cell block preparation. After aspiration we did a CECT-thorax. Lung window of her CT thorax (Fig. 97A) showed multiple nodular and ground-glass opacities of nonuniform sizes mainly distributed along the septal and fissure lines in right lung field. Some ill-defined centriacinar nodules were also visible along with few subpleural nodules. There was a conglomerated nodule anteriorly. Left lung was conspicuously normal. Mediastinal window (Fig. 97B) showed a conglomerated nodule in right lung anteriorly close to prevascular space. There were encysted effusion in right pleura and oblique fissure. CT-guided FNAC was done from the mass lesion. The report came as plenty of lymphocytes with hyperchromatic nuclei

Fig. 97A: Lung window of CT-thorax showing irregular nodular opacities and ground-glass opacities in right lung with right-sided pleural thickening

377

378

101 Cases in Respiratory Medicine

Fig. 97B: Mediastinal window of CECT-thorax showing right-sided loculated pleural effusion with fluid in oblique fissure and a mass lesion in the prevascular area

suggestive of lymphoproliferative disease most likely lymphoma. Latter on pleural biopsy and cell block report came as negative.

Discussion Despite some negative point, in our case treatment with antitubercular medicines were justifiable because of adolescent age, absence of manifestations of other disease, lymphocytic exudative effusion and high ADA (>40 U/L). The age went against malignancies. Connective tissue diseases were not considered due to age of patient and absence of joint symptoms or localized nature of illness. CT findings in our patient were also confusing. There were nodules and ground-glass opacities along with fissure line and that findings were suggestive of sarcoidosis. Pleural effusion is found in less than 1% cases of sarcoidosis, moreover effusion as a presentation is most unlikely for sarcoidosis. Some ill-defined nodules and ground-glass opacities were centrilobular. Ill-defined centrilobular nodules are suggestive of hypersensitivity pneumonia but that could be excluded by absence of exposure to organic material and pleural effusion. Some nodules were pleural-based suggestive of pleural metastasis. All together we should classified nodules as random nodules. Random nodules and ground-glass opacities were distributed throughout right lungs and totally spearing left lung. Random nodules may be seen in miliary TB and malignant metastases. Different sizes of nodules were suggestive of metastases. In both conditions, nodules are distributed bilaterally. So, our diagnosis was local spread of malignancy mainly through lymphatic. Fortunately, we got a conglomerated nodular mass close to anterior mediastinum and FNAC gave a clue to diagnosis. But the problem remained

Cases

as lymphoma needed histopathological examination of tissue to confirm diagnosis. Structural changes were more important than cellur changes for the diagnosis of lymphoma. In pure intrathoracic lymphoma, getting tissue for histological examination remains a major challenge. Radiologist was not willing to take biopsy as the conglomerated nodule was close to blood vessels.

Step III We searched for peripheral, mediastinal, abdominal lymphadenopathy by more vigorous clinical examination of lymph node areas, CECT-thorax, CECT and USG abdomen. All investigations were negative. Bone marrow aspiration was done and it was again non-informative. We did FOB with transbronchial lung biopsy (TBLB). The diagnosis came as non-Hodgkin’s lymphoma and further classification was not possible.

Discussion The problem with TBLB is that it gets tiny bits of tissues and that is often crushed causing structural changes. TBLB may be helpful for few diseases like tuberculosis, sarcoidosis, infections and malignancies. TBLB is not helpful in most of the interstitial lung diseases. Pure intrathoracic lymphoma is a diagnostic challenge to all pulmonologist as there was no peripheral lymphadenopathy. In presence of pleural effusion, pleural fluid flow-cytometry may be helpful, but it is costly and not always available. EBUS may be an option for mediastinal lymphadenopathy if EBUSguided biopsy facility is available. About 19% patients with non-Hodgkin’s lymphoma were found to have intrathoracic disease. In about 10–11% of histiocytic lymphoma, the disease was completely confined within chest. Most common manifestations of intrathoracic lymphoma are paratracheal and mediastinal lymphadenopathy (in 25% cases), and hilar lymphadenopathy (in 17% cases). Isolated pulmonary or pleural involvement are less common. Pulmonary lesions are found in about 20% cases and pleural effusion occurs in 18% cases. The intrathoracic lymphomas are usually at an advanced clinical stage. Because of the rapid progression of this entity, early recognition of its intrathoracic manifestations is mandatory for effective therapy.

Management We referred the case to hematology department for opinion. They refused to start treatment without proper investigation. We started chemotherapy with CHOP (cyclophosphamide, doxorubicin, vincristine and prednisolone) after consultation with an oncologist. Patient responded partially. Her dyspnea decreased. We continued other supportive treatment like diet, oxygen inhalation, bronchodilators and antibiotics. We discharged her after 2 cycles of chemotherapy. She continued 6 cycles of chemotherapy. We lost her from follow-up after 1 year.

379

380

101 Cases in Respiratory Medicine

Discussion After diagnosis, lymphoma should be properly classified as different histological types have different outcome and treatment regimen. For staging of lymphoma, the following steps are used: (i) physical examination, (ii) documentation of B symptoms, (iii) laboratory evaluation with complete blood counts, liver function tests, uric acid, calcium, serum protein electrophoresis, serum β-2 microglobulin, (iv) chest radiograph, (v) CT scan of abdomen, pelvis, and chest, and vi) bone marrow biopsy. Five clinical risk factors that determine the prognosis are (i) age >60 years, (ii) elevated serum lactate dehydrogenase levels, (iii) less performance status, (iv) advanced stage of disease, and (v) more than one site of extranodal involvement. Combination chemotherapy with CVP (cyclophosphamide, vincristine, and prednisone) or CHOP (cyclophosphamide, hydroxyurea, vincristine, and prednisone) are most frequently used. With adequate treatment, 50–75% of patients will achieve a complete remission. About 20% of remission cases will remain in remission for >10 years. Other patients relapse usually on average after 2 years. The addition of rituximab to CHOP and other effective combination chemotherapy improves survival and decrease the risk of histological progression. Complete remissions can be My Learning achieved in 85% or more of patients with median remission durations about 6 or 7 Pure intrathoracic lymphoma remains a diagnostic challenge. years.

CASE 98: ADOLESCENT GIRL WITH ANTERIOR MEDIASTINAL MASS CASE REPORT Step I A 16-year-old girl presented with chest pain and cough for 3 weeks. Chest pain was insidious in onset and dull aching over precordium without any referred pain that responded partially to analgesics. Cough was occasional and nonproductive. She had no history of fever, hemoptysis, dyspnea or past history of illness. She had no addiction and her menstrual history was normal. On examination, general survey revealed no abnormality except mild pallor. Examination of respiratory system revealed dullness over manubrium sternum and left infraclavicular area. Her peripheral blood showed anemia (8 g/dL) and raised ESR (120 mm/h). Routine blood biochemistry was normal. Her chest X-ray PA view (Fig. 98A) showed a heterogeneous mediastinal mass extending to left upper and mid zones with sharp lateral border. Chest X-ray left lateral view (Fig. 98B) showed anterior mediastinal mass obliterating retrosternal lucency.

Cases

Fig. 98A: Chest X-ray PA view showing a large mass in left hemithorax with sharp lateral margin and untraceable medial margin

Fig. 98B: Chest X-ray left lateral view showing anterior mediastinal mass obscuring retrosternal space

Discussion In our case, the dull aching chest pain was probably ignored for long time allowing tumor size to increase and that presented as large mass. Pain arising from mediastinal structures are usually vague, dull aching and poorly localized (except pains arising from heart and vascular structures). Chest

381

382

101 Cases in Respiratory Medicine

pain is often ignored for long time as it is usually nonspecific. Mediastinal masses usually do not produce physical sign unless they become large to cause dullness or they compress any vital structure. X-ray chest is a useful tool to detect mediastinal mass. For classifying mediatinal tumor lateral view is essential. Mediastinal tumors are classified into anterior, middle and posterior mediastinal tumor. Causes of anterior mediastinal mass are retrosternal goiter, thymic tumor, germ cell tumors and lymphadenopathy. To exclude retrosternal goiter, examination to detect lower border of thyroid gland is important. Thymus normally causes opacity in the superior mediastinum with concavity laterally (the sail sign) in children and it reduces in size by puberty. Thymic hyperplasia and thymic tumor may present as anterior mediastinal mass in adult. Lymphadenopathy in anterior mediastinum is not common. Germ cell tumors are important causes of anterior mediastinal mass and it is characterized by its heterogeneity as it contains tissues of all three layers (ectoderm, mesoderm and endoderm). They may be benign or malignant. Their diagnosis depends on biopsy as well as levels of tumor markers.

Step II CT-scan thorax (Fig. 98C) showed anterior mediastinal tumor. Tumor was heterogeneous with sharp border. Multiple areas of fat attenuations were found within the tumor. Mediastinal fat line was partially obscured anteriorly that may be suggestive of invasion. We had percutaneous core biopsy of the mass and report came as teratoma containing components of all three embryonal layers. We tested for blood biomarkers and levels of human chorionic gonadotropin (HCG), alpha fetoprotein (AFP) and lactic dehydrogenase (LDH) were within normal limits. HCG was less than

Fig. 98C: CT-thorax showing a large heterogeneous mediastinal mass with areas of fat densities pushing heart to right

Cases

0.5 mIU/mL and alpha feto-protein was 0.95 IU/mL. USG whole abdomen and breast examination revealed no abnormality.

Discussion We started interpreting CT-thorax by determining the site from where the mass was arising like mediastinum, pleura, lungs, sternum and spine. It is very important to examine the interface with adjacent structures. A mass with speculated, nodular, obscured or irregular borders usually arises from lung. On the other hand, a well marginated mass with broader base against the mediastinum arises either from mediastinum or mediastinal pleura. The common tumors found in prevascular space are thyroid mass, thymic neoplasms, germ cell tumor, and lymphadenopathy. Parathyroid adenoma, cystic hygroma, pericardial cyst, aortic body paraganglioma, mesenchymal tumor and aneurism are uncommon prevascular tumors. Germ cell tumors comprises about 10–15% of anterior mediastinal mass. It is thought to be arising from remnants of embryonal cells. Germ cell tumor usually occurs in young adult. Mediastinal germ cell tumors are classified as benign (mature teratoma) and malignant (seminoma, malignant teratoma, embryonal carcinoma, choriocarcinoma and mixed tumor). More than 90% of malignant germ cell tumors occur in men. Benign tumors are equally distributed in both sexes. Malignant tumors usually secrete tumor biomarkers like LDH, AFP and beta HCG. Their serum levels are useful for diagnosis and monitoring malignant germ cell tumors. Teratomas are the most common germ cell tumor, usually arising from the cell nest within or in contact with thymus. Teratomas may be mature or malignant. Mature or benign teratomas contains predominantly ectodermal tissues along with some elements from endoderm and mesoderm. The term dermoid cyst is used when tumor are primarily contains ectodermal components like skin, sebaceous material, hair and calcification. Such lesions are usually unilocular. They are accountable for 60–70% germ cell tumor. Half of them are asymptomatic at the time of diagnosis. Chest pain, cough, dyspnea and fever are the usual symptoms. Pneumonia, hemoptysis, SVC symptoms, trichoptysis are uncommon symptoms. Tumor markers levels are usually normal. Complete removal is the usual treatment. Radiologically, mature teratomas are well-defined, round or lobulated mass that project to one side of the midline. The characteristic features of teratoma include areas of water attenuation, areas of fat attenuations, presence of calcification, ossification and even teeth. Fat-fluid level within tumor is virtually diagnostic of teratoma. A definite cyst wall with curvilinear calcification may be visible in CT-scan. Malignant tumors are usually symptomatic at presentation. Weight loss is usually more marked. Tumor markers are elevated in malignant teratoma but in pure seminoma both HCG and AFP levels may be normal. Elevated levels suggest the presence of non-seminomatous component. The tumor is usually quite heterogeneous contain cysts, necrotic material and hemorrhage. Adjacent mediastinal fat planes are often obliterated and extensive local invasion may be seen.

383

384

101 Cases in Respiratory Medicine

Management We referred the patient to thoracic surgery unit. They operated by anterior thoracotomy. They removed more than 95% of mass but could not remove some part of tumor due to adhesion with pericardium and vascular structures. Partial pericardiectomy was done. Biopsy report came as benign teratoma containing elements of all three germinal layers but predominantly ectodermal tissue, without evidence of malignancy. We referred the case to oncology department and they gave local radiotherapy. Patient was followed-up with monthly chest X-ray and quarterly CT-thorax. Patient showed no sign of relapse during 2 years of follow-up.

Discussion There was utter confusion regarding nature of tumor. Radiological picture of partial loss of mediastinal fat line was suggestive of malignant invasion and operative findings confirmed local invasion. On the other hand, normal levels of biomarkers in teratoma were suggestive of benign nature. Core biopsy and operative specimen biopsy did not show any evidence of malignancy. We discussed the case with oncologist in detail and ultimately My Learning we came to a conclusion to give him local Heterogeneous tumor in anterior radiotherapy. The decision was taken with mediastinum is usually teratoma the assumption that slides might have made and marked heterogeneity in from non-representative areas. Surgical clinical and histological behavior is the characteristic of teratoma. findings were given more importance.

CASE 99: YOUNG MAN WITH OPAQUE LEFT HEMITHORAX CASE REPORT Step I A 30-year-old man presented with cough for 1 month, chest pain for 20 days and breathlessness for 15 days. As per patient’s statement he had insidious onset of cough. Cough was progressive with scanty mucoid expectoration. Cough was intermittent and was increased at night and during exertion and talking. Patient then developed dull aching central chest pain that was persistent and progressing and partly relieved with pain killers. For last 2 weeks, he noticed dyspnea while climbing stairs and during carrying weight. He denied any history of wheeze, hemoptysis, fever, palpitation or night awakening. He was laborer by occupation, he had two daughters and one son and all of them were healthy. He was nonsmoker but habituated with chewing tobacco. He denied any past history of any medical or surgical illness. Positive findings in general survey were presence of pallor; BP—100/70 mm of Hg; pulse—78/min; RR—28/min; and pedal edema present. Examination of respiratory system revealed fullness in the left anterior chest wall; movement of upper part of left hemithorax decreased; trachea shifted to the right; apical

Cases

impulse could not be localized; dull percussion notes in the left anterior chest wall continuous with mediastinum; Traube’s space was normally resonant; and diminished vesicular breath sound over left infraclavicular and mammary areas. Direct percussion over sternum was dull. Other areas of chest revealed no abnormalities. Other systems were normal.

Discussion A young man had insidious onset of disease with sequential development of cough, chest pain and dyspnea. As he was laborer by occupation he was expected to detect dyspnea earlier than persons with sedentary habit. It should be remembered that lung parenchyma do not have pain fibers. Pain should come from pleura, chest wall or mediastinum. Dull aching central chest pain was indicative of mediastinal involvement. Persistent and progressive chest pain might be suggestive of malignancy and should be investigated. Cough with expectoration indicated lung parenchymal or airway disease. We thought of bronchogenic carcinoma involving mediastinum but his age and absence of smoking history were not conducive of that idea. Examination findings were suggestive of a mass lesion that was continuous with mediastinum. Normally direct percussion over manubrium should be resonant. Normal percussion of Traube’s space excluded pleural pathology. So, our clinical diagnosis was either lung mass invading mediastinum or mediastinal mass extending to anterior chest wall.

Step II Peripheral blood examination showed Hb%—8 g/dL; WBC—8500/mm3, N—60, L—30, E—10 and ESR—40 mm in 1st hour. His fasting blood glucose was 50 mg/dL; urea was 32 mg/dL and creatinine was 0.8 mg/dL. Sputum for AFB was negative. His X-ray chest PA view (Fig. 99A) showed homogenous opacity involving

Fig. 99A: Chest X-ray PA view showing opaque left hemithorax with obliteration diaphragmatic angles and shifting of mediastinum to right

385

386

101 Cases in Respiratory Medicine

almost whole left hemithorax except most upper part with a convex upper margin. Mediastinum was deviated towards right side. CT-thorax (Fig. 99B) showed a large almost homogenous mass with irregular but distinct anterior, lateral and posterior border. Medial border was continuous with mediastinum. There was a pleuralbased nodule with thickening in the adjacent pleura. Serum levels of beta human chorionic gonadotropin level and alpha fetoprotein were below detectable levels. We tried unguided FNAC, and that revealed high yellow color fluid with plenty of lymphocytes. USG of the thorax showed a mass partly solid, partly cystic continuous with mediastinum. USGguided FNAC from solid area (Fig. 99C) showed biphasic cells containing thymocytes and lymphoid cells suggestive of thymoma. He had anemia, but he did not have evidence of myasthenia gravis, pure red cell aplasia and hypogammaglobulinemia.

Discussion The thymus is a normal anterior mediastinal organ in children that regresses in adult. It contains epithelial and stromal cells derived from pharyngeal pouch and lymphoid precursors derived from mesodermal cells. Thymoma is the most common anterior mediastinal tumor in adults. Other causes of anterior mediastinal tumor include lymphomas, germ cell tumors, and substernal thyroid tumors, carcinoid tumors, lipomas and thymic cysts. Thymic hyperplasia and granulomatous inflammatory diseases (tuberculosis, sarcoidosis) may mimic thymoma. Thymoma may be asymptomatic or present with nonspecific chest symptoms. Myasthenia gravis, pure red cell aplasia and hypogammaglobulinemia are found in about 30%, 5–8% and 5% of thymoma patients, respectively. On the other hand, thymoma is present in about 10–15% patients with myasthenia gravis, 30–50% patients with pure red cell aplasia and 10% patients with hypogammaglobulinemia. CT-thorax is helpful in finding invasive nature of tumor; MRI is helpful for staging posterior mediastinal tumors, and somatostatin receptor imaging with indium-labeled somatostatin analogues may be of value. FNAC cannot differentiate between thymomas and lymphomas. Thymomas are potentially malignant epithelial tumors. It is not useful to differentiate benign and malignant thymoma as cytological features are not reliable predictors of biologic behavior. The noninvasive (in 65% cases) or invasive nature of tumor will help in prognosticate the disease.

Management We referred the case to thoracic surgeon and they resected the tumor as much as possible. They could not remove the tumor totally as there was pleural and pericardial invasion. Preoperative staging was stage III. We referred him to oncology department they started chemotherapy with cisplatin, doxorubicin and cyclophosphamide along with local radiation. Patient survived for one and half years.

Cases

Fig. 99B: CT-thorax, cut at a upper level, showing large homogeneous mass with lobulated margin and a pleural-based nodule

Fig. 99C: FNAC from mediastinal mass showing thymocytes and lymphoid cells suggestive of thymoma

Discussion Prognosis of thymoma depends on invasive nature and resectibility of tumor. Totally encapsulated tumor are staged as stage I and total resection of the tumor at that stage achieve cure rate about 96%. In microinvasive stage II, thymoma treatment is total resection followed by postoperative

387

388

101 Cases in Respiratory Medicine

radiation therapy. In macroinvasive stage III and IV thymoma treatment should be neoadjuvant chemotherapy followed by radical surgery, with/ without additional radiation therapy. Chemotherapy in thymoma should contain My Learning a platinum compound and an anthracycline It is the biological behavior with or without c yclophosphamide, (invasive nature) rather than histological differentiation of vincristine or prednisone. In our case, we persuaded the thoracic benign or malignant nature surgeon to operate the case for confirming that will dictate the outcome of thymoma. the diagnosis as well as for radical surgery.

CASE 100: ATRIAL SEPTAL DEFECT WITH MULTIPLE NODULAR OPACITIES CASE REPORT Step I A 45-year-old man presented with recurrent respiratory tract infection. As per patient he had recurrent attacks of cough, fever and expectoration since his adulthood. He used to go to a quack and was treated with antibiotics. He was habituated with that and he never took it seriously. For last 6 months, he observed dull aching central chest pain that increased in exertion. He had an attack of hemoptysis and consulted a local physician. A chest X-ray was done and that showed bilateral opacities. Sputum was negative for AFB. He was referred to us. He attended our outpatient department. According to his statement, he had recurrent attacks of cough with mucoid/mucopurulent expectoration with fever at least once in every year since puberty. He could not recall any illness before that. He noticed a change in his problem and had dull aching central chest pain during exertion. He accommodated that by avoiding exertion. He had few episodes of blood streak expectorations that worried him. He denied history of dyspnea but confessed that he could not work at the same level with his coworkers. He was farmer by occupation. He took more than 20 biries daily for more than 25 years. He did not have any other significant past, personal and family history. General survey was within normal limits except RR was 28/min. Examination of respiratory system was normal. Cardiovascular system examination revealed wide splitting of second heart sound that did not changes with phases of respiration. Examination of other systems was normal. His peripheral blood and routine blood biochemistry were normal. ECG showed partial right bundle branch block. His X-ray chest PA view (Fig. 100A) showed bulging of pulmonary bay; dilatation of right main and right lower lobar artery and dilated arteries could be stressed up to periphery. Echocardiography confirmed the diagnosis of atrial septal defect (ASD).

Cases

Discussion Unlike ventricular septal defect that usually presents in childhood, ASD usually presents in adulthood. Respiratory tract infection is a common mode of presentation. A careful clinical examination focusing on second heart sound splitting and its relation with phases of respiration is sufficient to make a diagnosis. ASD is a developmental defect in atrial septum that may be presented in adulthood and is more common in females. Ostium secundum defect is the commonest type that involves the fossa ovalis. Patients with ASD are usually asymptomatic in early life. ASD is associated with increased tendency for respiratory infections. The pulmonary arterial hypertension in ASD is a result of increased blood flow through pulmonary circulation due to left-to-right shunt. By the fourth decade, most of patients develop pulmonary arterial hypertension, bidirectional and then right-to-left shunting of blood, right heart failure and atrial arrhythmias. Physical examination will reveal signs of pulmonary arterial hypertension and right ventricular enlargement/failure. A widely spitted and relatively fixed in relation to respiration is the characteristic sign of ASD. ECG characteristically shows an rSr’ pattern in the right precordial leads. Chest X-ray findings are enlarged right atrium, right ventricle, pulmonary artery and its branches. Echocardiography shows pulmonary arterial and right ventricular and right atrial dilatation with abnormal (paradoxical) ventricular septal motion and the septal defect. The defect can be directly visualized by two-dimensional imaging, color-flow imaging, or echo-contrast. Treatment is surgical repair of defect with a patch of pericardium or with prosthetic material.

Step II A careful examination of chest X-ray showed that all opacities in the lung field could be stressed with pulmonary arterial branches except one opacity in the left mid-lower zone junction (arrow head in Fig. 100A). We advised contrast enhance CT-thorax. CECT-thorax (Fig. 100B) showed pulmonary arterial hypertension and multiple vascular opacities (taking contrast) all over both lung fields. Within those glittering opacities, there was single opacity in the left lower lobe that did not take contrast (non-vascular opacity). CT-guided FNAC from that non-enhanced lesion was done and the report came as adenocarcinoma of lung.

Discussion Pulmonary arterial hypertension (PAH) in CT-scan can be diagnosed by measuring the diameter of pulmonary artery and aorta. Diameter of pulmonary artery more than that of ascending aorta is confirmatory of PAH. The causes of PAH are many. They are usually grouped into hypoxic vasoconstriction; obliteration of vascular bed; occlusion of pulmonary artery;

389

390

101 Cases in Respiratory Medicine

Fig. 100A: Chest X-ray showing dilated pulmonary arteries and bilateral nodular opacities

Fig. 100B: CECT-thorax showing dilated pulmonary arteries (presenting like lung nodules) and an irregular nodule in left lower lobe without contrast enhancement

secondary to parenchymal lung diseases; idiopathic and increase blood flow through pulmonary circulation. In all those conditions except in condition of increased blood flow, there is peripheral narrowing of blood vessels. In condition with increased pulmonary blood flow, like ASD and VSD, blood vessels could be stressed up to peripheral one-third of lung field. The vascular opacities in chest X-ray cause two-directional problems: (i) peripheral vascular opacities may be confused with pulmonary nodules or mass unless vessels are stressed carefully and (ii) FNAC from vascular

Cases

opacity may be dangerous. Opacity of other disease may be taking as vascular opacities. The differentiation of vascular nodules from non-vascular nodules is an essential step. In our case, only careful examination of chest X-ray identified a nonvascular opacity. That was confirmed by CECT-thorax. Identification of non-vascular nodules within multiple vascular nodules was difficult but an essential step. Otherwise, the diagnosis of lung cancer would be missed.

Management We referred the case to the cardiothoracic surgeon for lobectomy. He was investigated with cardiac catheterization. Fortunately, he had not developed reversal of shunt. They did left lower lobectomy and repaired the defect of ASD. He recovered after a stormy postoperative period.

Discussion Adenocarcinoma is classified within non-small cell lung cancer. Unlike squamous cell carcinoma, adenocarcinoma generally responds poorly to chemotherapy or radiotherapy. I am not sure how far the simplistic generalization is effective. Chemotherapy or radiotherapy kills a percentage of cells not the absolute number of cells. As per two log cell kill hypothesis, a tumor containing 1010 cells after a cycle of treatment the number of cell will be 108. Chemotherapy or radiotherapy will not only kill malignant cells but also normal cells. Normal cells can multiple quicker than malignant cells when needed but that has physiochemical regulation. On the other hand, malignant cells usually grow slower but steadily without any regulation (autonomy). In between cycles normal cells will replenish themselves whereas malignant cells will grow slowly to about 109 cells. After next cycle, the number of cell will be 107. So, after cycles of chemotherapy or radiotherapy, the number of cells will be minimum but never be zero. It is assumed that rest of the cells will be taken care of body My Learning immunity. From this above discussion, I want Within glittering stars (vascular to emphasize on the fact that surgery should nodules) the quiet star (nonbe the treatment of choice in any solid tumor vascular nodule) needs more attention when it is resectable and operable.

CASE 101: A YOUNG MAN WITH NODULES BEHAVING DIFFERENTLY CASE REPORT Step I A 20-year-old man had right-sided chest pain and blood streak expectoration for 3 months, and respiratory distress even at rest for 1 month. He smoked 2–3 biries/day for 2 years. He went to a local physician. His chest X-ray showed a

391

392

101 Cases in Respiratory Medicine

Fig. 101A: Scanogram showing a large mass right upper zone with multiple cannon ball opacities

large mass lesion in the right upper zone and multiple cannon ball opacities of different sizes. CT-guided FNAC was done from the mass lesion in right upper zone and the report came as poorly differentiated non-small cell carcinoma of lung. He was referred to us. In outpatient department, we examined him and we get following findings. Patient had signs of dyspnea with tachypnea (RR—36/min); accessory muscles of respiration working; suprasternal and intercostal suction. He had signs of superior vena caval (SVC) obstruction with engorged nonpulsatile jugular veins and prominence of few upper chest wall veins with flow above downwards. There was sign of mass lesion in right upper lobe (impaired movement, dull percussion note and diminished vesicular breath sounds in right infraclavicular area) with trachea shifted towards left. There was no other significant finding in general survey and systemic examination. His routine blood and biochemistry were within normal limits. Scanogram of CT-thorax (Fig. 101A) showed a large heterogeneous mass covering onethird of right hemithorax and multiple bilaterally distributed cannon ball opacities of different sizes. CECT-thorax at upper cut (Fig. 101B) showed a large heterogeneous mass in right upper lobe. CECT-thorax at lower cut (Fig. 101C) showed multiple contrast enhanced masses of different sizes.

Discussion A young man, who smoked less than 5 biries for 2 years, had a mass lesion in right upper lobe with multiple cannon ball metastases, respiratory distress and SVC syndrome. CT-guided FNAC report came as non-small cell lung cancer. The diagnosis of bronchogenic carcinoma had been established. Cannon ball opacities could be explained as lung to lung metastases.

Cases

Fig. 101B: CECT-thorax (upper cut) showing a large heterogeneous mass in right upper lobe

Fig. 101C: CECT-thorax at lower cut showing multiple contrast-enhanced masses of different sizes

As cancer cells are showered into circulation at different spells the sizes of cannon ball opacities usually vary. I had some problem with the diagnosis. Firstly, the age of patient was too early for lung cancer and secondly, he was practically non-smoker. In FNAC, a small number of cells are examined. Expertise required for FNAC is definitely more than biopsy where tissue has been examined. Moreover, poorly differentiated malignancies sometimes required immune-histochemistry for classification of their character.

393

394

101 Cases in Respiratory Medicine

Step II We specifically searched for other primary sources of lung metastasis. We found that the left testis was enlarged to a size of a tennis ball and that was nontender. On enquiring, the patient said that the swelling was present for last one year. As there was a history of trauma, the swelling was diagnosed as hematocele by a local surgeon. USG of left testis showed heterogeneous lesion. FNAC showed hyperchromatic cells suggestive of malignancy. Testicular FNAC is usually avoided but we did that as there was confusion regarding diagnosis. Left orchiectomy was done. Section of the left testis showed a large, solid, mostly necrotic tumor mass macroscopically. Histopathological examination of tissue revealed necrotic material; loose clusters of atypical, hyperchromatic, slightly pleomorphic cells; immature glands; both mature and immature cartilage; immature stromal tissue; and trophoblastic tissue with atypical syncytiotrophoblastic giant cells. The diagnosis was immature teratoma with choriocarcinoma. We examined for biomarker of germ cell tumor. The report came as α fetoprotein was more than 1,000 IU/mL; β-human chorionic gonadotropin level was more than 10,000 mIU/mL and LDH was 550 U/L. USG whole abdomen was normal. We diagnosed the tumor as non-seminoma testis and we graded the patient as intermediate risk group. Lung metastasis is not a poor risk factor for germ cell tumor.

Discussion The testicular tumors are divided into seminoma and non-seminoma group. Metastases are classified as pulmonary and non-pulmonary visceral metastasis. Pulmonary metastases are not considered as bad prognostic sign. The International Germ Cell Cancer Consensus Group developed criteria to classify patients into good, intermediate and poor risk groups. The tumor markers are included along with TNM (primary tumor, regional nodes and metastasis) staging. Seminomas are classified as good or intermediate risk depending on the absence or presence of non-pulmonary visceral metastases. The non-seminoma germ cell tumor is considered as intermediate risk when (i) primary site is gonadal or retroperitoneal; (ii) absent non-pulmonary visceral metastases; (iii) α fetoprotein 1000–10,000 ng/mL, β-human chorionic gonadotropin 5000–50,000 mIU/mL and LDH 1.5 to 10 times of upper normal limit. Lesser biomarker levels are considered as good risk factor. Bad risk factors include a higher biomarker levels and non-pulmonary metastasis. Tumor markers are the biochemical substances those are liberated from tumor and those represent the biological behavior of tumor. Tumor markers of germ cell tumors include AFP, β-HCG and LDH. Their levels are important for diagnosis of germ cell tumor; differentiating seminoma from non-seminoma; staging and assessing prognosis; monitoring response to treatment and detection of relapse.

Cases

Management We treated him as non-seminoma testis intermediate risk group with lung metastasis. After consultation with oncology department, we started chemotherapy with cisplatin—30 mg and etoposide—100 mg intravenously from day 1 to day 5. After three cycles of chemotherapy, patient improved significantly. Dyspnea was almost absent, he could perform his normal activities and SVC syndrome disappeared. Follow-up chest X-ray PA view (Fig. 101D) showed complete disappearance of cannon ball metastases but the mass lesion in right upper zone showed little regression. Tumor marker levels decreased and the level of α fetoprotein came down to 109.5 IU/mL. We continued treatment. At the end of 4th cycle, patient developed hoarseness of voice. ENT examination showed right vocal cord palsy. After 5th cycle, patient developed severe frontal headache. His CT scan brain (Fig. 101E) showed mixed density inhomogeneous, enhancing space occupying lesions with lobulated outline in right frontoparietal and parieto-occipital region with perilesional edema. Blood biomarker levels were measured and they were within normal limits. We started cranial irradiation, but the patient’s condition deteriorated and he expired.

Discussion The peculiarity of the case was that right upper lobe tumor behaved differently from other lung metastases. While other metastases disappeared and tumor marker level decreased, that mass showed no change. In fact that tumor behaved aggressively despite no increase of biomarker levels. It involved right recurrent laryngeal nerve producing hoarseness of voice. Right recurrent laryngeal nerve hooks round right subclavian artery. Unlike left

Fig. 101D: Follow-up chest X-ray showing disappearance of nodules/masses, and persistence of right upper lobe tumor

395

396

101 Cases in Respiratory Medicine

Fig. 101E: CT-brain showing multiple metastases

recurrent laryngeal nerve, it is rarely involved in lung diseases. Very rarely right recurrent laryngeal nerve may be involved by direct invasion of right apical tumor. The aggressive nature of right apical tumor was evident from brain metastases. Brain metastases were not considered from germ cell tumor because of normal level of biomarkers without any upward surge. We were surprised to find that there were no rises of biomarker levels at the time of brain metastases. Shortly, we lost the patient.

Step III Clinical manifestations of testicular metastases and right upper lobe tumor were different. On one hand, testicular metastasis disappeared along with decreases of tumor biomarker levels to normal levels. On the other hand, right upper lobe mass showed aggressive behavior with invasion of right recurrent laryngeal nerve and brain metastases without raise of blood biomarker levels. That made us think whether we were dealing with two different malignancies or they were different expression of same tumor where different cell lines behaved differently. That problem could be solved by immune-histochemistry but that facility was not available then. We compared the high power pictures of FNAC from right upper lobe tumor with that of testicular tumor. We found gross differences in their microscopic appearance. FNAC from right upper lobe mass (Fig. 101F) showed clusters of medium to large size, atypical cells, containing round to oval pleomorphic, hyperchromatic nuclei with nucleoli having coarse chromatin. On the other hand, FNAC from testicular tumor (Fig. 101G) showed occasional clusters of atypical hyperchromatic slightly pleomorphic cells with abundant necrosed material.

Cases

Fig. 101F: FNAC from right upper lobe mass showing clusters of medium to large size, atypical cells, containing round to oval pleomorphic, hyperchromatic nuclei with nucleoli having coarse chromatin

Fig. 101G: FNAC from testicular tumor showing occasional clusters of atypical hyperchromatic slightly pleomorphic cells with abundant necrosed material

Discussion Simultaneous or sequential appearance of two or more malignancies, demonstrably different by histopathology/immune-histochemistry and biological behavior are called second malignancy. Second malignancies are classified as synchronous (second tumor appeared within 6 months of

397

398

101 Cases in Respiratory Medicine

detection of first tumor) and metachronous (second tumor detected after 6 months of first tumor diagnosis). Metachronous second malignancies are more common and occur as a side effect of chemotherapy or radiotherapy. Synchronous second malignancies are less common. The possible explanation may be that the genetic mutations (augmentation of oncogenes or deletion of tumor suppressor gene) may affect different organs or cell lines. To diagnose second malignancy the demonstration of different histopathology by immune-histochemistry was essential. We could demonstrate that one tumor My Learning (in right upper zone) clinically behaved differently from other metastases, and they Even in diagnosed cases meti­ were different under high power microscopy. culous physical examination may surprise you and bring new But we could not confirm second malignancy insight into the case. by immune-histochemistry.

Index Page numbers followed by f refer to figure, and t refer to table

A Abdominal diseases 257 Abdominal lymphadenopathy 379 Abdominal viscera 61 Abdomino-thoracic respiration 282 Abnormal chest X-ray 2 Abnormal inhomogeneity 204 Absorption collapse 332 Accidental discovery of irregular calcification 41 case report 41 discussion 41 right sided pleural calcification 42f Accumulation of fluid 257 Acellular collagen 42 Acetyl hydrazine 213 Acetylators group 213 Acid fast bacilli 48, 110, 218 Acinar dysplasia 112 Acinetobacter species 147 Acquired immunodeficiency syndrome 5 Actinomycosis 352 Acute exacerbation of COPD 120, 126 Acute inflammatory demyelinating polyneuropathy 144 Acute myocardial infarction 2, 118, 235 Acute onset of dyspnea in SLE 156 bilateral dense centrilobular nodules 158f bilateral diffuse nonhomogeneous opacities 157f case report 156 discussion 156, 158, 159 management 159 methylprednisolone sodium succinate 159 Acute respiratory distress syndrome 148 Acute respiratory failure life-threatening 113 non-life-threatening 113 Acute silicoproteinosis 71 Adenocarcinoma 364 Adenocarcinoma of lung 39, 344, 346, 362, 364, 389

Adenomatoid proliferation 111 Adhesive collapse 332 Aeroallergens 89, 91 Air-bronchogram 152, 285, 286, 337, 338 Air hunger 11 Alae nasi 14, 294 Albendazole 65-66, 292-293, 312 Alkaline phosphatase 321 Allergic angiitis granulomatosis 93, 198 bronchopulmonary 88, 94, 198 rhinitis 7, 31 Alpha fetoprotein 382 Alpha-1 antitrypsin deficiency 1, 102 Alprazolam 110, 347 Alveolar macrophages 34 Alveolar microlithiasis 45 Alveolar proteinosis 152 Amikacin 147, 327 Aminoglycosides 298 Amosite 43 Amoxicillin–clavulanic acid 297 Amphiboles 43 Ampicillin-sulbactam 298 Amyloidoma 37 Anaerobic empyema 264 Analysis of symptoms 2 Anaplasia 267 Anaplasia of tumor 133 Anchovy sauce 271 Anhidrosis 366 Ankylosing spondylitis 11, 78 Anorexia 5 Anterior mediastinal mass 380 case report 380 discussion 381, 383, 384 fat densities pushing heart to right 382f large mass in left hemithorax 381f management 384 obscuring retrosternal space 381f Anticoagulation agents 235 Anti-cyclic citrullinated peptides 202 Antifibrotic effects 173

400

101 Cases in Respiratory Medicine Antiphospholipid antibody syndrome 232 Antipseudomonal fluoroquinolones 298 Anti-pseudomonas antibiotics 117, 265 Antipsychotic drugs for obsessive compulsive disorder 249 Antiretroviral treatment 256 Antitubercular drugs 79, 92, 231, 236 ethambutol 230 isoniazid 230 pyrazinamide 230 rifampin 230 Antitubercular medicine with INH 212 Antitubercular treatment 221 Anxiolytic drug 224 Aortic body paraganglioma 383 Apical fibrosis 57 impulse 16 Apnea 12 Apoptosis 96 Appetite 5 Arabino-galactan layer 34 Arterial blood gas 114 Arteriovenous malformation 36 Arthralgia 156, 162, 164 Asbestosis 171 Aspergilloma 47, 78 Aspergillus fumigatus 93, 94 Asplenia 298 Asthmatic patient came with hemoptysis 90 bilateral central bronchiectasis 94 bilateral perihilar opacity 93f case report 90 discussion 91, 92, 93 management 94 Asymptomatic lady wants to go home 285 case report 285 discussion 285, 286 management 286 Asymptomatic man with hilar calcification 33 case report 33 management 35 primary complex with large hilar 33f Asymptomatic SPN in smoker 38 case report 38 discussion 39, 40 heterogeneous SPN in right 39 management 40

Asystole 63 Atelectasis 338, 367 Atorvastatin 341 Atrial septal defect 44 Atrial septal defect with multiple nodular opacities 388 case report 388 CECT-thorax dilated pulmonary arteries 390f dilated pulmonary arteries 390f discussion 389, 391 management 391 Atypical bacterial coverage 217 Atypical carcinoma 373 Atypical mycobacteria 111 Atypical mycobacterial infection 209, 240 Auramine-Rhodamine staining 225 Auscultation 20 added sound 22 breath sound 20 vocal resonance 21 Autoamputation 164 Autoinflammatory disease 96 Autologous bone marrow transplantation 202 Autophagy 96 Axillary lymph node 277 Azathioprine 157, 178, 358 Aztreonam 328 Azygoesophageal recess 101

B Bacilli 34 Back examination of 14 scapula 15 shoulder 14 spine 15 Bacteroides fragilis 319 Bagassosis 182 Banyan tree 41 Barrel chest 18 Benign pattern of calcification 37 teratoma 384 tumor 38, 383 Berylliosis 211 Beta-blocker atenolol 165 Beta-human chorionic gonadotropin 394 Bilateral airspace opacities 158

Index basal 176 consolidation 159 bilateral lower zone consolidation 160f case report 159 discussion 161, 162 lower lobe consolidations 162f management 162 treatment and outcome 160 diaphragmatic palsy 283 diminished thoracic expansion 354 empyema thoracic 281 boil in the back 284f case report 281 discussion 282, 283, 284 management 283 extensive miliary opacities 226 hilar lymphadenopathy 209, 211 case report 48 discussion 49, 50, 51 management 50 with ill-defined parenchymal opacities 49f hyperresonant nodes 354 perihilar 152 alveolar opacity 89 opacities 152 pneumonia 162 pneumothorax 285 polyphonic rhonchi 99 reticulonodular dense 44 reticulonodular opacities 80, 188 Bi-level positive airway pressure 113 Biliary surgery 321 Bilious expectoration 309f case report 309 cavity with fluid level in right lower zone 311f discussion 310 lung abscess in right lower lobe 311f management 312 multiple fluid-filled cysts 312 Biot breathing 11, 12 Biphasic cells 386 Biphasic crepitation 66 Bleomycin 165 Blood biomarker levels 395, 396 Blood pressure 6 Blood-stained expectoration 157 Blouse 219 Blunt non-penetrating trauma 60 Body mass index 6

Borders of lung lobules 363 Bradypnea 11, 12 Brain metastases 396 Breathing, different patterns of 12 Bromocriptine 42 Bronchial adenoma 328, 373 Bronchial breath sound 21 amphoric breath sound 21 high-pitched bronchial 21 low-pitched bronchial 21 Bronchial carcinoid tumor 373 Bronchiectatic cavity 111 segments 22 Bronchiolar walls 204 Bronchiolitis obliterance - organizing pneumonia 185, 204 Bronchioloalveolar cell carcinoma 2, 37, 326 Bronchoalveolar lavage 50, 116, 159, 228, 262 Bronchoalveolar lavage 50 Bronchobiliary fistula 310 Bronchogenic carcinoma 6, 85, 226, 328, 341, 360, 368 Bronchogenic cysts 290 Bronchophony 21 Bronchopleural fistula 109, 238, 279, 316, 326 Bronchopulmonary segment 171 Bronchorrhea 326 Bronchoscopic biopsy 37, 307f, 357 Bronchoscopic lavage 185, 247 Bronchoscopy 82 Bronchovesicular 21 Buccal cavity 7 Bullae 288 Bullous emphysema 290 Bupropion 340

C Cachectic 299 Calcification in solitary pulmonary nodule 35 case report 35 discussion 35 management 37 solitary pulmonary nodule 36f Calcinosis cutis 165 Calf muscle tenderness 232 Candidiasis 8

401

402

101 Cases in Respiratory Medicine Candle wax 41 Capillary blood glucose 62 Capitonnage 65 Carbapenems 320 Carboplatin 343, 367, 375 Carcinoid syndrome 374 Carcinoid tumors 373, 386 Cardiac diseases 98 Cardiac percussion 19 Cardiac silhouette 362 Cardiogenic pulmonary edema 90, 152 Cardiophrenic angle 53, 362 Cardiothoracic surgeon 391 Cardiovascular, abnormalities palpation of 17 Carina 265, 362 Cartridge based nucleic acid amplification test 92, 185, 376 Caseating epithelioid granuloma 376 Caseating necrosis 218, 219 Caseous necrosis 34 Castleman disease 202 Catalase-peroxidase 215 Cavernous 21 Cavitary tuberculosis 78, 209 CECT-thorax mediastinal window 39f Cefepime 147 Cefixime 225 Cefoperazone 280, 327 Cefoperazone sulbactam 110, 160 Cell division 29 Cell mediated immunity 34 Cell wall 34 Central caseous necrosis 34 Central mediastinum 354 Central venous catheterization 59 Centriacinar emphysema 195 Centrilobular 71 Centrilobular nodules 178, 182, 218, 378 Cervical lymphadenopathy and pneumothorax 187 atypical histiocytes 190f bilateral reticulonodular opacities 189f case report 187 cystic spaces of bizarre shaped 190f discussion 189, 191 extensive scarring and sinus formations 188f management 192 Cervical spondylitis 85, 358, 364 case report 364

discussion 366, 367 management 367 MRI scan of cervical spine 365f Changing diagnosis 263 case report 263 discussion 264, 266 management 266 nodule in the trachea at the level of carina 265f Chest and deformity shape of 11 barrel chest 11 chest wall bulging 13 funnel chest 13 pigeon chest 12 thoracic kyphoscoliosis 13 Chest pain 3, 330 Chest radiology 33 Chest wall expansion 17, 18, 282 Chest wall in the axillary area 258 Chest wall swelling case report 349 discussion 350, 351, 352, 353 homogeneous opacity in right mid zone 350f management 353 tumors 352 Cheyne–Stokes breathing 11, 12 Chlamydia pneumoniae 296 Chlamydia psittaci 296 Chloroquine 157 Cholecystectomy 33 Cholelithiasis 320 Choriocarcinoma 383 Choroidal tubercle 228 Chronic aspiration 171 Chronic berylliosis 210 Chronic cholecystisis 38 Chronic hypersensitivity pneumonia 171 Chronic infections 226 Chronic nonspecific symptoms 51 case report 51 discussion 52 management 54 mass lesion in right cardiophrenic angle 54f right lower zone opacity 52f Chronic obstructive pulmonary disease 2, 28, 170, 368 Churg-Strauss syndrome 88, 93, 187 Cicatricial collapse 332 Ciliospinal reflex 366

Index Ciprofloxacin 147, 247, 298 Cirrhosis 13, 25, 270 Cisplatin 165, 367, 374, 375 based regimes 332 Clarithromycin 183, 247, 297 Classical tuberculous pleural effusion 257 case report 257 discussion 257 left-sided pleural effusion with leftsided transverse fissure 259f management 259 uniqueness 259 Clindamycin 264, 298 Clinical data analysis eighth step clinical expertise 30 fifth step positive and negative clues 28 first step collection of clinical information 26 fourth step course of clinical manifestation 28 fundamental governing theories of clinical medicine clinical bias 31 diseases flock together 31 evidence-based medicine 31 explain by single pathology 31 ninth step clinical reasoning 30 second step piercing through individual manifestation 27 seventh step clinical setting 29 seventh step epidemiology of disease 29 sixth step pattern identification 29 tenth step clinical judgment 30 third step grouping of clinical data 27 Clonidine 340 Clubbing 7 Coamoxyclav 299 Coarse crackles 22 Coccidioidomycosis 211 Coin test 24 Collapse lung border 57 Common presentation of a rare disease in adult 108 Community-acquired pneumonia 296, 314 Comorbidities 31 Concave lateral margins 221 Concentric calcification 37 Congenital cystic adenomatoid malformation 111

Connective tissue diseases 171 Contralateral chest wall metastasis 352 COPD patient with type II respiratory failure case report 123 discussion 124, 125, 126 management 125 refined ABCD assessment tool 124f with raised left dome of the diaphragm 353 case report 353 discussion 354, 356 management 357 raised left dome of the diaphragm 355f, 356f with rapid change in chest radiology 112 absence of nodular opacity 115f bilateral reticulonodular opacities 114f case report 112 discussion 113, 115 management 116 Cor pulmonale 6 Costochondral joints 15 Costophrenic angle 66, 260, 279, 333 Costophrenic sulci 195 Cotrimoxazole 247 Cough 3 Covert clue 28 Coxiella burnetii 296 Crackles biphasic crackles 23 bronchiectasis 23 early inspiratory crackles 23 late inspiratory crackles 23 post-tussive crackles 23 Craniocaudal predilection 186 Crepitation 22 Crocidolite 43 Crowding of ribs 17 Cryotherapy 375 Cryptogenic organizing pneumonia 186 Cuirass 120 Cutaneous lesions 13 Cyanosis 7, 8 Cyclophosphamide 163, 192, 375, 379, 380 Cycloserine 238 Cysplatin 266 Cystic adenomatoid malformation 292

403

404

101 Cases in Respiratory Medicine Cystic fibrosis 5, 107 Cystic hygroma 383 Cytochromal enzyme 213 Cytokines 282

D Decortication 238 Decubitus position 6, 20, 78, 316, 317, 326 Deep hyperchromatic nucleus 335 Deep vein thrombosis 7, 118-119, 232234f Deglutition 85 Delayed-type hypersensitivity 34 Dendritic cells 34 Dense reticulonodular opacities 70 Dermatome nerve supply 258 Dermatomyositis 185 Dermoid cyst 63, 64, 383 Dermoids 371 Desquamative interstitial pneumonia 200 Dexamethasone suppression test 131 Dextrocardia 19 Diabetes insipidus 191 Diabetic ketoacidosis 145 Diaphragmatic hernia 288 Diffuse alveolar hemorrhage 158 Diffuse interstitial lung disease with severe chest pain 357 bilateral basal reticular opacities 357f case report 357 discussion 358, 360 erosions of left 2nd and 3rd ribs 359f honey combing at both lung bases 358f management 360 opacity encircling left apex of lung 359f Diffuse parenchymal lung disease 164, 166, 169 bilateral reticular patterns with fuzzy cardiac 169f case report 169 discussion 169 discussion 172, 173 management 172 predominantly reticular pattern 171 presented like tuberculosis 183 bilateral mainly reticular opacities 184f

bilateral reticular opacities 186f case report 183 discussion 184, 185 management 187 step II 185 with changing diagnosis 174 case report 174 discussion 174, 177, 178 management 179 relatively clear lung bases 177f reticular opacities 175f with systemic manifestation 164 bilateral ground-glass opacities 166f bilateral reticulonodular opacities 166f case report 164 discussion 164, 167 management 168 sclerodactyly and autoamputation 164f Discoid rash 156, 206 Domperidone 273 Dopamine 139 Doxorubicin 375, 379 Doxycycline 247 Draining bronchioles 247 Drug-induced lung diseases 171 Drugs causing PIE chlorpropamide 198 gold salts 198 hydralazine 198 indomethacin 198 isoniazid 198 nitrofurantoin 198 penicillin 198 sulfonamides 198 thiazides 198 tricyclic antidepressants 198 Dry-powder inhaler 97 Dyslipidemias 97 Dysmotile cilia syndrome 107, 108 Dysphagia 60 Dyspnea 4, 6, 183, 208, 267, 294, 299, 330, 333, 368

E Early onset COPD or late onset asthma 98 case report 98 discussion 99, 100

Index management 102 paraseptal emphysema 101f Ecchynococcus multilocularis 292 Ectoderm 382 Edema 7 Efavirenz 155, 256 Eggshell calcification 71 Egophony 21 Elderly COPD patient with leukocytosis 117 case report 117 discussion 117, 118, 119 management 119 Elderly lady had a concealed truth 266 case report 266 discussion 267, 269 management 269 Elderly lady with respiratory acidosis case report 137 discussion 137, 138, 139 final outcome 139 Elderly man with acute over chronic respiratory insults case report 66 discussion 66, 68 fibrocalcified lesion in right upper zone 67 insults management 68 Elderly man with metabolic alkalosis 131 case report 131 discussion 131, 133, 134 management 133 PET positive nodule in right upper lobe 132f small nodule in right upper lobe 132 Elderly man with right lung collapse case report 330 discussion 330, 332 management 332 with metastases in left lung 331f Elevated serum lactate 380 Embolectomy 235 Embryonal carcinoma 383 Empyema fluid 254 Empyema necessitatis 14 Emtricitabine 155, 256 Encysted pleural effusion 283, 335 Encysted pleural effusion with liver cyst 274 case report 274 discussion 275, 277

management 277 massive encysted pleural effusion 276f right-sided encysted pleural effusion 274f Endobronchial hamartomas 37 spread of tuberculosis 218 tuberculosis 291, 328 ultrasonography 50, 209, 251 Endocyst 63 Endoderm 382 Endogenous cytokines 4 Endothelium 150 Endotracheal aspirate 147 electrocautery 123 Enophthalmos 366 Enoxaparin 233, 235 Entamoeba histolytica 272 Enterobacteriaceae 147 Enucleation 65 Enzyme-linked immunosorbent assay 64 Eosinophilia 291 Eosinophilic granuloma of bone 191 inflammation 100 lung disease 187 Epidemiologic clues 296 Epigastric suction 14 Epithelioid cells 34 Epithelioid granuloma 218 Epitrochlear 6 Epituberculosis 35 Erythematous circular 206 Escherichia coli 148 Esophagoscopic procedures 59 Etamsylate 328 Etanercept 154 Ethambutol 305 Ethionamide 238 Ethmoidal sinus 8 Etoposide 192, 266, 375 Excessive mineralocorticoid secretion 133 Exertional dyspnea 184, 202 Exogenous pyrogens 4 Expectoration 3 Expiratory positive airway pressure 120 Extended-spectrum beta-lactamases 328

405

406

101 Cases in Respiratory Medicine Extensive ground glass abnormality 170 Extensive peribronchiolar macrophage 201 Extrapulmonary tuberculosis 214, 231, 245 Extravascular compartment 139, 300 Extrinsic allergic alveolitis 182 Exudative pleural effusion 215

F Facial anhidrosis 366 FDG-PET 37 Fever 4 Fiberoptic bronchoscopy 59, 265, 328, 332, 355, 373 Fibrinolytic therapy 234, 235 Fibrocavitary disease 316, 326 Fibrocavitary lung disease 106 Fibro-optic bronchoscopy 122 Fibrosing mediastinitis 307, 369 Fibrothorax 360 case report 360 discussion 362 encysted pleural effusion 363f lung mass the level of carina 364f lung window of CT-thorax 363f management 364 opacity 361f Fibrotic sarcoidosis 78 Fibrous layer 63 Fine-needle aspiration cytology 39 Flattening 13 Fluroquinolone 237 Fluticasone 113, 205 Fondaparinux 235 Forced expiratory time 24 Formoterol 97 Fossa ovalis 389 Fractioned radiotherapy 375 Fracture neck femur 83 case report 83 discussion 84, 86 homogeneous opacity 85f management 85 surrounding rarefaction of bones 84f Frank blood 372 Frank empyema 254 Frequent exacerbations of COPD bronchiectatic changes 104f case report 102

discussion 103, 104 management 105 Frontoparietal 395 Fundal gas 61, 107 Fungal antigens 198 granuloma 37 spores 182 Fusobacterium 319 Fusobacterium nucleatum 319 Fuzzy cardiac 169

G Gallbladder surgery 35 Ganglions 366 Gas under the diaphragm 320 case report 320 discussion 321, 323 management 322 obliterating right costophrenic angle 322f Gastroesophageal reflux disease 88, 291 Gentamicin 147 Germ cell tumor 220, 383, 386 Glucocorticosteroids 157 Glycopyrrorium combination 97 Goodpasture’s syndrome 158 Granulomatous inflammatory diseases 386 Group A Streptococcus 265 Guillain-Barré syndrome with ventilatory failure 144 case report 144 discussion 144, 146, 147 management 146 Gum bleeding 8 hypertrophy 8 Gynecomastia 14

H Haemophilus influenzae 155, 265, 296, 313 Hamartoma 37 Hampton’s hump 233 Hand-Schüller-Christian disease 191 Healthcare-associated pneumonia 296 Hematogenous 63 Hemithorax 18, 360

Index Hemodynamic disturbance 288 instability 122 Hemoptysis 3 Hemorrhagic pleural effusion 261 Hemorrhagic pleural effusion with low adenosine deaminase 260 case report 260 discussion 260, 262 management 262 Henoch-Schönlein purpura 158 Hepatitis 25, 155, 29, 212-213 WHO graded drug-induced 212 Hepatobilliary fistula 272 Hepatocellular carcinoma 214 Hepatomegaly 19 Hepatosplenomegaly 261, 333 Hepatotoxic drugs 213, 237 Hereditary hemorrhagic telangiectasia 75 Heterogeneous opacities 110f, 237 High density lipoprotein 141 High-frequency ventilation 150 Highly active antiretroviral therapy 155 Hilar enlargement 211 Hilar glandular 35 Hilar lymph nodes 34, 35 Hilar lymphadenopathy 49 Histoplasma antigen 308 Histoplasma capsulatum 306 Histoplasma yeasts 308 Hoarseness of voice 343 Holter monitoring 62 Honeycombing 176 Horner’s syndrome 366 Hospital-acquired pneumonia 296 Humidifier lung 182 Hydatid cyst 63, 276 Hydatidosis 292 Hydrocortisone 335 Hydrophilic molecules 34 Hydropneumothorax 16, 55, 56, 57 Hydroxychloroquine 157, 160 Hydroxyurea 380 Hypercarbia 114 Hyperchromatic cells 394 nuclei 396 nucleus 275 Hypercoagulability state 232 Hyperhomocysteinemia 232 Hyperlucency 287

Hyperlucent right lung 287 case report 287 discussion 288, 289 management 290 Hyperpnea 12 Hypogammaglobulinemia 107, 386 Hypokalemia 131 Hypothyroidism 4 Hypoxia with bilateral lung infiltrates 151 bilateral parahilar opacities 153f case report 148 case report 151 discussion 151, 152, 154, 155 management 155 respiratory alkalosis 152 Hypoxic respiratory failure discussion 148 lower zone paracardiac opacity 148f management 150

I Idiopathic interstitial fibrosis 171, 176, 200 IgE-mediated hypersensitivity 198 Illicit drug use 296 Imipenem 147, 298, 320 Immature stromal tissue 394 Immunosuppressant drugs 157 Indacaterol 97 Infarction or granuloma 37 Inflam-aging 31 Inflammatory granulomas 37 Infliximab 154 Influenza 29 Influenza-like prodrome 186 Infraaxillary 335 Infraclavicular areas 16, 179 Inframammary 74, 270, 335 Infrascapular areas 17, 253, 270, 278 Inguinal 6 Inhaled corticosteroids 96 Inhaled salmeterol-fluticasone combination 137, 140 Inspection 8 Inspiratory flow resistance property 150 muscles 14 phase 21 positive airway pressure 120 Intercostal muscle paralysis 11

407

408

101 Cases in Respiratory Medicine Intercostal suction 6 Interleukin-1 282 Interlobular lines 363 Interlobular septal thickening 71 Intermediate neuroendocrine carcinoma 373 Interscapular area 278 Interstitial alveolar infiltrate 182 Interstitial lung disease 2, 169, 283, 358 Interstitial pneumonia 1, 177 Interventricular septum 118 Intervertebral disk 279 Intrabronchial hamartoma 37 Intracellular signaling 173 Intraluminal blockage 329 causes 328 growth 362 lesion 39, 110 organizing fibrosis 187 tumor 375 Intrathoracic lymphoma 263, 379 Intrathoracic malignancy 43 Intravascular compartment 300 metastasis 219 Intravenous methylprednisolone 140 Invasive aspergillosis 78 Invasive mechanical ventilation 124 Ipratropium 113, 120, 135 Ipsilateral hilar lymphadenopathy 226 prominence 226 Ipsilateral lung metastases 352 nodules 352 Irinotecan 375 Iron lung 120 Ischemic heart diseases 97 Isoniazid 305 Itraconazole 168, 308

J Jacket ventilator 120 Japanese scientist for panbronchiolitis 219 Jaundice 7, 322, 368 J receptor 4 Jugular venous pressure 139 Juxtaphrenic lung 195

K Kanamycin 238 Keratoconjunctivitis 51 Keratotic scaling 206 Ketoacidosis 11, 12t Klebsiella infection 3 Klebsiella pneumoniae 109, 280, 296, 327, 328, 337 Kronig’s isthmus 19, 79 Kussmaul breathing 11, 12 Kyphosis 11, 15

L Lactic dehydrogenase 382 Laminated hyaline membrane 63 Lamivudine 155, 256 Landouzy septicemia 228 Langerhan giant cells 34 Langerhans cell histiocytosis 190 Langerhans cells 191 Langerhans histiocytosis 201 Laparotomy 320 Larynx 8 Laser therapy 375 Lassitude 261 Latissimus dorsi 14 Left hemithorax 259, 260, 285, 335 Left hilar lymphadenopathy 351, 352 Left infrascapular area 327, 372 Left lower interscapular area 257 Left lower lobectomy 111 Left midaxillary line 257 Left midclavicular line 257, 372 Left sided air-fluid level case report 55, 56 discussion 55, 57 management 58 stomach in the thorax 57f Left ventricular ejection fraction 97 failure 88 function 3 Left-sided pleural effusion 333 case report 333 discussion 334, 335 left-sided homogeneous opacity 333f management 335 Legionella pneumonia 222 Legionella pneumophila 296, 337

Index Legionella species 296 Leprosy 210 Letterer-Siwe disease 191 Leukamoid blood picture 313 Leukocytosis 138, 139, 147 Leukopenia 147 Leukoplakia 8 Leukotriene 134 Levofloxacin 146, 147, 238, 297, 298 Levosalbutamol 127 Life-threatening episode associated with tuberculosis 229 bilateral patchy opacities 230f case report 229 discussion 230, 232, 233 extensive deep vein thrombosis 234f final diagnosis 235 management 233 pleural-based triangular opacity 234f Ligamentum arteriosum 341 Ligeonella 296 Linezolid 147, 284 Linezolid 298 Lingular segment 176 Lipase levels 261 Little mucoid expectoration 353 Liver disease 13 enzyme levels 321 function test 249 metastasis 25 Lobectomy 65, 133, 391 Lobulation 221 Localized rhonchi in adult 344 air-tapping in right upper lobe with right hilar mass 345f case report 344 discussion 344, 346 hyperlucency in right upper zone 345f Localized rhonchi in adult management 346 Loeffler’s syndrome 187, 197, 198 Long-acting beta-2 agonist 96 Long journey seeking treatment 236 case report 236 discussion 236, 237, 238 management 238 Low-pitched rhonchi 17 Luminal block 328 Lung abscess 326

cancer 13, 39 infections 294 interstitium 203 malignancy 363 parenchyma 82 parenchymal lesion 35, 222 window 362 Lymph node 63 Lymphangiomyomatosis 182 Lymphocyte neutrophil 259, 261 Lymphocytic 267 exudative 376 pleural effusion 254 pleural fluid 260 Lymphohematogenous route 228 Lymphoid interstitial pneumonia 202 Lymphoma 25 Lymphoplasmacytes 172 Lymphoreticular system 25, 267

M Macleod’s syndrome 81, 288 Macroaspiration 7, 317 Macronodule 214 Malar rash 156, 167 Malignancy 37 Malignant germ cell tumors 383 lymph nodes 6 teratoma 383 tumor 366 Mallampati classification 8 Man with chest wall mass 346 case report 346 discussion 347, 349 heterogeneous mass in left hemithorax 348f homogeneous opacity in left mid zone 348f management 349 Mandible 191 Mantoux test 50, 82, 209, 220, 249 Manubrium 368 Masked metabolic acidosis in ICU 129 case report 129 discussion 129, 130 final outcome 130 Massive hemoptysis with past history of pulmonary TB 75 case report 75 change of position of air crescent 77f

409

410

101 Cases in Respiratory Medicine discussion 76, 77 management 78 non-homogeneous opacity in left upper zone 76f Mature teratoma 383 Mean arterial pressure 63 Mediastinal adenopathy 307 emphysema 13 fat line 382 germ cell tumor 335, 383 lymph nodes 307 lymphadenopathy 222 mass 382 pain 3, 85 percussion 20 pleura 367, 383 shifting 253, 258 window 53 Mediastinum 16 Mediatinal tumor 382 Melioidosis 29 Mendelson’s syndrome 317 Meninges 229 Meropenem 127, 320 Mesenchymal tumor 383 Mesoderm 382 Mesothelial cell 254, 258, 277 Mesothelioma 261, 267, 360, 364 Metabolic alkalosis 131, 145 Metabolic syndrome 141 Metachronous 398 Metamyelocyte 313 Metastatic liposarcoma 37 Metformin 140 Methicillin resistant Staphylococcus aureus 298 Methotrexate 42, 157, 192, 308 Methyl prednisolone 172 Methylprednisolone 129, 159, 162 Methysergide 42 Metronidazole 273, 320, 322 Microaspiration 7 Microbiologically confirmed tuberculosis 230 Microconidia 307 Microliths 45 Midclavicular line 263, 310, 368 Middle-aged lady with a round opacity 222 case report 222 discussion 224

management 224 with central lucency 223f Middle-aged man presented with consolidation 336 case report 336 discussion 337, 338 management 338 sharp lateral margin in right lower zone 336 Mild centrilobular nodules 201 Miliary opacities 228 Miliary tuberculosis 69, 179, 228, 229 Mimic thymoma 386 Mineralocorticoid excess syndrome 133 Minocycline 247 Miosis 366 Mitomycin 42 Mitral stenosis 44, 184 Mixed connective tissue disease 167 Mixed density inhomogeneous 395 Monitoring intrinsic peep 125 Mononuclear cells or neutrophils 203 Monophonic rhonchi 344 wheezing 22 Montelukast 134 Moraxella catarrhalis 296, 313 Morgagni hernia 53, 63 Mosaic attenuation pattern 204 Movement of chest 16 Moxifloxacin 247, 298 Mucoid expectoration 219, 229, 245, 247, 263, 330, 336 Mucopurulent expectoration 326, 388 Müller’s muscles 366 Multidrug resistant (MDR) TB 238 amikacin 238 cycloserine 239 ethionamide 239 fluoroquinolone 239 kanamycin 238 kapreomycin 238 Multilobar involvement 112 Multiorgan dysfunction syndrome 139 Multiple air-fluid levels 313 case report 313 discussion 313, 314, 315 final diagnosis 315 management 314 multiple air-fluid levels in right hemithorax 314 multiple bulla 315f

Index Multiple bullae 316 Multiple cannon ball metastases 392 Multiple cystic spaces 111 Multiple discharging sinuses 188 Multiple fluid-filled cysts 291 case report 291 discussion 291, 292 management 292 Multiple hydatid cysts 292 Multiple myeloma 352 Multiple rib erosions 242 case report 242 discussion 243, 244 final diagnosis 245 healed rib erosions 244f management 244 posterior end of right 2nd rib and left 6th rib 242f Multisystemic involving lungs 191 Multisystemic presentations 156 Multivitamine tablet 309 Myalgia 261 Myasthenia gravis 386 Mycobacterial growth indicator tube 240 Mycobacterial protein 215 Mycobacterium intracellulare 247 Mycobacterium tuberculosis 34, 92, 208, 211, 238, 240, 376 Mycolic acids 34 Mycoplasma pneumoniae 296 Myocardial pain 3

N Nasal cavity 7 polyp 7 speculum 7 Nasogastric tube suction 59 National Tuberculosis Control Program 79 Nebulized bronchodilator 113, 135 Necrotizing pneumonia 217, 218, 319 Neucleotide reverse transcriptase inhibitors 155 Neuroendocrine tumors 373 Neutralize acidosis 130 Neutralize gastroesophageal reflux 173 Neutrophil 137 Neutrophilic leukocytosis 103, 237, 270 Nevirapine 155, 256

Nintedanib 173 Nintedanib inhibits tyrosine kinases 173 Nitrofurantoin 171 Nocardia infection 209 Nocardiosis 218 Nodular opacities 43, 363 case report 43 dense nodules of alveolar microlithiasis 45f discussion 43 ill-defined parahilar opacities 44f lung opacities with relatively lucent heart 45f management 46 Nodules behaving differently 391 case report 391 CT-brain, multiple metastases 396f discussion 392, 394, 395, 397 FNAC from right upper lobe mass 397f large heterogeneous mass in right upper lobe 393 management 395 multiple cannon ball opacities 392 multiple contrast-enhanced masses of different sizes 393f persistence of right upper lobe tumor 395f testicular tumor 397f Non-invasive ventilation 89, 120 Nonnucleoside reverse transcriptase inhibitors 155 Nonresolving pneumonia 338 Non-responding dyspnea 87 case report 87 disappearance of round tumor 90 discussion 87, 88 management 90 round opacity in right mid-zone 89f Non-small cell lung cancer 332 Nonspecific interstitial pneumonia 167, 177 Nontender lymph nodes 276 Normal alveolar-arterial oxygen 146 Nortriptyline 340 Nose and sinuses 7 Nuclear-cytoplasmic ratio 29 Nucleolar pattern 206

O Obese lady with metabolic syndrome 140 case report 140

411

412

101 Cases in Respiratory Medicine discussion 140, 143 management 143 right lower zone homogenous opacity 141f Obstructive airway diseases 87 Obstructive lung disease 96 Obstructive sleep apnea 143 Old man with suspected lung cancer 323 Old man with suspected lung cancer aortic indentation over esophagus 325f case report 323 discussion 324, 325 management 324 right paratracheal opacity 324f Omazulimab 134 Omeprazole 172 Oncogenes 398 Opaque left hemithorax 384 case report 384 discussion 385, 386, 387 large homogeneous mass 387f lymphoid cells suggestive of thymoma 387f management 386 shifting of mediastinum to right 385f Oral cavity 8 Orchiectomy 335 Ornidazole 273 Oropharyngeal secretions 317 Osteomyelitis 352 Osteomyelitis of distal phalanges 165 Osteosarcoma 6, 37 Oval pleomorphic 396 Overt clue 28

P Paclitaxel 367 Palliative radiotherapy 343 Pallor 7, 368 Panbronchiolitis 115 Pancoast syndrome 366 Pancoast tumor 366, 367 Pancreatitis 1, 261 Pancytopenia 162 Pantoprazol 309 Paracetamol 159, 217, 281, 309, 313, 320-321 Paradoxical movement 14 Paraneoplastic syndrome 133 Parapneumonic effusion 217, 267, 279

Paraseptal emphysema 101 Parasitic cysts 290 Parathyroid adenoma 383 mass 53 Parenteral anticoagulant 235 Parietal pericardium 367 pleura 42 Parieto-occipital region 395 Paroxysmal nocturnal dyspnea 91 Partial eventration 196 Peak expiratory flow rate 134 Peak inspiratory pressure 150 Pectus excavatum 16 Pedal edema 4, 368 Pegion shaped 106 Penetrating chest trauma 59 Penicillamine 204 Peptidoglycan layer 34 Percussion 18 hydropneumothorax 20 in Kronig’s isthmus 19 liver dullness 19 of back 19 through midaxillary line 19 traube’s space 19 Periadenitis 6 Peribronchial interstitial space 89 noncaseating granulomas 182 Peribronchovascular 50 predominance 170 Pericardial cyst 383 effusion 19, 35 invasion 386 involvements 179 pain 85 Pericarditis 206 Pericardium 3, 16, 85, 229 Perilymphatic 363 Perilymphatic and random 182 Peripheral blood examination 109 Peripheral cyanosis 7 Peripheral lung opacities 195 bilateral peripheral opacities 196f bilateral peripheral patchy consolidations 197f case report 195 discussion 196, 197 management 198

Index Peripheral lymph nodes 6 Peripheral lymphadenopathy 261, 267 Peripheral vascular opacities 390 Persistent cough in rheumatoid arthritis 202 case report 202 discussion 203 management 205 mosaic attenuation 204f smoker bilateral nonspecific opacities 200f case report 199 cystic spaces 201f discussion 199, 201 management 202 Persisting cavity despite treatment 245 case report 245 discussion 246, 248 management 248 thin-walled cavity in right midzone 246f Phagolysosomes 34 Phagosomes 34 Phantom tumor 90 Photodynamic therapy 375 Physiochemical regulation 391 Pinpricking 187 Piperacillin-tazobactam 129, 140, 264 Pirfenidone 173 Planning invasive investigation 261 Plateau pressure 125 Pleural diseases 257 effusion 73f, 310 fibrosis 41, 276 fluid examination 257 fluid favored malignancy 254 fluid glucose 162 plaque 42 rub 23 Pleuritic chest pain 261 Pleuritis 206 Pleuropericardial cyst 53, 63 Pleuropneumonectomy 238 Pleuropulmonary amebiasis 272 Pleuro–pulmonary blastoma 290 Pleuropulmonary disease 253 case report 253 discussion 254, 255, 256 left-sided pleural effusion 255f management 256

Pneumatocele 300f, 301 Pneumococcal pneumonia 3, 29, 296 Pneumocystis jirovecii 154-155 Pneumocystis pneumonia 152, 154 Pneumocystis cysts 154 Pneumonia management of antibiotics for outpatient treatment 297 duration of therapy 298 in patient admitted in ICU ward 298 in patient admitted in non- ICU ward 298 outcome of treatment 298 Pneumonia severity index 294 Pneumothorax with reticulonodular opacities 68 bilateral reticulonodular opacities 69f case report 68 discussion 69, 70 eggshell calcification in right hilum 71 management 71 Polycythemia 7, 98, 118, 341-342 Polygonal lines 363 Polymorphonuclear leukocytes 162, 182 Polymyositis 185 Polyphonic wheezing 22 Polyserositis 162 Polysomnography 143 Popliteal 6 Porphyromonas 319 Post-cardiopulmonary bypass 149 Postobstructive pneumonia 291 Post-tussive crepitation 67 Potential pathogenic microorganisms 103 Praziquantel 65 Precordium 16 Precordium intermittently 344 Prednisolone 159, 162, 174, 202, 379 Prevotella 319 Profuse micronodules 170 Profuse serous expectoration 326 Progressing pneumonia 216 case report 216 consolidation in posterior segment 218f consolidation right midzone 216f deteriorating radiological opacities 217f discussion 217, 218 management 219

413

414

101 Cases in Respiratory Medicine Progressive dyspnea 179 Progressive massive fibrosis 71, 209 Prophylactic cotrimoxazole 256 Proteinuria 206 Prothrombin 233 Proton pump inhibitors 52 Proximal extent 338 Proximal intraluminal growth 262 Pseudomonas aeruginosa 264, 265, 266, 296 Pseudotumor 214 Ptosis 366 Puerperal fever 219 case report 219 discussion 220, 221 management 221 superior mediastinal widening with patchy opacities 220f Pulmonary arterial hypertension 167, 389 Pulmonary capillary wedge pressure 149 Pulmonary edema 152, 153 Pulmonary infarct 36 Pulmonary Langerhan’s cell histiocytosis 171, 191, 200-201 Pulmonary lymphangioleiomyomatosis 193 Pulmonary neoplasms 373 Pulmonary thromboembolism 2, 28, 73, 118, 232 Pulmonary thromboendarterectomy 235 Pulmonary toxicity 375 Pulsation 14 Pulse 6 Pure intrathoracic lymphoma 261 Pure red cell aplasia 386 Purpuric spot 13 Purse-lip breathing 14 Pyloric obstruction 341 Pyogenic cavities 318 Pyogenic empyema 281 Pyogenic infection 260, 318, 342 Pyogenic liver abscesses 214 Pyogenic respiratory infection 4 Pyrazinamide 305

Q Quantitative bacterial cultures 296 Quantitative-culture approach 147 Quinacrine 157

R Radiation-induced lung diseases 171 Radiolucent 287 Radiolucent area 321 Radiolucent left hemithorax 60 Rare cause of shock 62 case report 62 discussion 63, 65 empty cyst at right cardiophrenic angle 62f filled up cyst at right cardiophrenic angle 65f Rasmussen’s aneurysm 47 Raynaud’s phenomenon 165, 167, 168 Recurrent hemoptysis 371 case report 371 complete clearance of opacities 374f discussion 372, 373, 375 heterogeneous opacities in left lower lung field 372f management 374 opacity in left hilum 373f Recurrent pneumothorax 192 bilateral lower zone opacities 194f bilateral thin-walled round cysts 194f case report 192 discussion 193 management 195 Recurrent right paratracheal lymphadenopathy 249 case report 249 discussion 249, 251, 252 large partly necrotic lesion 252f management 251 paratracheal mass with areas of low attenuation 250f paratracheal opacity 252f right paratracheal opacity 250 Relaxation collapse 332 Respiration 11 respiratory depth 11 respiratory rate 11 special character 11 types of respiration 11 Respiratory alkalosis 145, 232 bronchiolitis 200, 201, 204 diseases 33 disorder 125 fluoroquinolone 297 problems in intensive care unit 117

Index system clinical approach 1 system different patterns of breathing 12 system examination of 7, 25 gastrointestinal 25 genital system 25 lower respiratory system 8 lymphoreticular 25 nervous system 25 upper respiratory tract 7 system general survey 5 system history taking 1 chief complaints 2 family history 5 history of present illness 2 past history 5 personal history 5 Reticulonodular opacities 169 pattern 182 Retrosternal chest pain 330 extension 371 goiter 220, 221, 382 thyroid 371 Revised National Tuberculosis Control Program 219, 230 Revisiting a diagnosed case 205 bilateral nodular opacities 207f case report 205 centrilobular in distribution 207f conglomerated opacity 208f discussion 205, 206, 208, 209 management 209 Rheumatoid arthritis 161, 171, 202 pleural effusion 29 pleuritis 254 Rheumatologic symptoms 307 Rhinophyma 7 Rhodamine stain 224 Rhonchi 22, 89 Rhonchus 22 Rifampicin 305 Right costophrenic angle 300 Right hilar lymphadenopathy 344 Right lower lobe lung abscess 316 bodyache 316 case report 316 discussion 316, 317, 319 large thick-walled cavity 318f management 320

Right midclavicular line 354 Right-sided loculated empyema 269 case report 269 discussion 270, 272, 273 management 273 reddish-brown fluid 271f right-sided multiloculated hydropneumothorax 271 subphrenic abscess 270 Rigid bronchoscope 222 Rupture hydatic cyst 63 Rupture of caseous materials 247

S Salbutamol 135, 174, 255 Salbutamol inhalation 99 Salmeterol 113 Salmeterol-fluticonazole inhaler 357 Sarcoid granulomas 215 Sarcoidosis 7, 25, 50, 51, 171, 210, 221 treatment of 51 with pleural effusion 209 bilateral hilar lymphadenopathy 210f case report 209 discussion 209, 212, 214 large liver abscess 214f management 213 Satellite lesions with tree in bud pattern 218 Scaleni group of muscles 14 Scanty mucoid expectoration 187, 384 Sclerodactyly 165 Sclerosis of finger 165 Scolices 312 Scolicidal agent 292 Scoliosis 15 Segmental collapse 373 Segmentectomy 65 Seminoma 335, 383, 394 Serositis 157 Serum alkaline phosphatase level 214 Serum angiotensin-converting enzyme 50 Severe pulmonary arterial hypertension 142 Severe sepsis 138 Shifting dullness 20 Shock with pleural effusion 72 case report 72 discussion 73, 74

415

416

101 Cases in Respiratory Medicine management 75 MRI-angiogram 74f right sided massive pleural effusion 72f Sigh breathing 12t Sign of air crescent 77 airway obstruction 103 cor pulmonale 99 d’Espine 23 dyspnea 14, 268, 282, 285 empty pericyst 64 Grave 234 Hallo 224 Hamman’s 24 hemorrhage 25 Homans’ 232 Hoover’s 15 hyperinflated lung 103 inflammation 8 Knuckle sign 15 long-standing pain 46 case report 46 discussion 47 final diagnosis 48 management 48 pellets inside and outside lungs apart 47f McConnell’s sign 232 metastasis 25, 39 Mimicking hydropneumothorax 58 Palla’s sign 233 pleural fibrosis 67 reverse d’Espine 23 Scratch sign 24 Signet-ring sign 106 Silhouette 318 thoracic operation 13 thorax in respiratory pathology/ diseases 9t Tripod sign 15 underlying lung collapse 275 Westermark’s sign 233 Sildenafil citrate 168 Silicosis 70 Silicotuberculosis 209 Simple case suddenly got astonishing attention case report 105 dextrocardia 107

discussion 106, 108 management 108 Simple parapneumonic effusion 281 Sjögren syndrome 202, 204 Small cell lung cancer 373 Small thoracotomy 176 Small-cell neuroendocarcinoma 373 Solitary calvarial lesion 191 Specular 206 Spider telangiectasia 13 Spirometry 35 Splenomegaly 19, 270 Splenomegly 25 Spontaneous breathing trial 125 Spontaneous pneumothoraxes 193 Sputum examination 331 Squamous cell carcinoma 251, 268, 337 Squamous epithelial cells 296 Staphylococcus aureus 147, 188, 284, 296, 301 Staphylococcus infection 100 Stereotactic body radiation therapy 339 Sternal tenderness 25 Sternomastoids 14 Stidor 24 Stone-blusting industry 207 Streptococcus pneumoniae 155, 265, 296, 313, 337 Subcostal angle 18 Subcutaneous crepitation 24 lesions 13 Subphrenic abscess 321, 323 Subpleural areas 363 Subsegmental bronchi 277 Substernal thyroid tumors 386 Subtrochanteric fracture 133 Succusion splash 23, 55 Sudden onset respiratory distress 81 case report 81 discussion 82 foreign body in right intermediate bronchus 83f management 83 Superinfections 264 Superior mediastinal opacity 221 widening 220 Superior vena cava syndrome 367 case report 367

Index discussion 368, 371 management 371 mass taking radioactive iodine 370f mediastinal widening 369f round mass pushing trachea 370f Superior vena caval obstruction 368 Supraclavicular 14 Suprasternal angle 16 suction 6 Swine flu 29 Swyer–James syndrome 290 Syndrome of 15 acquired immunodeficiency 5 acute respiratory distress 146, 148, 228 anterior mediastinal 25 antiphospholipid antibody 232 bronchogenic 25 carcinoid 374 Churg-Strauss 88, 93, 187, 197, 198 clinic-pathological 186 dyskinetic cilia 108 dysmotile cilia 107, 108 endocrine paraneoplastic 133 gastroesophageal reflux 199 Goodpasture 158, 200 Guillain-Barré 144-146 hepatopulmonary 151 Horner’s 365, 366 hypereosinophilic 197, 198 immotile cilia 108 immune reconstruction 256 Kartagener 108 Loffler 187, 197, 198 Macleod’s 81, 288, 290 Mendelson’s 371 metabolic 97, 140-143 Miller-Fisher 144 mineralocorticoid excess 131, 133 multiorgan dysfunction 139 obese lady with metabolic 140 open negative 247 Pancoast’s 366 paraneoplastic 25, 133 pulmonary infiltrates with eosinophilia 197 Rendu-Osler-Weber 75 Sjögren 202, 204 superior vena cava (SVC) 4, 6, 307, 349, 353, 367, 371, 392, 395

Swyer–James 290 systemic inflammatory response 138 Tietz’s 15, 52 upper airway cough 88, 93, 199, 291 Young 108 Systemic lupus erythematosus 205 Systemic spillage of inflammatory cytokines 356

T Tachyarrhythmias 63 Tachycardia 118 Tachypnea 12 Tachypnea 6, 11, 139 Tazobactam 147 TB bacilli 34 Telangiectasia 165 Telmisartan 315, 341 Tenofovir 155, 256 Tension pneumothorax 288 Teratoma 53, 371, 383 Terrible lymphoma 53 Tertiary syphilis 210 Testicular tumor 335, 394 Theophylline 335 Thermophilic bacteria 182 Thoracentesis 59 Thoracic malignancy 330 movement 15 sequel of past SIN 278 sequel of past SIN bronchiectasis 281 case report 278 discussion 279, 280, 281 homogeneous opacity in right mid and lower zones 278 lung abscess 281 management 280 opacity with air-fluid level posteriorly 280f Thoracoabdominal 11 Thoracotomy 371 Thrombocytopenia 206 Thrombophilia 232 Thymic cysts 386 hyperplasia 386 mass 220

417

418

101 Cases in Respiratory Medicine Thymoma 53, 386-388 Thyroid abation therapy 371 hormone 371 tissues 371 Tidal percussion 19, 354 Tinidazole 273 Tiny nodular opacities 226 Tissue hypoxia 130 Tissue plasminogen activator 235 Tissue-binding autoantibodies 156 T lymphocytes 34 Tobramycin 147 Topoisomerase antibody 166 Tortuous vessels 53 Toxemia 225, 299 case report 225, 299 discussion 226, 228, 229 management 229 miliary nodules with few areas of conglomerated lesions 227f miliary opacities 227f right hilar prominence with illdefined 226f Toxic granules 299 Tracheal movement 16 position 16 tug 16 Tracheostomy tube 121 Traction bronchiectasis 176 Tramadol 347 Tranexamic acid 110 Transbonchial lung biopsy 202 Transbronchial 176 fine needle aspiration 304 lung biopsies 228 lung biopsy reports 50 Transdiaphragmatic 63 Transforming growth factor 173 Transient ischemic attack 63 Transthoracic core needle biopsy 37 Trapezii 14 Traube’s space 19, 28, 257, 341, 354, 355, 385 Trendelenburg position 78 Triangular opacity behind heart 326 case report 326 discussion 326, 327, 328 management 328

triangular opacity behind the heart 327f Tricuspid regurgitation 161 Trilobed left lung 259 Trimethoprim-sulfamethoxazole 155 Triotropium bromide 113 Trophoblastic tissue 394 Troponine t test 235 Tube thoracostomy 263 Tubercle bacilli sensitive 251 Tubercular liver abscess 214 lymph nodes 6 pleural effusion 42 pleural management 43 Tuberculine test 35 Tuberculoma 214 Tuberculosis 2, 216 drugs 92 drugs classified 241 treatment 79 with sudden onset of dyspnea 78 bilateral nodular lesion 80f case report 78 discussion 79, 80, 81 management 81 pleural effusion 254 Tubular 21 Tumor doubling time 36 node-metastasis 39 suppressor gene 398 Tympanitic note 18 Typical lobar pneumonia 294 air-bronchogram in apical segment of left lower lobe 295f assessment of severity 294 case report 294 discussion 296, 297 examination 294 homogeneous opacity in left lower and mid zone 295f investigation 295 management 297

U Ulnar surface 17 Uncommon prevascular tumors 383 Unfractionated heparin 235

Index Unilateral pleuroparenchymal disease 376 case report 376 discussion 376, 378, 379, 380 lung window of CT-thorax 377f management 379 mediastinal window of CECT-thorax 378f Unresolved pneumonia 301 approach to nonresponding 303 case report 301 chest radiology 304 clinical manifestation 307 diagnosis 308 discussion 302, 306 examination 301 final outcome 308 invasive procedures 305 investigation 301 management 308 progressive disseminated histoplasmosis 308 progressive pneumonia 303 right paratracheal and subcarinal lymphadenopathy 305 right-sided pleural effusion with right paratracheal opacity 305 treatment 302 Unresponsive COPD 95 case report 95 discussion 96, 97 management 97 Unusual associate disease in COPD 119 case report 119 discussion 120, 123 management 123 tracheal tumor obstructing tracheal lumen 122 Upper abdominal pain 33 Upper airway cough syndrome 88, 291 Upper lobe cavity 339 case report 339 discussion 340, 341, 343 heterogeneous opacity in left upper zone 340f management 343 thick walled cavity 342f

Upper respiratory tract 147 Uremia 11 Urticarial rash 64 Usual interstitial pneumonia 27, 167, 172

V Vancomycin 146, 147, 315 Varenicline 340 Vascular anomalies 13 marking 81 Vasoactive mediators 232 Venous flow direction of 13 Ventilated patient with opaque left hemithorax 126 case report 126 collapse of left lung and endotracheal tube 128f discussion 127 management 129 Ventilator associated pneumonia 126, 146 Ventilator-associated lung injury 149 Venturi mask 124 Vesicular with prolong expiration 21 Vietnam 29 Vinblastin 192 Vincristine 375, 379, 380 Viral infections 29 Visceral pleural invasion 338 Vivax malaria 30 Vocal cord dysfunction 88 Vocal fremitus 17

W Warfarin sodium 233 Wegener’s granulomatosis 7, 78, 158 Weight loss 4 Wheeze 4, 169 Whispered pectoriloquy 21 Wright-Giemsa stains 154

X XDR tuberculosis 239, 256, 298 Xpert gene TB 251

419

420

101 Cases in Respiratory Medicine

Y Young asthmatic with deteriorating ABG bilateral intercostal tube drainage 136f case report 134 discussion 134, 135, 137 management 136 subcutaneous emphysema 136f Young man had a road traffic accident 59 case report 59 discussion 59, 60 management 61

stomach in thorax 61f tip of nasogastric tube 60 Young man with non-responsive TB 239 case report 239 discussion 239, 241 final diagnosis 241 management 241

Z Zidovudine 155, 256 Ziehl-Neelsen (ZN) stain 185, 219, 225, 239, 296